Nclex Practice Questions Nutrition

Question: Which statement made by an adult patient demonstrates an understanding of healthy nutrition teaching?

A. I need to stop eating red meat.
B. I will increase the servings of fruit juice to four a day.
C. I will make sure that I eat a balanced diet and exercise regularly.
D. I will not eat so many dark green vegetables and eat more yellow vegetables.

Answer: C. I will make sure that I eat a balanced diet and exercise regularly.

==================================================

Question: The nurse teaches a patient who has had surgery to increase which nutrient to help with tissue repair?

A. Fat
B. Protein
C. Vitamin
D. Carbohydrate

Answer: B. Protein

==================================================

Question: The nurse is caring for a patient experiencing dysphagia. Which interventions help decrease the risk of aspiration during feeding? (Select all that apply.)

A. Sit the patient upright in a chair.
B. Give liquids at the end of the meal.
C. Place food on the strong side of the mouth.
D. Provide thin foods to make it easier to swallow.
E. Feed the patient slowly, allowing time to chew and swallow.
F. Encourage the patient to lie down to rest for 30 minutes after eating.

Answer: A. Sit the patient upright in a chair.
C. Place food on the strong side of the mouth.
E. Feed the patient slowly, allowing time to chew and swallow.

==================================================

Question: The nurse suspects that the patient receiving parenteral nutrition (PN) through a central venous catheter (CVC) has an air embolus. What action does the nurse need to take first?

A. Raise the head of the bed to 90 degrees
B. Turn patient to left lateral decubitus position
C. Notify health care provider immediately
D. Have the patient perform the Valsalva maneuver

Answer: B. Turn patient to left lateral decubitus position

==================================================

Question: The acceptable macronutrient distribution range for carbohydrates is
a. 20-30% of cal
b. 45-65% of cal
c. 10-25% of cal
d. 30-45% of cal

Answer: b. 45-65% of cal

==================================================

Question: Which action is initially taken by the nurse to verify the correct position of a newly placed small-bore feeding tube?

A. Placing an order for x-ray film examination to check the position
B. Confirming the distal mark on the feeding tube after taping
C. Testing the pH of the gastric contents and observing the color
D. Auscultating over the gastric area as air is injected into the tube

Answer: A Placing an order for x-ray film examination to check position

==================================================

Question: The catheter of the patient receiving parenteral nutrition (PN) becomes occluded. Place the steps for caring for the occluded catheter in the order in which the nurse would perform them.

A. Attempt to aspirate a clot.
B. Temporarily stop the infusion.
C. Flush the line with saline or heparin.
D. Use a thrombolytic agent if ordered or per protocol.

Answer: B, C, A, D
B. Temporarily stop the infusion.
C. Flush the line with saline or heparin.
A. Attempt to aspirate a clot.
D. Use a thrombolytic agent if ordered or per protocol.

==================================================

Question: Which correctly defines the total fiber adequate intake?

a. 25 g/day for women and 38 g/day for men
b. 10 g/day for women and 15 g/day for men
c. 30 g/day for women and 35 g/day for men
d. 28 g/day for women and 33 g/day for men

Answer: a. 25 g/day for women and 38 g/day for men

==================================================

Question: Based on the knowledge of peptic ulcer disease (PUD), the nurse anticipates the presence of which bacteria when reviewing the laboratory data for a patient suspected of having PUD?

A.Micrococcus
B.Staphylococcus
C.Corynebacterium
D.Helicobacter pylori

Answer: D. Helicobacter pylori

==================================================

Question: AMDR (acceptable macronutrient distribution range) for fats is

a. 20-30% of cal
b. 45-65% of cal
c.35-45% of cal
d. 20-35% of cal

Answer: d. 20-35% of cal

==================================================

Question: AMDR (acceptable macronutrient distribution range) for protein is

a. 10-35% of cal
b.20-35% of cal
c. 15-25% of cal
d. 35-45% of cal

Answer: a. 10-35% of cal

==================================================

Question: The nurse is assessing a patient receiving enteral feedings via a small-bore nasogastric tube. Which assessment findings need a further intervention?

A. Gastric pH of 4.0 during placement check
B. Weight gain of 1 pound over the course of a week
C. Active bowel sounds in the four abdominal quadrants
D. Gastric residual aspirate of 350 mL for the second consecutive time

Answer: D. Gastric residual aspirate of 350 mL for the second consecutive time

==================================================

Question: The home care nurse is seeing the following patients. Which patient is at greatest risk for experiencing inadequate nutrition?

A. A 55-year-old obese man recently diagnosed with diabetes mellitus
B. A recently widowed 76-year-old woman recovering from a mild stroke
C. A 22-year-old mother with a 3-year-old toddler who had tonsillectomy surgery
D. A 46-year-old man recovering at home following coronary artery bypass surgery

Answer: B. A recently widowed 76-year-old woman recovering from a mild stroke

==================================================

Question: What are the water-soluble vitamins?

Answer: C and B-complex

==================================================

Question: Which statement made by a patient of a 2-month-old infant requires further education?

A. I'll continue to use a formula for the baby until he is a least a year old.
B. I'll make sure that I purchase iron-fortified formula.
C. I'll start feeding the baby cereal at 4 months.
D. I'm going to an alternate formula with whole milk starting next month.

Answer: D. I'm going to an alternate formula with whole milk starting next month.

==================================================

Question: What are the fat-soluble vitamins?

Answer: D, E, K, A

==================================================

Question: The nurse is checking the feeding tube placement. Place the steps in the proper sequence.

A. Draw 5-10 mL gastric aspirated into a syringe.
B. Flush tube with 30 mL air.
C. Mix aspirate in syringe and place in a medicine cup.
D. Observe the color of gastric aspirate.
E. Perform hand hygiene and put on clean gloves
F. Dip pH strip into gastric aspirate.
G. Compare strip with the color chart from the manufacturer.

Answer: E, B, A, D, C, F, G
E. Perform hand hygiene and put on clean gloves
B. Flush tube with 30 mL air.
A. Draw 5-10 mL gastric aspirated into a syringe.
D. Observe the color of gastric aspirate
F. Dip pH strip into gastric aspirate.
G. Compare strip with the color chart from the manufacturer.

==================================================

Question: Which vitamin aids in iron absorption?

a. vitamin B
b. vitamin C
c. vitamin E
d. vitamin D

Answer: vitamin C

==================================================

Question: The nurse is teaching a program on healthy nutrition at the senior community center. Which points should be included in the program for older adults? (Select all that apply.)
A. Avoid grapefruit and grapefruit juice, which impair drug absorption.
B. Increase the amount of carbohydrates for energy.
C. Take a multivitamin that includes vitamin D for bone health.
D. Cheese and eggs are good sources of protein.
E. Limit fluids to decrease the risk of edema.

Answer: A. Avoid grapefruit and grapefruit juice, which impair drug absorption.
C. Take a multivitamin that includes vitamin D for bone health.
D. Cheese and eggs are good sources of protein.

==================================================

Question: Vegan diets can lack which vitamin, due to not naturally occurring in plants?

Answer: B12

==================================================

Question: The nurse sees the nursing assistive personnel (NAP) perform the following for a patient receiving continuous enteral feedings. What intervention does the nurse need to address immediately with the NAP? The NAP:

A. Fastens the tube to the gown with tape.
B. Places the patient supine while giving a bath.
C. Performs oral care for the patient.
D. Elevates the head of the bed 45 degrees.

Answer: B. Places the patient supine while giving a bath.

==================================================

Question: What are the major actions of Vitamin C?

Answer: Antioxidant, tissue building, absorption

==================================================

Question: The patient receiving total parenteral nutrition (TPN) asks the nurse why his blood glucose is being checked since he does not have diabetes. What is the best response by the nurse?

A. TPN can cause hyperglycemia, and it is important to keep your blood glucose level in an acceptable range.
B. The high concentration of dextrose in the TPN can give you diabetes; thus you need to be monitored closely.
C. Monitoring your blood glucose level helps to determine the dose of insulin that you need to absorb the TPN.
D. Checking your blood glucose level regularly helps to determine if the TPN is effective as a nutrition intervention.

Answer: A. TPN can cause hyperglycemia, and it is important to keep your blood glucose level in an acceptable range.

==================================================

Question: Major sources of Vitamin A

Answer: orange/yellow fruits and vegetables, fatty fish, dairy

==================================================

Question: The nurse is performing blood glucose monitoring for a patient receiving parenteral nutrition. Place the steps of the procedure in the correct sequence.

A. Clean puncture site with an antiseptic solution.
B. Identify patients using two identifiers.
C. Check code on test strip vial.
D. Wick blood drop into the test strip.
E. Gently squeeze fingertip until the drop of blood appears.
F. Assess the area of skin to be used as a puncture site.
G. Read results and document in the medical records.

Answer: F, B, C, A, E, D, G
F. Assess the area of skin to be used as a puncture site.
B. Identify patients using two identifiers.
C. Check code on test strip vial.
A. Clean puncture site with an antiseptic solution.
E. Gently squeeze fingertip until the drop of blood appears.
D. Wick blood drop into the test strip.
G. Read results and document in the medical records.

==================================================

Question: Major sources of Vitamin D

Answer: sunlight, fish, fortified dairy products, egg yolk

==================================================

Question: Major sources of Vitamin E

Answer: Vegetable oils, grains, nuts, dark green vegetables

==================================================

Question: Vitamin C & B complex.

A. Water-soluble vitamins
B. Trace elements
C. Basal metabolic rate
D. Amino acid

Answer: A. Water-soluble vitamins

==================================================

Question: Inorganic elements that act as catalysts in biochemical reactions.
A. Water-soluble vitamins
B. Trace elements
C. Basal metabolic rate
D. Amino acid

Answer: B. Basal metabolic rate

==================================================

Question: Major sources of Vitamin K

Answer: Green leafy vegetables, eggs

==================================================

Question: The energy needed to maintain life-sustaining activities for a specific period of time at rest.

A. Water-soluble vitamins
B. Trace elements
C. Basal metabolic rate
D. Amino acid

Answer: C. Basal metabolic rate

==================================================

Question: _____ is used for synthesis of thyroxine, the thyroid hormone that helps regulate metabolism.
a. Iron
b. Sulfur
c. Iodine
d. Phosphorus

Answer: c. Iodine

==================================================

Question: Major sources of Potassium

Answer: oranges, dried fruits, tomatoes, avocado, dried peas, meats, broccoli, bananas, dairy products, meats, whole grains

==================================================

Question: The simplest form of a protein.

A. Water-soluble vitamins
B. Trace elements
C. Basal metabolic rate
D. Amino acid

Answer: D. Amino acid

==================================================

Question: Made up of three fatty acids attached to a glycerol.

A. Triglycerides
B. Nitrogen balance
C. Polyunsaturated fatty acids
D. Resting energy expenditure (REE)

Answer: A. Triglycerides

==================================================

Question: What is the minimum daily amount of water intake needed?

Answer: 1500 mL

==================================================

Question: Under normal conditions, the AI for adult fluid intake is

a. 2 L/day for men and 1.1 L/day for women
b. 3 L/day for men and 3.5 L/day for women
c. 3 L/day for men and 2.2 L/day for women
d. 2.2 L/day for men and 3 L/day for women

Answer: c. 3 L/day for men and 2.2 L/day for women

==================================================

Question: Carbohydrate units.
A. Lipids
B. Nutrient density
C. Saccharides
D. Dispensable amino acids

Answer: C. Saccharides

==================================================

Question: A medication that decreases BMR is

a. Epinephrine
b. Levothyroxine
c. Amitriptyline

Answer: c. Amitriptyline

==================================================

Question: When calculating % weight change, greater than ____% in 1 week indicates significant weight loss.

a. 2
b. 4
c. 6

Answer: a. 2%

==================================================

Question: When calculating a % weight change, greater than ____% in 3 months indicates significant weight loss.

a. 5.5
b. 6.5
c. 7.5

Answer: c. 7.5%

==================================================

Question: Overweight is defined as a BMI of ____ to ____.

Answer: 25 to 29.9

==================================================

Question: Patient symptoms are projectile vomiting, fever, myalgia, watery diarrhea, and headache. Consumed contaminated fruits and vegetables, salads prepped by an infected person, oysters, and/or contaminated water. Foodborne illness is ____. The onset of 24-48 hours.

a. Listeria
b. E. coli
c. Norovirus
d. Salmonella

Answer: c Norovirus

==================================================

Question: Mrs. Nelson is talking with the nurse about the dietary needs of her 23-month-old daughter, Laura. Which of the following responses by the nurse would be appropriate?

A. "Use skim milk to cut down on the fat in Laura's diet."
B. "Laura should be drinking at least1 quart of milk per day."
C. "Laura needs less protein in her diet now because she isn't growing as fast."
D. "Laura needs fewer calories in relation to her body weight now than she did as an infant."

Answer: D "Laura needs fewer calories in relation to her body weight now than she did as an infant."

==================================================

Question: All of the following patients are at risk for alteration in nutrition except:

A. Patient L, whose weight is 10% above his ideal body weight.
B. Patient J, who is 86 years old, lives alone and has poorly fitting dentures.
C. Patient M, a 17-year-old girl who weighs 90 pounds and frequently complains about her baby fat.
D. Patient K, who has been allowed nothing by mouth (NPO) for 7 days after bowel surgery and is receiving 3000 mL of 10% dextrose per day.

Answer: A. Patient L, whose weight is 10% above his ideal body weight.

==================================================

Question: A patient who has been hospitalized after experiencing a heart attack will most likely receive a diet consisting of:

A. Low fat, low sodium, and low carbohydrates
B. Low fat, low sodium, and high carbohydrates
C. Low fat, high protein, and high carbohydrates
D. Liquids for several days, progressing to a soft and then a regular diet

Answer: B. Low fat, low sodium, and high carbohydrates

==================================================

Question: Acceptable foods for the Full liquid diet

Answer: liquids from clear liquid diet, all forms of milk, soups, vegetable/fruit juices, eggnog, plain ice cream and sherbet, refined/strained cereal, puddings

==================================================

Question: Indications for full liquid diet

Answer: Transition from liquid to soft diets, Post-op, acute gastritis, febrile conditions, intolerance of solid foods

==================================================

Question: Contraindications for full liquid diet

Answer: lactose intolerance or hypercholestrolemia

==================================================

Question: The slenderized liquid (pureed) diet consists of...

Answer: liquids and foods that are pureed to liquid form

==================================================

Question: Indications of slenderized liquid (pureed) diet

Answer: chewing/swallowing difficulty, oral/facial surgery, wired jaws

==================================================

Question: Soft (bland, low-fiber) diet consists of...

Answer: whole foods that are low in fiber, lightly seasoned, and easily digested

==================================================

Question: Indications of soft (bland, low-fiber) diet

Answer: Transitioning between full liquid and regular diets, acute infections, chewing difficulties, GI disorders

==================================================

Question: The mechanical soft diet consists of...

Answer: Foods that require minimal chewing before swallowing (ground meats, canned fruits, softly cooked vegetables)

==================================================

Question: Indications of mechanical soft diet

Answer: limited chewing ability, dysphagia, poorly fitting dentures, clients who are edentulous (without teeth), surgery to head, neck, or mouth; stricures of the intestinal tract

==================================================

Question: What is the dysphagia diet used for?

Answer: prescribed when swallowing is impaired, such as following a stroke

==================================================

Question: Signs of dysphagia

Answer: drooling, pocketing food, choking, gagging

==================================================

Question: What are the 3 levels of solid textures of food used in the dysphagia diet? Provide examples of each

Answer: Level 1: pureed- pureed fruits, veg, meats, soups, scrambled eggs, pudding, custard, applesauce
Level 2: mechanically altered- ground meat with gravy, chicken or tuna salad, moistened pancakes and syrup, poached eggs, soft canned or cooked fruit
Level 3: advanced- moist foods such as soups, rice, stuffing, tender meats or casseroles, breads that are not crusty

==================================================

Question: What are the 4 levels or liquid consistencies in the dysphagia diet? Provide examples of each

Answer: 1. Thin: clear juices, froyo, ice seam, milk, soda, broth
2. Necar-like: thicker than water but can sip through a straw; consistency of heavy syrup. ex) nectars, cream soups, buttermilk and thin milkshakes
3. Honey-like: pourable and thickened, can be eating with a spoon but not sipped through a straw. ex) honey, tomato sauce, yogurt
4. Spoon-thick: maintain shape and eaten with a spoon. ex) pudding, custard, hot cereals

==================================================

Question: Indications for the Regular diet (normal or house diet)

Answer: For clients who do not need dietary restrictions

==================================================

Question: Examples of "honey-like" liquid consistencies

Answer: Honey, tomato sauce, yogurt

==================================================

Question: Examples of "spoon-thick" liquid consistencies

Answer: pudding, custard, hot cereals

==================================================

Question: Examples of "nectar-like" liquid consistencies

Answer: nectars, cream soups, buttermilk, thin milkshakes

==================================================

Question: This form of nutrition administration is used when a client cannot consume adequate nutrients and calories orally, but maintains a PARTIALLY functional GI system.

Answer: Enteral Nutrition (EN)

==================================================

Question: 2 reasons enteral nutrition might be administered

Answer: Medical condition (burns, trauma, radiation therapy, demo, liver/renal dysfunction, infection, inflammatory bowel disease) or when a client is neuromuscular impaired and cannot chew or swallow food

==================================================

Question: What does enteral nutrition consist of?

Answer: commercial formula administered by a tube into the stomach or small intestine

==================================================

Question: What is gavage feeding for an infant used for?

Answer: AKA EN feeding- for and infant who is too weak for sucking, unable to swallow, and lacks a gag reflex

==================================================

Question: What are the two general enteral feeding routes?

Answer: Nasoenteric tubes and Ostomies

==================================================

Question: In this enteral feeding method, the formula is administered at a CONTINUOUS rate over a 12- to 24-hr period.

a. Cyclic feeding
b. Intermittent tube feeding
c. Continuous infusion method
d. Bolus feeding

Answer: c. Continuous infusion method

==================================================

Question: This enteral feeding delivery method is recommended for critically ill clients, because of its association with smaller residual volumes and lower risk or aspiration and diarrhea.
a. Intermittent tube feeding
b. Bolus feeding
c. Cyclic feeding
d. Continuous infusion method

Answer: d. Continuous infusion method

==================================================

Question: When using the continuous infusion method of enteral feeding, residual volumes should be measured every _______

a. 2-4 hr
b. 4-6 hr
c. 6-8 hr

Answer: b. 4-6 hr

==================================================

Question: In the continuous infusion method of enteral feeding, the feeding tubes should be flushed with water every ___ hr.
a. 1
b. 4
c. 8

Answer: b. 4 hr

==================================================

Question: For this method in enteral feeding, formula is administered at a CONTINUOUS rate over an 8- to 12-hr time period, often during SLEEPING hours.
a. Bolus feeding
b. Cyclic feeding
c. Continuous infusion
d. Intermittent tube feeding

Answer: b. Cyclic feeding

==================================================

Question: This EN delivery method is often used for NONcritical clients, home tube feedings, and clients in rehab. It mostly resembles a NORMAL patter of nutrient intake.

Answer: Intermittent tube feeding

==================================================

Question: This EN feeding method administers a LARGE volume of formula (700mL max, usual 250-500mL) over a SHORT period of time, usually in 15-30min, 4-6x daily
a. Bolus feeding
b. Intermittent tube feeding
c. Cyclic feeding
d. Continuous infusion method

Answer: a. Bolus feeding

==================================================

Question: Contraindication for bolus feedings in EN

Answer: tube placed into the jejunum or duodenum.

==================================================

Question: What is the temperature of feeding solution when administering EN feedings?

Answer: Room temp

==================================================

Question: What are some gastrointestinal complications that can result from EN feedings?

Answer: GI complications- constipation, diarrhea, cramping, pain, abdominal distention, dumping syndrome, nausea, vomiting

==================================================

Question: What are some mechanical complications that can result from EN feedings?

Answer: Tube misplacement/dislodgment, aspiration, irritation and leakage at the insertion site, irritation of the nose/esophagus/mucosa, clogging of the feeding tube

==================================================

Question: What are some metabolic complications that can result from EN feedings?

Answer: Dehydration, hyperglycemia, electrolyte imbalances, fluid overload, refeeding syndrome, rapid weight gain

==================================================

Question: To avoid food poisoning in EN, replace the feeding bag, administration tubing, and any equipment used to mix the formula ever ____ hr.

Answer: 24

==================================================

Question: To avoid food poisoning in EN, fill generic bags with only ____ hr worth of formula.

Answer: 4

==================================================

Question: What does TPN stand for?

Answer: Total Parenteral Nutrition

==================================================

Question: What does PPN stand for?

Answer: Peripheral Parenteral nutrition

==================================================

Question: What are 2 reasons parenteral nutrition would be used?

Answer: 1. clients GI tract is not functioning
2. client cannot physically/psychologically consume sufficient nutrients orally/enterally

==================================================

Question: A client's caloric needs are very high and anticipated duration of therapy is greater than 7 days. Which PN would be administered, TPN or PPN?

Answer: TPN

==================================================

Question: TPN can only be administered in a ________

Answer: central vein

==================================================

Question: TPN is used when the solution to be administered is _____, meaning it is composed of >10% dextrose.
a. isotonic
b. hypertonic
c. hypotonic

Answer: b. hypertonic

==================================================

Question: Peripheral PN is administered into a ______, resulting in a limited nutritional value

Answer: Peripheral vein

==================================================

Question: A client requires short-term nutritional support with fewer calories per day. What PN should be administered, TPN or PPN?

Answer: PPN

==================================================

Question: PN IV lipids are contraindicated for clients who are allergic to _____, ______, and ______.

Answer: soybean oil, eggs, safflower oil

==================================================

Question: PN IV lipids are contraindicated for clients who have these 2 diseases.

Answer: hyperlipidemia, severe hepatic disease

==================================================

Question: PN protein solutions are available in concentrations of ___ to ___%.

Answer: 3.5% to 15%

==================================================

Question: What two medications can be added to PN solution by pharmacy services?

Answer: insulin and heparin

==================================================

Question: Why would insulin be added to a patient's PN solution?

Answer: To reduce the potential of hyperglycemia

==================================================

Question: Why would heparin be added to a patient's PN solution?

Answer: To prevent fibrin buildup on the catheter tip

==================================================

Question: What are 3 desired therapeutic outcomes of PN?

Answer: -improved nutritional status
-weight maintenance/gain
-positive nitrogen balance

==================================================

Question: What is the expected reference range of albumin levels?

Answer: 3.5 to 5.0 g/dL

==================================================

Question: What is the expected reference range of pre albumin levels?

Answer: 15 to 36 mg/dL

==================================================

Question: Change the PN tubing and bag every _____ hr or per facility protocol

Answer: 24hr

==================================================

Question: To avoid hypoglycemia, an IV of dextrose ___ to ___% in water is administered if the PN solution is unavailable.

Answer: 10% to 20%

==================================================

Question: A nurse is developing a teaching plan for a client taking insulin? Which of the following foods should the client carry with him as a minor snack to help treat hypoglycemia?

A. Crackers
B. Almonds
C. White Chocolate
D. Life Saver

Answer: D. Life saver

Although chocolate does contain sugar, it also provides a substantial amount of fat that can delay the absorption of sugar. The best choices for treating hypoglycemia are absorbable pure sugars including soft drinks, fruit juices, or lifesavers. Almonds and crackers do not contain simple sugars

==================================================

Question: A nurse is educating a prediabetic client about the Mediterranean diet. Which of the following statements indicates that the teaching was successful?

A. "On this diet, I can consume wine instead of beer with no limits"
B. "I can still eat pizza each day because of the cheese, meat, and vegetables for my toppings"
C. "With this diet, a 12 oz steak is still a viable dinner option"
D. "This diet will likely minimize my waistline"

Answer: D. "This diet will likely minimize my waistline"

==================================================

Question: A client expresses an understanding of seasoning food when he states (Select all that apply).

A. "The preferences for salty taste eventually will decrease"
B. "Salt substitutes replace sodium with potassium or calcium"
C. "Reducing sodium intake will lower my blood pressure"
D. "I should always prepare food from scratch"
E. "Sea salt is better for you than table salt"

Answer: A. "The preferences for salty taste eventually will decrease" C. "Reducing sodium intake will lower my blood pressure"

==================================================

Question: A client with Diabetes Mellitus states, "I cannot eat big meals; I prefer to snack throughout the day." The nurse should carefully explain that:

A. Regulated food intake is basic to controlling blood glucose levels.
B. Salt and sugar restriction is the main concern.
C. Small, frequent meals are better for digestion.
D. Large meals can contribute to a weight problem.

Answer: A. Regulated food intake is basic to controlling blood glucose levels.

==================================================

Question: The healthcare provider is teaching a client diagnosed with Celiac disease about the disease process. Which of the following statements made by the client would indicate a correct understanding of the teaching?

A. "I'm glad that I can still eat bread made with rye flour"
B. "I have an allergy to the proteins that are found in wheat"
C. "My immune system reacts to gluten and damages my gut"
D. "The bacteria in my gut are not able to ferment the gluten"

Answer: C. "My immune system reacts to gluten and damages my gut"

==================================================

Question: Which of the following are likely to increase your chances of getting chronic kidney disease?

A. Ethnicity
B. Genetics
C. Mental Status
D. Access to Healthcare

Answer: A. Ethnicity
B. Genetics
D. Access to Healthcare

==================================================

Question: Which foods are included in a Lacto-Ovo vegetarian diet?

A. Eggs, Milk, Honey, Plant food
B. Eggs, Milk, Plant food
C. Eggs, Honey, Plant food
D. Honey, Plant food

Answer: A. Eggs, Milk, Honey, Plant food

==================================================

Question: Which of the following are functions of the kidneys? (Select all that apply)

A. Secretion of renin
B. Secretion of aldosterone
C. Excretion of nitrogenous waste
D. Secretion of erythropoietin

Answer: A. Secretion of renin
C. Excretion of nitrogenous waste
D. Secretion of erythropoietin

==================================================

Question: Which of the following is an appropriate lunch for a patient on a level 1 dysphagia diet?

A. Pureed meatloaf, pureed broccoli, and chocolate pudding
B. PB and J sandwich on white bread, applesauce, cooked carrots, and tea with honey
C. Chicken salad, a fruit cup, and a thin chocolate milkshake
D. Tomato soup with a turkey sandwich, orange, and brownies without nuts

Answer: The correct answer is A. A level 1 dysphagia diet requires all of the food to be pureed.

==================================================

Question: A nurse is conducting teaching with a client regarding nutrition therapy for gastroparesis. Does the nurse know the teaching is successful when the client states that the following are ways to increase nutrition? (Select 3 that apply.)

a. Lay down for 1 to 2 hours after eating
b. Chew foods thoroughly
c. Control blood glucose levels for all patients not just those with diabetes
d. Consume fats in liquids if fat in solids is not tolerated
e. Consume smaller, more frequent meals
f. Be sure to eat only 3 large meals a day with no snacks in between

Answer: b. Chew foods thoroughly
d. Consume fats in liquids if fat in solids is not tolerated
e. Consume smaller, more frequent meals

==================================================

Question: When recommending a healthy after school snack, the nurse should suggest which of the following? Select all that apply.

A. Low-fat yogurt topped with fresh fruit
B. Peanut butter and jelly on white bread
C. Apple slices with cottage cheese
D. Air-popped popcorn
E. Graham crackers with chocolate hazelnut spread

Answer: A. Low-fat yogurt topped with fresh fruit
C. Apple slices with cottage cheese
D. Air-popped popcorn

==================================================

Question: The nurse knows that children who are overweight or obese are at risk for developing what comorbidities? Select all that apply.

A. Type I diabetes
B. Coronary artery disease
C. Hypertension
D. Chronic obstructive pulmonary disease (COPD)
E. Gastroesophageal reflux disease (GERD)

Answer: B. Coronary artery disease
C. Hypertension
E. Gastroesophageal reflux disease (GERD)

==================================================

Question: 1) Darius has a son who is 30 and is curious if type 2 diabetes can affect him. What would be the most appropriate response?

a) "diabetes runs in families; he can expect to have it when he is older"
b) "diabetes runs in the family, but is a preventable disease with proper nutrition and exercise"
c) "diabetes is not contagious, so he does not have to worry"
d) "he does not have to worry because he is young"

Answer: b) "diabetes runs in the family, but is a preventable disease with proper nutrition and exercise"

==================================================

Question: 2) Darius just came out of the nutrition class. which would show that he understood the nutrition class? Select all that apply (select 2)

a) I must limit sodium to 2300 per day
b) I can never eat cake or drink coke again because it contains too many carbohydrates
c) I do not have to watch my carbohydrates because I am taking insulin
d) I cannot drink any alcohol because it can react with my insulin

Answer: a) I must limit sodium to 2300 per day
c) I do not have to watch my carbohydrates because I am taking insulin

==================================================

Question: 3)The nurse is educating a client with hypertension who is obese and prediabetic about foods with cardiometabolic benefits. The client lives alone and is not used to cooking. the nurse knows the client understands the teaching when he states which of the following:

a) I will use stick margarine when cooking my vegetables on the stovetop
b) I will continue eating sausage with my breakfast sandwiches for extra protein
c) eating fruits and vegetables will help improve my blood pressure
d) I should not eat nuts because they have large amounts of sodium

Answer: c) eating fruits and vegetables will help improve my blood pressure

==================================================

Question: 4) The physician has advised a client to lose 50lbs due to the risk factors associated with his weight. The client does not want to give up his eating and drinking habits. He asks the nurse why it is so important for him to lose weight. The best response is:

a) Your current BMI is too high for your age, you need to lower it to a healthy level
b) Individuals who are overweight are at risk for cardiovascular disease and heart failure
c) If you continue to eat the way you do, you will put yourself at risk for stroke
d) Losing weight will help promote your overall health and decrease the risk for future health problems

Answer: d) Losing weight will help promote your overall health and decrease the risk for future health problems

==================================================

Question: 7) A woman in her mid-40's decides to lose weight to improve blood pressure. she starts following a diet reducing sodium, incorporating fruits and vegetables, and eliminating all meat. she lost 30 lbs. last month and feels "more motivated than ever." She presents to the clinic for a wellness checkup and is found to be anemic. What is the best priority nursing intervention?

a) recommend iron pills to improve anemia
b) praise the women for meeting her weight loss goal
c) ask the women what she is eating at home and assess diet
d) diagnose the woman with an eating disorder

Answer: c) ask the women what she is eating at home and assess diet

==================================================

Question: 8) An 18-year-old male presents to the hospital with severe pain later attributed to multiple large kidney stones. He is treated with surgery but needs to stay in a post-op recovery unit, and while he is there, he continues to request milk with every meal and every snack. The nurse should first:

a) Allow him to have milk anytime because calcium is good for the healing process
b) advise him to drink less milk to improve his recovery
c) Confront the dietitian and tell them to stop letting him have milk
d) Ignore the situation because this is the dietitian's job, not the nurse's

Answer: b) advise him to drink less milk to improve his recovery

==================================================

Question: An 80-year-old male is admitted to the ER after an episode of hyperglycemia. He has been stabilized. His daughter comes to visit him and scolds him for not taking his insulin. What is the priority intervention for the nurse?

A. Instruct the man on how to take insulin properly and send him home.
B. Ask him why he did not take his insulin.
C. Refer him to an assisted living facility.
D. Scold him along with his daughter

Answer: B. Ask him why he did not take his insulin.

==================================================

Question: A local nursing home receives a new admission this morning. She has been diagnosed with gastroparesis. The dietitian is unavailable this morning and you, an experienced RN, are instructed to plan meals for her. Which of the following eating patterns is most appropriate?

A. 3 large, well-rounded meals per day.
B. 4 high-fat, high-protein meals per day.
C. Multiple small meals per day
D. Tube feedings.

Answer: C. Multiple small meals per day.

==================================================

Nclex Predictor

Question: Which of these instructions should a nurse include in the teaching plan for a client who had removal of a cataract in the left eye?
a. "Forcefully cough and take deep breaths every two hours to keep your airway clear."
b. "Perform the prescribed eye exercises each day to strengthen your eye muscles."
c. "Rinse your eyes with saline each morning to prevent postoperative infection."
d. "Take the prescribed stool softener to avoid increasing intraocular pressure."

Answer: d. "Take the prescribed stool softener to avoid increasing intraocular pressure."

==================================================

Question: A client vomits during a continuous nasogastric tube feeding. A nurse should stop the feeding and take which of these actions?
a. Suction the nasogastric tube.
b. Flush the tube with 30 mL of sterile water.
c. Remove the nasogastric tube.
d. Check the residual volume.

Answer: d. Check the residual volume.

==================================================

Question: Which of these actions best demonstrates cultural sensitivity by a nurse?
a. The nurse talks in a slow-paced speech.
b. The nurse asks clients about their beliefs and practices toward pregnancy.
c. The nurse uses charts and diagrams when teaching pregnant clients.
d. The nurse can speak several different languages.

Answer: b. The nurse asks clients about their beliefs and practices toward pregnancy.

==================================================

Question: Which of these manifestations should a nurse expect to observe in a 3-month-old infant who is diagnosed with dehydration?
a. Hyperreflexia.
b. Tachycardia.
c. Bradypnea.
d. Agitation.

Answer: b. Tachycardia.

==================================================

Question: When assessing a client's risk of developing nosocomial infection, a nurse plans to determine potential entry portals, which include:
a. the urinary meatus.
b. vomitus.
c. contaminated water.
d. sexual intercourse.

Answer: a. the urinary meatus.

==================================================

Question: A client who is on the inpatient psychiatric unit has a history of violence. Which of these actions should a nurse take if the client is agitated?
a. Encourage the client to verbalize feelings.
b. Lock the client in a secluded room.
c. Ask the other clients to give feedback regarding the client's behavior.
d. Ignore the client's inappropriate behavior.

Answer: a. Encourage the client to verbalize feelings.

==================================================

Question: Which of these measures should a nurse include when planning care for a school-aged child during a sickle cell crisis episode?
a. Monitoring for signs of bleeding.
b. Providing pain relief.
c. Administering cool sponge baths to reduce fevers.
d. Offering a high calorie diet.

Answer: b. Providing pain relief.

==================================================

Question: Which of these instructions should a nurse include in the plan of care for a 32-week gestation client who had an amniocentesis today?
a. "Drink at least six glasses of fluids during the next six hours after the test."
b. "Call the clinic if you experience any abdominal cramps."
c. "Don't be concerned if you have some vaginal spotting in the next 12 hours."
d. "When you get home, stay on bed-rest for the next 48 hours."

Answer: b. "Call the clinic if you experience any abdominal cramps."

==================================================

Question: An adolescent has a nursing diagnosis of fatigue related to inadequate intake of iron-rich foods. Selection of which of these lunches by the client indicates a correct understanding of foods high in iron content?
a. Peanut butter and jam sandwich.
b. Chicken nuggets with rice.
c. Tuna salad sandwich.
d. Beefburger with cheese.

Answer: d. Beefburger with cheese.

==================================================

Question: A client has been admitted with acute pancreatitis. Which of these laboratory test results supports this diagnosis?
a. Elevated serum potassium level.
b. Elevated serum amylase level.
c. Elevated serum sodium level.
d. Elevated serum creatinine level.

Answer: b. Elevated serum amylase level.

==================================================

Question: Which of these manifestations, if assessed in a client who is two-hours postoperative after abdominal surgery, should a nurse report immediately?
a. Vomiting and a pulse rate of 106/minute.
b. Respiratory rate of 12/minute and urine dribbling.
c. Blood pressure of 100/60 mm Hg and wound discomfort.
d. Urine output of 100 mL/hr and flushed skin.

Answer: a. Vomiting and a pulse rate of 106/minute.

==================================================

Question: Which of these observations of a student nurse's behavior while interacting with a client who is crying indicates a correct understanding of therapeutic communication?
a. The student maintains continuous eye contact with the client.
b. The student places one arm around the client's shoulder?
c. The student sits quietly next to the client.
d. The student leaves the room to provide privacy for the client.

Answer: c. The student sits quietly next to the client.

==================================================

Question: Which of these actions should a nurse take initially if a client who is diagnosed with diabetes mellitus develops tremors and ataxia?
a. Measure the client's blood sugar level.
b. Administer a concentrated form glucose to the client.
c. Administer a prn dose of insulin.
d. Measure the client's urine for ketones.

Answer: a. Measure the client's blood sugar level.

==================================================

Question: An elderly client is at increased risk of developing drug toxicity to prescribed medications due to declining hepatic and renal functioning. Which of these strategies should a nurse plan to decrease this risk?
a. Increasing the time interval between medication doses.
b. Limiting the client's oral fluid intake.
c. Administering the medications with meals.
d. Encouraging the client to void every three to four hours.

Answer: a. Increasing the time interval between medication doses.

==================================================

Question: A client has persistent paranoid delusions that the food on the unit is poisoned. Which of these measures should a nurse include in the client's care plan?
a. Explaining that staff does not poison clients.
b. Focusing on how the hospital staff helps clients.
c. Allowing the client to eat food from sealed containers.
d. Telling the client that not eating the food that is served will result in privilege restrictions.

Answer: c. Allowing the client to eat food from sealed containers.

==================================================

Question: Thrombophlebitis is a complication that may result due to surgery. Which of these actions should a nurse take in the operating room to prevent this complication from occurring?
a. Gatch the knee of the bed.
b. Administer anticoagulants preoperatively.
c. Apply sequential compression devices.
d. Maintain the legs in a dependent position.

Answer: c. Apply sequential compression devices.

==================================================

Question: When discussing weigh gain during pregnancy, a nurse should recommend that the total weight gain for a pregnant client who is at ideal body weight for her height is:
a. at least 15 pounds.
b. 15 to 20 pounds.
c. 25 to 35 pounds.
d. at least 45 pounds.

Answer: c. 25 to 35 pounds.

==================================================

Question: Which of these manifestations, if reported by a client who is 10-weeks-pregnant, supports the diagnosis of ruptured tubal pregnancy.
a. Sharp unilateral abdominal pain.
b. Uncontrollable vomiting.
c. Marked abdominal distention.
d. Profuse vaginal bleeding.

Answer: a. Sharp unilateral abdominal pain.

==================================================

Question: Which of these assignments, if made by a nurse to a nursing assistant, indicates that the nurse needs additional instructions regarding the principles of delegation?
a. "Please bathe the client in room 12, and then bring the client to the dining room for breakfast by 9 A.M."
b. "Please bathe the client in room 10, administer a back rub, and then evaluate if the back rub eased the client's discomfort."
c. "Please measure the intake and output for the client's in rooms 8. 9. and 10, and record each on the intake/output sheets by 2 P.M."
d. "Please toilet the clients in rooms 11, 12, and 13 mid-morning and after lunch."

Answer: b. "Please bathe the client in room 10, administer a back rub, and then evaluate if the back rub eased the client's discomfort."

==================================================

Question: A client has the following order for regular insulin (Humulin R) on a sliding scale:
Blood sugar 150-180 mg: Give 2 units regular insulin
Blood sugar 181-200 mg: Give 4 units regular insulin
Blood sugar 201-220 mg: Give 6 units of regular insulin
Blood sugar above 220 mg: Call MD
At 11 A.M., a nurse obtains a finger stick glucose of 198 mg. The only syringe is a three milliliter one. Regular insulin is available as 100 units per milliliter. How many milliliters should the nurse administer?
a. 0.04
b. 0.4
c. 4
d. 40

Answer: a. 0.04

==================================================

Question: Which of these nursing diagnosis is the priority for a client who is one-hour postoperative after extensive abdominal surgery?
a. Risk for impaired physical mobility.
b. Risk for deficient fluid volume.
c. Risk for ineffective airway clearance.
d. Risk for infection.

Answer: c. Risk for ineffective airway clearance.

==================================================

Question: A nurse should recognize that which of these occupations increases a person's risk of developing hepatitis B?
a. Sanitation worker.
b. Nursery school teacher.
c. Hemodialysis nurse.
d. Fish market sales person.

Answer: c. Hemodialysis nurse.

==================================================

Question: Which of these assessments is the priority for a client who sustained second-degree burns of the face and neck?
a. Respiratory status.
b. Renal function.
c. Level of pain.
d. Signs of infection.

Answer: a. Respiratory status.

==================================================

Question: A nurse should place a child who is two hours post-tonsillectomy and adenoidectomy in which of these positions?
a. Supine, flat.
b. Orthopneic.
c. Trendelenberg.
d. Side-lying.

Answer: d. Side-lying.

==================================================

Question: Which of these instructions should a nurse include in the discharge teaching for a client who has diabetes mellitus?
a. "Soak your feet in hot water once a day."
b. "Cut your toenails in an oval shape weekly."
c. "Avoid using any soap on your feet."
d. "Apply lotion to your feet each day."

Answer: d. "Apply lotion to your feet each day."

==================================================

Question: A nurse inadvertently administers an incorrect medication to a client. Which of these actions should the nurse take first?
a. Assess the client.
b. Notify the physician.
c. Contact the nurse manager.
d. Complete an incident report.

Answer: a. Assess the client.

==================================================

Question: An elderly client who is receiving a blood transfusion develops a rapid bounding pulse and an elevated blood pressure. Which of these actions should a nurse take?
a. Add a 5% dextrose solution to the line.
b. Raise the head of the bed.
c. Stop the transfusion.
d. Measure the client's temperature.

Answer: c. Stop the transfusion.

==================================================

Question: When caring for a client who has hepatitis B, a nurse should wear:
a. gloves when administering oral medications to the client.
b. a gown when changing the client's position.
c. gloves when removing the intravenous cannula.
d. a gown when emptying the client's used bath water.

Answer: c. gloves when removing the intravenous cannula.

==================================================

Question: Which of these outcome criteria is appropriate for a client who has a nursing diagnosis of ineffective airway clearance?
a. Absence of wheezing throughout the lung fields.
b. Clear lung sounds on auscultation.
c. Pulse oximetry level of 80%.
d. Frequent coughing throughout the day.

Answer: b. Clear lung sounds on auscultation.

==================================================

Question: A doctor prescribes liquid oral iron medication for a 4-year-old child. Which of these questions should a nurse ask the child's mother to determine if the medication is being administered correctly?
a. "Are you using a straw to administer the medicine?"
b. "Has your child been urinating more frequently?"
c. "Have you increased your child's milk intake each day?"
d. "Is there a change in the color of your child's skin?"

Answer: a. "Are you using a straw to administer the medicine?"

==================================================

Question: Which of these assessment findings, if present in a 4-month-old infant who has severe diarrhea, should a nurse recognize as suggestive that the infant is dehydrated?
a. Bulging anterior fontanel.
b. Pulse rate of 120/minute.
c. Decreased urine output.
d. Cyanosis of the mucus membrane.

Answer: c. Decreased urine output.

==================================================

Question: Which of these instructions should be included in the teaching plan for the parents of a 10-month-old infant who is admitted to the hospital for failure to thrive?
a. Advise the mother to make sure the infant drinks the entire bottle at each feeding.
b. Encourage the mother to feed the infant slowly in a quiet environment.
c. Teach the mother to position the infant on the abdomen following feedings.
d. Instruct the mother to play actively with the infant during bottle feedings.

Answer: b. Encourage the mother to feed the infant slowly in a quiet environment.

==================================================

Question: When a newborn is 48 hours old, a nurse notes that the child is jaundiced. The nurse should recognize which of these conditions as a probable cause of the newborn's jaundice?
a. Dehydration.
b. Liver immaturity.
c. ABO incompatibility.
d. Gallbladder immaturity.

Answer: b. Liver immaturity.

==================================================

Question: Which of these items should a nurse removed from the food tray of a client who is on a sodium-restricted diet?
a. Packet of a salt substitute.
b. Grapefruit juice.
c. Container of jelly.
d. Ketchup.

Answer: d. Ketchup.

==================================================

Question: Which of these statements, if made by a client who had a total hip replacement, would indicate a correct understanding of the postoperative instructions?
a. "I will stoop carefully to pick up items from the floor."
b. "I will use a raised toilet seat in the bathroom."
c. "I will bend forward when tying my shoes."
d. "I will put my leg through the full range of motion each day."

Answer: b. "I will use a raised toilet seat in the bathroom."

==================================================

Question: Which of these measures should a nurse include when planning care for an 88-year-old client who is admitted to the hospital with pneumonia?
a. Restricting visitors to the client's immediate family members.
b. Limiting the client care activities to no more than five minutes each.
c. Allowing the client to perform self-care as tolerated.
d. Providing the client with a non-stimulating environment.

Answer: c. Allowing the client to perform self-care as tolerated.

==================================================

Question: A client, who is newly diagnosed with cancer says to anurse, "I suppose I need to complete all unfinished business as soon as possible." Which of these responses is appropriate?
a. "Yes, you should do this immediately.
b. "Don't you think you should stay focused on your treatment for now?
c. "Exactly what things are you talking about?"
d. "It sounds like you are concerned with your diagnosis."

Answer: d. "It sounds like you are concerned with your diagnosis."

==================================================

Question: Which of these interventions should plan for a child who is receiving chelation therapy for lead poisoning?
a. Keeping an accurate record of intake and output.
b. Instituting measures to prevent skeletal fractures.
c. Maintaining isolation precautions.
d. Maintaining strict bed rest.

Answer: a. Keeping an accurate record of intake and output.

==================================================

Question: A nurse obtains these vital signs on an adult client. Which finding should the nurse follow-up first?
a. Heart rate, 60/minute and regular.
b. Respiration, 30/minute and deep.
c. Temperature, 97.1 °F (36.2 °C)
d. Blood pressure, 136/86 mm Hg

Answer: b. Respiration, 30/minute and deep.

==================================================

Question: When determining the duration of a uterine contraction, a nurse should measure the contraction from the:
a. beginning of one contraction to the end of that contraction.
b. end of one contraction to the beginning of the next contraction.
c. beginning of one contraction to the beginning of the next contraction.
d. strongest point of one contraction to the strongest point of the next contraction.

Answer: a. beginning of one contraction to the end of that contraction.

==================================================

Question: A nurse should recognize which of these signs is a probably sign of pregnancy?
a. Frequency of urination.
b. Positive pregnancy test.
c. Nausea in the morning.
d. Abdominal distention.

Answer: b. Positive pregnancy test.

==================================================

Question: All of these clients are on bed rest. Which one is the most at risk to develop skin breakdown?
a. An 82-year-old client who bathes once a week.
b. An 83-year-old client who applies powder after drying the skin.
c. An 84-year-old client who has been NPO for four days.
d. An 85-year-old client who has coronary artery disease.

Answer: c. An 84-year-old client who has been NPO for four days.

==================================================

Question: A client diagnosed with type 1 diabetes mellitus has a glycosylated hemoglobin A1c of 4.2%. A nurse should interpret this to mean that the client has:
a. had a period of sustained hyperglycemia.
b. been non-compliant with home management.
c. been in relatively good diabetic control.
d. eaten a high carbohydrate snack just prior to testing.

Answer: c. been in relatively good diabetic control.

==================================================

Question: A nurse is caring for a client with burns and in reverse isolation. Which measures should the nurse include?
a. Wearing disposable gloves when chaging the dressings.
b. Having the client wear goggles when staff is in the room.
c. Wearing a gown, mask, and gloves when providing care to the client.
d. Disposing of the client's soiled laundry in a red bag.

Answer: c. Wearing a gown, mask, and gloves when providing care to the client.

==================================================

Question: A physician has ordered 100 mg of Amoxicillin po for a child. The available liquid amoxicillin is 250 mg/5 mL. How many milliliters should a nurse administer?
a. 1.0
b. 1.5.
c. 2.0
d. 2.5

Answer: c. 2.0

==================================================

Question: A nurse charts on all assigned clients at 2:00 P.M. The nurse then remembers something that happened at 9:00 A.M. to a client who was not charted. Which of these actions should the nurse take?
a. Include the 9:00 A.M. scenario in the shift report.
b. Enter the scenario after the original 2:00 P.M. charting and mark it as a "late entry".
c. Put the information in the margin and indicate the accurate time placement by drawing an arrow.
d. Draw a line through the previous charting with "error" and then re-record everything, including the new information.

Answer: b. Enter the scenario after the original 2:00 P.M. charting and mark it as a "late entry".

==================================================

Question: While giving a bath to a client, a nurse notices that the client's back appear reddened. Which of these interpretations and additional assessments should the nurse make?
a. The client's skin is sensitive to touch; lightly rub the client's chest area.
b. The client has decreased circulation; palpate the peripheral pulses.
c. The client is showing signs of pressure; press on the skin and observe for a return of color.
d. The client is allergic to the soap; check the extremities for discoloration.

Answer: c. The client is showing signs of pressure; press on the skin and observe for a return of color.

==================================================

Question: A newborn is placed under fluorescent light as part of the treatment for physiologic jaundice. During the duration of the newborn's treatment, a nurse should:
a. cover the newborn's closed eyes with patches.
b. measure the newborn's pulse and respirations every two hours.
c. keep the newborn under the light at all times, even during the feedings.
d. notify the physician if the newborns stools become greenish yellow.

Answer: a. cover the newborn's closed eyes with patches.

==================================================

Question: Which of these symptoms should a nurse expect to assess in a client who develops hypoglycemia?
a. Fruity breath odor.
b. Polyuria.
c. Diaphoresis.
d. Flushed skin.

Answer: c. Diaphoresis.

==================================================

Question: A client is eight hours postoperative after a transurethral resection of the prostate (TURP). Which of these observations, if noted by a nurse, indicates a complication?
a. Hourly urine output of 90 mL.
b. Reports of bladder spasms.
c. BP 92/60 mm Hg, pulse rate 118/minute.
d. Pink-tinged urine output.

Answer: c. BP 92/60 mm Hg, pulse rate 118/minute.

==================================================

Question: A nurse should assess a child who has diabetes mellitus (type 1) for symptoms of hyperglycemia, which include:
a. flushed skin and thirst.
b. irritability and hunger.
c. sweating and jitteriness.
d. lethargy and tremors.

Answer: a. flushed skin and thirst.

==================================================

Question: Which of these laboratory test results should a nurse monitor for a client who is receiving intravenous heparin therapy at a rate of 1,500 units per hour for the treatment of an acute pulmonary embolism?
a. Partial thromboplastin time.
b. Clot retraction time.
c. Platelet levels.
d. Bleeding time.

Answer: a. Partial thromboplastin time.

==================================================

Question: Which of these techniques should a nurse use to assess for correct placement of a nasogastric tube prior to administering a feeding?
a. Aspirate 10 mL contents and measure the pH.
b. Slowly inject 50 mL of saline and observe for resistance.
c. Inject 20 mL of water and listen for gurgling sounds.
d. Observe for bubbles after submerging the end of the tube in a cup of water.

Answer: a. Aspirate 10 mL contents and measure the pH.

==================================================

Question: A client has shortness of breath when lying down and usually assumes an upright or sitting position in order to breathe more comfortably. A nurse should document this observation as:
a. dyspnea.
b. bradypnea.
c. orthopnea.
d. apnea.

Answer: c. orthopnea.

==================================================

Question: Which of these instructions should a nurse give to a client when collecting a sputum specimen?
a. "Take a deep breath, then cough and spit into this container."
b. "Gargle with antiseptic mouthwash before you spit into this container.
c. "Spit whatever sputum you have in your mouth into this container."
d. "Drink some fluids to loosen your secretions and the spit into this container."

Answer: a. "Take a deep breath, then cough and spit into this container."

==================================================

Question: A client who is receiving radiation therapy has a nursing diagnosis of imbalanced nutrition: less than body requirements related to diminished taste perception and nausea. Which of these additional nursing diagnoses should a nurse consider for the client?
a. Risk for aspiration.
b. Ineffective protection.
c. Risk for deficient fluid volume.
d. Altered tissue perfusion.

Answer: c. Risk for deficient fluid volume.

==================================================

Question: Which of these menus, if chosen by a parent of a child who has celiac disease, would indicate to a nurse that the parent understands the teaching about a gluten-free diet?
a. Broiled steak, baked potato, and spinach.
b. Pork chop, egg noodles, and green peas.
c. Fried chicken, white roll, and mixed vegetables.
d. Baked macaroni with cheddar cheese and corn.

Answer: a. Broiled steak, baked potato, and spinach.

==================================================

Question: Which of these statements, if made by a nurse, is non-therapeutic because it disregards a client's feelings and concerns?
a. "You appear anxious and tense."
b. "Everything will be okay."
c. "I notice you're biting your nails."
d. "I'm not sure I understand what you're saying."

Answer: b. "Everything will be okay."

==================================================

Question: A client tells a nurse, "I am so scared about the interview tomorrow. I just know I will say the wrong thing and not get the job." Which of these responses, if made by the nurse, will create a communication barrier?
a. "Would you like to practice the interview?"
b. "Have you thought about some possible questions that may be asked in the interview?"
c. "Tell me more about your concerns."
d. "You need to relax, and everything will be fine."

Answer: d. "You need to relax, and everything will be fine."

==================================================

Question: A young healthy adult, who has been exercising in hot weather, has fatigue, loss of appetite, and lightheadedness. Which of these assessments should a nurse make?
a. Determine the client's preferred diet.
b. Measure the client's body temperature.
c. Auscultate the lungs.
d. Ascertain the client's typical sleep pattern.

Answer: b. Measure the client's body temperature.

==================================================

Question: Which of these nursing measures is the priority for a child who has hemophilia and who sustains a leg injury?
a. Ensuring adequate hydration for the child.
b. Soaking the child's injured leg in warm water.
c. Administering the missing factor VIII to the child.
d. Transfusing one unit of whole blood to the child.

Answer: c. Administering the missing factor VIII to the child.

==================================================

Question: Which of these outcomes should a nurse focus on for a client who had a bronchoscopy two hours ago?
a. Preventing hemorrhage.
b. Preventing pneumonia.
c. Preventing aspiration.
d. Preventing dehydration.

Answer: c. Preventing aspiration.

==================================================

Question: A client who had a coronary artery bypass graft four days ago suddenly develops sinus tachycardia and reports shortness of breath and dizziness. Which of these interpretations and actions should a nurse take?
a. This is an expected occurrence following bypass surgery; continue to monitor the client.
b. This indicates normalization of the blood pressure; hold all anti-hypertensive medications.
c. This may be an early sign of heart failure; notify the physician.
d. This indicates hypoxia; administer oxygen at 5/L per minute.

Answer: c. This may be an early sign of heart failure; notify the physician.

==================================================

Question: Which of these lunch selections, if made by a client who has congestive heart failure, should a nurse recognize as indicative of a need for additional instructions?
a. Cottage cheese with fresh fruit salad, whole wheat bread, and herbal tea.
b. Baked chicken with brown rice, mixed green salad, and iced coffee.
c. Egg salad sandwich with mayonnaise, pickles, and seltzer water.
d. Beef tenderloin, carrots, mashed potatoes, and a baked apple.

Answer: c. Egg salad sandwich with mayonnaise, pickles, and seltzer water.

==================================================

Question: Which of the statements if made by a client who is take furosemide (Lasix), supports a nursing diagnosis of knowledge deficit?
a. "This medication will increase the amount and frequency of my urination."
b. "This medication must be taken, even on days when I fell well."
c. "I will need to add more salt to my diet because this medication will increase its excretion."
d. "I should change my position slowly to avoid dizziness related to this medication."

Answer: c. "I will need to add more salt to my diet because this medication will increase its excretion."

==================================================

Question: Which of these statements, if made by a client who has chronic obstructive pulmonary disease, indicates improvement?
a. "I hope to attend my grandson's graduation next month."
b. "I can now walk one more block than I could last month."
c. "I take several quick breaths when I begin to cough."
d. "I do my breathing exercises in the evening after I eat dinner."

Answer: b. "I can now walk one more block than I could last month."

==================================================

Question: An 8-month-old infant is admitted to the hospital because of failure to thrive. Which of these actions should a nurse plan?
a. Limit the parents' interactions with the infant.
b. Consistently assign the care of the infant to the same staff.
c. Rotate assignments so that all staff can evaluate the infant.
d. Limit the infant's activity until the cause of the problem is identified.

Answer: b. Consistently assign the care of the infant to the same staff.

==================================================

Question: Which of these actions should a nurse include to enhance the effectiveness of client teaching sessions?
a. Include all content in one session so as not to overwhelm the client.
b. Initially demonstrate and explain the procedure to the client.
c. Avoid repetition of content.
d. Include all clients on the unit in the sessions.

Answer: b. Initially demonstrate and explain the procedure to the client.

==================================================

Question: Which of these laboratory test results is more important for a nurse to assess for a client who reports chest pain?
a. WBC count.
b. PTT level.
c. Troponin level.
d. Hemoglobin.

Answer: c. Troponin level.

==================================================

Question: A nurse should explain to a primigravida that urine tests will be done at each prenatal visit throughout the pregnancy to measure:
a. specific gravity and pregnancy hormones.
b. culture and white blood cell count.
c. glucose and protein.
d. bacteria and red blood cell count.

Answer: c. glucose and protein.

==================================================

Question: Which of these manifestations should a nurse expect to observe in a client who is diagnosed with paranoid schizophrenia?
a. Regression.
b. Suspiciousness.
c. Catatonia.
d. Hyperactivity.

Answer: b. Suspiciousness.

==================================================

Question: Which of these measures should an emergency room nurse include when speaking with a family experiencing the loss of an infant from Sudden Infant Death Syndrome (SIDS)?
a. Explaining to the parents how SIDS could have been predicted.
b. Discouraging the parents from viewing the infant's body.
c. Encouraging the parents to take the opportunity to say goodbye.
d. Interviewing the parents in-depth about the circumstances of the infants death.

Answer: c. Encouraging the parents to take the opportunity to say goodbye.

==================================================

Question: Which of these assessments is the priority for a client who is admitted with recurrent depression?
a. Previous episodes of depression.
b. Compliance with prescribed medications.
c. Presence of a suicide plan.
d. Problems with communication.

Answer: c. Presence of a suicide plan.

==================================================

Question: Which of these changes in the assessment data of a child who has congestive heart failure should a nurse recognize as indicative of a therapeutic response to prescribed medication therapy?
a. Increased weight.
b. Increased urine output.
c. Increased respiratory rate.
d. Increased heart size.

Answer: b. Increased urine output.

==================================================

Question: Which of these assignments, if delegated to unlicensed assistive personnel (UAP) by a nurse, is appropriate?
a. The UAP is assigned to measure a client's intake and output.
b. The UAP is assigned to assess a client's lung sounds.
c. The UAP is assigned to teach a client about diet restrictions.
d. The UAP is assigned to change a client's postoperative wound dressing.

Answer: a. The UAP is assigned to measure a client's intake and output.

==================================================

Question: A client who has a history of asthma develops an acute asthma attack. Which of these questions should a nurse ask when assessing the etiology of this attack?
a. "Have you eaten any new foods recently?"
b. "How many hours did you sleep last night?"
c. "Are you exercising every day?"
d. "Have you reduced your fluid intake recently?"

Answer: a. "Have you eaten any new foods recently?"

==================================================

Question: Which of these foods should a nurse suggest that a client who is diagnosed with iron-deficiency anemia choose for dinner?
a. Cooked dry beans, green leafy vegetables, and dried fruits.
b. Raw cabbage, tomato juice, and cantaloupe.
c. Fresh fish, peanut butter, and oatmeal.
d. Cheddar cheese, enriched bread, and yellow vegetables.

Answer: a. Cooked dry beans, green leafy vegetables, and dried fruits.

==================================================

Question: A nurse in a prenatal clinic performs Leopold's maneuvers on a client who is 8-months-pregnant primarily to:
a. turn the fetus in the uterus.
b. ease the fetus into the true pelvis.
c. assessment of the location of the placenta.
d. determine the fetal presentation.

Answer: d. determine the fetal presentation.

==================================================

Question: A child is brought to the clinical for serum lead screening because of ingestion of lead-based paint. Which of these manifestations, if present in the child, would indicate early signs of lead toxicity?
a. Convulsive seizures.
b. Behavior changes.
c. Bleeding tendencies.
d. Low-grade fever.

Answer: b. Behavior changes.

==================================================

Question: Which of these recommendations should a nurse make when teaching a client who is to start taking oral prednisone (Deltasone)?
a. "Take this medicine at bedtime, on an empty stomach."
b. "Take this medicine with a hot beverage in the evening."
c. "Take this medicine in the morning, one hour before breakfast."
d. "Take this medicine in the morning with food or milk."

Answer: d. "Take this medicine in the morning with food or milk."

==================================================

Question: Which of these actions should a nurse take prior to initiating prescribed antibiotic therapy for a client who has a urinary tract infection?
a. Measure the body temperature.
b. Cleanse the perineum.
c. Weigh the client.
d. Obtain a urine culture specimen.

Answer: d. Obtain a urine culture specimen.

==================================================

Question: When caring for a client who is receiving oxygen therapy via nasal cannula, a nurse should instruct the client:
a. to inhale through the mouth.
b. to breathe through the nose.
c. to hold the catheter when coughing.
d. to take quick, shallow breaths.

Answer: b. to breathe through the nose.

==================================================

Question: Each of these clients has impaired mobility related to knee surgery. Which client should a nurse assess first?
a. A 20-year-old who has a sports-related injury.
b. A 37-year-old who reports limited mobility.
c. A 59-year-old who has a history of hypertension.
d. A 70-year-old who has bilateral cataracts.

Answer: c. A 59-year-old who has a history of hypertension.

==================================================

Question: The mother of a 2-month-old tells a nurse that the baby is consuming six ounces of plain commercial formula seven times a day, plus one ounce of cereal in the morning and at bedtime. Based on this information, the nurse should conclude that the baby's diet is:
a. too high in calories.
b. too high in iron content.
c. deficient in calcium.
d. insufficient for the baby's age and weight.

Answer: c. A 59-year-old who has a history of hypertension.

==================================================

Question: A nurse plans to assess a client's recent memory. Which of these questions should the nurse include?
a. "Who is your closest friend?"
b. "What was the name of the school you attended?"
c. "What day were you admitted to the unit?"
d. "What did you have for breakfast?"

Answer: d. "What did you have for breakfast?"

==================================================

Question: A client who has a breast tumor says to a nurse, "I am so anxious. Why did I have to get sick now?" Which of these responses, if made by the nurse, is therapeutic?
a. "You will need to find someone to talk over your fears on a regular basis."
b. "What do you think is making you feel so anxious now?"
c. "Are you aware that there are newer, more effective treatments for breast cancer?"
d. "Tell me more about your concerns."

Answer: d. "Tell me more about your concerns."

==================================================

Question: Which of these actions, if taken by a nurse who is transferring a client from the bed to the chair, is correct?
a. The bed is raised to a comfortable working height for the nurse.
b. The wheelchair is placed perpendicular to the bed.
c. The nurse stands behind the client during the transfer.
d. The nurse supports the client in an upright standing position for a few moments.

Answer: d. The nurse supports the client in an upright standing position for a few moments.

==================================================

Question: A nurse should assist a pregnant client who is in the first trimester to achieve the developmental task of this stage of pregnancy, which is:
a. accepting the fact that she is pregnant.
b. accepting the fact that the fetus is a separate being.
c. accepting that she will soon deliver the child.
d. accepting that her body image has changed.

Answer: a. accepting the fact that she is pregnant.

==================================================

Question: When interacting with a client who is paranoid, a nurse should:
a. use touch to place the client at ease.
b. maintain a caring facial expression.
c. stand close to the client.
d. maintain a professional attitude towards the client.

Answer: d. maintain a professional attitude towards the client.

==================================================

Question: Which of these tasks is appropriate for a nurse to delegate to a nursing assistant in an acute care unit?
a. Feeding a client who was admitted with a stroke yesterday.
b. Ambulating a client who was admitted with a myocardial infarction yesterday.
c. Measure the blood pressure of a client who was admitted with an asthma attack yesterday.
d. Suctioning the tracheostomy that was performed on a client yesterday.

Answer: c. Measure the blood pressure of a client who was admitted with an asthma attack yesterday.

==================================================

Question: Which of these techniques should a nurse plan to use with a client who is delusional?
a. Explore the delusion so the client will know it is false.
b. Explain clearly why the client's belief is incorrect.
c. Focus on reality-based topics.
d. Avoid speaking with the client when he/she is delusional.

Answer: c. Focus on reality-based topics.

==================================================

Question: Which of the following manifestations should a nurse recognize as suggestive of right-sided heart failure?
a. Cool extremities and frothy sputum.
b. Jugular vein distention and pedal edema.
c. Orthopnea and frequent cough at night.
d. Weight loss and lower calf pains.

Answer: b. Jugular vein distention and pedal edema.

==================================================

Question: Which of these statements, if made by a nursing student prior to a sterile dressing change, is correct?
a. "I understand that if objects touch other objects on the sterile field they are considered contaminated."
b. "I understand that sterile objects that are below my waist are considered contaminated."
c. "I understand that all objects in the sterile field must be dry."
d. "I understand that contaminated objects can be used if rinsed with an antimicrobial solution."

Answer: b. "I understand that sterile objects that are below my waist are considered contaminated."

==================================================

Question: A nurse reviews a client's prenatal record and notes that the client's last menstrual period (LMP) was on September 18th. Using the Naegele's rule, the nurse should calculate that the client's expected date of delivery (EDD) will be:
a. May 11th.
b. May 25th.
c. June 11th.
d. June 25th.

Answer: d. June 25th.

==================================================

Question: Which of these instructions should a nurse give to a client who has venous insufficiency regarding the use of elastic stockings (TEDs)?
a. "Bunch the TEDs up and pull them on like socks."
b. "Lower the TEDs to your ankles if your legs ache."
c. "Keep the TEDs on at all times."
d. "Put the TEDs on before you get up in the morning."

Answer: d. "Put the TEDs on before you get up in the morning."

==================================================

Question: A nurse assesses a client who is scheduled for a total abdominal hysterectomy at 10:00 A.M. WHich of the factors should the nurse recognize as most likely to influence the outcome of the surgery?
a. The client has voided two times since 5:00 A.M.
b. The client is not able to demonstrate leg exercises because of osteoarthritis.
c. The client takes one acetylsalicylic acid (baby Aspirin) daily.
d. The client reports mouth dryness.

Answer: c. The client takes one acetysalicylic acid (baby Aspirin) daily.

==================================================

Question: A client's urine output is 500 mL in 24 hours. Which of these actions should a nurse take?
a. Report the findings to the physician.
b. Obtain an order for a diuretic.
c. Encourage the client to limit fluid intake.
d. Record the finding and continue to monitor the client.

Answer: a. Report the findings to the physician.

==================================================

Question: A nurse should question an order for a potassium chloride intravenous infusion for which of these clients?
a. A client who has hypoxia.
b. A client who is obese.
c. A client who has anuria.
d. A client who is congested.

Answer: c. A client who has anuria.

==================================================

Question: A 22-year-old college student has a heart rate that is 48/minute and regular during a routine physical examination. Which of these questions should a nurse consider when analyzing this heart rate?
a. Is this student an athlete?
b. Does this student smoke?
c. How much alcohol does this student drink?
d. Is this student feeling anxious?

Answer: a. Is this student an athlete?

==================================================

Question: Which of the following clients should a nurse recognize is most likely to develop diabetic ketoacidosis?
a. A 23-year-old who has type 1 diabetes mellitus and is being treated for a tooth abscess.
b. A 31-year-old gestational diabetic who has occasional bout of nausea.
c. A 55-year-old who has type 2 diabetes mellitus and is adjusting well to the lifestyle changes.
d. A 72-year-old who has type 2 diabetes mellitus and is managed with diet and exercise.

Answer: a. A 23-year-old who has type 1 diabetes mellitus and is being treated for a tooth abscess.

==================================================

Question: Which of these postoperative complications in the first hour after surgery requires immediate intervention?
a. Serous draining on the dressing.
b. Swelling of an extremity under a cast.
c. Vomiting.
d. Dehiscence of a wound.

Answer: d. Dehiscence of a wound.

==================================================

Question: Which of these assessments should a nurse make of a client who had a knee replacement this morning?
a. Pain.
b. Signs of infection.
c. Bowel movement frequency.
d. Range of motion.

Answer: a. Pain.

==================================================

Question: Which of these actions should a nurse take prior to assisting an elderly client to shave his face?
a. Have the client sign a consent form.
b. Determine what medications the client takes.
c. Soften the client's skin by applying lotion.
d. Cleanse the face with a bactericidal solution.

Answer: b. Determine what medications the client takes.

==================================================

Question: Which of these factors should a nurse consider when delegating tasks to unlicensed assistive personnel (UAP)?
a. The UAP's relationship with clients.
b. The UAP's willingness to perform tasks.
c. The UAP's previous experiences on the unit.
d. The UAP's duration of employment on the unit.

Answer: c. The UAP's previous experiences on the unit.

==================================================

Question: Which of these nursing diagnoses is the priority for a young adult client who has first-degree burns of the legs and smoke inhalation from a fire in the home?
a. Pain.
b. Risk for infection.
c. Impaired gas exchange.
d. Body image disturbance.

Answer: c. Impaired gas exchange.

==================================================

Question: A child who has cystic fibrosis is receiving pancrelipase (Pancrease MT) with meals and snacks. To determine if the desired effects of the Pancrease are achieved, a nurse should consider which of these questions?
a. Is the child's blood sugar level within normal limits?
b. Has the child's appetite improved with the medications?
c. Are the child's stools of normal consistency?
d. Does the child report increased belching and flatus?

Answer: c. Are the child's stools of normal consistency?

==================================================

Question: When assessing a group of children, a nurse should recognize which child is at increased risk of developing acute glomerulonephritis?
a. A 3-year-old who has multiple urinary tract anomalies.
b. A 4-year-old who had a streptococcal infection a week ago.
c. A 5-year-old who has recurrent enuresis at night.
d. A 6-year-old who had chicken pox infection two weeks ago.

Answer: b. A 4-year-old who had a streptococcal infection a week ago.

==================================================

Question: A client says to a nurse, "I am Alexander the Great. I am a world leader and must return to my kingdom. I am not taking any medications. I do not want anyone to come near me. I need to protect myself if they do." Which of these problems should the nurse focus on first?
a. Risk for violence.
b. Delusions of grandeur.
c. Disturbed personal identity.
d. Risk for noncompliance.

Answer: a. Risk for violence.

==================================================

Question: When a client who has a diagnosis of depression is taking a monoamine oxidase (MAO) inhibitor, which of these dieatry instructions should a nurse give to the client?
a. "Increase your intake of foods that are high in vitamin C, such as oranges."
b. "Avoid foods that contain tyramine, such as aged cheeses."
c. "Increase your intake of foods high in tryptophan, such as fish."
d. "Restrict foods high in sodium, such as canned soups."

Answer: b. "Avoid foods that contain tyramine, such as aged cheeses."

==================================================

Question: Which of these strategies should a nurse plan for a client who is manic and has lost 30 pounds?
a. Nutritious finger foods.
b. Low-protein diets.
c. Limiting fluids in between meals.
d. Daily weights.

Answer: a. Nutritious finger foods.

==================================================

Question: A 15-year-old child who has type I diabetes mellitus receives an injection of regular insulin 5 units and isophane (NPH) insulin 15 units subcutaneously at 7:00 A.M. before eating breakfast. At 10:30 A.M., the child tells the school nurse, "I am sweating and feel weak." Which of these actions should the nurse take first?
a. Measure the blood sugar.
b. Determine what the child ate for breakfast.
c. Give a simple carbohydrate.
d. Contact the physician.

Answer: a. Measure the blood sugar.

==================================================

Question: A client who has a head injury is drowsy and lethargic, and has clear nasal discharge. Which of these actions should a nurse take?
a. Obtain a specimen of the drainage for culture and sensitivity.
b. Test the drainage for glucose.
c. Cover the nares with sterile gauze.
d. Cleanse the nostrils with sterile saline solution.

Answer: b. Test the drainage for glucose.

==================================================

Question: Which of these actions, if taken by a nursing assistant, should a nurse recognize as increasing the client's risk of developing a nosocomial infection?
a. Wearing non-sterile gloves while emptying the Foley drainage bag.
b. Taping a paper bag to the side rail for tissue disposal.
c. Placing the Foley catheter drainage bag on the bed while transferring the client.
d. Using the same cuff to measure the blood pressures of all the clients on the unit.

Answer: c. Placing the Foley catheter drainage bag on the bed while transferring the client.

==================================================

Question: A nurse is preparing a client for a vaginal examination. Which of these statements should the nurse make?
a. "Go into the bathroom and empty your bladder."
b. "Cleanse your perineal area with betadine solution."
c. "Hold your breath while the speculum remains in place."
d. "Push down as the doctor inserts the speculum."

Answer: a. "Go into the bathroom and empty your bladder."

==================================================

Question: A licensed practical nurse (LPN) is assigned to care for all of these clients. Which client should the nurse assess first?
a. A 25-year-old client who is terminally ill with metastatic testicular cancer.
b. A 37-year-old client who has second-degree burns on both feet.
c. A 49-year-old client who has an acute myocardial infarction related to cocaine ingestion.
d. A 68-year-old client who is bed bound related to severe Parkinson's disease.

Answer: c. A 49-year-old client who has an acute myocardial infarction related to cocaine ingestion.

==================================================

Question: Which of these preventative measures should a nurse manager in a long-term care facility plan to institute to decrease clients' risks for falls?
a. Monitoring clients frequently for evidence of activity intolerance.
b. Placing all client personal items in the bedside drawers.
c. Raising the side rails for all clients who have memory impairment.
d. Maintaining all client beds in the highest position.

Answer: a. Monitoring clients frequently for evidence of activity intolerance.

==================================================

Question: Which of these assessment findings, if present in a primigravida, indicates that the client is experiencing true labor?
a. The pains are felt in the lower abdomen, back, and groin.
b. The Braxton-Hicks contractions have become stronger and more frequent.
c. There is an increased amount of white mucus discharge.
d. There is a progressive increase in effacement and cervical dilatation.

Answer: d. There is a progressive increase in effacement and cervical dilatation.

==================================================

Question: A client is admitted for opiate detoxification for the fifth time. Which of these statements, if made by a staff member, indicates a biased view of the client?
a. "I feel so frustrated when clients are re-admitted."
b. "Addicts relapse because they don't try hard enough."
c. "I think this client needs to consider long-term placement after detoxification."
d. "The team really needs to discuss this client's treatment plan."

Answer: b. "Addicts relapse because they don't try hard enough."

==================================================

Question: Which of these women, each of whom is in labor, should a nurse recognize as in need of immediate attention?
a. A woman who is having contractions every 6 to 8 minutes of mild to moderate intensity.
b. A woman who is receiving oxytocin augmentation and who has contractions lasting 60 to 70 seconds.
c. A woman who is in the active phase of labor and who insists she needs to use the bedpan to have a bowel movement.
d. A woman whose uterine contractions frequency is every two to give minutes.

Answer: c. A woman who is in the active phase of labor and who insists she needs to use the bedpan to have a bowel movement.

==================================================

Question: A nurse has received a report on these assigned clients. Which client should the nurse follow-up first?
a. A client, admitted with acute diverticulitis, who has a white blood cell count (WBC) of 10,000 mm3.
b. A client, admitted with acute pancreatitis, who has a fasting serum glucose of 130 mg/dL today, and had a reading of 160 mg/dL yesterday.
c. A client, admitted with hepatitis, who has jaundice and tea-colored urine.
d. A client who is currently receiving cancer chemotherapy and who has a white blood cell count of 500 mm3 today.

Answer: d. A client who is currently receiving cancer chemotherapy and who has a white blood cell count of 500 mm3 today.

==================================================

Question: Which of these statements, if made by a client who is taking a diuretic, should a nurse recognize as indicative of the need for additional instructions?
a. "I take all of my medications at bedtime so I don't forget them."
b. "I eat one or two bananas every day."
c. "I weigh myself every day in the morning."
d. "I will call my doctor if I have muscle weakness."

Answer: a. "I take all of my medications at bedtime so I don't forget them."

==================================================

Question: A nurse is monitoring a client who had a cystoscopy six hours ago. The nurse should inform the physician of which these manifestations?
a. The client has pink-tinged urine.
b. The client reports burning on urination.
c. The client's white blood cell count is 15,000 mm3.
d. The client appears drowsy.

Answer: c. The client's white blood cell count is 15,000 mm3.

==================================================

Question: Which of these actions should a nurse perform prior to a client's scheduled hemodialysis?
a. Administer prophylactic antibiotics.
b. Weigh the client.
c. Give the client normal saline solution to drink.
d. Measure the urine specific gravity.

Answer: b. Weigh the client.

==================================================

Question: Which of these behaviors, if taken by a staff nurse on a psychiatric unit, indicates a correct understanding of therapeutic techniques?
a. A nurse smiles when speaking with clients who are manic.
b. A nurse uses touch to communicate concern with a depressed client.
c. A nurse sets consistent limits with manipulative clients.
d. A nurse shares own anxiety reduction techniques with a client who has panic attacks.

Answer: c. A nurse sets consistent limits with manipulative clients.

==================================================

Question: A client has been in bed for the past three days. Which of these measures should a nurse include before assisting the client out of bed?
a. Having the client drink a glass of water.
b. Raising the head of the bed.
c. Flexing the client's knees.
d. Assessing the lung sounds.

Answer: b. Raising the head of the bed.

==================================================

Question: A client who has insulin-dependent diabetes mellitus asks a nurse, "What should I do when I feel nervous, sweaty, and hungry?" The nurse should give the client which of these instructions?
a. "Lie down and rest."
b. "Eat a carbohydrate snack."
c. "Take your prn dose of insulin."
d. "Add a slice of bread to your next meal."

Answer: b. "Eat a carbohydrate snack."

==================================================

Question: Which of these tasks should a licensed practical nurse (LPN) delegate to a nursing assistant?
a. Checking the 11 A.M. blood sugar for a client who has ketoacidosis.
b. Measuring the pulse oximetry level for a client who has status asthmaticus.
c. AMbulating a client who had a hip replacement three days ago.
d. Changing the dressing for a client who had wound debridement last week.

Answer: c. AMbulating a client who had a hip replacement three days ago.

==================================================

Question: A 36-week-pregnant woman awakens to find she is having profuse, red vaginal bleeding. A nurse should prepare the woman to have an immediate sonogram to determine the:
a. location of the placenta.
b. uterine response to labor.
c. the fetus's current weight.
d. condition of the uterine vascular bed.

Answer: a. location of the placenta.

==================================================

Question: A nurse is planning to interview a client who speaks limited English. Which of these strategies should the nurse include?
a. Smile frequently during the interview interview to reduce the client's anxiety.
b. Observe the client for indicators of confusion or not understanding questions.
c. Maintain constant eye contact throughout the interview.
d. Keep the interview short to decrease the client's fatigue.

Answer: b. Observe the client for indicators of confusion or not understanding questions.

==================================================

Question: A nurse takes the weight of a normal 2-year-old child who comes in to the pediatric clinic for a well-child visit. If the child weighted 7 lbs, 2 oz. at birth, how much should the nurse expect the child to weight at this visit?
a. 14 lbs, 2 oz.
b. 18 lbs, 6 oz.
c. 28 lbs, 8 oz.
d. 45 lbs, 10 oz.

Answer: c. 28 lbs, 8 oz.

==================================================

Question: A nurse has been discussing the nutritional needs of children with a group of parents in a clinic. Which of these statements, if made by the parent of a 2-year-old child, should the nurse follow up?
a. "I give my child slices of cheese as an afternoon snack."
b. "I give my child a cup of skim milk as an afternoon snack."
c. "I give my child some popcorn as an afternoon snack."
d. "I give my child some yogurt as an afternoon snack."

Answer: c. "I give my child some popcorn as an afternoon snack."

==================================================

Question: Which of these client care situations has the greatest potential for presenting an ethical dilemma for a nurse?
a. Participating in pregnancy termination procedures.
b. Counseling a client who is terminally ill with AIDS.
c. Discussing contraception options with adolescents.
d. Caring for a client who is from a different culture than the nurse.

Answer: a. Participating in pregnancy termination procedures.

==================================================

Question: Which assessment information should a nurse obtain first when a pregnant woman and her husband arrive at the Labor and Delivery Unit?
a. Whether the couple attended birthing classes.
b. The frequency and intensity of labor contractions.
c. The number of previous pregnancies and outcomes.
d. The amount and time of the client's last food intake.

Answer: b. The frequency and intensity of labor contractions.

==================================================

Question: A client who has Parkinson's disease has been identified as being at risk for falls. Which of these actions by a nurse is most likely to reduce the client's risk of falling?
a. Monitor the client's blood pressure after ambulation.
b. Ensure the client wears socks when ambulating.
c. Encourage frequent weight-bearing exercise.
d. Assign an assistant to remain with the client when ambulating.

Answer: d. Assign an assistant to remain with the client when ambulating.

==================================================

Question: A nurse determines that the therapeutic effectiveness of magnesium sulfate (MgSO4) for client who has preeclampsia is achieved when there is increased:
a. urinary output.
b. blood pressure.
c. respiratory rate.
d. uterine movement.

Answer: a. urinary output.

==================================================

Question: Which of these assessments is the initial priority of a client who is one-hour postoperative after an exploratory laparotomy?
a. The appearance of the client's surgical incision.
b. The client's level consciousness.
c. The adequacy of the client's respiratory function.
d. The client's fluid and electrolyte status.

Answer: c. The adequacy of the client's respiratory function.

==================================================

Question: Which of these client reports should a nurse recognize as suggestive of hypothyroidism?
a. "My hands shake whenever I reach for anything."
b. "I feel cold and tired all the time."
c. "I sweat whenever I walk more than one block."
d. "My head aches each evening."

Answer: b. "I feel cold and tired all the time."

==================================================

Question: A nurse is monitoring a client who is taking acetylsalicylic acid (Aspirin) 975 mg daily for adverse effects, which include:
a. loss of joint mobility.
b. increased serum calcium levels.
c. increasing heart failure.
d. occult blood in the stools.

Answer: d. occult blood in the stools.

==================================================

Question: Which of these rationales explains the purpose of nasogastric tube with suction for a client who had abdominal surgery?
a. Prevention of gastric decompression.
b. Removal of secretions from the stomach.
c. Provision of postoperative nutrition.
d. Promotion of abdominal distention.

Answer: b. Removal of secretions from the stomach.

==================================================

Question: A 75-year-old client who is newly admitted to a long-term care facility has all these nursing diagnoses. Which one is the priority?
a. Risk of injury.
b. Anxiety.
c. Sleep pattern disturbance.
d. Chronic.

Answer: a. Risk of injury.

==================================================

Question: A 12-month-old child is playing with the father. Which of these behaviors indicates that the child is demonstrating object permanence?
a. The child transfers a toy to the other hand when given another one.
b. The child returns a block to the same spot on the table.
c. The child looks for a toy that the father has hidden under the table.
d. The child recognizes that a ball of clay is the same when flattened out.

Answer: c. The child looks for a toy that the father has hidden under the table.

==================================================

Question: A nurse should recognize that a client's selection of which of these foods demonstrates a correct understanding of a high-fiber diet for colon cancer prevention?
a. Corn muffin.
b. Bran flakes.
c. Raising muffin.
d. Green salad.

Answer: b. Bran flakes.

==================================================

Question: Which of these discharge instructions should a nurse include for a client who has a ruptured tympanic membrane that occurred during a fall?
a. "No showers or washing of the hair for the next month."
b. "Avoid yawning or holding your head down."
c. "Do not allow any water to enter the ear until healing is confirmed by direct visualization."
d. "Avoid swallowing and coughing until your ear has healed."

Answer: c. "Do not allow any water to enter the ear until healing is confirmed by direct visualization."

==================================================

Question: Which of these nursing measures is appropriate for a client who has recurrent renal calculi?
a. Weighing the client daily before breakfast.
b. Measuring the blood pressure every four hours.
c. Encouraging a daily intake of three liters of fluids.
d. Testing the urine for protein each shift.

Answer: c. Encouraging a daily intake of three liters of fluids.

==================================================

Question: When auscultating the lungs of a woman who is admitted for severe pregnancy-induced hypertension, a nurse notes the presence of crackles and moist respirations. These assessment findings most likely indicate which of these complications?
a. A convulsion is imminent.
b. Pulmonary edema has developed.
c. Bilateral lobar pneumonia is present.
d. Respiratory failure is evident.

Answer: b. Pulmonary edema has developed.

==================================================

Question: A licensed practical nurse (LPN) is assigned to care for all of these clients. Which client should the nurse assess first?
a. A client who is eight-hours postoperative after a hip replacement.
b. A client who is drowsy after falling out a third story window.
c. A client who is four hours post-colonoscopy and polyp removal.
d. A client who is dysphasic after a transient ischemic attack.

Answer: b. A client who is drowsy after falling out a third story window.

==================================================

Question: Which of these clients is at the highest risk of developing osteoporosis?
a. An obese African-American adolescent who does not exercise.
b. A pregnant Asian client who is a vegetarian.
c. A middle-aged Native-American male who is quadriplegic.
d. A thin, elderly Caucasian female who lives alone.

Answer: d. A thin, elderly Caucasian female who lives alone.

==================================================

Question: A nurse is obtaining the health history of a client who is admitted for surgical repair of an inguinal hernia. Which of these factors should the nurse recognize as having the greatest impact on the outcome of the surgery?
a. The client takes several acetylsalicylic acid (Aspirin) tablets daily for knee pain.
b. The client drinks one glass of beer every evening with dinner.
c. The client had a knee replacement six months prior to this admission.
d. The client is allergic to all penicillin-type antibiotics.

Answer: a. The client takes several acetylsalicylic acid (Aspirin) tablets daily for knee pain.

==================================================

Question: A nurse should recognize that a client who has chronic obstructive pulmonary disease (COPD), needs additional instructions if the client makes which of these statements?
a. "I will try to take slow, deep breaths when I feel short of breath."
b. "I will use the albuterol (Proventil) nebulizer before I eat.
c. "I will drink most of my fluids between meals."
d. "I will turn up the oxygen flow rate if I have difficulty breathing."

Answer: d. "I will turn up the oxygen flow rate if I have difficulty breathing."

==================================================

Question: A woman is treated in the emergency room for a broken arm and multiple facial bruises caused by her spouse. Which of these statements, if made by a nurse, is therapeutic?
a. "You should leave this relationship now or you will be sorry."
b. "Are you aware that women who remain in abusive relationships eventually are killed?"
c. "This type of abuse typically recurs after a period of remorse by the abuser."
d. "Can you think of what you did to cause this abuse?"

Answer: c. "This type of abuse typically recurs after a period of remorse by the abuser."

==================================================

Nclex Prioritization

Question: 1. A client admitted to the hospital with a diagnosis of cirrhosis has massive ascites and difficulty breathing. The nurse performs which intervention as a priority measure to assist the client with breathing?

a) repositions side to side every 2 hours
b) elevates the head of the bed 60 degrees
c) auscultates the lung field every 4 hours
d) encourages deep breathing exercises every 2 hours

Answer: 1) B
- The client is having difficulty breathing because of upward pressure on the diaphragm from the ascitic fluid. Elevating the head of the bed enlists the aid of gravity in relieving pressure on the diaphragm. The other options are general measures to promote lung expansion in the client with ascites, but the priority measure is the one that relieves diaphragmatic pressure.

==================================================

Question: 2. A nurse is scheduling a client for diagnostic studies of gastrointestinal (GI) system. Which of the following studies, if ordered, should the nurse schedule last?

a) ultrasound
b) colonoscopy
c) barium enema
d) computed tomography

Answer: 2) C
- When barium is instilled into the lower GI tract, it may take up to 72 hours to clear the GI tract. The presence of barium could cause interference with obtaining clear visualization and accurate results of the other tests listed, if performed before the client has fully excreted the barium. For this reason, diagnostic studies that involve barium contrast are scheduled at the conclusion of other medical imaging studies.

==================================================

Question: 3. A nurse is formulating a plan of care for a client receiving enteral feedings. The nurse identifies which nursing diagnosis as the highest priority for this client?

a) diarrhea
b) risk for aspiration
c) risk for deficient flid volume
d) imbalanced nutrition, less than body requirements

Answer: 3) B
- Any condition in which gastrointestinal motility is slowed or esophageal reflux is possible places a client at risk for aspiration. Although options 1, 3, and 4 may be a concern, these are not the priority.

==================================================

Question: 4. A client arrives at the emergency department with upper gastrointestinal (GI) bleeding and is in moderate distress. The priority nursing action is to:

a) obtain vital signs
b) ask the client about the precipitating events
c) complete an abdominal physical assessment
d) insert a nasogastric (NG) tube and Hematest the emesis

Answer: 4) A
- The priority action is to obtain vital signs to determine whether the client is in shock from blood loss and to obtain a baseline by which to monitor the progress of treatment. The client may not be able to provide subjective data until the immediate physical needs are met. Insertion of an NG tube may be prescribed but is not the priority action. A complete abdominal physical assessment needs to be performed but is not the priority.

==================================================

Question: 5. A client with a history of suicide attempts is admitted to the mental health unit with the diagnosis of depression. Upon the client's arrival, the client's therapist reports to the nurse that the clients telephoned the therapist earlier in the evening and reported having a overwhelming suicidal thoughts. Keeping this information in mind, the priority of the nurse is to assess for:

a) interaction with peers
b) the presence of suicidal thoughts
c) the amount of food intake for the past 24 hours
d) information regarding the past medication regimen

Answer: 5) B
The critical information from the therapist is that the client is having thoughts of self-harm; therefore, the nurse needs further information about present thoughts of suicide so that the treatment plan may be as appropriate as possible. The nurse must make sure the client is safe. The items in options A, C, and D should be assessed; however, evaluation for suicide potential is most important

==================================================

Question: 6. A group of health nurse is caring for a group of homeless people. When planning for the potential needs of this group, what is the most immediate concern?

a) peer support through structured groups
b) finding affordable housing for the group
c) setting up a 24-hour crisis center and hotline
d) meeting the basic needs to ensure that adequate food, shelter, and clothing are available

Answer: 6) D
- The question asks about the immediate concern. The ABCs of community health are always attending to people's basic needs of food, shelter, and clothing. Options A, B, and C are other activities that may be completed at a later time.

==================================================

Question: 7. A community health nurse is working with older residents who were involved in a recent flood. Many of the residents are emotionally despondent, and they refused to leave their homes for days. When planning forth rescue and relocation of these older residents, what is the first item that the nurse needs to consider?

a) contacting the older resident's families
b) attending to the emotional needs of the older residents
c) arranging for ambulance transportation for the oldest residents
d) attending to the nutritional status and basic needs of the older residents

Answer: 7) D
- The question asks about the first thing that the nurse needs to consider. The ABCs of community health are always attending to people's basic needs of food, shelter, and clothing. Options A, B, and C are other activities that may or may not be needed at a later date.

==================================================

Question: 8. A client is scheduled for an arteriogram using a radiopaque dye. The nurse assesses which most critical item before the procedure?

a) vital signs
b) intake and output
c) height and weight
d) allergy to iodine or shellfish

Answer: 8) D
- Allergy to iodine or seafood is associated with allergy to the radiopaque dye that is used for medical imaging examinations. Informed consent is necessary, because an arteriogram requires the injection of a radiopaque dye into the blood vessel. Although options A, B, and C are components of the preprocedure assessment, the risks of allergic reaction and possible anaphylaxis are the most critical.

==================================================

Question: 9. A client in a long-term care facility has had a series of gastrointestinal (GI) diagnostic tests, including an upper GI series and endoscopies. Upon return to the long-term care facility, the priority nursing assessment should focus on:

a) the comfort level
b) activity tolerance
c) the level of consciousness
d) the hydration and nutrition status

Answer: 9) D- Many of the diagnostic studies to identify GI disorders require that the GI tract be cleaned (usually with laxatives and enemas) before testing. In addition, the client most often takes nothing by mouth before and during the testing period. Because the studies may be done over a period that exceeds 24 hours, the client may become dehydrated and/or malnourished. Although options A, B, and C may be components of the assessment, option D is the priority.

==================================================

Question: 10. A nurse is assessing a 39 year old Caucasian female client. The client has a blood pressure (BP) of 152/92 mm Hg at rest, a total cholesterol of level of 190 mg/dL, and a fasting blood glucose level of 110 mg/dL. The nurse would place priority on which risk factor for coronary heart disease (CHD) in this client?

a) age
b) hypertension
c) hyperlipidemia
d) glucose intolerance

Answer: 10) B
- Hypertension, cigarette smoking, and hyperlipidemia are major risk factors for CHD. Glucose intolerance, obesity, and response to stress are also contributing factors. An age of more than 40 years is a nonmodifiable risk factor. A cholesterol level of 190 mg/dL and a blood glucose level of 110 mg/dL are within the normal range. The nurse places priority on major risk factors that need modification.

==================================================

Question: 11. A labor room nurse is caring for a client in labor with a known history of sickle cell anemia. Which priority action would the nurse implement to assist in preventing a sickle cell crisis from occurring during labor?

a) continually reassure and coach the client
b) administer the prescribed oxygen throughout labor
c) maintain strict asepsis throughout the labor process
d) increase the intravenous (IV) fluids if the client complains of feeling thirsty

Answer: 11) B
- During the labor process the client is at high risk for being unable to meet the oxygen demands of labor and is at high risk for sickle cell crisis. An intervention to prevent sickle cell crisis during labor includes administering oxygen. Options A and C are appropriate interventions during labor but are not specific to sickle cell anemia. Intravenous fluids may need to be increased, but a physician's order is needed to do so.

==================================================

Question: 12. A nurse is caring for a client with preeclampsia who suddenly progresses to an eclamptic state. The initial nursing action would be to:

a) check the fetal heart rate
b) check the maternal blood pressure
c) maintain an open airway
d) administer oxygen to the mother by face mask

Answer: 12) C
- The initial nursing action when a client progresses to an eclamptic state (has a seizure) is to maintain an open airway. Options A, B, and D are procedures that may be implemented but option 3 identifies the initial action.

==================================================

Question: 13. A nurse is caring for a client who has wrist restraints applied. Which nursing intervention would receive highest priority regarding the wrist restraints?

a) providing range-of-motion exercises to the wrists
b) removing the restraints periodically per agency guidelines
c) applying lotion to the skin under the restraints
d) assessing color, sensation, and pulses distal to the restraint

Answer: 13) D
- Assessing color, sensation, and pulses distal to the restraint determines the presence of neurovascular compromise that is associated with the use of restraints. All of the other interventions should be implemented, but option 4 is the priority.

==================================================

Question: 14. A registered nurse (RN) has delegated care of a newly postoperative client to a licensed practical nurse (LPN). The LPN notifies the RN that the client's blood pressure and respirations are elevated from the baseline readings and that the client is complaining of pain and dyspnea. The RN takes which action next?

a) the RN need not to carry out further assessment because the LPN is very experienced and trustworthy
b) the RN requests that the LPN offer the client a opioid analgesic, which has ordered postoperatively
c) the RN places a call to the attending surgeon and reports that the client is having pain and dyspneic
d) the RN assesses the client, checks the client's surgical notes, and gathers addition data before calling the surgeon

Answer: 14) D
- The RN must not depend exclusively on the judgment of an LPN because the RN is responsible for supervising those to whom client care has been delegated. The client has recently had surgery, and there is the potential for complications, which may be signaled by alterations in vital signs and respiratory status. An analgesic may be needed, but in order to make that determination, the RN must have more information. A call to the surgeon may be warranted, but the RN has insufficient data at this time. In order to provide the client with the degree of care required, the nurse must assess the client, gather additional information, and analyze that information before notifying the surgeon.

==================================================

Question: 15. A primigravida is admitted to the labor unit. During the assessment of the client, her membranes rupture spontaneously. The priority nursing action is which of the following?

a) monitor the contraction pattern
b) assess the fetal heart rate
c) note the amount, color, and odor of the amniotic fluid
d) check maternal vital signs

Answer: 15) B
- When the membranes rupture in the birth setting, the nurse immediately checks the fetal heart rate to detect changes associated with prolapse or compression of the umbilical cord. Options A, C, and D may be a component of care but are not the priority action.

==================================================

Question: 16. A nurse is assigned to provide care to a client in labor and will care for the client throughout labor and into the postpartum period. The nurse assists in developing a plan of care and determines that the priority assessment in the fourth stage of labor is which of the following?

a) assessing the uterine fundus and lochia
b) checking the mother's temperature
c) encouraging food and fluid intake
d) providing privacy for the parents and their newborn infant

Answer: 16) A
- The fourth stage of labor is the stage of physical recovery for the mother and newborn infant. It lasts from the delivery of the placenta through the first 1 to 4 hours after birth. A potential complication following delivery is hemorrhage. The most significant source of bleeding is the site where the placenta is implanted. It is critical that the uterus remain contracted and that vaginal blood flow is monitored every 15 minutes for the first 1 to 2 hours. Although options B, C, and D are also interventions during this stage, they are not the priority.

==================================================

Question: 17. A client is brought to the emergency department by the police after having lacerated both wrists in a suicide attempt. The nurse should take which initial action?

a) examine and treat the wound sites
b) obtain and record a detailed history
c) encourage and assist the client to ventilate feelings
d) administer an anti-anxiety agent

Answer: 17) A
- The client has a physiological injury, and the nurse would initially examine and treat the wound sites because of bleeding. Although options B,C, and D may be appropriate at some point, the initial action would need to be to treat the wounds.

==================================================

Question: 18. A nurse has just administered a dose of hydralazine hydrochloride (Apresoline) intravenously to a client. Based on the action of this medication, the nurse would initially assess the client's:

a) cardiac rhythm
b) oxygen saturation
c) blood pressure
d) respiratory rate

Answer: 18) C
Hydralazine is a powerful vasodilator that exerts it action on the smooth muscle walls of arterioles. After an intravenous dose is administered, the nurse should check the client's blood pressure every 5 minutes until stable and every 15 minutes thereafter (or per agency procedure). Although options A, B, and D are a component of the assessment, based on the action of the medication the nurse would initially check the client's blood pressure

==================================================

Question: 19. A client is 3 hours postoperative following a right upper lobectomy. The collection chamber of the closed pleural drainage system contains 400 ml of bloody drainage. The client's vital signs are blood pressure 100/50 mmHg, heart rate of 100 beats per minute, and respiratory rate 26 breaths per minute. There is intermittent bubbling in the water seal chamber. One hour following the initial assessment, the nurse notes that the bubbling in the water seal chamber is now constant and the client appears dyspneic. The nurse should first check:

a) lung sounds
b) vital signs
c) the chest tube connections
d) the amount of drainage

Answer: 19) C
- Constant bubbling in the water seal chamber indicates an air leak. This is most likely related to an air leak caused by a loose connection. Other causes might be a tear or incision in the pulmonary pleura, which requires physician intervention. Although the items in options A, B, and D need to be assessed, they should be performed after initial attempts to locate and correct the air leak.

==================================================

Question: 20. A client with mania will be placed in seclusion after overturning two tables and throwing a chair against the wall. Before placing the client in seclusion, the nurse would first:

a) inspect the client for injuries resulting from the incident and initiate appropriate treatment
b) document the behavior leading to seclusion
c) document the time and the client is placed in seclusion
d) make sure that there is a written order by the physician allowing for the seclusion

Answer: 20) A
- The primary concern of the nurse should be to ascertain that the client is injury free or to attend to any injuries that may have resulted. Options B, C, and D are all important tasks for the nurse, but they do not refer to assessment of the client and would not be the nurse's first action.

==================================================

Question: 21. A nurse in a postanesthesia care unit (PACU) receives a client transferred from the operating room. The PACU nurse assesses the client for which of the following first?

a) active bowel sounds
b) adequate urine output
c) orientation to the surroundings
d) a patent airway

Answer: 21) D
- After a transfer from the operating room, the PACU nurse performs an assessment of the client. The ABCs'airway, breathing, and circulation'must be assessed first. Urine output and orientation to the surroundings might also be assessed, but these are not the first actions. The client might not have active bowel sounds at this time because of the effects of anesthesia.

==================================================

Question: 22. A nurse manager is planning the client assignments for the day. Which of the following clients would the nurse assign to the nursing assistant?

a) a 2-day postoperative client who had a below-the-knee amputation
b) a client on a 24-hour urine collection who is on strict bedrest
c) a cleint scheduled to be discharged after coronary artery bypass surgery
d) a client scheduled for a cardiac catheterization

Answer: 22) B
- The registered nurse is legally responsible for client assignments and must assign tasks on the basis of the guidelines of nurse practice acts and the job description of the employing agency. A 2-day postoperative client who had a below-the-knee amputation will require both physiological and psychosocial care. A client scheduled to be discharged after coronary artery bypass surgery will require reinforcement of home care management. A client scheduled for a cardiac catheterization requires physiological needs and frequent nursing assessments. The nursing assistant has been trained to care for a client on bedrest and on urine collections. The nurse manager would provide instructions to the nursing assistant regarding the tasks, but the tasks required for this client are within the role description of a nursing assistant.

==================================================

Question: 23. A registered nurse (RN) must determine how best to assign coworkers (another RN and one licensed practical nurse LPN) to provide care to a group of clients. Which of the following is the appropriate assignment?

a) the RN is assigned to care for an unemployed 26-year old woman, newly diagnosed with acquired immunodeficiency syndrome (AIDS), who has four school-age children
b) the LPN is assigned to care for a 41-year old male, postresection of an acoustic neuroma 2 days ago, transferred from the intensive care unit (ICU) this morning
c) the LPN is assigned to provide discharge teaching about medications and maintenance of nephrostomy tube to a 35-year old man
d) the RN is assigned to care for a 65-year old woman hospitalized because of chest pain, being discharged today to home with no medication

Answer: 23) A
- In order to determine what can and cannot be delegated to a co-worker, several factors need to be considered. The nurse must carefully consider what level of care each client requires immediately and potentially in the future, what competencies are possessed by co-workers, and what legal limitations there are on the practice of those co-workers. In option 2, the client has undergone a serious neurosurgical procedure that can impair swallowing and gag reflexes, and there is significant risk of increased intracranial pressure in the first few days postoperatively. This and the fact that the client has been transferred from the ICU this morning make this an inappropriate assignment for an LPN. The LPN is also not able to provide discharge teaching on medications and treatments to a client. Teaching is a professional responsibility, which the RN cannot delegate to anyone except another RN, making option 3 incorrect. Although under some circumstances the RN might care for a client being discharged following chest pain, the question tells you that there is an LPN available. The RN would be best used to care for the client with more critical or complicated needs. Option 4 is therefore incorrect. The woman newly diagnosed with AIDS, who is unemployed and with small children, is likely to be in need of the skills of an RN in terms of both physiological and psychosocial needs, making option 1 an appropriate assignment.

==================================================

Question: 24. A nurse manager of a medical-surgical unit returns to work after being on vacation for a week. It is the beginning of the shift, and the nurse manager is faced with several activities that need attention. Which activity will the nurse manager attend to first?

a) a crash cart needs checking
b) client assignments for the day
c) a phone message that indicates that the charge nurse of the next shift is ill and will not be reporting to work
d) a stack of mail from the education department and administrative services

Answer: 24) B
- The nurse manager needs to attend to the client assignments first. Client care is the priority. In addition, the nursing staff needs assignments so that they can begin client assessments and begin delivering client care. The nurse manager should next check the crash cart (which is normally done every shift) to ensure that needed equipment is available in the event of an emergency. The nurse manager could also delegate this task to another registered nurse while client assignments are being planned. The nurse manager would next begin the problem-solving process related to finding a charge nurse for the next shift. Because this activity directly affects client care, this would be done before reading the stack of mail.

==================================================

Question: 25. A nurse responds to an external disaster that occurred in a large city when a building collapsed. Numerous victims require treatment. Which victim will the nurse attend to first?

a) an alert victim who has numerous bruises on the arms and legs
b) a victim with a partial amputation of a leg who is bleeding profusely
c) a hysterical victim who received a head injury
d) a victim who sustained multiple serious injuries and is deceased

Answer: 25) B
- The nurse determines which victim will be attended to first on the basis of the acuity level of the victims involved in the disaster. The priority victim is the one who must be treated immediately or life, limb, or vision will be threatened. This victim is categorized as emergent (option 2). The victim who requires treatment, but life, limb, or vision is not threatened if care can be provided within 1 to 2 hours is considered urgent and is the second priority (option 3). The victim who requires evaluation and possible treatment but for whom time is not a critical factor is categorized as nonurgent and is the third priority (option 1).

==================================================

Nclex Priority Questions Examples

Question: 1. A client admitted to the hospital with a diagnosis of cirrhosis has massive ascites and difficulty breathing. The nurse performs which intervention as a priority measure to assist the client with breathing?

a) repositions side to side every 2 hours
b) elevates the head of the bed 60 degrees
c) auscultates the lung field every 4 hours
d) encourages deep breathing exercises every 2 hours

Answer: 1) B
- The client is having difficulty breathing because of upward pressure on the diaphragm from the ascitic fluid. Elevating the head of the bed enlists the aid of gravity in relieving pressure on the diaphragm. The other options are general measures to promote lung expansion in the client with ascites, but the priority measure is the one that relieves diaphragmatic pressure.

==================================================

Question: 2. A nurse is scheduling a client for diagnostic studies of gastrointestinal (GI) system. Which of the following studies, if ordered, should the nurse schedule last?

a) ultrasound
b) colonoscopy
c) barium enema
d) computed tomography

Answer: 2) C
- When barium is instilled into the lower GI tract, it may take up to 72 hours to clear the GI tract. The presence of barium could cause interference with obtaining clear visualization and accurate results of the other tests listed, if performed before the client has fully excreted the barium. For this reason, diagnostic studies that involve barium contrast are scheduled at the conclusion of other medical imaging studies.

==================================================

Question: 3. A nurse is formulating a plan of care for a client receiving enteral feedings. The nurse identifies which nursing diagnosis as the highest priority for this client?

a) diarrhea
b) risk for aspiration
c) risk for deficient flid volume
d) imbalanced nutrition, less than body requirements

Answer: 3) B
- Any condition in which gastrointestinal motility is slowed or esophageal reflux is possible places a client at risk for aspiration. Although options 1, 3, and 4 may be a concern, these are not the priority.

==================================================

Question: 4. A client arrives at the emergency department with upper gastrointestinal (GI) bleeding and is in moderate distress. The priority nursing action is to:

a) obtain vital signs
b) ask the client about the precipitating events
c) complete an abdominal physical assessment
d) insert a nasogastric (NG) tube and Hematest the emesis

Answer: 4) A
- The priority action is to obtain vital signs to determine whether the client is in shock from blood loss and to obtain a baseline by which to monitor the progress of treatment. The client may not be able to provide subjective data until the immediate physical needs are met. Insertion of an NG tube may be prescribed but is not the priority action. A complete abdominal physical assessment needs to be performed but is not the priority.

==================================================

Question: 5. A client with a history of suicide attempts is admitted to the mental health unit with the diagnosis of depression. Upon the client's arrival, the client's therapist reports to the nurse that the clients telephoned the therapist earlier in the evening and reported having a overwhelming suicidal thoughts. Keeping this information in mind, the priority of the nurse is to assess for:

a) interaction with peers
b) the presence of suicidal thoughts
c) the amount of food intake for the past 24 hours
d) information regarding the past medication regimen

Answer: 5) B
The critical information from the therapist is that the client is having thoughts of self-harm; therefore, the nurse needs further information about present thoughts of suicide so that the treatment plan may be as appropriate as possible. The nurse must make sure the client is safe. The items in options A, C, and D should be assessed; however, evaluation for suicide potential is most important

==================================================

Question: 6. A group of health nurse is caring for a group of homeless people. When planning for the potential needs of this group, what is the most immediate concern?

a) peer support through structured groups
b) finding affordable housing for the group
c) setting up a 24-hour crisis center and hotline
d) meeting the basic needs to ensure that adequate food, shelter, and clothing are available

Answer: 6) D
- The question asks about the immediate concern. The ABCs of community health are always attending to people's basic needs of food, shelter, and clothing. Options A, B, and C are other activities that may be completed at a later time.

==================================================

Question: 7. A community health nurse is working with older residents who were involved in a recent flood. Many of the residents are emotionally despondent, and they refused to leave their homes for days. When planning forth rescue and relocation of these older residents, what is the first item that the nurse needs to consider?

a) contacting the older resident's families
b) attending to the emotional needs of the older residents
c) arranging for ambulance transportation for the oldest residents
d) attending to the nutritional status and basic needs of the older residents

Answer: 7) D
- The question asks about the first thing that the nurse needs to consider. The ABCs of community health are always attending to people's basic needs of food, shelter, and clothing. Options A, B, and C are other activities that may or may not be needed at a later date.

==================================================

Question: 8. A client is scheduled for an arteriogram using a radiopaque dye. The nurse assesses which most critical item before the procedure?

a) vital signs
b) intake and output
c) height and weight
d) allergy to iodine or shellfish

Answer: 8) D
- Allergy to iodine or seafood is associated with allergy to the radiopaque dye that is used for medical imaging examinations. Informed consent is necessary, because an arteriogram requires the injection of a radiopaque dye into the blood vessel. Although options A, B, and C are components of the preprocedure assessment, the risks of allergic reaction and possible anaphylaxis are the most critical.

==================================================

Question: 9. A client in a long-term care facility has had a series of gastrointestinal (GI) diagnostic tests, including an upper GI series and endoscopies. Upon return to the long-term care facility, the priority nursing assessment should focus on:

a) the comfort level
b) activity tolerance
c) the level of consciousness
d) the hydration and nutrition status

Answer: 9) D- Many of the diagnostic studies to identify GI disorders require that the GI tract be cleaned (usually with laxatives and enemas) before testing. In addition, the client most often takes nothing by mouth before and during the testing period. Because the studies may be done over a period that exceeds 24 hours, the client may become dehydrated and/or malnourished. Although options A, B, and C may be components of the assessment, option D is the priority.

==================================================

Question: 10. A nurse is assessing a 39 year old Caucasian female client. The client has a blood pressure (BP) of 152/92 mm Hg at rest, a total cholesterol of level of 190 mg/dL, and a fasting blood glucose level of 110 mg/dL. The nurse would place priority on which risk factor for coronary heart disease (CHD) in this client?

a) age
b) hypertension
c) hyperlipidemia
d) glucose intolerance

Answer: 10) B
- Hypertension, cigarette smoking, and hyperlipidemia are major risk factors for CHD. Glucose intolerance, obesity, and response to stress are also contributing factors. An age of more than 40 years is a nonmodifiable risk factor. A cholesterol level of 190 mg/dL and a blood glucose level of 110 mg/dL are within the normal range. The nurse places priority on major risk factors that need modification.

==================================================

Question: 11. A labor room nurse is caring for a client in labor with a known history of sickle cell anemia. Which priority action would the nurse implement to assist in preventing a sickle cell crisis from occurring during labor?

a) continually reassure and coach the client
b) administer the prescribed oxygen throughout labor
c) maintain strict asepsis throughout the labor process
d) increase the intravenous (IV) fluids if the client complains of feeling thirsty

Answer: 11) B
- During the labor process the client is at high risk for being unable to meet the oxygen demands of labor and is at high risk for sickle cell crisis. An intervention to prevent sickle cell crisis during labor includes administering oxygen. Options A and C are appropriate interventions during labor but are not specific to sickle cell anemia. Intravenous fluids may need to be increased, but a physician's order is needed to do so.

==================================================

Question: 12. A nurse is caring for a client with preeclampsia who suddenly progresses to an eclamptic state. The initial nursing action would be to:

a) check the fetal heart rate
b) check the maternal blood pressure
c) maintain an open airway
d) administer oxygen to the mother by face mask

Answer: 12) C
- The initial nursing action when a client progresses to an eclamptic state (has a seizure) is to maintain an open airway. Options A, B, and D are procedures that may be implemented but option 3 identifies the initial action.

==================================================

Question: 13. A nurse is caring for a client who has wrist restraints applied. Which nursing intervention would receive highest priority regarding the wrist restraints?

a) providing range-of-motion exercises to the wrists
b) removing the restraints periodically per agency guidelines
c) applying lotion to the skin under the restraints
d) assessing color, sensation, and pulses distal to the restraint

Answer: 13) D
- Assessing color, sensation, and pulses distal to the restraint determines the presence of neurovascular compromise that is associated with the use of restraints. All of the other interventions should be implemented, but option 4 is the priority.

==================================================

Question: 14. A registered nurse (RN) has delegated care of a newly postoperative client to a licensed practical nurse (LPN). The LPN notifies the RN that the client's blood pressure and respirations are elevated from the baseline readings and that the client is complaining of pain and dyspnea. The RN takes which action next?

a) the RN need not to carry out further assessment because the LPN is very experienced and trustworthy
b) the RN requests that the LPN offer the client a opioid analgesic, which has ordered postoperatively
c) the RN places a call to the attending surgeon and reports that the client is having pain and dyspneic
d) the RN assesses the client, checks the client's surgical notes, and gathers addition data before calling the surgeon

Answer: 14) D
- The RN must not depend exclusively on the judgment of an LPN because the RN is responsible for supervising those to whom client care has been delegated. The client has recently had surgery, and there is the potential for complications, which may be signaled by alterations in vital signs and respiratory status. An analgesic may be needed, but in order to make that determination, the RN must have more information. A call to the surgeon may be warranted, but the RN has insufficient data at this time. In order to provide the client with the degree of care required, the nurse must assess the client, gather additional information, and analyze that information before notifying the surgeon.

==================================================

Question: 15. A primigravida is admitted to the labor unit. During the assessment of the client, her membranes rupture spontaneously. The priority nursing action is which of the following?

a) monitor the contraction pattern
b) assess the fetal heart rate
c) note the amount, color, and odor of the amniotic fluid
d) check maternal vital signs

Answer: 15) B
- When the membranes rupture in the birth setting, the nurse immediately checks the fetal heart rate to detect changes associated with prolapse or compression of the umbilical cord. Options A, C, and D may be a component of care but are not the priority action.

==================================================

Question: 16. A nurse is assigned to provide care to a client in labor and will care for the client throughout labor and into the postpartum period. The nurse assists in developing a plan of care and determines that the priority assessment in the fourth stage of labor is which of the following?

a) assessing the uterine fundus and lochia
b) checking the mother's temperature
c) encouraging food and fluid intake
d) providing privacy for the parents and their newborn infant

Answer: 16) A
- The fourth stage of labor is the stage of physical recovery for the mother and newborn infant. It lasts from the delivery of the placenta through the first 1 to 4 hours after birth. A potential complication following delivery is hemorrhage. The most significant source of bleeding is the site where the placenta is implanted. It is critical that the uterus remain contracted and that vaginal blood flow is monitored every 15 minutes for the first 1 to 2 hours. Although options B, C, and D are also interventions during this stage, they are not the priority.

==================================================

Question: 17. A client is brought to the emergency department by the police after having lacerated both wrists in a suicide attempt. The nurse should take which initial action?

a) examine and treat the wound sites
b) obtain and record a detailed history
c) encourage and assist the client to ventilate feelings
d) administer an anti-anxiety agent

Answer: 17) A
- The client has a physiological injury, and the nurse would initially examine and treat the wound sites because of bleeding. Although options B,C, and D may be appropriate at some point, the initial action would need to be to treat the wounds.

==================================================

Question: 18. A nurse has just administered a dose of hydralazine hydrochloride (Apresoline) intravenously to a client. Based on the action of this medication, the nurse would initially assess the client's:

a) cardiac rhythm
b) oxygen saturation
c) blood pressure
d) respiratory rate

Answer: 18) C
Hydralazine is a powerful vasodilator that exerts it action on the smooth muscle walls of arterioles. After an intravenous dose is administered, the nurse should check the client's blood pressure every 5 minutes until stable and every 15 minutes thereafter (or per agency procedure). Although options A, B, and D are a component of the assessment, based on the action of the medication the nurse would initially check the client's blood pressure

==================================================

Question: 19. A client is 3 hours postoperative following a right upper lobectomy. The collection chamber of the closed pleural drainage system contains 400 ml of bloody drainage. The client's vital signs are blood pressure 100/50 mmHg, heart rate of 100 beats per minute, and respiratory rate 26 breaths per minute. There is intermittent bubbling in the water seal chamber. One hour following the initial assessment, the nurse notes that the bubbling in the water seal chamber is now constant and the client appears dyspneic. The nurse should first check:

a) lung sounds
b) vital signs
c) the chest tube connections
d) the amount of drainage

Answer: 19) C
- Constant bubbling in the water seal chamber indicates an air leak. This is most likely related to an air leak caused by a loose connection. Other causes might be a tear or incision in the pulmonary pleura, which requires physician intervention. Although the items in options A, B, and D need to be assessed, they should be performed after initial attempts to locate and correct the air leak.

==================================================

Question: 20. A client with mania will be placed in seclusion after overturning two tables and throwing a chair against the wall. Before placing the client in seclusion, the nurse would first:

a) inspect the client for injuries resulting from the incident and initiate appropriate treatment
b) document the behavior leading to seclusion
c) document the time and the client is placed in seclusion
d) make sure that there is a written order by the physician allowing for the seclusion

Answer: 20) A
- The primary concern of the nurse should be to ascertain that the client is injury free or to attend to any injuries that may have resulted. Options B, C, and D are all important tasks for the nurse, but they do not refer to assessment of the client and would not be the nurse's first action.

==================================================

Question: 21. A nurse in a postanesthesia care unit (PACU) receives a client transferred from the operating room. The PACU nurse assesses the client for which of the following first?

a) active bowel sounds
b) adequate urine output
c) orientation to the surroundings
d) a patent airway

Answer: 21) D
- After a transfer from the operating room, the PACU nurse performs an assessment of the client. The ABCs'airway, breathing, and circulation'must be assessed first. Urine output and orientation to the surroundings might also be assessed, but these are not the first actions. The client might not have active bowel sounds at this time because of the effects of anesthesia.

==================================================

Question: 22. A nurse manager is planning the client assignments for the day. Which of the following clients would the nurse assign to the nursing assistant?

a) a 2-day postoperative client who had a below-the-knee amputation
b) a client on a 24-hour urine collection who is on strict bedrest
c) a cleint scheduled to be discharged after coronary artery bypass surgery
d) a client scheduled for a cardiac catheterization

Answer: 22) B
- The registered nurse is legally responsible for client assignments and must assign tasks on the basis of the guidelines of nurse practice acts and the job description of the employing agency. A 2-day postoperative client who had a below-the-knee amputation will require both physiological and psychosocial care. A client scheduled to be discharged after coronary artery bypass surgery will require reinforcement of home care management. A client scheduled for a cardiac catheterization requires physiological needs and frequent nursing assessments. The nursing assistant has been trained to care for a client on bedrest and on urine collections. The nurse manager would provide instructions to the nursing assistant regarding the tasks, but the tasks required for this client are within the role description of a nursing assistant.

==================================================

Question: 23. A registered nurse (RN) must determine how best to assign coworkers (another RN and one licensed practical nurse LPN) to provide care to a group of clients. Which of the following is the appropriate assignment?

a) the RN is assigned to care for an unemployed 26-year old woman, newly diagnosed with acquired immunodeficiency syndrome (AIDS), who has four school-age children
b) the LPN is assigned to care for a 41-year old male, postresection of an acoustic neuroma 2 days ago, transferred from the intensive care unit (ICU) this morning
c) the LPN is assigned to provide discharge teaching about medications and maintenance of nephrostomy tube to a 35-year old man
d) the RN is assigned to care for a 65-year old woman hospitalized because of chest pain, being discharged today to home with no medication

Answer: 23) A
- In order to determine what can and cannot be delegated to a co-worker, several factors need to be considered. The nurse must carefully consider what level of care each client requires immediately and potentially in the future, what competencies are possessed by co-workers, and what legal limitations there are on the practice of those co-workers. In option 2, the client has undergone a serious neurosurgical procedure that can impair swallowing and gag reflexes, and there is significant risk of increased intracranial pressure in the first few days postoperatively. This and the fact that the client has been transferred from the ICU this morning make this an inappropriate assignment for an LPN. The LPN is also not able to provide discharge teaching on medications and treatments to a client. Teaching is a professional responsibility, which the RN cannot delegate to anyone except another RN, making option 3 incorrect. Although under some circumstances the RN might care for a client being discharged following chest pain, the question tells you that there is an LPN available. The RN would be best used to care for the client with more critical or complicated needs. Option 4 is therefore incorrect. The woman newly diagnosed with AIDS, who is unemployed and with small children, is likely to be in need of the skills of an RN in terms of both physiological and psychosocial needs, making option 1 an appropriate assignment.

==================================================

Question: 24. A nurse manager of a medical-surgical unit returns to work after being on vacation for a week. It is the beginning of the shift, and the nurse manager is faced with several activities that need attention. Which activity will the nurse manager attend to first?

a) a crash cart needs checking
b) client assignments for the day
c) a phone message that indicates that the charge nurse of the next shift is ill and will not be reporting to work
d) a stack of mail from the education department and administrative services

Answer: 24) B
- The nurse manager needs to attend to the client assignments first. Client care is the priority. In addition, the nursing staff needs assignments so that they can begin client assessments and begin delivering client care. The nurse manager should next check the crash cart (which is normally done every shift) to ensure that needed equipment is available in the event of an emergency. The nurse manager could also delegate this task to another registered nurse while client assignments are being planned. The nurse manager would next begin the problem-solving process related to finding a charge nurse for the next shift. Because this activity directly affects client care, this would be done before reading the stack of mail.

==================================================

Question: 25. A nurse responds to an external disaster that occurred in a large city when a building collapsed. Numerous victims require treatment. Which victim will the nurse attend to first?

a) an alert victim who has numerous bruises on the arms and legs
b) a victim with a partial amputation of a leg who is bleeding profusely
c) a hysterical victim who received a head injury
d) a victim who sustained multiple serious injuries and is deceased

Answer: 25) B
- The nurse determines which victim will be attended to first on the basis of the acuity level of the victims involved in the disaster. The priority victim is the one who must be treated immediately or life, limb, or vision will be threatened. This victim is categorized as emergent (option 2). The victim who requires treatment, but life, limb, or vision is not threatened if care can be provided within 1 to 2 hours is considered urgent and is the second priority (option 3). The victim who requires evaluation and possible treatment but for whom time is not a critical factor is categorized as nonurgent and is the third priority (option 1).

==================================================

Nclex Question Of The Day Ncsbn

Question: An 18 month-old weighing 22 pounds is admitted to the pediatric unit with a diagnosis of dehydration. A replacement bolus of normal saline at 20 mL/kg is ordered to be administered intravenously over 40 minutes.
In mL/hour, what will be the setting for the IV delivery system?

Answer: 300
Using ratio proportion:First, convert 22 pounds to kilograms (22/2.2) = 10 kg20 mL/kg = 20 x 10 kg = 200 mL200 mL/40 minutes = x mL/60 minutes (in an hour)200 x 60 = 12000/40 = 300 mL/hrUsing dimensional analysis:20 mL/kg x 1 kg/2.2 lb x 22 lb x 60 min/hr x 1/40 min = 300 mL/hr

==================================================

Question: The mother of a 2 month-old baby calls a pediatrician's nurse two days after the first DTaP, inactivated polio vaccine (IPV), Hepatitis B and Haemophilus influenzae type B (HIB) immunizations. She reports that the baby feels very warm, cries inconsolably for as long as three hours, and has had several shaking spells. Which immunization would the nurse expect to be primarily responsible with these findings?

A. DTaP
B. IPV
C. Hepatitis B
D. HIB

Answer: A
DTaP immunization is a vaccine that protects against diptheria, tetanus and pertussis (whooping cough). The majority of reactions described in this question occur with the administration of the DTaP vaccination. Contraindications to giving repeat DTaP immunizations include the occurrence of severe side effects after a previous dose, as well as signs of encephalopathy within seven days of the immunization.

==================================================

Question: A client diagnosed with angina has been instructed about the use of sublingual nitroglycerin. Which statement made by the client is incorrect and indicates a need for further teaching?

A. "I'll call the health care provider if pain continues after three tablets five minutes apart."
B. "I will rest briefly right after taking one tablet."
C. "I understand that the medication should be kept in the dark bottle."
D. "I can swallow two or three tablets at once if I have severe pain."

Answer: D
Clients must understand that just one sublingual tablet should be taken at a time and placed under the tongue. After rest and a five-minute interval, a second and then eventually a third tablet may be necessary.

==================================================

Question: The nurse is working with victims of domestic abuse. The nurse should understand which of these factors is a reason why domestic violence or emotional abuse remains extensively undetected?

A. The expenses due to police and court costs are prohibitive
B. Little knowledge is known about batterers and battering relationships
C. There are typically many series of minor, vague complaints
D. Few people who have been battered seek medical care

Answer: C
Signs of domestic violence or emotional abuse may not be clearly manifested and include many series of a minor complaints such as headache, abdominal pain, insomnia, back pain and dizziness. These may be covert indications of violence or abuse that go undetected. These complaints may be vague and reflect ambivalence about the disclosure of any violence or abuse.

==================================================

Question: The nurse is obtaining an aerobic wound culture from a client with stage two pressure injury. The nurse first removes a gauze dressing and observes a moderate amount of purulent drainage on the dressing and then the nurse performs hand hygiene. What is the next correct step in the procedure?

A. Swab the gauze dressing that was removed from the wound
B. Irrigate the wound with normal saline
C. Obtain a culture by rotating a sterile swab in the open wound
D. Remove wound exudate from the wound edges with a cotton tip applicator

Answer: B
After removing the dressing and performing hand hygiene, the wound needs to be irrigated to remove surface pathogens before the nurse can obtain a wound culture. Cultures are not obtained from wound exudate on the dressing or wounds that have not been irrigated since the exudate may be contaminated with normal skin flora.

==================================================

Question: The nurse is caring for a client who is experiencing frightening hallucinations that are markedly increased at night. The client's partner asks to stay a few hours beyond the visiting time, in the client's private room. What would be the best response by the nurse?

A. "Yes, staying with the client and orienting the client to the surroundings may decrease any anxiety."
B. "No, your presence may cause the client to become more anxious."
C. "No, it would be best if you brought the client some reading material that the client could read at night."
D. "Yes, would you like to spend the night when the client's behavior indicates that the client is or will be frightened?"

Answer: A
Encouragement of a family member or a close friend to stay with the client in a quiet surrounding cannot only help increase orientation, but can also minimize confusion and anxiety. The visitor could also report to the nurse any unusual findings of the client. This would be the most supportive approach for this client.

==================================================

Question: The RN, who is functioning as the charge nurse, needs to determine shift assignments. How will the charge nurse determine which client assignments are appropriate for the licensed practical nurse (LPN)?

A. Ask the LPN about prior experience caring for clients with similar diagnoses
B. Determine how many nursing assistants are available to help the LPN with client care
C. Refer to the list of technical tasks LPNs are trained to perform
D. Review the procedure manual with the LPN prior to making an assignment

Answer: A
The definition of assignment is the routine care, activities and procedures that are within the authorized scope of practice of the RN or LPN/LVN. The RN must determine the needs of the clients and make assignments not only based on scope of practice, but also education, demonstrated competency and skill level. Regardless if the LPN received education and training to perform specific skills, the RN needs to determine the LPN's experience with caring for clients with similar diagnoses. While the RN is responsible for ensuring an assignment given to a delegatee is carried out completely and correctly, the LPN must be able to perform the skills or tasks independently.

==================================================

Question: The nurse is caring for a school-aged child with a diagnosis of secondary hyperparathyroidism after treatment for chronic renal disease. Which serum lab data should receive priority attention by the nurse?

A. Osmolality and sodium
B. Blood urea nitrogen and magnesium
C. Calcium and phosphorus
D. Glucose and potassium

Answer: C
The parathyroid regulates the calcium and phosphorus serum levels. Calcium and phosphorous levels will be elevated in hyperfunction of this gland until the client is stabilized. To recall this information think of a see-saw. Associate that calcium is first in the alphabet and thus calcium follows the direction of the abnormality - hyper or hypo function - of the parathyroid. Put the calcium on one side and the phosphorus on the other side of the see-saw.

==================================================

Question: The nurse is caring for a client who just had a central venous catheter line inserted at the bedside. Which of these assessments requires immediate attention by the nurse?

A. Pallor in the extremities
B. Increased temperature by one degree
C. Involuntary coughing spells
D. Dyspnea at rest

Answer: D
Complications of central catheter insertion include pneumothorax and hemothorax. Air embolism is another potential complication. Dyspnea, shallow respirations, sudden sharp chest pain that worsens with coughing or deep breathing are indications of pneumothorax. Other potential complications of central catheters may include thrombosis, local or systemic infection, or even cardiac tamponade (if the central line perforates the heart). When considering the options listed, the client who is dyspneic after central line insertion would be the greatest concern for the nurse.

==================================================

Question: The nurse is providing preprocedural education to the client preparing for a barium enema. What statement made by the client indicates a need for further education?

A. "I will need to drink plenty of fluids and eat foods high in fiber after the procedure."
B. "I will use the prescribed laxative before the procedure."
C. "I will not eat or drink anything after midnight before the procedure."
D. "A barium enema is used to examine the upper and lower GI tracts."

Answer: D
A barium enema involves filling the large intestine (lower GI tract) with diluted barium liquid while x-ray images are taken. After the procedure, a small amount of barium will be immediately expelled and the remainder will be excreted in the stool. Because barium liquid may cause constipation, clients should eat foods high in fiber and drink plenty of fluids to help expel the barium from the body.

==================================================

Question: A client admitted with heart failure is experiencing severe shortness of breath and states, "I feel like something is terribly wrong!" The client is restless and begins to cough up large amounts of pink frothy sputum. The client's skin is a dusky grayish color and the oxygen saturation levels have decreased from 92% to 76% in the last hour. What is the first action the nurse should take?

A. Check vital signs
B. Administer the PRN ordered oxygen
C. Call the health care provider
D. Place the bed in high Fowler's position

Answer: B
When dealing with a medical emergency, the rule is to assess airway first, then breathing, and then circulation. Starting oxygen is the priority. The other actions should also be implemented as quickly as possible, including activation of the rapid response team. The client is experiencing an acute episode of fulminant pulmonary edema, likely as a result of a new and severe cardiac event and possible cardiogenic shock. Emergency assessment and intervention is indicated to prevent cardiac arrest and possible death.

==================================================

Question: There is an order for a continuous lidocaine infusion at a rate of 4 mg/minute to treat PVCs. The IV solution contains 2 grams of lidocaine in 500 mL of D5W. The infusion pump delivers 60 microdrops/mL.
What rate in microdrops/minute would deliver 4 mg of lidocaine/minute? Report the response using a whole number.

Answer: 60
Dimensional analysis (DA): Remember in DA, you always want to start your equation with what's called for in the solution. In this case, you want to know microdrops/minute.microdrops/minute = 4 mg/min X 1 g/1000 mg X 500 mL/2 g X 60 microdrops/mL = 4 X 500 X 60/1000 X 2 = 120000/2000 = 60 microdrops/mLAnother way to solve for X:What you have: 2 grams (2000 mg) lidocaine in 500 mL AND you are using a microdrip set (60 microdrops/mL)What you want/need: 4 mg lidocaine to infuse/minute4 mg/min X 500 mL/2000 mg X 60 (microdrops)/min = 60 microdrops/minute

==================================================

Question: The nurse is reviewing client assignments at the beginning of the shift. Which task could be safely assigned to an unlicensed assistive person (UAP)?

A. Stay with a client during the self-administration of insulin
B. Clean and apply a dressing to a small pressure ulcer on the leg
C. Empty a client's colostomy bag
D. Monitor a client's response to passive range of motion exercises

Answer: C
If the UAP has demonstrated competency in the task, s/he may empty a client's colostomy bag. This is an uncomplicated, routine task with an expected outcome. The other tasks involve one or more parts of the nursing process and cannot be assigned to an UAP.

==================================================

Question: The school nurse is screening the children for scoliosis. At what time of development should the nurse expect to see early findings of scoliosis?

A. During the years when children begin to run and jump
B. During a preadolescent growth spurt
C. In early infancy before 8 months of age
D. When a child begins to play competitive sports

Answer: B
Idiopathic scoliosis is seldom apparent before 10 years of age and is most noticeable at the beginning of the preadolescent growth spurt. It is more common in females than in males.

==================================================

Question: The home care nurse is admitting a new client with a diagnosis of COPD, atrial fibrillation and gout. After reviewing the client's medication list, the nurse would arrange for periodic monitoring of blood drug levels for which of the following medications? (Select all that apply.)

A. Beclomethasone inhaled (Qvar)
B. Digoxin (Lanoxin)
C. Theophylline (Elixophyllin, Theo-24, Uniphyl)
D. Allopurinol (Aloprim, Zyloprim)
E. Glipizide (Glucotrol)

Answer: B,C
It is necessary to monitor blood levels for the client taking theophylline and digoxin to prevent the client from developing toxicity.

==================================================

Question: The nurse is working with clients who are diagnosed with eating disorders. Which eating disorder would the nurse expect to cause the greatest fluctuation in serum potassium levels?

A. Dysthymic disorder
B. Anorexia nervosa
C. Binge eating disorder
D. Bulimia nervosa

Answer: D
Hypokalemia can be caused by overuse of laxatives and by prolonged fasting and starvation. But the greatest fluctuation in potassium levels is associated with bulimia, due to the purging process that causes dehydration and potassium loss. Low potassium levels can cause weakness, abdominal cramping and irregular heart rhythms. Dysthymic disorder is associated with poor appetite or overeating.

==================================================

Question: The nurse has an order to insert an indwelling urinary catheter for a male client. What is the best reason for lubricating the tip of the catheter prior to insertion?

A. Reduce the friction within the urethra
B. Diminish the leakage of urine around the catheter
C. Minimize risk for infection
D. Prevent bladder distention

Answer: A
Due to the somewhat long length of the male urethra, lubrication reduces potential discomfort and localized tissue irritation as the catheter is passed.

==================================================

Question: A client asks the nurse about including her 2 year-old and 12 year-old sons in the care of their newborn sister. Which response is an appropriate initial statement by the nurse?

A. "Focus on your sons' needs during the first days at home."
B. "Suggest that your partner spend more time with the boys."
C. "Tell each child what he can do to help with the baby."
D. "Ask the children what they would like to do for the newborn."

Answer: A
In an expanded family, it is important for parents to reassure older children that they are loved and as important as the newborn.

==================================================

Question: The nurse is caring for a client who is exhibiting a panic attack. What should the nurse do for this client?

A. Assist the client to describe the experience in detail
B. Develop a trusting relationship
C. Maintain safety for the client
D. Teach the client to control behaviors

Answer: C
Clients who display signs of severe anxiety in the form of a panic attack need to be supervised closely until the anxiety is lessened. They may harm themselves or others because during panic attacks perception is narrowed and thinking is flawed.

==================================================

Question: The nurse is to review the topic of caring for clients with Guillain-Barré syndrome with other staff members at a monthly meeting. Which of these findings should the nurse include in the discussion? (Select all that apply.)

A. Weakness, tingling or loss of sensation in legs and feet occur first
B. Rapidly progressive ascending paralysis of the legs, arms, respiratory muscles and face
C. Difficulty with bladder control or intestinal functions
D. Hypertension
E. Difficulty with eye movement, facial movement, speaking, chewing or swallowing
F. Numbness, tingling, prickling sensation or moderate pain throughout the body

Answer: A,B,C,E,F
Guillian-Barré is an autoimmune disease. The symptoms of weakness or tingling sensation begins in the legs and progresses to the arms and upper body, resulting in almost complete paralysis. The client is often put on a ventilator during the worst part of the disease to assist breathing. The client may have low blood pressure or poor blood pressure control.

==================================================

Question: A 1 year-old child is receiving temporary total parental nutrition (TPN) through a central venous line. This is the first day of TPN therapy. Although all of the following nursing actions must be included in the plan of care of this child, which one would be a priority at this time?

A. Use aseptic technique during dressing changes
B. Check results of liver enzyme tests
C. Maintain central line catheter integrity
D. Monitor serum glucose levels

Answer: D
Hyperglycemia may occur during the first day or two as the child adapts to the high-glucose load of the TPN solution. Thus, a priority nursing responsibility is blood glucose testing.

==================================================

Question: The nurse is teaching diet restrictions to a client diagnosed with Addison's disease. The client indicates an understanding of the dietary restrictions when making which of these statements?

A. "I will increase fluids and restrict sodium and potassium."
B. "I will increase sodium and fluids and restrict potassium."
C. "I will increase sodium, potassium and fluids."
D. "I will increase potassium and sodium and restrict fluids."

Answer: B
The manifestations of Addison's disease (also called adrenal insufficiency or hypocortisolism) are due to mineralocorticoid deficiency that results in renal sodium wasting and potassium retention. Other findings are dehydration, hypotension, hyponatremia, hyperkalemia and metabolic acidosis.

==================================================

Question: A nurse is working in an inpatient psychiatric setting. The nurse understands what reason touching clients should be limited to a quick handshake?

A. A handshake allows the use of therapeutic touch while maintaining boundaries.
B. Touching a client, other than a handshake, can set off a violent episode.
C. Refraining from touching signals the termination of the nurse-client relationship.
D. A handshake will not be misinterpreted as an invitation to more sexual behavior.

Answer: A
The therapeutic use of touch is a basic part of the nurse-client relationship. However, in a psychiatric setting, the extent of physical contact should be limited to handshakes. Some facilities may even have a no-touch policy, especially when working with clients who have a history of sexual trauma. Even reassuring touching can be misinterpreted by the client.

==================================================

Question: Upon completion of the admission documents, the nurse identifies that an elderly client does not have an advance directive. What action should the nurse take?

A. Document this information on the chart
B. Refer this issue to the nurse manager and the risk manager
C. Give the client written information about advance directives
D. Assume that the client wishes full resuscitation efforts

Answer: C
For each admission, nurses should request a copy of a client's current advance directive. If there is none, the nurse must provide written information about what an advance directive implies. It is then the client's choice to sign the forms. Note that a standard is for non-direct care providers to witness these forms; a social worker or other health care professional would need to witness a client's signature.

==================================================

Question: The clinic nurse is examining a 15 month-old child with suspected otitis media. Which group of findings should the nurse anticipate?

A. Vomiting, pulling at ears and pearly white tympanic membrane
B. Periorbital edema, absent light reflex and translucent tympanic membrane
C. Diarrhea, retracted tympanic membrane and enlarged parotid gland
D. Irritability, rhinorrhea, and bulging tympanic membrane

Answer: D
Clinical manifestations of otitis media include irritability, rhinorrhea, bulging tympanic membrane, and pulling at the ears.

==================================================

Question: The client is diagnosed with a large spontaneous pneumothorax and will have a chest tube inserted. The nurse understands that the chest tube is needed for which of the reasons listed below?

A. Increase intrathoracic pressure to allow both lungs to expand equally
B. Drain the purulent drainage from the empyema that caused the problem
C. Prevent an accumulation of blood and other drainage into the pleural cavity
D. Drain air from the pleural cavity and restore normal intrathoracic pressure

Answer: D
There are no clinical signs or symptoms in primary spontaneous pneumothorax until a cyst or small sac (bleb) ruptures. When air enters the pleural space, the pressure in the space equals the pressure outside the body; the vacuum is lost and the lung collapses. This causes acute onset chest pain and shortness of breath. A small pneumothorax without underlying lung disease may resolve on its own. A larger pneumothorax requires aspiration of the free air and/or placement of a chest tube to evacuate the air.

==================================================

Question: A nurse practicing in a maternity setting has a client whose fetus is post-mature. The nurse recognizes that the fetus is at risk due to what factor?

A. Excessive fetal weight
B. Low blood sugar levels
C. Progressive placental insufficiency
D. Depletion of subcutaneous fat

Answer: C
The placenta functions less efficiently as pregnancy continues beyond 42 weeks. Immediate and long-term effects may be related to hypoxia. These newborns are typically meconium stained.

==================================================

Question: A client is scheduled for a transesophageal echocardiogram (TEE). Prior to the procedure, which activity could be delegated to the unlicensed assistive person (UAP)?

A. Assess the client's psychological state
B. Provide basic instructions about the procedure
C. Obtain a signed consent
D. Remove the pitcher of water from the bedside table

Answer: D
Removing the water pitcher would be an appropriate task because the client would be NPO. The health care provider is responsible for instructions about the procedure and needs to address client questions or concerns. The nurse is typically responsible to obtain a signed consent form and to assess the client both physically and psychologically before the procedure.

==================================================

Question: The nurse and a student nurse are discussing the health issues related to a laboring HBsAg-positive client. Which of these comments by the student is incorrect and indicates a need for further instruction?

A. "The infant will receive the hepatitis B vaccine within 12 hours after birth."
B. "The HBsAg-positive mother should be reported to the state or local health department."
C. "The HBsAg-positive mother should not breastfeed her baby."
D. "The infant will receive the hepatitis B immune globulin within 12 hours after birth."

Answer: C
All persons with HBsAg-positive laboratory results should be reported to the state or local health department. The newborn should receive the hepatitis B immune globulin and hepatitis B vaccine within 12 hours after birth, using different sites (the second vaccine is given between 1 and 2 months; the last vaccine is given between 6 and 18 months). HBV is not spread by breastfeeding, kissing, hugging, coughing, or casual contact.

==================================================

Question: The nurse is planning care for a 12 year-old child diagnosed with sickle cell disease who is in a vaso-occlusive crisis of the elbow. Which intervention should be included in the plan of care?

A. Passive range of motion exercise
B. Pain management
C. Cold compresses to elbow
D. Fluid restriction

Answer: B
Management of a sickle cell crisis is directed towards supportive and symptomatic treatment. The priority of care is pain relief. In a 12 year-old child, patient-controlled analgesia promotes maximum comfort. Fluid are usually increased and range of motion exercises are avoided in the acute phase of the crisis. Cold is avoided because it constricts the vessels and may result in increased pain.

==================================================

Question: A nurse is assessing the growth of children during their school-age years. What would the nurse expect to see during this assessment?

A. Decreasing amounts of body fat and muscle mass
B. Little change in body appearance from year to year
C. Yearly weight gain of about 5 1/2 pounds per year
D. Progressive height increase of 4 inches each year

Answer: C
School-age children gain about 5 1/2 pounds each year and increase about 2 inches in height.

==================================================

Question: The nurse is assessing a child with suspected lead poisoning. Which assessment should a nurse expect to find?

A. Auditory wheezes with expiration
B. Numbness and tingling in feet
C. Excessive perspiration
D. A history of difficulty sleeping

Answer: B
A child who has unusual neurologic complaints, such as neuropathy or footdrop that cannot be attributed to other causes, may be affected by lead poisoning. This may occur when a child ingests or inhales paint chips from lead-based paint or dust during remodeling in older buildings. Other findings of lead poisoning are appearance of bluish gum line, hyperactivity and developmental delays.

==================================================

Question: The nurse assesses delayed gross motor development in a 3 year-old child. The inability of the child to do which action confirms this finding?

A. Catch a ball
B. Ride a bicycle
C. Skip on alternate feet
D. Stand on one foot

Answer: D
At this age, gross motor development allows a child to balance on one foot.

==================================================

Question: A client has a history of chronic obstructive pulmonary disease (COPD). The nurse enters the client's room to find that the nasal cannula is in proper position with the oxygen set at 6 liters per minute, the client's color is flushed and the respirations are 8 per minute. What should the nurse do first?

A. Remove the nasal cannula for at least five minutes
B. Lower the oxygen's flow rate
C. Place client in a higher sitting position
D. Check the client's pulse for strength and rate

Answer: A
The client has findings of oxygen toxicity so the nurse should first remove the cannula for a least five minutes. Then the nurse should perform these next sequence of actions: pulse assessment, change of position and then lower the oxygen flow rate and reapply if respirations are within normal parameters. A higher concentration of supplemental oxygen removes the hypoxic drive to breathe and leads to increased hypoventilation, respiratory decompensation, and the development or worsening of respiratory acidosis.

==================================================

Question: A client who is 12 hours postop becomes confused and says: "Giant sharks are swimming across the ceiling." Which assessment is necessary by the nurse to adequately identify the source of this client's behavior?

A. Peripheral glucose stick
B. Cardiac rhythm strip
C. Pupillary response
D. Pulse oximetry

Answer: D
A sudden change in mental status in any postop client should trigger a nursing intervention directed toward evaluation of the client's respiratory status. Pulse oximetry would be the initial assessment. If available, arterial blood gases would be better. Acute respiratory failure is the sudden inability of the respiratory system to maintain adequate gas exchange, which may result in hypercapnia and/or hypoxemia. Clinical findings of hypoxemia include these finding, which are listed in order of initial to later findings: restlessness, irritability, agitation, dyspnea, disorientation, confusion, delirium, hallucinations, and loss of consciousness. While there may be other factors influencing the client's behavior, the first nursing action should be directed toward maintaining oxygenation. Once respiratory or oxygenation issues are ruled out, then significant changes in glucose would be evaluated.

==================================================

Question: A client continuously calls out to the nursing staff when anyone passes the client's door and asks them to do something in the room. The charge nurse should take which approach for this client?

A. Reassure the client that a staff person will check frequently to see if the client needs anything
B. Arrange for each staff member to go into the client's room to check on needs every hour on the hour
C. Keep the client's room door cracked to minimize the distractions of people passing by the room
D. Assign a nursing staff member to visit the client at regular intervals

Answer: D
Regular, frequent, planned contact by a designated staff member is the best approach to provide a continuity of care and communicate to the client that care will be available as needed.

==================================================

Question: A nurse working at a clinic is reviewing a client's blood sugar log and recognizes that the client is not consistently monitoring blood sugar. Which of the following diagnostic tests would assist the nurse in evaluating the client's overall management of diabetes?

A. Hemoglobin
B. Fasting blood sugar
C. Hemoglobin A1C
D. White blood cell count

Answer: C
The hemoglobin A1C is the best indicator of glycemic control because it reflects an average of the blood sugar over the life of a red blood cell (approximately 90 to 120 days). The fasting blood sugar will only evaluate the client's blood sugar at that specific testing time. Hemoglobin and a white blood cell count are not used to determine blood sugar levels.

==================================================

Question: A client who has returned from surgery reports feeling nauseated and later has an emesis. The nurse administers promethazine per standing orders. In addition to relief from nausea, what other effects of this medication does the nurse expect? (Select all that apply.)

A. Dry mouth
B. Sedation
C. Pinpoint pupils
D. Heart palpitations
E. Runny nose

Answer: A,B,D
Promethazine (Phenergan) is used as an antihistamine, sedative and antiemetic. It produces anticholinergic effects, such as dry mouth and nasal congestion, dilated pupils and urinary retention. Although promethazine is a sedative, the nurse should understand that it can cause some people to have heart palpitations and to feel restless and unable to sleep.

==================================================

Question: The nurse suspects that the client is in cardiogenic shock. Which of the following findings supports this information?

A. Bradycardia
B. Increased cardiac output
C. Decreased or muffled heart sounds
D. Bounding pulses

Answer: C
Cardiogenic shock involves decreased cardiac output and evidence of tissue hypoxia in the presence of adequate intravascular volume; it is the leading cause of death in acute MI. Findings of cardiogenic shock include hypotension, rapid and faint peripheral pulses, distant-sounding heart sounds, cool and mottled skin, oliguria and altered mental status.

==================================================

Question: The client is admitted with a pressure ulcer that's two inches in diameter with no tunneling. It is a shallow open ulcer with loss of dermis and a red/pink wound bed. The nurse observes some serous drainage. What intervention does the nurse anticipate will be ordered to treat this wound?

A. Hydrogel dressing
B. Whirlpool treatment and debridement
C. Alginate dressing with silver added
D. Alternating pressure pad overlay for the bed

Answer: A
This ulcer is a partial thickness wound. These types of wounds heal by tissue regeneration, which is why the nurse would expect a gel dressing to be ordered. This dressing will keep the wound moist, provide protection from infection and promote healing; also, the cool sensation provided by the gel offers pain relief. Pink/red wound edges are considered normal in the inflammatory stage of healing; the wound does not require debridement. There is nothing to indicate that there's an infection, which is why the alginate with silver is not needed; also, alginate dressings are better for wounds with moderate-to-heavy drainage and are good for filling cavities or tracts. An alternating pressure pad overlay would not treat the wound.

==================================================

Question: The nurse is evaluating a stage III pressure ulcer while performing a dressing change. Which wound assessment findings indicate that the prescribed treatment is appropriate to support wound healing? (Select all that apply.)

A. The wound base is moderately moist, shiny and red
B. Clumps of soft yellow tissue adhere to the wound bed
C. The size of the wound is decreasing
D. The periwound texture is moist and soft
E. The edge of the wound appears rolled or curled under
F. A fruity odor is noted on the dressing

Answer: A,C
A wound base that's moist, shiny and "beefy" red indicates good blood flow, new tissue growth and healing. Slough is clumps or strings of moist and soft tissue and can be yellow, tan or green in color - slough will impede healing. A fruity odor indicates infection. Soft and denuded tissues in the periwound indicate tissue breakdown due to excessive moisture from wound drainage. Curled or rolled wound edges (epibole) prevents epithelial cells from migrating to close the wound.

==================================================

Question: A client with anemia has a new prescription for ferrous sulfate. When teaching the client about diet and iron supplements, what should the nurse emphasize about taking an iron supplement?

A. Lie down for about 10 minutes after taking the pill
B. Take the iron tablet with a glass of orange juice
C. Take an antacid with the iron supplement to reduce stomach upset
D. Take the iron tablet with a glass of low-fat milk

Answer: B
Iron is best taken on an empty stomach, one hour before or two hours after meals, with a full glass of water or orange juice (ascorbic acid enhances the absorption of iron.) The client should not take the medication with antacids, dairy products, coffee or tea because these will decrease the effectiveness of the medicine. The client should not lie down for at least 10 minutes after taking the medicine.

==================================================

Question: A client has been given a prescription for alendronate. Which of the following statements indicate the client understands how to safely take this medication? (Select all that apply.)

A. "I will notify the health care provider if I have any difficulty swallowing."
B. "I will take the pill immediately preceding weight-bearing exercise."
C. "I will swallow it with 8 ounces of water."
D. "I will stand or sit quietly for 30 minutes after taking it."
E. "I will always eat breakfast before taking it."

Answer: A,C,D
Alendronate (Fosamax) can cause esophagitis or esophageal ulcers unless precautions are followed. The client must be able to sit upright or stand for at least 30 minutes after taking the tablet. The client should take the tablet first thing in the morning, with a full glass of water, at least 30 minutes before eating or drinking anything or taking any other medication.

==================================================

Question: A nurse, who is assigned for five days to a client who has exhibited manipulative behaviors, becomes aware of feeling reluctance to interact with the client. The nurse should take what action next?

A. Discuss the feelings of reluctance with an objective peer or supervisor within the next 24 hours
B. Develop a behavior modification plan for the client that will promote more functional behavior within the next week
C. Limit contacts with the client to avoid reinforcement of the manipulative behavior during the work times
D. Talk with the client about the negative effects of manipulative behaviors on other clients and staff within the next few days

Answer: A
The nurse who experiences stress in a therapeutic relationship can gain objectivity through discussion with other professionals. The nurse may wish to have a peer observe the nurse-client interactions with this client for a shift and then have a debriefing of positive and negative actions. The nurse must attempt to discover attitudes and feelings in the self that influence the nurse-client relationship in positive and negative ways.

==================================================

Question: A nurse is reviewing the nutritional needs for a child diagnosed with cystic fibrosis. The nurse should anticipate that this client would be deficient in which vitamins?

A. B12, D and K
B. A, D and K
C. A, C and D
D. A, B1 and C

Answer: B
The uptake of fat-soluble vitamins, A, D and K, is decreased in children with cystic fibrosis. Vitamin B12 is deficient in clients who have had bariatric surgery or various degrees of a gastrectomy. Vitamin B1 is often deficit in clients who have an alcohol addiction. These clients are given a thiamine (B1) injections daily times three to prevent Korsakoff syndrome. Vitamin D may be deficient in people who do not get at least 10 to 15 minutes of sunlight on the arms each day. Vitamin C deficit is associated with less than the needed intake of foods with vitamin C.

==================================================

Question: An older adult client, admitted after a fall at home, begins to seize and loses consciousness. What action by a nurse is appropriate to do next?

A. Stay with client and monitor the condition
B. Collect pillows and pad the side rails of the bed
C. Place an oral airway in the mouth and suction
D. Announce a cardiac arrest and plan to assist with intubation

Answer: A
For the client's safety, remain at the bedside and observe respirations, the movements of the extremities and level of consciousness. Prepare to clear the airway or suction if obstructed. If suction equipment is not at the bedside, request that someone else get it for you, rather than leaving the client. Do not place anything in the client's mouth. For safety, do not leave the client unattended. A cardiac arrest should only be announced if pulse or respirations are absent after the seizure.

==================================================

Question: In response to a call for assistance by a client in labor, the nurse notes that a loop of the umbilical cord is protruding from the vagina. What is the priority action?

A. Put the client into a knee-chest position
B. Apply oxygen by mask
C. Check for a fetal heart beat
D. Call the health care provider

Answer: A
Immediate action is needed to relieve pressure on the cord to prevent the risk of fetal hypoxia. A Trendelenburg or knee-chest position accomplishes this. The exposed cord should be covered with saline soaked gauze and not reinserted. The fetal heart rate should be checked rapidly, the health care provider should be called immediately and the client should be prepared for immediate vaginal or C-section birth. A prolapsed umbilical cord is a medical emergency, which can result in brain damage or death to the fetus if not treated promptly and properly.

==================================================

Question: A client is admitted directly from surgery in skeletal traction for a fractured femur. Which of these nursing interventions should be the priority?

A. Maintain proper body alignment
B. Apply an overhead trapeze to assist with movement in bed
C. Inspect the pin sites for evidence of drainage or inflammation
D. Perform frequent neurovascular assessments of the affected leg

Answer: D
The priority postoperative action is to assess the neurovascular status of the leg after a fracture. Nursing management of a client in skeletal traction also includes assessing and caring for pin sites, and educating the client and family about skeletal traction. The overhead trapeze helps the client move in bed and proper body alignment is important, but these are not the priority.

==================================================

Question: If a client is stated to have a dual diagnosis. The nurse should understand that this indicates a substance abuse problem as well as what other type of problem?

A. Medical problem
B. Mental disorder
C. Disorder of any type
D. Cross addiction

Answer: B
A dual diagnosis is the concurrent presence of a major psychiatric disorder and chemical dependence.

==================================================

Question: The nurse notes cloudy drainage two days post-insertion of an abdominal catheter for peritoneal dialysis. What other data does the nurse need to collect before reporting this finding to the provider?

A. Breath sounds
B. Bowel sounds
C. Temperature
D. Urine output

Answer: C
Cloudy drainage may indicate a peritoneal infection, so it is essential to evaluate the client's temperature before notifying the health care provider. In a client on dialysis for renal failure little to no urine output would be an expected finding.

==================================================

Question: A nurse is educating parents on accidental poisoning in children. Which type of accidental poisoning is expected to occur in children under age six?

A. Topical contact
B. Oral ingestion
C. Inhalation
D. Eye splashes

Answer: B
The greatest risk for young children is from oral ingestion. While children under age six may come in contact with other poisons or inhale toxic fumes, these are not as common.

==================================================

Question: A 2-year-old child has just been diagnosed with cystic fibrosis. The child's parent asks the nurse what the most important concerns are at this time. Which is the appropriate response from the nurse?

A. "Thick, sticky secretions from the lungs are a constant challenge."
B. "Cystic fibrosis results in nutritional concerns that can be dealt with."
C. "You will work with a team of experts and have access to a support group."
D. "There is a high probability of life-long complications."

Answer: A
The primary factor, and the one responsible for many of the clinical manifestations of cystic fibrosis, is mechanical obstruction caused by the increased viscosity of mucous gland secretions.Because of the increased viscosity of bronchial mucus, there is greater resistance to ciliary action (probably secondary to infection and ciliary destruction), a slower flow rate of mucus and incomplete expectoration, which also contributes to the mucus obstruction. This retained mucus serves as an excellent medium for bacterial growth. Reduced oxygen-carbon dioxide exchange causes variable degrees of hypoxia, hypercapnia and acidosis.In severe cases, progressive lung involvement, compression of pulmonary blood vessels and progressive lung dysfunction frequently lead to pulmonary hypertension, cor pulmonale, respiratory failure and death. Pulmonary complications are present in almost all children with cystic fibrosis, but the onset and extent of involvement are variable.

==================================================

Question: A nursing assistant is taking care of a 2 year-old child with Wilm's tumor. The assistant asks the nurse why there is a sign above the bed that says DO NOT PALPATE THE ABDOMEN. Which statement by the nurse would be the best response?

A. "Touching the abdomen could cause cancer cells to spread."
B. "Pushing on the stomach might contribute to a bowel obstruction."
C. "Examining the area would be painful."
D. "Placing any pressure on the abdomen may cause the tumor to rupture."

Answer: A
Manipulation of the abdomen can lead to dissemination of cancer cells to nearby and distant areas. Bathing and turning the child should be done carefully.

==================================================

Question: The nurse manager identifies that time spent charting is excessive. The nurse manager states that "staff will form a task force to investigate and develop potential solutions to the problem and then report on this at the next staff meeting." What is the nurse manager's leadership style?

A. Affiliative
B. Autocratic
C. Transformational
D. Dynamic

Answer: C
A transformational style of management involves staff members in the decision-making processes. Staff members review current policies and provide feedback to their leader in the pursuit of the common good.

==================================================

Question: A nurse is caring for a 4 year-old two hours after a tonsillectomy and adenoidectomy. Which of these assessments must be reported immediately?

A. Complaints of throat pain
B. Apical heart rate of 110
C. Increased restlessness
D. Vomiting of dark emesis

Answer: C
Increased restlessness with increased respiratory and heart rates are often early signs of active bleeding. The other options are expected findings at this time in the postop period for this surgery. The dark emesis indicates old blood that most likely was swallowed during surgery.

==================================================

Question: The client had an open reduction and internal fixation (ORIF) of a femur fracture. During a routine assessment 36 hours after surgery, the nurse finds the client disoriented, short of breath and warm to the touch. The client's temperature is 102.4 F (39 C). What assessment should the nurse perform next?

A. Measure oxygen saturation using a pulse oximeter
B. Assess orientation to time, person and place
C. Remove the splint and inspect the incision
D. Perform a neurologic check of bilateral distal extremities

Answer: A
Based on the client's history and assessment findings, the nurse should suspect fat embolism syndrome (FES). Neurologic changes and respiratory distress are two of the classic findings of FES (the third finding is a characteristic petechial rash.) The nurse should activate the rapid response team. While waiting for the team, the nurse will measure the client's SpO2, as well as pulse and blood pressure, and auscultate the lungs. The nurse will also administer supplemental oxygen and ensure venous access.

==================================================

Question: A hospitalized child has a seizure while the family is visiting. The nurse notes the child's whole body is rigid, followed generalized jerking movements of the extremities. The child vomits immediately after the seizure. What is a priority nursing diagnosis for the child at this time?

A. Risk for airway obstruction related to aspiration
B. Fluid volume deficit related to vomiting
C. Risk for infection related to vomiting
D. Altered family processes related to chronic illness and hospitalization

Answer: A
The tonic-clonic seizure appears suddenly and often leads to brief loss of consciousness. The greatest risk for this child is from airway obstruction due to aspiration of the vomit.

==================================================

Question: A nurse is caring for a client with left ventricular heart failure with an ejection fraction (EF) of 40%. Which assessment finding is an early indication of inadequate tissue perfusion?

A. Use of accessory muscles
B. Crackles in the lungs
C. Distended jugular veins
D. Confusion and restlessness

Answer: D
Neurological changes, including impaired mental status, are early signs of inadequate tissue perfusion and decreased oxygenation of the brain tissues. Other signs of low EF are shortness of breath, dependent edema and arrhythmias. The low EF indicates that this client has severe damage to the left ventricle (normal EF is about 55-70%).

==================================================

Question: A nurse is caring for a client who is receiving procainamide intravenously. It is important for the nurse to monitor which of these parameters?

A. Serum potassium levels
B. Hourly urinary output
C. Continuous ECG readings
D. Neurological signs

Answer: C
Procainamide is used to suppress cardiac arrhythmias. When administered intravenously, it must be accompanied by continuous cardiac monitoring.

==================================================

Question: A Hispanic couple confide in the nurse about their concern with staff giving their newborn the "evil eye." What should the nurse communicate to the other personnel who are involved in the care of this family?

A. Avoid touching the infant above the waist
B. Talk very slowly while speaking to him
C. Look only at the parents and not the newborn
D. Touch the baby after looking at him

Answer: D
In many cultures, an "evil eye" is cast when looking at a person without touching. Thus, the spell is broken by touching while looking or assessing. Remember that quotations in the stem of the question are often the most important content in the question (evil eye). You should make the association between the words "looking" and "seeing"(eye). Also note that the answer needs to refer to the newborn, not the parents ("give the newborn the evil eye"). To only look at the parents is an unrealistic approach.

==================================================

Question: The client returns from the post anesthesia care unit (PACU) in stable condition following abdominal surgery. While planning immediate postoperative care, the nurse identifies the nursing diagnoses listed below. Prioritize these diagnoses by placing them in order of importance (with 1 being the most important).

A. Impaired mobility related to invasive equipment
B. Acute pain related to surgical procedure
C. Risk for ineffective airway clearance related to anesthesia
D. Risk for imbalanced nutrition: less than body requirements related to NPO satus

Answer: C,B,A,D
Airway is the highest priority, especially in the immediate postoperative period. Pain control is the next priority because this client will most likely experience significant pain. Although impaired mobility is expected, it does increase the client's risk for postoperative complications. The client's risk for nutrition imbalance is the lowest priority and is to be expected for a client who has had abdominal surgery; hydration is provided intravenously.

==================================================

Question: The nurse who is caring for clients over the age of 70, implements a teaching plan about diet. Using knowledge based on age-related changes, the nurse will emphasize which of the following factors?

A. Add high protein supplements to your diet
B. Make at least half your grains whole grain
C. Follow the DASH eating plan
D. Look for foods fortified with iron and other minerals

Answer: B
Anyone, regardless of age, should eat a balanced diet of nutrient-packed foods. However, the diet of the older adult without other chronic health issues should include an increase of fiber and whole grains. The DASH diet is recommended to reduce blood pressure, but there is nothing to indicate this client is hypertensive. Older adults should eat lean proteins but don't necessarily need protein supplements. They should also look for foods fortified with vitamins B12 and D, as well as calcium.

==================================================

Question: A newborn born prematurely is to be fed breast milk through a nasogastric tube. Why is breast milk preferred over formula for premature infants?

A. Is higher in calories/ounce
B. Contains less lactose
C. Provides antibodies
D. Has less fatty acids

Answer: C
Breast milk is ideal for the preterm baby who needs additional protection against infection through maternal antibodies. It is also much easier to digest. Therefore, less residual is left in the infant's stomach.

==================================================

Question: A client was recently released from a locked psychiatric facility. During a scheduled outpatient follow up appointment, the client states to the nurse, "I'm afraid I am going to get sick again." Which of the following responses by the nurse is a priority in preventing relapse?

A. "I will provide you with a bus pass and referral to a support group that will help you learn about managing your illness and medications."
B. "If you take your medications exactly as your health care provider instructed, you won't get sick again."
C. "I think you are doing well but you can call for an appointment with your health care provider if you think you need help."
D. "You shouldn't fear a relapse because it can happen to anyone and we will be here to help you."

Answer: A
Relapse prevention is a priority focus for clients recovering from an acute mental illness episode. Since education plus peer and community support rank high in helping prevent relapse, the priority is to refer the client to after-care and support groups. Additionally, since continuity of care involves access to care, the nurse should address the client's transportation needs by offering him a bus pass so he can attend these meetings. Continuing to take medications is important, but advice and reassurance without tangible follow up is not helpful to clients in early recovery from an acute event. Reassurance and referral to a health care provider may also be inadequate and does not demonstrate the nurse's concrete role in relapse prevention. Telling the client not to fear relapse and providing false reassurance is non-therapeutic.

==================================================

Question: A client being treated for hypertension returns to the community clinic for a follow-up. The client says, "I know these pills are important, but I just can't take these water pills anymore. I drive a truck for a living, and I can't be stopping every 20 minutes to go to the bathroom." Which nursing diagnosis should the nurse select for this client?

A. Defensive coping related to chronic illness
B. Knowledge deficit related to misunderstanding of disease state
C. Altered health maintenance related to occupation
D. Noncompliance related to medication side effects

Answer: D
The client kept the appointment and stated knowledge that the pills were important. The client is unable to comply with the regimen due to side effects, not because of a lack of knowledge about the disease process.

==================================================

Question: A nurse is caring for a client suspected to have a diagnosis of active tuberculosis (TB). Which diagnostic tests is essential for the nurse to obtain for the determination of the presence of active TB?

A. Sputum culture for cytology
B. Tuberculin skin testing
C. White blood cell count
D. Chest x-ray anterior/posterior and lateral

Answer: A
The sputum culture is the method for the determination if active TB is present. This test takes one to two weeks to get the results. Thus, these clients would need to be in isolation or on medication and not coughing during the wait.

==================================================

Question: The client is taking bupropion to treat depression and is worried about taking the medication. The client tells the nurse a friend said the medication was removed from the market because it caused seizures. What is an appropriate response by the nurse?

A. "Omit the next doses until you talk with the health care provider."
B. "Your health care provider knows the best drug for your condition."
C. "Ask your friend about the source of this information."
D. "There were problems, but the recommended dose is changed."

Answer: D
Bupropion (Budeprion, Buproban, Wellbutrin, Zyban) was introduced in the United States in 1985 and then withdrawn because of the occurrence of seizures in some clients who took the drug. The drug was reintroduced in 1989 with specific recommendations about dose ranges to limit the occurrence of seizures. The risk of seizure appears to be strongly associated with higher dosages.

==================================================

Question: The nurse is caring for a 17 month-old child diagnosed with acetaminophen poisoning. Which of these lab reports should the nurse review first?

A. Aspartate aminotransferase (AST) and Alanine transaminase (ALT)
B. Prothrombin Time (PT) and partial thromboplastin time (PTT)
C. Red blood cell and white blood cell counts
D. Blood urea nitrogen (BUN) and creatinine clearance

Answer: A
Acetaminophen is toxic to the liver and causes hepatic cellular necrosis. This causes the liver enzymes AST and ALT to be released into the blood stream, which elevates serum levels. The next lab values to review are those associated with coagulation, then the blood counts and lastly the renal-associated labs, including BUN and creatinine.

==================================================

Question: The client undergoes a laparoscopic removal of the appendix. Which postoperative instructions will the nurse reinforce? (Select all that apply.)

A. No showering for 48 hours after surgery
B. Maintain bedrest for 24 hours before gradually resuming regular activities
C. Some shoulder discomfort can be expected
D. Use 2 tablespoons of Milk of E. Magnesia if no bowel movement 3 days after surgery
E. Restrict diet to bland, easily digestible food for a few days
F. Gently scrub off the "skin glue" when you feel able

Answer: A,C,D,E
Laparoscopic surgery involves using carbon dioxide gas to open the inside of the abdomen, which pushes up the diaphragm; this may cause shoulder discomfort postoperatively. Clients should keep the dressings clean and dry for 48 hours before they can shower, but no tub baths for a few weeks. If "skin glue" is used over the incision(s), the client should not try to scrub it off because it will wear off on its own. Clients may resume normal activities as soon as they are able but no heavy lifting or aerobic exercise for about 2 weeks. If they do not have a BM after 2-3 days, clients can take 2 tablespoons of MOM several times a day until they have a BM. Diet can be advanced as tolerated but it's best to stick to non-greasy, non-spicy foods for a few days.

==================================================

Question: The home health nurse observes the client change an ileostomy pouch. Which action is best to help prevent skin breakdown?

A. Use deodorant soaps the contain lotion to clean the stoma
B. Change the stoma pouch daily
C. Apply antiseptic cream to reddened stoma
D. Make sure the skin around the stoma is wrinkle-free

Answer: D
The ileostomy pouch should be changed approximately every 5 to 7 days; the bag should be emptied about every 4 to 6 hours. Before applying a pouch, the stoma and skin around the stoma should be gently cleaned using mild soap and water and allowed to dry. A skin barrier powder or other skin prep can be applied to intact skin around the stoma - but not to the stoma. The skin around the stoma should be dry and wrinkle-free before applying a new pouch or wafer to ensure a tight, leak-free seal.

==================================================

Question: A nurse is caring for a client who is receiving a blood transfusion and develops urticaria one-half hour after the transfusion has begun. What is the first action the nurse should take?

A. Slow the rate of infusion
B. Stop the infusion
C. Take vital signs and observe for further deterioration
D. Administer Benadryl and continue the infusion

Answer: B
This is an indication of an allergy to the plasma protein. The priority action of the nurse is to stop the transfusion by disconnecting at the IV insertion site. The nurse should then start a saline line at the IV insertion site and notify the health care provider.

==================================================

Question: A client has end-stage renal disease. Which of these statements made by the client indicates a correct understanding of the issues related to this disease?

A. "I can expect to have periods of little urine and then sometimes a lot of urine."
B. "I have to go for epoetin (Procrit) injections at the health department."
C. "I know I have a high risk of clot formation since my blood is thick from too many red cells."
D. "My bones will be stronger with this disease since I will have higher calcium than normal."

Answer: B
Anemia in end-stage renal failure is caused by reduced endogenous erythropoietin production in the kidney. Anemia in primary end-stage renal disease is treated with subcutaneous injections of Procrit or Epogen to stimulate the bone marrow to produce red blood cells. With kidney failure, too much phosphorus can build up in the blood and calcium is pulled from the bones, resulting in weakened bones. The statement about producing variable amounts of urine is incorrect, as the client will produce little to no urine at this stage of the disease.

==================================================

Question: The ICU nurse works in a rural hospital that has a remote electronic ICU monitoring system (eICU.) What is one of the best reasons for having access to an eICU?

A. An ICU nurse and intensivist remotely monitor ICU clients around the clock
B. An ICU nurse is on-call to answer questions when needed
C. Clients can ask the intensivist for a second opinion
D. Less staff is needed on site when a remote eICU is available

Answer: A
Using cameras, microphones, and high-speed computer data lines, the eICU involves having an experienced ICU nurse and practicing intensivist monitoring ICU clients in remote locations around the clock. The eICU does not change the ratio of nurses to clients at the bedside, but it does make the nurse's bedside time more productive and assistance from their remote colleagues is only a push button away.

==================================================

Question: The client is diagnosed with tuberculosis (TB). The nurse understands that the treatment plan for this client will involve what type of drug therapy?

A. Administering two antituberculosis drugs
B. Aminoglycoside antibiotics
C. An anti-inflammatory agent
D. High doses of B complex vitamins

Answer: A
In order to prevent drug-resistant strains of TB, clients are always prescribed at least two different antitubercule medications. Rifampin and isoniazid are the most effective drugs used to treat TB and are always used together, for at least six months. Additional medications, such as pyrazinamide and either streptomycin or ethambutol, may also be prescribed. Vitamin B6 is usually prescribed to help prevent expected side effect of isoniazid.

==================================================

Question: While working a 12-hour night shift, the nurse has a "near miss" and catches an error before administering a new medication to the client. Which factors could have contributed to the near miss? (Select all that apply.)

A. The nurse works in the intensive care unit (ICU)
B. The nurse has worked on the same unit for five years
C. The nurse is assigned more clients than usual due to staffing issues
D. The nurse was interrupted when preparing the medication
E. The nurse has worked four 12-hour night shifts in a row

Answer: A,C,D,E
There are a number of reasons for near misses and making medication errors, including heavy workload and inadequate staffing, distractions, interruptions and inexperience. Fatigue and sleep loss are also factors, especially for nurses working in units with high acuity clients such as the ICU.

==================================================

Question: A 4 month-old infant is being given digoxin. The client's blood pressure is 92/78 mm Hg; resting pulse is 78 BPM; respirations are 28 BPM; and the serum potassium level is 4.8 mEq/L (4.8 mmol/L). The client is irritable and has vomited twice since the morning dose of digoxin. Which finding is most indicative of digoxin toxicity?

A. Irritability
B. Vomiting
C. Bradycardia
D. Dyspnea

Answer: C
The most common sign of digoxin toxicity in children is bradycardia which is a heart rate below 100 BPM in an infant. Normal resting heart rate for infants 1-11 months-old is 100-160 BPM.

==================================================

Question: A client is receiving total parenteral nutrition (TPN) via a tunneled catheter. The catheter accidentally becomes dislodged from the site. Which action by the nurse should take priority?

A. Monitor respiratory status
B. Apply a pressure dressing to the site
C. Assess for mental status changes
D. Check that the catheter tip is intact

Answer: B
The client is at risk of bleeding or developing an air embolus if the catheter exit site is not covered with a pressure and occlusive dressing. An occlusive dressing is one that is totally covered by adhesive tape around the edges, as well as over the entire dressing.

==================================================

Question: The oncology client reports pain, and the provider orders hydromorphone IM 0.015 mg/kg right away. How many milligrams does the nurse administer? The nurse checks the chart and determines the client weighs 119 pounds.
How many milligrams of hydromorphone (Dilaudid, Exalgo) will the nurse administer? (Report your answer to one decimal point and write only the number.)

Answer: 0.8
Using dimensional analysis, the final units will be milligrams, so begin the equation with milligrams on top, then multiply to cancel unwanted units until only the milligrams remain.(0.015 mg/kg) X (1 kg/2.2 lbs) X (119 lb/1) = 1.79/2.2 = 0.82 = 0.8

==================================================

Question: The nurse is caring for a client admitted with a diagnosis of Meniere's disease. When teaching the client about the disease, the nurse should explain that the client should avoid foods high in which substance?

A. Fiber
B. Carbohydrates
C. Calcium
D. Sodium

Answer: D
The client with Meniere's disease has an alteration in the balance of the fluid in the inner ear (endolymph). A low-sodium diet will aid in reduction of the fluid. Sodium restriction is commonly ordered as adjunct to diuretic therapy in the acute and chronic treatment.

==================================================

Question: The nurse is assessing a 4 year-old child who is in skeletal traction 24 hours after surgical repair of a fractured femur. The child is crying and reports having severe pain. The right foot is pale and there is no palpable pulse. What action should the nurse take first?

A. Notify the health care provider
B. Administer the ordered PRN medication
C. Reassess the extremity in 15 minutes
D. Readjust the traction for comfort

Answer: A
Pain and absence of a pulse within 48-72 hours after a severe injury to an extremity suggests acute compartment syndrome. This condition occurs when there's a build up of pressure within the muscles; this pressure decreases blood flow and can cause muscle and nerve damage. Acute compartment syndrome is a medical emergency. Surgery is needed immediately; delaying surgery can lead to permanent damage to the extremity.

==================================================

Question: An 80 year-old client diagnosed with pneumonia is exhibiting new onset confusion. The client is pulling at tubes and items near the bed and trying to get out of bed. Which intervention would be most appropriate?

A. Request an order for restraints
B. Frequently remind the client to stay in bed
C. Request an order for antianxiety medication
D. Arrange for a sitter to stay with the client

Answer: D
Clients treated for pneumonia often develop new cognitive impairments; confusion or delirium is common. Although no one wants someone to fall out of bed or pull out tubes, restraints should always be used as a last resort. A less restrictive approach would be to arrange for a sitter to stay with the client. Use of antianxiety medications, such as benzodiazepines, should be avoided in the elderly because they increase the risk of cognitive impairment, delirium and falls.

==================================================

Question: A woman in early labor puts her call light on and tells the nurse "I think my water bag just broke and I feel like something came out with the water." A visual exam by the nurse reveals a prolapsed umbilical cord. List in order of priority the actions the nurse should perform in this obstetrical emergency.

A. Glove and place two fingers into the cervical opening, beside the umbilical cord, to relieve pressure
B. Administer oxygen to the mother via mask at 10 L/min
C. Call for assistance, asking that the health care provider is notified
D. Place the client in a knee-chest position on the bed

Answer: A,C,D,B
A prolapsed cord is a medical emergency; the blood flow from the placenta to the fetus will be occluded with each contraction if the umbilical cord is compressed against the presenting part of the fetus and the dilated cervix which is why the priority intervention is to apply gloves and place two fingers to one side of the cord (or entire hand) to relieve pressure. The nurse is also calling for assistance so that someone can notify the health care provider and staff can prepare for emergent cesarean. Placing the client in a modified Sims or knee-chest position will allow gravity to help decrease pressure on the cord from the presenting part, but the primary relief from pressure on the umbilical cord is the gloved fingers. Oxygen administration will help once the circulation of blood to the fetus is re-established.

==================================================

Question: A client tells a nurse: "I have decided to stop taking sertraline (Zoloft) because I don't like the nightmares, sex dreams and obsessions I have had since starting on the medication." What is an appropriate response by the nurse?

A. "Side effects and benefits should be discussed with your health care provider."
B. "Many medications have potential side effects."
C. "This medication should be continued despite unpleasant symptoms."
D. "It is unsafe to abruptly stop taking any prescribed medication."

Answer: D
Abrupt withdrawal the short-acting SSRI sertraline (Zoloft) causes SSRI Discontinuation Syndrome. A slow tapering of the medication will be prescribed to avoid the symptoms associated with this syndrome, which may include insomnia, headache, dry mouth, nausea and diarrhea.

==================================================

Question: A 57 year-old male client has a hemoglobin of 10 g/dL (6.21 mmol/L) and a hematocrit of 32% (0.32). What would be the most appropriate follow-up by a home care nurse?

A. Ask the client if the client has noticed any bleeding or dark stools
B. Call 911 and send the client to the emergency department
C. Refer the client to schedule an appointment with a hematologist
D. Schedule a repeat hemoglobin and hematocrit in one month

Answer: A
Normal hemoglobin for males is 14 - 18 g/dL (8.69 - 11.17 mmol/L). Normal hematocrit for males is 42 - 52% (0.42-0.52). The lab values for this client are below normal and indicate mild anemia. The nurse should ask if the client has noticed any bleeding or change in stools that could indicate bleeding from the GI tract.

==================================================

Question: A nurse is providing information to a client who is newly diagnosed with tuberculosis (TB). The nurse should be sure to include which statement when teaching the client about managing this disease?

A. "Isolate yourself from others until you are finished taking your medication."
B. "Follow up with your primary care provider in three months."
C. "Continue to take your medications even when you are feeling fine."
D. "Continue to get yearly tuberculin skin tests."

Answer: C
The client with TB needs is to understand the importance of medication compliance, even when the client is no longer having any symptoms. TB treatment usually requires a combination of medications with treatment for at least six months. Stopping treatment or skipping doses can lead to a drug-resistant form of TB. Clients are most infectious early in the course of therapy but the numbers of acid-fast bacilli are greatly reduced as soon as two weeks after therapy begins. Once clients no longer have a productive cough, they are not considered contagious.

==================================================

Question: While assessing the vital signs in children, the nurse should know that the apical heart rate is preferred until the radial pulse can be accurately assessed at about what age?

A. Four years
B. Three years
C. One year
D. Two years

Answer: D
A child should be at least 2 years old to use the radial pulse to assess heart rate.

==================================================

Question: A parent asks the nurse about a Guthrie Bacterial Inhibition test that was ordered for her newborn. Which of the following points should the nurse discuss with the client prior to this test? (Select all that apply.)

A. The test will be delayed if the baby's weight is less than 5 pounds
B. Positive tests require dietary control for prevention of brain damage
C. This test identifies an inherited disease
D. The urine test can be done after six weeks of age
E. Best results occur after the baby has been breast-feeding or drinking formula for two full days
F. Routine screening of newborn infants is not mandatory in the United States

Answer: A,B,C,D,E
Screening for PKU is mandated in all 50 states, though methods of screening vary. The Guthrie Bacterial Inhibition Assay (BIA) is one test used to diagnose phenylketonuria (PKU), a disease characterized by an enzyme deficiency. A blood sample is taken from the baby's heel shortly after birth, with a follow-up test 7 to 10 days later. Test results are more accurate if the baby weighs more than 5 pounds and has been regularly drinking milk for more than 24 hours. A urine test is normally done after six weeks of age if a baby did not have the blood test.

==================================================

Question: A nurse is assigned to care for a client who has been diagnosed with an intracranial aneurysm that has now stopped leaking. To minimize the risk of another bleeding episode, or rupture, the nurse should plan to take which of these actions?

A. Keep the client in a upright sitting position
B. Treat any elevation in blood pressure
C. Apply a warming blanket for temperatures of 98 F (36.6 C) or less
D. Avoid arousal of the client except for family visits

Answer: B
Treating any blood pressure elevation and reducing stress by maintaining a quiet environment, including during family visits, will assist in minimizing the risk of a cerebral bleed. An upright sitting position with the pressure on the hip area can lead to increased intracranial pressure; this position should be avoided. A warming blanket is inappropriate to use.

==================================================

Question: The nurse is assessing a client in the emergency department. Which statement suggests that the client is experiencing acute cardiac ischemia?

A. "I've got a pressure deep in my chest behind my breast bone."
B."As I take a deep breath the pain gets worse."
C. "When I sit up the pain gets worse."
D. "The pain is right here in my stomach area."

Answer: A
Pain that gets worse with deep breaths may be related to a disorder of the lungs. Pain that gets worse with movement is probably from the muscles or bones in the chest, but is not cardiac ischemia. Pain that worsens in the supine position and is relieved with sitting up is characteristic of pericarditis. Although pain in the stomach, especially after a meal, may actually be angina, a person most typically will feel pain, aching or pressure in the middle of the chest, just beneath the sternum. Many people describe the sensation as discomfort or heaviness instead of pain, so the term discomfort should be used when asking clients about their findings.

==================================================

Question: client is scheduled to have a blood test for cholesterol and triglycerides the next day. What statement should the nurse include in the directions for the client?

A. "Be sure and eat a fat-free diet until the test."
B. "Do not eat or drink anything but water for 12 hours before the test."
C. "Stay at the laboratory so two blood samples can be drawn an hour apart."
D. "Have the blood drawn within two hours of eating breakfast."

Answer: B
Serum lipid levels should be obtained from clients who have been fasting for at least 12 hours.

==================================================

Question: A 3 year-old child has findings that may suggest a neuroblastoma. While listening to the concerns of the parents, which finding is consistent with this diagnosis and requires follow-up by the health care provider?

A. "He seems to be getting weaker and weaker and is sometimes unsteady on his feet."
B. "We keep having to buy him larger size pants because he's growing so big around the waist."
C. "He doesn't seem to be going to the bathroom as much and his urine is dark yellow in color."
D. "Our child has been quieter than normal lately and has lost weight."

Answer: B
One of the most common signs of neuroblastoma is increased abdominal girth due to the mass or tumor in the abdomen. The mass can cause pain and/or a feeling of fullness and the pressure may affect the child's bladder or bowel. Although the child with a neuroblastoma may not want to eat (which can lead to weight loss), this finding could have many causes. A more significant finding would be if the parents reported that child keeps outgrowing clothing or that clothing is tight around the abdomen.

==================================================

Question: After placement of a ventriculoperitoneal (VP) shunt as a treatment for hydrocephalus of their infant, the parents ask a nurse: "Why is there a small incision in the abdomen?" Which response would be the best for explaining the purpose of the incision?

A. "That's what is used for insertion of the catheter into the stomach."
B. "It's used to pass the catheter into the abdominal cavity."
C. "It's used to visualize the abdominal organs for correct catheter placement."
D. "It's there so the tubing can be inserted into the urinary bladder."

Answer: B
The preferred procedure in the surgical treatment of hydrocephalus is the placement of a ventriculoperitoneal shunt. This shunt procedure provides primary drainage of the cerebrospinal fluid from the ventricles to an extracranial compartment, which is commonly the peritoneum. A small incision is made in the upper quadrant of the abdomen so the shunt tip can be guided into the peritoneal cavity.

==================================================

Question: The nurse is assessing a 1 month-old infant. Which finding should the nurse report immediately?

A. Irregular breathing rate
B. Increased heart rate with crying
C. Inspiratory grunt
D. Abdominal respirations

Answer: C
Inspiratory grunt is an abnormal finding and indicates respiratory distress in infants. Other signs of respiratory distress in this age group are nasal flaring, often the initial finding, as well as sternal and intracostal retractions. Abdominal breathing is a normal expected breathing process for infants. The other findings are also normal in infants.

==================================================

Question: The nurse is teaching the parents of a child with sickle cell disease about ways to prevent complications and crises. What information would be a priority for the nurse to emphasize to the family?

A. The child may not be able to follow routine immunization schedules
B. The child should avoid becoming overheated or dehydrated during physical activity and exercise
C. The child can maintain normal activity with some restrictions
D. The child should be cautious of being exposed to people with a cold or fever

Answer: B
The goal of sickle cell treatment is to manage and control symptoms and to prevent sickle cell crisis. Fluid loss caused by overheating and dehydration can trigger a sickle cell crisis. People with sickle cell anemia need to keep their immunizations up-to-date, treat infections quickly, and avoid too much sun exposure.

==================================================

Question: A 10-month old infant is admitted with a diagnosis of bacterial meningitis. Several hours after admission, during a planning conference, which of the actions suggested to the registered nurse (RN) by the practical nurse (PN) would be appropriate to add to the plan of care?

A. Provide an over-the-crib protective top
B. Measure head circumference
C. Initiate droplet precautions
D. Provide passive range of motion

Answer: B
In meningitis, assessment of neurological signs should be done frequently. Head circumference is measured because subdural effusions and obstructive hydrocephalus can develop as a complication of meningitis. The client would have already been placed on droplet precautions and had a crib top applied to the bed when he was admitted to the unit.

==================================================

Question: A 52 year-old postmenopausal woman asks the nurse how frequently she should have a mammogram. How should the nurse respond?

A. "Unless you had previous problems, every two years is best."
B. "Your health care provider will advise you about your risks and the frequency."
C. "Yearly mammograms are advised for any women over 35."
D. "Once a woman reaches 50, she should have a mammogram yearly."

Answer: D
The American Cancer Society recommends a screening mammogram by age 40, every one to two years for women 40 to 49, and every year from age 50 onward. If there are family or personal health risks, other more frequent and additional assessments may be recommended.

==================================================

Question: A child is injured on the school playground and appears to have a fractured leg. Which of the following is the first action a school nurse should take?

A. Call for emergency transport to the hospital
B. Assess the child and the extent of the injury
C. Immobilize the limb and joints above and below the injury
D. Apply cold compresses to the injured area

Answer: B
Application of the nursing process dictates that assessment is the first step in the provision of care. The 6 Ps of vascular impairment (pain, pulse, pallor, paresthesia, paralysis and poikilothermia (coolness) can be used as a guide for assessment of the injured leg. The other options would be done in this sequence - immobilize, call 911 and then apply ice as indicated.

==================================================

Question: A client with considerable pain asks a nurse, "What is your opinion regarding acupuncture as a drug-free method for alleviating pain?" The nurse responds, "I'd forget about it; those weird non-Western treatments can be scary." The nurse's response is an example of what perspective?

A. Ethnocentrism
B. Discrimination
C. Prejudice
D. Cultural insensitivity

Answer: A
Ethnocentrism is the universal unconscious tendency of human beings to think that their ways of thinking, acting, and believing are the only right, proper and natural ways. It can be a major barrier to the provision of culturally conscious care. Ethnocentrism perpetuates an attitude that beliefs that differ greatly from one's own are strange, bizarre or unenlightened, and therefore wrong. At a more complex level, ethnocentric people regard others as inferior or immoral and believe their own ideas are intrinsically good, right, necessary, and desirable, while remaining unaware of their own value judgments.

==================================================

Question: The nurse checks lab results for an adult client with suspected cancer prior to a liver biopsy. Which finding requires immediate notification of the health care provider?

A. Elevated blood urea nitrogen (BUN) and creatinine
B. Hemoglobin of 11 g/dL (110 g/L)
C. Increased serum ammonia
D. Activated partial thromboplastin time (aPTT) of 50 seconds

Answer: D
Because the liver is a vascular organ and a biopsy is an invasive procedure, bleeding is one of the risks. An elevated aPTT increases the risk of bleeding. Abnormal findings in the other labs would not increase the client's risk of complications following a liver biopsy.

==================================================

Question: The nurse observes a coworker removing a narcotic from the electronic medication dispensing machine and self-medicating. Which action is required for the nurse who observes this behavior?

A. Report the coworker to the nursing supervisor
B. Ask other staff members if they have observed the coworker diverting drugs
C. Schedule an intervention to confront the coworker
D. Encourage the coworker to talk to someone about getting help

Answer: A
Nurses who divert drugs pose a threat to client safety and are a legal liability for the facility, which is why the behavior must first be reported to the unit manage or other nursing supervisor. The nurse practice act in some states also mandate reporting unsafe nursing practice to the board of nursing. Drug diversion is often a symptom of substance use disorder but the coworker may not necessarily be an addict. An intervention may be scheduled after specific examples of destructive behaviors are collected (from other staff and medication audits) and the coworker is confronted with the evidence.

==================================================

Question: The client returned from the cardiac catheterization lab four hours ago. The groin was used as the insertion site. Which of the assessment findings would the nurse immediately report to the health care provider? (Select all that apply.)

A. Capillary refill 6 seconds on the affected toes
B. Pale color of the affected limb
C. Trace amount of serosanguineous drainage on the groin dressing
D. Bruising or lump at the insertion site
E. Nonpalpable pedal pulse on the affected limb

Answer: A,B,E
A trace of serosanguineous drainage on the dressing is common. Some bruising or a small lump is expected at the insertion site. Reportable conditions include significant reports of pain; abnormal lab values; abnormal ECG strip; post-procedure bleeding or swelling; color, temperature or pulse changes, especially to the affected limb. Capillary refill should be about 3 seconds.

==================================================

Question: A client who had surgery is discharged on warfarin. Which statement by the client is incorrect and indicates a need for further teaching?

A. "I will report any bruises or unusual bleeding."
B. "I know I must avoid crowds."
C. "I plan on using an electric razor for shaving."
D. "I will keep all laboratory appointments."

Answer: B
There are no specific reasons for the client on warfarin to avoid crowds. Clients should not use a straight edge razor, should report any unusual bleeding and must keep all laboratory appointments when taking the blood thinner warfarin.

==================================================

Question: The registered nurse is teaching a childbirth education class about postpartum depression. Which statement, made by a class member, indicates that more teaching is needed?

A. "I will make an effort to talk with someone about my feelings if I start to feel overwhelmed."
B. "It's common for women with postpartum depression to have delusions about the infant."
C. "Women with postpartum depression have feelings of guilt and worthlessness."
D. "I may experience postpartum depression up to a year after delivery."

Answer: B
Postpartum depression symptoms include sleep and appetite disturbances, uncontrolled crying, with feelings of guilt and/or worthlessness. Although postpartum depression typically occurs within the first three months after delivery, it can occur up to a year later. A new mother who has symptoms of postpartum depression should take steps to get help right away. Delusions are associated with postpartum psychosis, not depression.

==================================================

Question: A nurse is giving instructions to the parents of a newborn infant with oral candidiasis. Which statement made by a parent is incorrect and indicates a need for more teaching?

A. "The therapy can be discontinued when the spots disappear."
B. "I will boil the nipples and pacifiers for 20 minutes."
C. "I will use a dropper to place the medicine on each side of my baby's mouth."
D. "Nystatin should be given four times a day after my baby eats."

Answer: A
The therapy should be continued for a week, even if lesions have disappeared within a few days. If the mother is breast-feeding, mother and baby should be treated at the same time to prevent re-infection.

==================================================

Question: An anxious parent of a 4 year-old consults the nurse for guidance on how to answer the child's question: "Where do babies come from?" What is the nurse's best response to the parent?

A. "Children ask many questions, but are not looking for answers."
B. "Full and detailed answers should be given to any questions."
C. "When a child asks a question, give a simple answer."
D. "This question indicates interest in sex beyond this age."

Answer: C
During discussions related to sexuality, honesty is very important. However, honesty does not mean imparting every fact of life associated with the question. When children ask one question, they are looking for one answer. When they are ready, they will ask for more detailed information.

==================================================

Question: The nurse auscultates bibasilar inspiratory crackles in a newly admitted 68 year-old client with systolic heart failure and an ejection fraction of 30%. Which other finding is most common with this diagnosis?

A. Fatigue
B. Nail clubbing
C. Peripheral edema
D. Chest pain

Answer: A
Systolic heart failure is the result of a pumping problem, which is why the ejection fraction is reduced (normal is 60%). Heart failure can be caused by a heart attack, but chest pain is not normally a finding in heart failure. Nail clubbing is usually associated with disorders of the lungs. Exertional dyspnea and fatigue are common in clients with left-sided (systolic) heart failure due to fluid backing up into the lungs and pulmonary congestion. Peripheral edema is more commonly seen with right-sided (diastolic) heart failure.

==================================================

Question: The client is diagnosed with infective endocarditis (IE) and has been receiving antibiotic therapy for four days. Which finding suggests that the antibiotic therapy has not been effective and must be reported to the health care provider (HCP) immediately?

A. Muscle tenderness
B. Nausea with vomiting
C. Streaks of red under the nails
D. Temperature of 103 F (39.5 C)

Answer: D
Findings of IE include skin rash (petechiae) and small areas of bleeding (splinter hemorrhages) under the fingernails. Muscle or joint pain or weakness are also common symptoms of IE. Nausea and vomiting may be side effects of the treatment; these findings probably would have appeared shortly after beginning treatment. Prolonged fever after 72 hours of antibiotic therapy indicates the antibiotic regime is not effective against the strain of microorganism - the nurse must call the HCP about this finding. Surgical intervention may be indicated for persistent sepsis after 72 hours of appropriate antibiotic treatment.

==================================================

Question: A nurse states, "I dislike caring for African-American clients because they are all so hostile." The nurse's statement is an example of which concept?

A. Stereotyping
B. Prejudice
C. Racism
D. Discrimination

Answer: A
Stereotyping refers to defining people and institutions, mentally or by attitudes, with narrow, fixed traits, rigid patterns, or with inflexible "boxlike" profile characteristics. Stereotyping is one of the common concerns of nurses when they begin to study different cultures and learn about transcultural nursing. Prejudice refers to preconceived ideas, beliefs, or opinions about an individual, group, or culture that limit a full and accurate understanding of the individual, culture, gender, race, event or situation. Discrimination is the unfair treatment of a person or group on the basis of prejudice. Racism is the belief that race is the primary determinant of human traits and capacities and that racial difference produces an inherent superiority of a particular race.

==================================================

Question: The nurse is teaching a group of clients about skin cancer. Which client statement indicates the need for further education about reducing the risk of skin cancer?

A. "I make sure to come inside between noon and 2 pm."
B. "I only tan in the controlled setting of a tanning booth."
C. "I wear sunglasses with ultraviolet protective lenses."
D. "I found a sunscreen with a sun protective factor of 30."

Answer: B
Tanning booths and sun lamps are no safer than the natural sun in terms of cellular damage and potential for developing skin cancer. The other self-help measures have positive effects on reducing the chance of damage from ultraviolet rays.

==================================================

Question: The nurse is caring for a client with hepatitis C. Which infection precautions should the nurse implement?

A. Blood-borne precautions
B. Droplet precautions
C. Transmission-based precautions
D. Standard precautions

Answer: D
Hepatitis C is transmitted via blood. Standard precautions are used for all blood-borne infections.Droplet precautions are a type of transmission-based precautions. Droplet precautions are used with pathogens that spread through the air with close contact and that affect the respiratory systems, such as influenza and pertussis (whopping cough).There is no precaution type called "blood-borne."

==================================================

Question: A nurse is teaching a newly diagnosed client with asthma how to use a peak flow meter. The nurse should explain that it is to be used to achieve which outcome?

A. Monitor atmosphere for presence of allergens
B. Provide metered doses for inhaled bronchodilator
C. Measure forced expiratory volumes
D. Determine the client's oxygen saturation

Answer: C
The peak flow meter is used to measure peak expiratory flow volumes. It provides useful information about the presence and/or severity of airway obstruction. If the result falls in the green, the client is good without any problems. If it falls into the yellow or red category, immediate action is required. The specific action should be determined with the health care provider ahead of time before this happens. Often the clients are advised to use a bronchodilator inhaler and then recheck for improvement. When teaching the colors for the peak flow meters, nurses often associated the colors and actions with those of a traffic light. Green = go; yellow = proceed with caution; and red = stop and get help.

==================================================

Question: A nurse is to administer meperidine (Demerol) 100 mg, atropine sulfate 0.4 mg, and promethazine (Phenergan) 50 mg IM to a preoperative client. Which action should the nurse take first?

A. Raise the side rails on the bed
B. Place the call bell within the client's reach
C. Assist the client to the bathroom
D. Instruct the client to remain in the bed

Answer: C
Meperidine is a narcotic analgesic and promethazine is an antihistamine; together they can potentiate CNS effects such as drowsiness, dizziness, lightheadedness and confusion. Although all of the options involve client safety, the first thing to do is to assist the client to the bathroom to void. After administering the preoperative medications, the nurse will instruct the client to remain in bed, place the call light in the client's hand and raise the side rails.

==================================================

Question: A nurse manager considers changing staff assignments from 8-hour shifts to 12-hour shifts. A staff-selected planning committee has approved the change, yet the staff are not receptive to the plan. The nurse manager should first take what action?

A. Design a different approach to deliver care with fewer staff
B. Retain the previous staffing pattern for another six months
C. Support the planning committee and post the new schedule
D. Explore how the planning committee evaluated barriers to the plan

Answer: D
A manager is ultimately responsible for delivery of care and yet has given a committee chosen by staff the right to approve or disapprove the change. Planned change involves exploring barriers and restraining forces before implementing change. To smooth acceptance of the change, restraining factors need to be evaluated. The manager wants to build the staff's skills at implementing change. Helping the committee evaluate its decision making is a useful step before rejecting or implementing the change. When possible, all affected by the change should be involved in the planning. The question is whether staff input has been thoroughly taken into consideration. This also illustrates the application of the nursing process to nonclient-care issues with assessment of the situation being the first step.

==================================================

Question: The client is one day post-op following a colon resection and there is an order to assist the client to walk in the hallway three times a shift while awake. Which instruction by the nurse is most appropriate when assigning this task to the unlicensed assistive person (UAP)?

A. "Have the client stand for at least two minutes before starting to walk."
B. "Apply a gait belt around the client's waist if the client reports feeling dizzy."
C. "Allow the client to sit on the side of the bed before assisting the client to stand and walk."
D. "When assisting the client, be sure to ask about the intensity of the pain."

Answer: C
The only appropriate statement is to allow the client to sit on the side of the bed first, before standing and walking. It is not necessary to stand up for two minutes before starting to walk. A gait belt should not be used since the client had abdominal surgery; besides, the UAP should not assist clients to stand and walk if they report feeling dizzy. The UAP cannot assess clients (ask about the intensity of the pain).

==================================================

Question: A nurse is discussing negativity with the parents of a 30 month-old child. How should the nurse tell the parents to best respond to this behavior?

A. Reprimand the child and give a 15-minute "time out"
B. Assert authority over the child through limit setting
C. Maintain a permissive attitude for this behavior
D. Use patience and a sense of humor to deal with this behavior

Answer: D
The nurse should help the parents see that negativity is a normal part of growth of autonomy in the toddler. They can best handle the negative toddler by using patience and humor.

==================================================

Question: A nurse is assessing several clients in a long-term health care facility. Which client is at highest risk for developing a pressure ulcer?

A. An 80 year-old ambulatory client with a history of diabetes mellitus
B. An obese client who uses a wheelchair
C. An incontinent client who has had three diarrhea stools in the past hour
D. A 79 year-old malnourished client on bed rest

Answer: D
Weighing significantly less than ideal body weight increases the number and surface area of bony prominences, which are susceptible to pressure ulcers. In addition, malnutrition is a major risk factor for pressure ulcers, from poor hydration and inadequate protein intake. Note that this is a priority question so that all of the clients are at risk for pressure ulcers. However, the question asks for the client with the highest risk.

==================================================

Question: A 12 year-old pediatric cancer client is distraught about the alopecia that occurred after the last chemotherapy treatment. Which nursing interventions are appropriate for this side effect of chemotherapy? (Select all that apply.)

A. Practice and teach thorough hand washing
B. Administer prescribed antiemetic medication before nausea is too severe
C. Encourage visits from friends before discharge from the hospital
D. Allow the child to choose a cap, scarf, wig or other head cover to use

Answer: C,D
Alopecia is the loss of hair, which is a frequent side effect of certain types of chemotherapy. Although it is not life-threatening, the body image change is difficult for many individuals, particularly children and adolescents. Encouraging visits from friends before discharge helps the young client and friends adjust. Wearing preferred forms of head cover-ups increases comfort and decreases embarrassment. The other options are proper interventions for chemotherapy, but do not help the client with hair loss.

==================================================

Question: A nurse asks a client with a history of alcoholism about recent drinking behavior. The client states, "I didn't hurt anyone. I just like to have a good time, and drinking helps me to relax." The client is using which defense mechanism?

A. Denial
B. Intellectualization
C. Rationalization
D. Projection

Answer: C
Rationalization is justifying illogical or unreasonable ideas, actions or feelings by the development of acceptable explanations for unacceptable actions. Both the teller and the listener find the rationalizations more satisfactory than the reality. Intellectualization is the use of reasoning in response to confrontation with unconscious conflicts and accompanying stressful emotions. Projection is the assignment of one's own feelings or thoughts to others.

==================================================

Question: The client has just had an enteral feeding tube inserted. What would be the most accurate method to verify initial placement of the feeding tube?

A. Abdominal x-ray
B. Aspiration for gastric contents
C. Flushing tube with saline
D. Auscultation with air insertion

Answer: A
The most objective and recommended approach to confirm correct tube placement after initial placement is radiography. This will determine if the tube is in the duodenum or jejunum and not in the airways of the lungs. After initial placement has been confirmed, the nurse can then verify placement by checking the pH of the aspirated gastric contents. Aspirates of pH 5.5 or below will indicate correct placement in most clients. The "whoosh" test is no longer recommended and should not be used.

==================================================

Question: A 16 month-old child has just been admitted to the hospital. As the nurse assigned to this child enters the hospital room for the first time, the toddler runs to the mother, clings to her and begins to cry. What should be an initial action by the nurse?

A. Arrange to change client care assignments
B. Explain that the child needs extra attention
C. Discuss the appropriate use of "time-out"
D. Explain that this behavior is expected

Answer: D
During normal development, fear of strangers becomes prominent and begins around age 6 to 8 months-old. Such behaviors include clinging to parent, crying and turning away from the stranger. These fears and behaviors extend into the toddler period. In the toddler period, separation anxiety is at its peak. As the child ages the behavior has a tendency to wane.

==================================================

Question: The nurse is taking the history of a pregnant woman. Which factor should the nurse recognize as the primary contraindication for breastfeeding?

A. Use of cocaine on weekends
B. Lactose intolerance
C. Age 40-years old
D. Family history of breast cancer

Answer: A
Binge use of cocaine can be just as harmful to the breast-fed newborn as regular daily use of cocaine.

==================================================

Question: The registered nurse (RN) is responsible for a client in isolation. Which nursing activity can be assigned to a licensed practical nurse (LPN)?

A. Reinforcing isolation precautions with visitors
B. Observing for and removing risks in the client's room
C. Assessing the client's attitude about infection control
D. Evaluating staff's compliance with infection control measures

Answer: A
LPNs (and unlicensed assistive persons) can reinforce information that was originally given by the RN. The other options are RN responsibilities and cannot be delegated.

==================================================

Question: The nurse has been teaching a client diagnosed with heart failure about proper nutrition. Which of these lunch selections indicates that the client has learned about sodium restriction?

A. Mushroom pizza and ice cream made from whole milk
B. Cheese sandwich with a glass of 2% milk
C. Cheeseburger and baked potato with butter
D. Sliced turkey sandwich with a side of canned pineapple

Answer: D
Sliced turkey sandwich is appropriate because it is not a highly processed food and canned fruits are low in sodium. All of the other choices contain one or more high-sodium foods.

==================================================

Question: A nurse is assessing a client two hours postoperatively after a femoral popliteal bypass to find that the upper leg dressing has become saturated with fresh blood. What should be the nurse's appropriate action?

A. Wrap the entire leg with elastic bandages
B. Apply pressure at the site of the bleeding
C. Reinforce the saturated dressing with a pressure dressing
D. Remove the old dressings with dressing reapplication

Answer: C
The fresh blood indicates active bleeding that need direct pressure with a pressure dressing. Because this type of surgery has long incisions the "site of the bleeding" may not be where the active bleeding is. Thus, this action is the best option of those given. The health care provider should be contacted next as the client undergoes continuous assessment for heart rate, blood pressure and respirations.

==================================================

Question: A nurse is caring for a 7 year-old child who is being discharged following a tonsillectomy. Which instruction is appropriate for the nurse to include during discharge teaching with the parents?

A. The child can return to school after being home for four days
B. Administer chewable aspirin for pain around the clock every six hours
C. The child may gargle with saline as necessary for discomfort
D. Report a persistent cough to the health care provider within 24 hours

Answer: D
Persistent coughing should be reported to the health care provider as this may indicate bleeding by a trickling of blood into the back of the throat. The other items are incorrect information especially the aspirin, which is not to be given to children. The saline may irritate the wound where the tonsils were removed.

==================================================

Question: A school nurse is called to the playground for an episode of mouth trauma. The nurse finds that the front tooth of a 9 year-old child has been avulsed ("knocked out"). After recovering the tooth, the initial response by the nurse should be to do what with the tooth?

A. Rinse the tooth in water before placing it into its socket
B. Hold the tooth by the roots until reaching the emergency room
C. Place the tooth in a clean plastic bag for transport to the dentist
D. Ask the child to replace the tooth even if the bleeding continues

Answer: A
Following avulsion of a permanent tooth, it is important to rinse the dirty tooth in water, saline solution or milk before re-implantation. If possible, replace the tooth into its socket within 30 minutes while avoiding contact with the root. The child should be taken to the dentist as soon as possible.

==================================================

Question: After surgery, a client with a nasogastric tube reports feeling nauseous. What action should the nurse take?

A. Call the health care provider to troubleshoot the problem
B. Put the head of the bed in a higher position
C. Administer an antiemetic that is ordered PRN
D. Check the patency of the nasogastric tube

Answer: D
An initial indication that the nasogastric tube is obstructed is a client's report of nausea. Nasogastric tubes may become obstructed by being kinked or with mucus or sediment.

==================================================

Question: The nurse is caring for a 1-year old client after heart surgery. The client weighs 22 pounds. The health care provider has given an order for morphine sulfate 4 mg IV every 3-4 hours as needed for pain. What should the nurse do next?

A. Administer the prescribed dose as ordered.
B. Hold the medication and contact the health care provider.
C. Give the dose every 6-8 hours.
D. Check with the pharmacist.

Answer: B
According to the Epocrates RX Online Reference (found under the Resources tab in the course), for Morphine prescribed parenterally (SQ/IM/IV), the recommended pediatric dose is: 0.1-0.2/kg (1-2 mg in this case) q2-4h. The prescribed dose falls outside of those guidelines (too high). Therefore, the nurse should hold the medication and contact the HCP for clarification.

==================================================

Question: A client is admitted to the hospital with a history of confusion. The client has difficulty remembering recent events and becomes disoriented when away from the home. Which statement would provide the best reality orientation for this client?

A. "Good morning. Do you remember where you are?"
B. "Hello. My name is Elaine Jones and I am your nurse for today."
C. "How are you today? Remember, you're in the hospital."
D. "Good morning. You're in the hospital. I am your nurse Elaine Jones."

Answer: D
As cognitive ability declines, the nurse should provide a calm, predictable environment for the client. This response establishes time, location and the caregiver's name.

==================================================

Question: A child is admitted to the hospital with findings consistent with rheumatic fever. During the admission process, which statement made by a parent would the nurse associate with this disease?

A. "Last week both feet had a fungal skin infection."
B. "Our child had a sore throat a month ago, which I treated with an herbal remedy."
C. "Our child is being tested for allergies and has reacted to some allergens."
D. "Both ears were infected when our child was 3 months-old."

Answer: B
Evidence supports a strong relationship between group A streptococcal infections and subsequent rheumatic fever (usually within two to six weeks). Therefore, the history of sore throat may have been an undiagnosed strep A infection. Appropriate antibiotic treatment of strep throat is the most effective way to reduce the risk of developing rheumatic fever.

==================================================

Question: A nurse is caring for a client who has developed cardiac tamponade. Which finding would the nurse anticipate?

A. Pleural friction rub
B. Widening pulse pressure
C. Bradycardia
D. Distended neck veins

Answer: D
In cardiac tamponade, intrapericardial pressures prevent adequate filling of the heart from the vena cava, and reduce cardiac output. As a result, venous pressures rise and the neck veins become distended.

==================================================

Question: A nurse is caring for an adult client on mechanical ventilation. Which nursing action will help prevent hypoxia when performing tracheostomy suctioning?

A. Hyperoxygenate the client with 100% O2 for 1 to 2 minutes before and after each suction pass
B. Time the duration of each suction pass to last 15 to 20 seconds
C. Flush the catheter using sterile normal saline before inserting it through the tracheostomy tube
D. Apply continuous suction while rotating and slowly removing the catheter

Answer: A
The nurse should administer supplemental 100% oxygen through the mechanical ventilator (or using a manual resuscitation bag) for 1 to 2 minutes before, after and between suctioning passes to prevent hypoxemia. The nurse first flushes the tip of the catheter using sterile normal saline to moisten it for easier insertion; however, this does not affect oxygenation. After flushing the catheter, the nurse inserts the catheter through the tracheostomy tube and applies suction intermittently for 5 to 10 seconds, while rotating and withdrawing the catheter.

==================================================

Question: The nurse is discussing dietary intake with an adolescent who has acne. What is the most appropriate statement by the nurse?

A. "Decrease fatty foods from your diet."
B. "Good nutritional habits promote healthy skin."
C. "Increase your intake of protein and vitamin A."
D. "Do not use caffeine in any form, including chocolate."

Answer: B
The exact cause of acne is not known, but genetics and hormones (androgens) play a role. Stress, picking or squeezing blemishes and harsh scrubbing can make acne worse. While poor nutrition may make acne-prone teens more susceptible to breakouts, chocolate or greasy foods don't cause acne. Vitamin A helps regulate the skin cycle, but too much can lead to toxic side effects. Teens should simply eat an age-appropriate, well-balanced diet.

==================================================

Question: The client is admitted with the diagnosis of chronic obstructive pulmonary disease (COPD). Which findings would require the nurse's immediate attention?

A. Low-grade fever and cough
B. Restlessness and confusion
C. Frequent productive cough with brownish sputum
D. Nausea and vomiting

Answer: B
Hypoxia and respiratory failure in COPD may be signaled by excessive somnolence, restless, aggressiveness, confusion, central cyanosis and shortness of breath. When these findings occur, the oxygen saturation and arterial blood gases (ABGs) should be assessed and oxygen should be rapidly titrated upward to correct the hypoxia. Signs of respiratory distress or failure may necessitate the use of ventilatory assistance BIPAP or emergent intubation and mechanical ventilation. Cough, discolored sputum, and fever may indicate a respiratory infection such as pneumonia, but this is a less urgent situation.

==================================================

Question: A nursing student asks the nurse manager to explain the forces that drive health care reform. The appropriate response by the nurse manager should include which approach?

A. Increased numbers of older adults and of the chronically ill of all ages
B. The escalation of fees with a decreased reimbursement percentage
C. High costs of diagnostic and end-of-life treatment procedures
D. A steep rise in provider fees and in insurance premiums

Answer: B
The percentage of the gross national product representing health care costs rose dramatically with reimbursement based on fee-for-service. Reimbursement for Medicare and Medicaid recipients based on fee-for-service also escalates health care costs.

==================================================

Question: A nurse is caring for a client who requires a mechanical ventilator for breathing. The high-pressure alarm goes off on the ventilator. What is an appropriate action for the nurse to take?

A. Perform a quick assessment of the client's overall condition along with respiratory effort
B. Call the respiratory therapist for help to troubleshoot the alarm
C. Press the alarm re-set button on the ventilator and observe the client
D. Disconnect the client from the ventilator and use a manual resuscitation bag

Answer: A
A number of situations can cause the high-pressure alarm to sound. It can be as simple as the client coughing. A quick assessment of the client will alert the nurse to whether it is a more serious or complex situation that might then require using a manual resuscitation bag and calling the respiratory therapist.

==================================================

Question: A nurse is teaching parents of an infant about the introduction of solid food to their baby. What is the first food that the nurse should teach the parents to add?

A. Vegetables
B. Fruit
C. Meats
D. Cereal

Answer: D
Cereal is usually introduced first because it is well-tolerated, easy to digest and fortified with iron. Then the meats or vegetables are introduced. The fruit is sweeter and often is recommended to be introduced last because of this; infants often like fruit the best.

==================================================

Question: A woman diagnosed with bipolar disorder is to take lithium as part of her treatment. What should the nurse discuss with the client as part of the teaching plan?

A. Weight reduction
B. Smoking cessation
C. Risk of concomitant use of oral contraceptives
D. Alcohol abstinence

Answer: D
Alcohol potentiates the effects of lithium, resulting in central nervous system depression and impairment of judgment, thinking and psychomotor skills. The client should be cautioned to avoid drinking alcoholic beverages.

==================================================

Question: There are new orders for the client's intravenous solution: "Decrease IV rate to 50 mL/hr and discontinue when infusion is complete." There is 250 mL remaining in the current 500 mL bag. The time is 10:30 am (1030 in military time).
At what time will the infusion be complete? State the answer in military time.

Answer: 1530
Solving with ratio proportion: 50 mL/1 hr = 250 mL/x hr x= 5 hr and 1030 + 5 = 1530. Or solving with logic: 250 mL remain/50 mL = 5 hours left 1030 (10:30 am) + 5 more hours = 1530 (using the 24-hour clock, or 3:30 pm)

==================================================

Question: A nurse is caring for a 5 year-old child whose left leg is in skeletal traction. Which activity would be an appropriate diversional activity?

A. Play "Simon Says"
B. Kick balloons with right leg
C. Throw bean bags
D. Play hand-held games

Answer: D
Immobilization with traction must be maintained until bone ends are in satisfactory alignment and with adequate regrowth of the bone. Activities that increase mobility interfere with the goals of treatment.

==================================================

Question: The nurse is reviewing age-appropriate diagnoses for older adults. Which nursing diagnosis would indicate that the client is at greatest risk for falling?

A. Impaired gas exchange related to retained secretions
B. Sensory perceptual alterations related to decreased vision
C. Altered patterns of urinary elimination related to nocturia
D. Alteration in mobility related to fatigue

Answer: C
Nocturia is especially problematic because many older adults fall when they rush to reach the bathroom during the night. They may be confused or not fully alert because of having been asleep. Inadequate lighting can increase their chances of stumbling, and then they may fall over furniture or carpets. Note that the question asks for the greatest risk, so that all of the options are correct and associated with falls. However, altered patterns of elimination are the most common risk for falls.

==================================================

Question: A client, admitted with palpitations and dyspnea, is diagnosed with atrial fibrillation (AF). Normal sinus rhythm is later restored using pharmacologic interventions. In addition to controlling cardiac rate and rhythm, the nurse understands that treatment for AF must include which of the following approaches?

A. Catheter ablation
B. Anticoagulation
C. Coronary artery bypass surgery
D. Cardioversion

Answer: B
In addition to rate and rhythm control, acute management of AF includes anticoagulation. Effective anticoagulation in clients with AF significantly reduces the risk of stroke and other thromboembolic events. When a client does not respond to pharmacologic interventions to restore sinus rhythm, cardioversion is used. Catheter ablation is used to disconnect the triggers for AF, but is not the first line of treatment. CABG is not used to treat AF.

==================================================

Question: A 15 month-old child comes to the clinic for a follow-up visit after hospitalization for treatment of Kawasaki disease and treatment involving immunoglobulins. The nurse should recognize which scheduled immunizations will be delayed?

A. Inactivated polio vaccine (IPV)
B. Haemophilus Influenzae Type b (Hib)
C. Mumps, measles, rubella (MMR)
D. Diptheria, tetanus, pertussis (DTaP)

Answer: C
Medical management of Kawasaki involves administration of immunoglobulins. Measles, mumps, rubella (MMR) is a live virus vaccine. Following administration of immunoglobulins, live vaccines should be held due to possible interference with the body's ability to form antibodies.

==================================================

Question: A nurse admits a client with hypertension who reports experiencing dizziness after taking diltiazem. Which focus is important for the nurse to assess?

A. Schedule for taking medication
B. Appearance of feet and ankles
C. Activity and rest patterns
D. Daily intake of potassium

Answer: A
A critical focus is whether the client has complied with the prescribed medication schedule and dose. Although diltiazem (Cardizem, Cartia, Dilacor, Diltia, Taztia, Tiazac) can be taken either in the morning or evening, taking the medication in the evening might help with this common side effect.

==================================================

Question: The client is scheduled for electroconvulsive therapy (ECT) in the morning. Which intervention must be completed prior to having this procedure?

A. Pre-anesthesia lab work
B. Electroencephalogram (EEG)
C. Blood type and crossmatch
D. Signed informed consent

Answer: D
Modern ECT is administered under general anesthesia. An electroencephalogram (EEG) is connected during the procedure but is not usually ordered pre-operatively. There's no need for a type and crossmatch. A basic metabolic panel (BMP) and complete blood count (CBC), as well as some other labs, may be ordered, as well as a ECG. But most importantly, the client has the right to be fully informed about the treatment and give written consent for the procedure.

==================================================

Question: A nurse is assigned to an adolescent unit. Which of these groups of needs would the nurse expect to have to deal with that day?

A. School performance, reading, journal writing
B. Privacy, autonomy, peer interactions
C. Interest in sports, competition, being right
D. Independence, confidence, narcissism

Answer: B
Adolescents display the need for privacy, autonomy and peer interaction concurrent with an evolving sense of identity.

==================================================

Question: A client who has been excessively drinking alcohol for five years states: "I drink when I get upset about ' things.' I have been unemployed. I feel like life is not leading anywhere." The nurse understands that the client is using alcohol as a way to deal with what issue?

A. Recreational/social needs
B. Feelings of anger
C. Issues of guilt and disappointment
D. Stressors in life

Answer: D
Alcohol is used by some people to manage anxiety and stress. The overall intent with this behavior is to decrease negative feelings and increase positive feelings. However, substance abuse, no matter what form or substance, eventually has an outcome of increased negative feelings.

==================================================

Question: The nurse is teaching the mother of a 5 month-old about nutrition for her baby. Which statement by the mother is incorrect and indicates the need for further teaching?

A. "When the baby wakes at night for a bottle, I give a feeding."
B. "I'm going to try feeding my baby some rice cereal this week."
C. "I keep formula made up ahead of time in the refrigerator for 24 hours."
D. "I dip the pacifier in honey so it is better taken."

Answer: D
The use of honey has been associated with infant botulism and should be avoided until after one year of age. Botulism effects the nervous system and often results in permanent damage. Older children and adults have digestive enzymes that kill the botulism spores.

==================================================

Question: A nurse is caring for elderly residents who live in a long-term care setting. Which activity would most effectively meet the growth and developmental needs for the elderly?

A. Regularly scheduled social activities
B. Reminiscence groups
C. Aerobic exercise classes
D. Transportation for shopping trips

Answer: B
According to Erikson's theory, older adults need to find and accept the meaningfulness of their lives, or they may become depressed, angry and fear death. Reminiscing contributes to successful adaptation by maintaining self-esteem, reaffirming identity, and working through loss. Erikson identifies this developmental challenge of the elderly as "ego integrity versus despair."

==================================================

Nclex Questions And Answers With Rationale

Question: Which of the following are agencies that focus on global health and development? Select all that apply.

1. World Health Organization
2. United Nations
3. National Institutes of Health
4. American Nurses Association

Answer: 1. World Health Organization
2. United Nations

Rationale: The NIH and ANA deal with local health issues within the United States while the WHO and United Nations deal with global health.

==================================================

Question: The number of deaths of infants younger than 28 days of age per 1000 live births is termed the:

1. infant death rate.
2. neonatal birth rate.
3. neonatal morbidity rate.
4. neonatal mortality rate.

Answer: 4. neonatal mortality rate.

Rationale: Neonatal mortality rate is defined as the number of deaths of infants under age 1 year per 1000 live births. Birth rate and neonatal birth rate refer to the number of births per 1000 population in a year. Neonatal morbidity rate refers to illness.

==================================================

Question: Which nursing action is the priority when a patient arrives at the clinic?

1. Prepare a plan of care.
2. Select the appropriate nursing diagnoses.
3. Administer medications as ordered.
4. Determine and document history and vital signs.

Answer: 4. Determine and document history and vital signs.

Rationale: Assessment is the first step of the nursing process and includes the collection of subjective and objective patient data. Diagnosis is the second step of the nursing process. Preparing a plan of care refers to the fourth step: planning. Administering medications relates to implementation, which is the fifth step of the nursing process.

==================================================

Question: Which of the following organizations sets standards of practice for nursing?

1. American Medical Association (AMA)
2. American Nurses Association (ANA)
3. Utilization review committee
4. American Academy of Pediatrics (AAP)

Answer: 2. American Nurses Association (ANA)

Rationale:The ANA develops standards of practice. The AMA is a medical association that is geared toward physicians. Utilization review committee reviews appropriateness of health care services and guidelines for physicians for treatment of illness, controlling management of care to achieve cost containment. The AAP is made up of pediatricians and establishes positions of leadership in setting standards of care for children.

==================================================

Question: The nurse is providing evidence-based care when he or she:

1. adheres to hospital procedures book guidelines.
2. carries out protocols learned in nursing school.
3. carries out the practice requested by the charge nurse or health care provider.
4. knows that the practice has been published in a professional journal or text.

Answer: 4. knows that the practice has been published in a professional journal or text.

Rationale: Evidence-based practice starts when the nurse uses the best evidence obtained from current, valid, published research. When the nurse combines that information with his or her critical thinking process, experiences, and patients' needs, it is then possible to plan safe, effective nursing care for the patient. The other answer options are not accurate.

==================================================

Question: which source would the nurse use to determine whether a specific nursing activity is within the scope of practice on an LPN/LVN?

1.Doctor's prescription record
2. Nursing procedure manual
3. Head nurse or nurse manager
4. The nurse practice act

Answer: 4. The nurse practice act

Rationale: Specific activities are listed by the state nurse practice act and nurses must practice within the limitations of the nurse practice act for their state. The other answer options are not sources that determine scope of practice.

==================================================

Question: Nursing organizations that suggest standards of care in maternal-child nursing are:

a. AWHONN
b. ANA
c. QSEN
d. AAP
1. c and d
2. a and b
3. b and c
4. b and d

Answer: 1. c and d
c. QSEN
d. AAP

Rationale: QSEN has to do with delivering quality care; American Academy of Pediatrics (AAP) is managed by physicians who set standards for pediatric care delivered by physicians. AWHONN and ANA set standards for nursing care.

==================================================

Question: Spermatozoa are produced in the:

1. vas deferens.
2. seminiferous tubules.
3. prostate gland.
4. Leydig cells.

Answer: 2. seminiferous tubules.

Rationale: Sperm are made in the convoluted seminiferous tubules of the testes. The other answer options are not correct because sperm are produced only in the testes.

==================================================

Question: A woman can keep a diary of her menstrual cycles to help determine her fertile period. She understands that after ovulation she will remain fertile for:

1. 2 hours.
2. 24 hours.
3. 3 to 5 days.
4. 7 to 14 days.

Answer: 2. 24 hours.

Rationale: The egg lives for only 24 hours after ovulation; therefore it is viable for fertilization only for 24 hours.

==================================================

Question: Which data indicate that a woman may have pelvic dimensions that would be inadequate for a normal vaginal delivery? A woman with a(n):

1. anthropoid-shaped pelvis with a history of pelvic inflammatory disease
2. gynecoid_shaped pelvis with a history of rickets
3. anthropoid-shaped pelvis that previously delivered a 9-lb infant
4. gynecoid-shaped pelvis with a history of poor nutrition

Answer: 2. gynecoid_shaped pelvis with a history of rickets

Rationale: Although the gynecoid-shaped pelvis is considered most favorable for a vaginal delivery, a woman with a history of rickets (a disease in which normal bone formation is disturbed when bone fails to mineralize and becomes soft and distorted) would be the most likely of the choices for inadequate normal vaginal delivery. An anthropoid-shaped pelvis with a history of PID, although not most favorable, allows for possible vaginal delivery, and history of PID might interfere with conception but not delivery. Previous delivery of a large infant is evidence of adequate pelvic dimensions for a normal vaginal delivery. Although poor maternal nutrition can affect the health of the newborn, delivery is not necessarily affected.

==================================================

Question: The muscular layer of the uterus that is the functional unit in pregnancy and labor is the:

1. perimetrium
2. myometrium
3. endometrium
4. cervix

Answer: 2. myometrium

Rationale: The myometrium is the middle muscular uterine layer and is functional in pregnancy and labor. The perimetrium is the outermost, or serosal, layer that envelops the uterus. The endometrium is the inner, or mucosal, layer that is functional during menstruation and implantation of the fertilized ovum. The cervix is the lower part of the uterus that lubricates the vagina, acts as a bacteriostatic agent, provides an alkaline environment, and produces a mucous plug.

==================================================

Question: During a prenatal clinic visit, a woman states that she probably will not plan to breastfeed her infant because she has very small breasts and believes she cannot provide adequate milk for a full-term infant, The best response of the nurse would be:

1. "Ask the physician if he or she will prescribe hormones to build up the breasts."
2. "I can provide you with exercises that will build up your breast tissue."
3. "The fluid intake of the mother will determine the milk output."
4. "The size of the breast has no relationship to the ability to produce adequate milk."

Answer: 4. "The size of the breast has no relationship to the ability to produce adequate milk."

Rationale: Breast size is primarily determined by the amount of fatty tissue and is unrelated to a woman's ability to produce milk. The other answer options are inaccurate.

==================================================

Question: The nurse is leading a class discussing ovulation and menstruation. The nurse explains that ovulation occurs:

a. 14 days after the last menstrual period
b. 14 days before the next menstrual period
c. at the 16th day of a 32-day menstrual cycle
d. 1 week before menses occurs
1. b and d
2. a and c
3. b only
4. d only

Answer: 3. b only

Rationale: Options A and C are related to the midcycle of menstruation when the egg matures and are released.

==================================================

Question: The child's sex is determined by the:

1. dominance of either the X or the Y chromosome.
2. number of X chromosomes in the ovum.
3. ovum, which contributes either an X or a Y chromosome.
4. sperm, which contains either an X or a Y chromosome.

Answer: 4. sperm, which contains either an X or a Y chromosome.

Rationale: The ovum always contributes an X chromosome (gamete), but the sperm can carry an X or a Y chromosome (gamete).

==================================================

Question: A woman who wants to become pregnant should avoid all medications unless they are prescribed by a physician who knows she is pregnant because:

1. the placenta allows most medications to cross into the fetus.
2. medications often have adverse effects when taken during pregnancy.
3. fetal growth is likely to be slowed by many medications.
4. the pregnancy is likely to be prolonged by some medications.

Answer: 1. the placenta allows most medications to cross into the fetus.

Rationale: The thin placental membrane provides some protection, but it is not a barrier to most substances ingested by the mother. Many harmful substances such as drugs, nicotine, and viral infectious agents are transferred to the fetus and may cause fetal drug addiction, congenital anomalies, and fetal infection.

==================================================

Question: When a couple has unprotected sexual intercourse 3 days before the woman ovulates, the risk of the woman becoming pregnant is:

1. limited because the ova lives only for 24 hours.
2. very high because both the ova and the sperm are capable of fertilizing at that time.
3. unknown.
4. very low because that is not the woman's "fertile period."

Answer: 4. very low because that is not the woman's "fertile period."

Rationale: The time during which fertilization can occur is brief because of the short life span of mature gametes. The ovum is estimated to survive for up to 24 hours after ovulation. The sperm remains capable of fertilizing the ovum for up to 5 days after being ejaculated into the area of the cervix. Therefore, the risk of pregnancy when unprotected intercourse occurs 4 days prior to ovulation is very low, because this is not the woman's fertile period.

==================================================

Question: The purpose of the foramen ovale is to:

1. increase fetal blood flow to the lungs.
2. limit blood flow to the liver.
3. raise the oxygen content of fetal blood.
4. reduce blood flow to the lungs.

Answer: 4. reduce blood flow to the lungs.

Rationale: The foramen ovale diverts most blood from the right atrium directly to the left atrium, rather than circulating it to the lungs. Therefore, blood flow to the lungs is reduced.

==================================================

Question: Why are twins often born early?

1. The uterus becomes overdistended.
2. The placenta becomes distended.
3. The woman's body cannot tolerate the weight.
4. The fetuses become too large to deliver vaginally.

Answer: 1. The uterus becomes overdistended.

Rationale: Many twin or higher multiples are born prematurely because the uterus becomes overly distended and irritable.

==================================================

Question: The nurse is responsible to examine the umbilical cord of the newborn infant. The nurse knows that:

a. the umbilical cord has 2 veins and 1 artery
b. the umbilical cord has 2 arteries and 1 vein
c. the umbilical cord has 2 arteries and 2 veins
d. umbilical arteries carry blood away from the fetus
e. umbilical arteries carry blood to the fetus
1. a and e
2. b and e
3. b and d
4. c and d

Answer: 3. b and d
b. the umbilical cord has 2 arteries and 1 vein
d. umbilical arteries carry blood away from the fetus

Rationale: The arteries and vein in the umbilical cord of the fetus function differently than arteries and veins in the rest of the body.

==================================================

Question: A woman arrives in the clinic for her prenatal visit. She states that she is currently 28 weeks pregnant with twins, she has a 5-year old son who was delivered at 39 weeks gestation and a 3-year old daughter delivered at 34 weeks gestation, and her last pregnancy terminated at 16 weeks gestation. The nurse will interpret her obstetric history as:

1. G4 T2 P2 A1 L4
2. G3 T2 P0 A1 L2
3. G3 T1 P1 A1 L2
4. G4 T1 P1 A1 L2

Answer: 4. G4 T1 P1 A1 L2

Rationale: G stands for gravida or how many pregnancies the woman has had. In this scenario there is a history of four pregnancies. The TPALM system is used to describe parity. T stands for term; this woman has had one child delivered at 39 weeks, which is considered term. P stands for preterm; this woman has had one child delivered at 34 weeks' gestation, which is considered preterm. A stands for abortion; this woman reports a pregnancy that terminated at 16 weeks' gestation. L stands for living; this woman has two living children. M stands for multiple, which is optional and not provided as a choice in this question.

==================================================

Question: Exercise during pregnancy should be practiced to achieve which of the following goals?

1. Maintaining physical fitness
2. Minimizing weight gain
3. Achieving weight loss
4. Improving physical fitness

Answer: 1. Maintaining physical fitness

Rationale: The goal of exercise during pregnancy should be maintenance of fitness, not improvement of fitness or weight loss.

==================================================

Question: During a prenatal examination at 30 weeks gestation, a woman is lying on her back on the examining table. She suddenly complains of dizziness and feeling faint. The most appropriate response of the nurse would be to:

1. reassure the woman and take measures to reduce her anxiety level.
2. offer the woman some orange juice or other rapidly absorbed form of glucose.
3. place a pillow under the woman's head.
4. turn the woman onto her side.

Answer: 4. turn the woman onto her side.

Rationale: Displacing the uterus to one side by turning the patient (preferably to the left) is all that is needed to relieve the pressure. If the woman must remain flat for any reason, then a small towel roll placed under one hip will also help prevent supine hypotension.

==================================================

Question: A woman being seen for her first prenatal care appointment has a positive home pregnancy test, and her chart shows a TPALM recording of 40120. The nurse would anticipate that:

1. minimal prenatal teaching will be required because this is her fourth pregnancy.
2. the woman will need help in planning the care of her other children at home during her labor and delivery.
3. the woman should experience minimal anxiety because she is familiar with the progress of pregnancy.
4. this pregnancy will be considered high risk, and measures to reduce anxiety will be needed.

Answer: 4. this pregnancy will be considered high risk, and measures to reduce anxiety will be needed.

Rationale: According to the TPALM system, these numbers indicate that the woman has had 4 children at term, has had 0 children at preterm, has 2 children now living, and has had 0 multiples. Because it is indicated that she had 4 children at term but only 2 are living now, the system indicates this pregnancy would be high risk, and anxiety-reduction techniques will be required.

==================================================

Question: A woman's LNMP was on April 1, 2019. She has been keeping her prenatal clinic appointments regularly but states she needs to alter the dates of a future appointment because she and her husband are going on an ocean cruise vacation for the New Year's celebration from December 30 through January 7, 2020. The best response of the nurse would be:

1. "Prenatal visits can never be altered. Every visit is important."
2. "Be sure to take antinausea medication when going on an ocean cruise."
3. "Perhaps you might consider rescheduling your vacation around the Thanksgiving holiday rather than the New Year's dates."
4. "I will reschedule your clinic appointment to accommodate your vacation plans.

Answer: 3. "Perhaps you might consider rescheduling your vacation around the Thanksgiving holiday rather than the New Year's dates."

Rationale: The woman's EDC (estimated date of confinement), using Naegele's rule, is calculated to be January 8, 2014. Scheduling a vacation that ends one day prior to the due date should be discouraged. Suggesting that the vacation be rescheduled is the best response by the nurse.

==================================================

Question: A nurse is explaining probable signs of pregnancy to a group of women. Probable signs of pregnancy include:

a. fetal heart beat
b. abdominal striae
c. amenorrhea
d. Braxton Hicks contractions
1. a and c
2. c and d
3. b and d
4. a and d

Answer: 3. b and d
b. abdominal striae
d. Braxton Hicks contractions

Rationale: Abdominal striae can be caused by stretching muscles due to weight gain and abdominal pain can be caused by gastrointestinal issues and both mistaken for signs of pregnancy. Absence of the menstrual period and the presence of a fetal heart beat are likely caused by pregnancy.

==================================================

Question: A woman has an incomplete abortion followed by vacuum aspiration. She is now in the recovery room with her husband and is crying softly. Select the most appropriate nursing action.

1. Leave the couple alone except for necessary recovery-room care.
2. Tell the couple that most abortions are for the best because the infant would have been abnormal.
3. Tell the couple that spontaneous abortion is very common and does not mean that they cannot have other children.
4. Express your regret at their loss and remain nearby if they want to talk about it.

Answer: 4. Express your regret at their loss and remain nearby if they want to talk about it.

Rationale: The woman experiencing abortion needs the nurse to listen and acknowledge the grief she and her partner feel. Open communication techniques such as providing a quiet presence, expressing sympathy, making open-ended statements, and providing reflection can accomplish this goal.

==================================================

Question: The health care provider gives magnesium sulfate intravenously to a woman with a diagnosis of preeclampsia. Which of the following nursing interventions are priority when caring for a patient who has received magnesium sulfate? (Select all that apply.)

a. Monitor uterine tone.
b. Monitor urine output.
c. Keep patient NPO.
d. Monitor respiratory rate.
1. a and b
2. c and d
3. b and d
4. a and c

Answer: 1. a and b
a. Monitor uterine tone.
b. Monitor urine output.
2. c and d
c. Keep patient NPO.
d. Monitor respiratory rate.
4. a and c
a. Monitor uterine tone.
c. Keep patient NPO.

Rationale: Priority nursing interventions for a patient receiving magnesium sulfate for preeclampsia include monitoring uterine muscle tone to assess for complications or signs of labor, as well as monitoring urine output and respiratory rate to assess for signs of toxicity. The woman receiving magnesium sulfate can have ice chips and sips of water during treatment.

==================================================

Question: A woman who has gestational trophoblastic disease (hydatidiform mole) should continue to receive follow-up medical care after initial treatment because:

1. choriocarcinoma sometimes occurs after intial treatment.
2. she has lower levels of immune factors and is vulnerable to infection.
3. anemia complications most cases of hydatidiform mole.
4. permanent elevation of her blood pressure is more likely.

Answer: 1. choriocarcinoma sometimes occurs after intial treatment.

Rationale: Hydatidiform mole may cause hemorrhage, clotting abnormalities, hypertension, and later development of cancer (choriocarcinoma).

==================================================

Question: Select the primary difference between the symptoms of placenta previa and abruptio placentae.

1. Fetal presentation
2. Presence of pain
3. Abnormal blood clotting
4. Presence of bleeding

Answer: 2. Presence of pain

Rationale: Manifestations of placenta previa include painless vaginal bleeding that is usually bright red. Bleeding accompanied by abdominal or low back pain is a typical characteristic of abruptio placentae.

==================================================

Question: During a prenatal clinic visit, your intervention with an abused woman is successful if you have assessed the status of the woman and:

1. persuaded her to leave her abusive partner.
2. informed her of her safety options.
3. convinced her to notify the police.
4. placed her in a shelter for abused women.

Answer: 2. informed her of her safety options.

Rationale: The woman being assessed for abuse is taken to a private area. The nurse determines whether there are factors that increase the risk for severe injuries or homicide such as drug use by the abuser, a gun in the house, prior use of a weapon, or violent behavior by the abuser outside the home. The woman should be referred to local shelters for her safety. The decision to leave her abuser is hers.

==================================================

Question: If a pregnant woman is admitted to the emergency department in shock after an accident, the nurse would help relieve the effect of shock by:

a. placing her in Trendelenburg position
b. placing her flat in bed in a supine position
c. placing a small pillow under left hip
d. closely observing and documenting fetal heart rate and contractions
1. c and d
2. a and d
3. b and d
4. d only

Answer: 1. c and d
c. placing a small pillow under left hip
d. closely observing and documenting fetal heart rate and contractions

Rationale: Placing a pregnant woman in supine or Trendelenburg's position places pressure on the uterine vessels and interferes with fetal circulation. Placing a small pillow under the left hip of the woman avoids this pressure. Close observation and documentation of the status of the mother and fetus is essential in all situations.

==================================================

Question: To determine the frequency of uterine contractions, the nurse should note the time from the:

1. beginning to end of the same contraction.
2. end of one contraction to the beginning of the next contraction.
3. beginning of one contraction to the beginning of the next contraction.
4. contraction's peak until the contraction begins to relax.

Answer: 3. beginning of one contraction to the beginning of the next contraction.

Rationale: Frequency is the time it takes from the beginning of one contraction to the beginning of the next contraction. Duration is from the beginning to the end of the same contraction. Interval is from the end of one contraction to the beginning of the next contraction.

==================================================

Question: Excessive anxiety and fear during labor may result in a(n):

1. ineffective labor pattern.
2. abnormal fetal presentation or position.
3. release of oxytocin from the pituitary gland.
4. rapid labor and uncontrolled birth.

Answer: 1. ineffective labor pattern.

Rationale: Anxiety can increase a woman's perception of pain and reduce her tolerance of it. Anxiety and fear also cause the secretion of stress compounds from the adrenal glands. These compounds, called catecholamines, inhibit uterine contractions and divert blood flow from the placenta.

==================================================

Question: A woman who is pregnant with her first child phones an intrapartum facility and says her "water broke." The nurse should tell her to:

1. wait until she has contractions every 5 minutes for 1 hour.
2. take her temperature every 4 hours and come to the facility if it is over 38 degrees C (100.4 degrees F).
3. come to the facility promptly, but safely.
4. call an ambulance to bring her to the facility.

Answer: 3. come to the facility promptly, but safely.

Rationale: When the amniotic sac ruptures, infection can become more likely if many hours elapse between the rupture of membranes and birth. In addition, the umbilical cord may slip down and become compressed. For these reasons, women should go to the birth facility when their membranes rupture, even if there are no other signs of labor.

==================================================

Question: A laboring woman suddenly begins making grunting sounds and bearing down during a strong contraction. The nurse should initially:

1. leave the room to find an experienced nurse to assess the woman.
2. look at her perineum for increased bloody show or perineal bulging.
3. ask her if she needs pain medication.
4. tell her that these are common sensations in late labor.

Answer: 2. look at her perineum for increased bloody show or perineal bulging.

Rationale: Grunting sounds and bearing down suggest that the woman is about to give birth. The nurse must stay with the woman and observe for other signs of impending birth, such as increased bloody show or perineal bulging, as well as monitor maternal and fetal well-being. The nurse may use the call bell to summon assistance or request the physician.

==================================================

Question: A woman in active labor has contractions every 3 minutes lasting 60 seconds, and her uterus relaxes between contractions. The electronic fetal monitor shows the FHR reaching 90 beats/min for periods lasting 20 seconds during a uterine contraction. The appropriate priority nursing action is to:

1. continue to monitor closely.
2. administer oxygen by mask at 10 L/min.
3. notify the health care provider.
4. prepare for a cesarean section.

Answer: 1. continue to monitor closely.

Rationale: Contractions every 3 minutes that last 60 seconds, a uterus that relaxes between contractions, and an FHR of 90 beats/minute that lasts 20 seconds during a uterine contraction all describe early decelerations, which result from compression of the fetal head and are a reassuring sign of fetal well-being.

==================================================

Question: The nurse is caring for a woman in labor. Which of the following observations require immediate nursing intervention?

a. FHR of 90 beats/min between contractions
b. maternal tachysystole
c. contractions lasting 60 seconds with an interval of 90 seconds
d. FHR baseline variability
1. b and c
2. a and d
3. a and b
4. c and d

Answer: 3. a and b
a. FHR of 90 beats/min between contractions
b. maternal tachysystole

Rationale: Fetal heart baseline variability is normal and contractions lasting 60 seconds with an interval of 90 seconds allow for adequate fetal circulation, and no change in nursing care is indicated. A FHR below 110 indicates fetal distress and maternal tachysystole indicates the uterine contractions are compromising fetal circulation and both require immediate intervention.

==================================================

Question: Which of the following is most appropriately used for pain relief during labor when the cervix is dilated less than 4 cm?

1. Naloxone (Narcan) via IM route
2. Meperidine (Demerol) via IM route
3. Promethazine (Phenergan) via IM route
4. Fentanyl (Sublimaze) via epidural route

Answer: 2. Meperidine (Demerol) via IM route

Rationale: Demerol (meperidine) along with Nubain is appropriate pain medication to use prior to 4 cm dilation. Epidurals are not usually given before 4 cm dilation. Phenergan is not given for pain relief.

==================================================

Question: Which technique is likely to be most effective for back labor?

1. Stimulating the abdomen by effleurage
2. Applying firm pressure in the sacral area
3. Blowing out in short breaths during each contraction
4. Rocking from side to side at the peak of each contraction

Answer: 2. Applying firm pressure in the sacral area

Rationale: Firm pressure against the lower back helps relieve some pain from back labor. Effleurage is performed on the abdomen in a circular movement during contractions. Blowing out prevents pushing before full dilation is reached. Rocking from side to side will not relieve back labor discomfort.

==================================================

Question: What drug should be immediately available for emergency use when a woman receives narcotics during labor?

1. Fentanyl (Sublimaze)
2. Diphenhydramine (Benadryl)
3. Lidocaine (Xylocaine)
4. Naloxone (Narcan)

Answer: 4. Naloxone (Narcan)

Rationale: Naloxone (Narcan) is used to reverse respiratory depression caused by opioid drugs.

==================================================

Question: Select the two most important nursing assessments immediately after a woman receives an epidural block.

1. Bladder distention
2. Condition of IV site
3. Respiratory rate
4. Blood pressure

Answer: 1. Bladder distention
4. Blood pressure

Rationale: The most common side effects of an epidural are maternal hypotension and urinary retention. After initiation of the epidural block, the fetal heart rate and blood pressure should be monitored and documented every 5 minutes for 15 minutes and then every 30 minutes for 1 hour. The nurse should palpate the suprapubic area for a full bladder every 2 hours or more often if a large quantity of IV solution was given.

==================================================

Question: A woman in labor states she wants to have epidural analgesia. When can this method of analgesia best be given?

1. Anytime during labor
2. During the transition phase of labor
3. During the first stage of labor
4. During the third stage of labor

Answer: 3. During the first stage of labor

Rationale: An epidural block is best given during the first stage of labor after the patient is 3 cm dilated and preferably before 7 cm of dilation. Timing of epidural insertion is essential because if it is inserted too early, it can slow down labor, and if it is inserted too late, it can interfere with the woman's ability to push effectively. Although transition is in the first stage of labor, it is usually the shortest phase. The third stage of labor is delivery of the placenta.

==================================================

Question: A woman who is in the early first stage of labor asks how she can relieve her discomforts. The nurse knows that nonpharmacological techniques that can relieve discomforts include:

a. sacral pressure
b. effleurage
c. sitz bath
d. laxatives
1. a and b
2. c and d
3. a and d
4. b and c

Answer: 1. a and b
a. sacral pressure
b. effleurage

Rationale: Laxatives are not given during labor and a sitz bath is a measure to relieve perineal injury postpartum. Sacral pressure and effleurage are effective non-pharmacological pain-relief techniques during labor.

==================================================

Question: The nurse notes that a woman's contractions during oxytocin induction labor are every 2 minutes; the contractions last 95 seconds, and the uterus remains tense between contractions. What action is expected based on these assessments?

1. No action is expected; the contractions are normal.
2. The rate of oxytocin administration will be increased slightly.
3. Pain medication or an epidural block will be offered.
4. Infusion of oxytocin will be stopped.

Answer: 4. Infusion of oxytocin will be stopped.

Rationale: Oxytocin is discontinued, or its rate reduced, if signs of fetal compromise or excessive uterine contractions occur. Excessive uterine contractions are most often evidenced by contractions closer than every 2 minutes, durations longer than 90 seconds, or resting intervals shorter than 60 seconds.

==================================================

Question: The nurse can anticipate that which of the following patients may be scheduled for induction of labor? A woman who is:

1. 38 weeks' gestation with fetus in transverse lie.
2. 40 weeks' gestation with fetal macrosomia.
3. 40 weeks' gestation with gestational hypertension.
4. 40 weeks' gestation with a fetal prolapsed cord.

Answer: 3. 40 weeks' gestation with gestational hypertension.

Rationale: Labor is induced if continuing the pregnancy is hazardous for the woman or the fetus. An indication for labor induction is gestational hypertension. Risk factors are too great for induction in the other choices.

==================================================

Question: A woman has an emergency cesarean delivery after the umbilical cord was found to be prolapsed. She repeatedly asks similar questions about what happened at birth. The nurse's interpretation of the woman's behavior is that she:

1. cannot accept that she did not have the type of delivery planned.
2. is trying to understand her experience and move on with postpartum adaptation.
3. thinks the staff is not telling her the truth about what happened at birth.
4. is confused about events because the effects of the general anesthetic are persisting.

Answer: 2. is trying to understand her experience and move on with postpartum adaptation.

Rationale: The nurse answers questions about events surrounding the birth and should encourage the woman and her partner to talk about the birthing situation so that they can integrate the experience. The focus should be on the birth rather than on the surgical aspects of cesarean delivery.

==================================================

Question: What nursing intervention during labor can increase space in the woman's pelvis?

1. Promote adequate fluid intake.
2. Position her on the left side.
3. Assist her to take a shower.
4. Encourage regular urination.

Answer: 4. Encourage regular urination.

Rationale: The most common soft-tissue obstruction during labor is a full bladder. The health care provider encourages urination and may catheterize a woman in labor. These interventions prevent trauma to her bladder and make more room in her pelvis.

==================================================

Question: A woman is being observed in the hospital because her membranes ruptured at 30 weeks gestation. While providing morning care, the nurse student notices that the draining fluid has a strong odor. The priority nursing action is to:

1. caution the woman to remain in bed until her physician visits.
2. ask the woman if she is having any more contractions than usual.
3. take the woman's temperature; report it and the fluid odor to the RN.
4. help to prepare the woman for an immediate cesarean delivery.

Answer: 3. take the woman's temperature; report it and the fluid odor to the RN.

Rationale: Amniotic fluid should be clear, possibly with flecks of vernix, and should not have a bad odor. The nurse should take the woman's temperature every 2 to 4 hours after her membranes rupture and observe, document, and report maternal temperature above 38° C (100.4° F), fetal tachycardia, tenderness over the uterine area, and foul-smelling fluid. These symptoms are suggestive of infection. A vaginal or cervical infection may cause membranes to rupture prematurely.

==================================================

Question: Following a vacuum extraction delivery, the nurse notices the newborn's head is not symmetrical with a chignon over the posterior fontanelle . The appropriate nursing action would be to:

a. apply cold compresses to the swollen area
b. notify the charge nurse or health care provider
c. document and continue routine observation
d. explain to the parents the swelling will resolve without treatment
1. a and b
2. a and c
3. c and d
4. all of the above

Answer: 3. c and d
c. document and continue routine observation
d. explain to the parents the swelling will resolve without treatment

Rationale: The vacuum extractor causes a harmless area of circular edema on the infant's scalp (chignon) where it was applied, does not necessitate intervention, and resolves quickly.

==================================================

Question: Which assessments are expected 24 hours after birth? (Select all that apply.)

1. Scant amount of lochia alba on the perineal pad.
2. Fundus firm and in the midline of the abdomen.
3. Breasts distended and hard with flat nipples.
4. Bradycardia.

Answer: 2. Fundus firm and in the midline of the abdomen.

Rationale: After 24 hours, the fundus begins to descend about 1 cm (one finger's width) each day.

==================================================

Question: Nursing the newborn promotes uterine involution because it:

1. uses maternal fat stores accumulated during pregnancy.
2. stimulates additional secretion of colostrum.
3. causes the pituitary to secrete oxytocin to contract the uterus.
4. promotes maternal formation of antibodies.

Answer: 3. causes the pituitary to secrete oxytocin to contract the uterus.

Rationale: Breastfeeding mothers may have more afterpains because infant suckling causes the posterior pituitary to release oxytocin, a hormone that contracts the uterus.

==================================================

Question: The best way to maintain the newborn's temperature immediately after birth is to:

1. dry the newborn thoroughly, including the hair.
2. give the newborn a bath using warm water.
3. feed 1 to 2 ounces of warmed formula.
4. limit the length of time that parents hold the newborn.

Answer: 1. dry the newborn thoroughly, including the hair.

Rationale: Newborns lose heat quickly after birth because amniotic fluid evaporates from their bodies, drafts move heat away, and they may contact cold surfaces. Evaporation is a mechanism of heat loss, and the amniotic fluid evaporates from the newborn's wet skin. Interventions to prevent heat loss from evaporation include drying the infant quickly and drying and covering the infant's head.

==================================================

Question: Eight hours postpartum the woman states she prefers the nurse to take care of the infant. The woman talks in detail about her birthing experience on the phone and to anyone who enters her room. She complains of being hungry, thirsty, and sleepy and is unable to focus on the infant care teaching offered to her. The nurse would interpret this behavior as:

1. inability to bond with the newborn.
2. development of postpartum psychosis.
3. inability to assume the parenting role.
4. the normal taking-in phase of the puerperium.

Answer: 4. the normal taking-in phase of the puerperium.

Rationale: Rubin described three phases of postpartum change that have been a framework of nursing care for 35 years. More recent studies have found that women progress through the same three phases. The first phase is "taking in." The mother is passive and willing to let others do for her. Conversation centers on her birth experience. The mother has an interest in her infant but is willing to let others handle the care and has little interest in learning. The primary focus is on recovery from birth and her need for food, fluids, and deep, restorative sleep.

==================================================

Question: Which of the following is a nursing intervention that does not require the written order of the health care provider? (Select all that apply.)

1. Administer an analgesic for pain.
2. Teach the patient how to perform perineal care.
3. Apply topical anesthetic for perineal suture pain.
4. Turn patient q2h.

Answer: 2. Teach the patient how to perform perineal care.
4. Turn patient q2h.

Rationale: Teaching the patient how to perform perineal care and turning the patient do not require a physician's order. Administration of analgesics and topical anesthetics would require a physician's order for implementation.

==================================================

Question: The earliest finding in postpartum hypovolemic shock is usually:

1. low blood pressure.
2. rapid pulse rate.
3. pale skin color.
4. soft uterus.

Answer: 2. rapid pulse rate.

Rationale: Tachycardia (rapid heart rate) is usually the first sign of inadequate blood volume (hypovolemia). The first blood pressure change is a narrow pulse pressure (a falling systolic pressure and a rising diastolic pressure). Skin and mucous membrane changes occur after tachycardia.

==================================================

Question: A bleeding laceration is typically manifested by:

1. a soft uterus that is difficult to locate.
2. low pulse rate and blood pressure.
3. bright red bleeding and a firm uterus.
4. profuse dark red bleeding and large clots.

Answer: 3. bright red bleeding and a firm uterus.

Rationale: Blood lost in lacerations is usually a brighter red than lochia, and it flows in a continuous trickle. Typically, the uterus is firm.

==================================================

Question: During the postpartum period the white blood cell (leukocyte) count is normally:

1. higher than normal.
2. lower than normal.
3. unchanged.
4. unimportant.

Answer: 1. higher than normal.

Rationale: White blood cells (leukocytes) are normally elevated during the early postpartum period to about 20,000 to 30,000 cells/dL, which limits the usefulness of the blood count to identify infection. Leukocyte counts in the upper limits are more likely to be associated with infection than lower counts.

==================================================

Question: A postpartum mother who is breastfeeding has developed mastitis. She states that she does not think it is good for her infant to drink milk from her infected breast. The best response from the nurse would be to:

1. instruct her to nurse the infant from only the unaffected breast until the infection clears up.
2. suggest that she discontinue breastfeeding and start the infant on formula.
3. encourage breastfeeding the infant to prevent engorgement.
4. apply tight breast binder to the infected breast until the infection subsides.

Answer: 3. encourage breastfeeding the infant to prevent engorgement.

Rationale: Antibiotics and continued removal of milk from the breast are the primary treatments for mastitis. The mother can usually continue to breastfeed unless an abscess forms.

==================================================

Question: A woman delivered her newborn several hours previously, and her uterus remains soft and boggy. Which of the following medications should the nurse anticipate that the health care provider would prescribe to increase uterine tone and firm the uterus? (Select all that apply.)

1. Methylergonovine (Methergine)
2. Carboprost (Hemabate)
3. Magnesium sulfate
4. Oxytoxin (Pitocin)

Answer: 1. Methylergonovine (Methergine)
2. Carboprost (Hemabate)
4. Oxytoxin (Pitocin)

Rationale: A dilute oxytocin (Pitocin) IV infusion is the most common drug ordered to control uterine atony. Other drugs to increase uterine tone include methylergonovine (Methergine) or prostaglandins such as Hemabate or Cytotec. Magnesium sulfate is a tocolytic and relaxes the smooth muscle of the uterus.

==================================================

Question: The nurse should be alert to subinvolution of the uterus as a cause of late postpartum bleeding. Signs to report and document include (select all that apply):

a. fundal height higher than expected for date
b. persistence of lochia rubra
c. low blood pressure
d. persistene of lochia rubra
1. c and d
2. a and d
3. a and b
4. b and c

Answer: 4. b and c
b. persistence of lochia rubra
c. low blood pressure

Rationale: Lochia rubra should only last for 3 days and prolonged lochia rubra may indicated subinvolution and low blood pressure may indicate loss of blood. Lochia alba does not occur in subinvolution and postpartum fundal height is not usually palpable in late postpartum bleeding.

==================================================

Nclex Questions On Diabetes Insipidus

Question: A client with primary diabetes insipidus is prescribed desmopressin (DDAVP). Which instruction should the nurse provide before the client is discharged?

1. "Administer desmopressin while the suspension is cold."

2. "Your condition isn't chronic, so you won't need to wear a medical identification bracelet."

3. "You may not be able to use desmopressin nasally if you have nasal discharge or blockage."

4. "You won't need to monitor your fluid intake and output after you start taking desmopressin."

Answer: 3

Desmopressin may not be absorbed if the intranasal route is compromised. Although diabetes insipidus is treatable, the client should wear medical identification and carry medication at all times to alert medical personnel in an emergency and ensure proper treatment. The client must continue to monitor fluid intake and output and receive adequate fluid replacement

==================================================

Question: Which of the following would indicate that a client has developed water intoxication secondary to treatment for diabetes insipidus?

1. Confusion and seizures

2. Sunken eyeballs and spasticity

3. Flaccidity and thirst

4. Tetany and increased blood urea nitrogen (BUN) levels.

Answer: 1

Classic signs of water intoxication include confusion and seizures, both of which are caused by cerebral edema. Weight gain will also occur. Sunken eyeballs, thirst, and increased BUN levels indicate fluid volume deficit. Spasticity, flaccidity, and tetany are unrelated to water intoxication.

==================================================

Question: A client is admitted for treatment of the syndrome of inappropriate antidiuretic hormone (SIADH). Which nursing intervention is appropriate?

1. Infusing I.V. fluids rapidly as ordered

2. Encouraging increased oral intake

3. Restricting fluids

4. Administering glucose-containing I.V. fluids as ordered

Answer: 3

To reduce water retention in a client with the SIADH, the nurse should restrict fluids. Administering fluids by any route would further increase the client's already heightened fluid load.

==================================================

Question: The nurse is developing a teaching plan for a client diagnosed with diabetes insipidus. The nurse should include information about which hormone lacking in clients with diabetes insipidus?

1. Antidiuretic hormone (ADH)

2. Thyroid-stimulating hormone (TSH)

3. Follicle-stimulating hormone (FSH)

4. Luteinizing hormone (LH)

Answer: 1

Clients with diabetes insipidus lack the hormone ADH. The client's TSH, FSH, and LH levels aren't affected.

==================================================

Question: When caring for a client with diabetes insipidus, the nurse expects to administer:

1. vasopressin (Pitressin Synthetic).

2. furosemide (Lasix).

3. regular insulin.

4. 10% dextrose.

Answer: 1

Because diabetes insipidus results from decreased antidiuretic hormone (vasopressin) production, the nurse should expect to administer synthetic vasopressin for hormone replacement therapy. Furosemide, a diuretic, is contraindicated because a client with diabetes insipidus experiences polyuria. Insulin and dextrose are used to treat diabetes mellitus and its complications, not diabetes insipidus

==================================================

Question: Which sign suggests that a client with the syndrome of inappropriate antidiuretic hormone (SIADH) secretion is experiencing complications?

1. Tetanic contractions

2. Neck vein distention

3. Weight loss

4. Polyuria

Answer: 2

SIADH secretion causes antidiuretic hormone overproduction, which leads to fluid retention. Severe SIADH can cause such complications as vascular fluid overload, signaled by neck vein distention. This syndrome isn't associated with tetanic contractions. It may cause weight gain and fluid retention (secondary to oliguria)

==================================================

Question: After falling off a ladder and suffering a brain injury, a client develops syndrome of inappropriate antidiuretic hormone (SIADH). Which findings indicate that the treatment he's receiving is effective?

Select all that apply:

1. Decrease in body weight

2. Rise in blood pressure and drop in heart rate

3. Absence of wheezes in his lungs

4. Increased urine output

5. Decreased urine osmolarity

Answer: 1, 4, 5

SIADH is an abnormality in which there is an abundance of the antidiuretic hormone. The predominant features are hyponatremia, oliguria, edema, and weight gain. Evidence of successful treatment includes a reduction in weight, an increase in urine output, and a decrease in the urine's concentration (urine osmolarity).

==================================================

Question: A client is diagnosed with syndrome of inappropriate antidiuretic hormone (SIADH). The nurse informs the client that the physician will prescribe diuretic therapy and restrict fluid and sodium intake to treat the disorder. If the client doesn't comply with the recommended treatment, which complication may arise?

1. Cerebral edema

2. Hypovolemic shock

3. Severe hyperkalemia

4. Tetany

Answer: 1

The Noncompliance with treatment for SIADH may lead to water intoxication from fluid retention caused by excessive antidiuretic hormone. This, in turn, limits water excretion and increases the risk of cerebral edema. Hypovolemic shock results from severe fluid volume deficit; in contrast, SIADH causes excessive fluid volume. The major electrolyte disturbance in SIADH is dilutional hyponatremia, not hyperkalemia. Because SIADH doesn't alter renal function, potassium excretion remains normal; therefore, severe hyperkalemia doesn't occur. Tetany results from hypocalcemia, an electrolyte disturbance not associated with SIADH.

==================================================

Question: When caring for a male client with diabetes insipidus, nurse Juliet expects to administer:
A. vasopressin (Pitressin Synthetic)
B. furosemide (Lasix).
C. regular insulin.
D. 10% dextrose.

Answer: A. vasopressin (Pitressin Synthetic)

Because diabetes insipidus results from decreased antidiuretic hormone (vasopressin) production, the nurse should expect to administer synthetic vasopressin for hormone replacement therapy. Furosemide, a diuretic, is contraindicated because a client with diabetes insipidus experiences polyuria. Insulin and dextrose are used to treat diabetes mellitus and its complications, not diabetes insipidus.

==================================================

Question: Nurse Louie is developing a teaching plan for a male client diagnosed with diabetes insipidus. The nurse should include information about which hormone lacking in clients with diabetes insipidus?

A. antidiuretic hormone (ADH).
B. thyroid-stimulating hormone (TSH).
C. follicle-stimulating hormone (FSH).
D. luteinizing hormone (LH).

Answer: A. antidiuretic hormone (ADH).

ADH is the hormone clients with diabetes insipidus lack. The client's TSH, FSH, and LH levels won't be affected.

==================================================

Question: A male client is admitted for treatment of the syndrome of inappropriate antidiuretic hormone (SIADH). Which nursing intervention is appropriate?

A. Infusing I.V. fluids rapidly as ordered
B. Encouraging increased oral intake
C. Restricting fluids
D. Administering glucose-containing I.V. fluids as ordered

Answer: C. Restricting fluids

To reduce water retention in a client with the SIADH, the nurse should restrict fluids. Administering fluids by any route would further increase the client's already heightened fluid load.

==================================================

Question: Which outcome indicates that treatment of a male client with diabetes insipidus has been effective?

A. Fluid intake is less than 2,500 ml/day.
B. Urine output measures more than 200 ml/hour.
C. Blood pressure is 90/50 mm Hg.
D. The heart rate is 126 beats/minute.

Answer: A. Fluid intake is less than 2,500 ml/day

Diabetes insipidus is characterized by polyuria (up to 8 L/day), constant thirst, and an unusually high oral intake of fluids. Treatment with the appropriate drug should decrease both oral fluid intake and urine output. A urine output of 200 ml/hour indicates continuing polyuria. A blood pressure of 90/50 mm Hg and a heart rate of 126 beats/minute indicate compensation for the continued fluid deficit, suggesting that treatment hasn't been effective.

==================================================

Question: A patient with SIADH is treated with water restriction and administration of IV fluids. The nurses evaluates that treatment has been effective when the patient experiences

A. Increased urine output, decreased serum sodium, and increased urine specific gravity

B. Increased urine output, increased serum sodium, and decreased urine specific gravity

C. Decreased urine output, increased serum sodium, and decreased urine specific gravity

D. Decreased urine output, decreased serum sodium, and increased urine specific gravity

Answer: B. increased urine output, increased serum sodium, and decreased urine specific gravity

The patient with SIADH has water retention with hyponatremia, decreased urine output and concentrated urine with high specific gravity. improvement in the patient's condition reflected by increased urine output, normalization of serum sodium, and more water in the urine, decreasing the specific gravity

==================================================

Question: The client diagnosed with a pituitary tumor developed syndrome of inappropriate antidiuretic hormone (SIADH). Which interventions should the nurse implement?

1. Assess for dehydration and monitor blood glucose levels.
2. Assess for nausea and vomiting and weigh daily.
3. Monitor potassium levels and encourage fluid intake.
4. Administer vasopressin IV and conduct a fluid deprivation test.

Answer: 2.

Early signs and symptoms are nausea and vomiting. The client has the syndrome of inappropriate secretion of antidiuretic (against allowing the body to urinate) hormone. In other words, the client is producing a hormone that will not allow the client to urinate.

1. The client has excess fluid and is not dehydrated, and blood glucose levels are not affected.
3. The client experiences dilutional hyponatremia, and the body has too much fluid already.
4. Vasopressin is the name of the antidiuretic hormone. Giving more increases the client's problem. Also, a water challenge test is performed, not a fluid deprivation test.

==================================================

Question: Which laboratory value should be monitored by the nurse for the client diagnosed with diabetes insipidus?

1. Serum sodium.
2. Serum calcium
3. Urine glucose.
4. Urine white blood cells.

Answer: 1

The client will have an elevated sodium level as a result of low circulating blood volume. The fluid is being lost through the urine. Diabetes means "to pass through" in Greek, indicating polyuria, a symptom shared with diabetes mellitus. Diabetes insipidus is a totally separate disease process.

2. Serum calcium is not affected by diabetes insipidus.
3. Urine glucose is monitored for diabetes mellitus.
4. White blood cells in the urine indicate the presence of a urinary tract infection.

==================================================

Question: The nurse is discharging a client diagnosed with diabetes insipidus. Which statement made by the client warrants further intervention?

1. "I will keep a list of my medications in my wallet and wear a Medic Alert bracelet."
2. "I should take my medication in the morning and leave it refrigerated at home."
3. "I should weigh myself every morning and record any weight gain."
4. "If I develop a tightness in my chest, I will call my health-care provider."

Answer: 2

Medication for DI is usually taken every 8 to 12 hours, depending on the client. The client should keep the medication close at hand.

1. The client should keep a list of medication being taken and wear a Medic Alert bracelet.
3. The client is at risk for fluid shifts. Weighing every morning allows the client to follow the fluid shifts. Weight gain indicates too much medication.
4. Tightness in the chest could be an indicator the medication is not being tolerated; if this occurs, the client should notify the health-care provider.

==================================================

Question: The client is admitted to the medical unit with a diagnosis of rule-out diabetes insipidus (DI). Which instructions should the nurse teach regarding a fluid deprivation test?

1. The client will be asked to drink 100 mL of fluid as rapidly as possible and then will not be allowed fluid for 24 hours.
2. The client will be administered an injection of antidiuretic hormone, and urine output will be measured for four (4) to six (6) hours.
3. The client will be NPO, and vital signs and weights will be done hourly until the end of the test.
4. An IV will be started with normal saline, and the client will be asked to try to hold the urine in the bladder until a sonogram can be done.

Answer: 3

The client is deprived of all fluids, and if the client has DI the urine production will not diminish. Vital signs and weights are taken every hour to determine circulatory status. If a marked decrease in weight or vital signs occurs, the test is immediately terminated.

1. The client is not allowed to drink during the test.
2. This test does not require any medications to be administered, and vasopressin will treat the DI, not help diagnose it.
4. No fluid is allowed and a sonogram is not involved.

==================================================

Question: The nurse is planning the care of a client diagnosed with syndrome of inappropriate antidiuretic hormone (SIADH). Which interventions should be implemented? Select all that apply.

1. Restrict fluids per health-care provider order.
2. Assess level of consciousness every two (2) hours.
3. Provide an atmosphere of stimulation.
4. Monitor urine and serum osmolality.
5. Weigh the client every three (3) day

Answer: 1, 2, 4

1. Fluids are restricted to 500 to 600 mL per 24 hours.
2. Orientation to person, place, and time should be assessed every two (2) hours or more often.
3. A safe environment, not a stimulating one, is provided.
4. Urine and serum osmolality are monitored to determine fluid volume status.
5. The client should be weighed daily.

==================================================

Question: The nurse is caring for a client diagnosed with diabetes insipidus (DI). Which intervention should be implemented?

1. Administer sliding-scale insulin as ordered.
2. Restrict caffeinated beverages.
3. Check urine ketones if blood glucose is >250.
4. Assess tissue turgor every four (4) hours.

Answer: 4

The client is excreting large amounts of dilute urine. If the client is unable to drink enough fluids, the client will quickly become dehydrated, so tissue turgor should be assessed frequently.

1. Diabetes insipidus is not diabetes mellitus; sliding-scale insulin is not administered to the client.
2. There is no caffeine restriction for DI.
3. Checking urine ketones is not indicated.

==================================================

Question: The unlicensed assistive personnel (UAP) complains to the nurse she has filled the water pitcher four (4) times during the shift for a client diagnosed with a closed head injury and the client has asked for the pitcher to be filled again. Which intervention should the nurse implement first?

1. Tell the UAP to fill the pitcher with ice cold water.
2. Instruct the UAP to start measuring the client's I & O.
3. Assess the client for polyuria and polydipsia.
4. Check the client's BUN and creatinine levels.

Answer: 3

The first action should be to determine if the client is experiencing polyuria
and polydipsia as a result of developing diabetes insipidus, a complication of the head trauma.

1. The client should have the water pitcher filled, but this is not the first action.
2. This should be done but not before assessing the problem.
4. This could be done, but it will not give the nurse information about DI.

==================================================

Question: The nurse is admitting a client diagnosed with syndrome of inappropriate antidiuretic hormone (SIADH). Which clinical manifestations should be reported to the health-care provider?

1. Serum sodium of 112 mEq/L and a headache.
2. Serum potassium of 5.0 mEq/L and a heightened awareness.
3. Serum calcium of 10 mg/dL and tented tissue turgor.
4. Serum magnesium of 1.2 mg/dL and large urinary output.

Answer: 1

A serum sodium level of 112 mEq/L is dangerously low, and the client is at risk for seizures. A headache is a symptom of a low sodium level.

2. This is a normal potassium level, and a heightened level of awareness indicates drug usage.
3. This is a normal calcium level and the client is fluid overloaded, not dehydrated, so there would not be tented tissue turgor.
4. This is a normal magnesium level, and a large urinary output is desired.

==================================================

Question: The male client diagnosed with syndrome of inappropriate antidiuretic hormone (SIADH) secondary to cancer of the lung tells the nurse he wants to discontinue the fluid restriction and does not care if he dies. Which action by the nurse is an example of the ethical principle of autonomy?

1. Discuss the information the client told the nurse with the health-care provider and significant other.
2. Explain it is possible the client could have a seizure if he drank fluid beyond the restrictions.
3. Notify the health-care provider of the client's wishes and give the client fluids as desired.
4. Allow the client an extra drink of water and explain the nurse could get into trouble if the client tells the health-care provider.

Answer: 3

This is an example of autonomy (the client has the right to decide for himself).

1. Discussing the information with others is not allowing the client to decide what is best for himself.
2. This could be an example of beneficence (to do good) if the nurse did this so the client has information on which to base a decision on whether to continue the fluid restriction.
4. This is an example of dishonesty and should never be tolerated in a health-care setting.

==================================================

Question: The nurse determines that demeclocycline (Declomycin) is effective for a patient with syndrome of inappropriate antidiuretic hormone (SIADH) based on finding that the patient's
A. weight has increased.
B. urinary output is increased.
C. peripheral edema is decreased.
D. urine specific gravity is increased.

Answer: B

Demeclocycline blocks the action of antidiuretic hormone (ADH) on the renal tubules and increases urine output. An increase in weight or an increase in urine specific gravity indicates that the SIADH is not corrected. Peripheral edema does not occur with SIADH. A sudden weight gain without edema is a common clinical manifestation of this disorder.

==================================================

Question: The nurse determines that additional instruction is needed for a 60-year-old patient with chronic syndrome of inappropriate antidiuretic hormone (SIADH) when the patient says which of the following?
A. "I need to shop for foods low in sodium and avoid adding salt to food."
B. "I should weigh myself daily and report any sudden weight loss or gain."
C. "I need to limit my fluid intake to no more than 1 quart of liquids a day."
D. "I will eat foods high in potassium because diuretics cause potassium loss."

Answer: A

Patients with SIADH are at risk for hyponatremia, and a sodium supplement may be prescribed. The other patient statements are correct and indicate successful teaching has occurred.

==================================================

Question: A 56-year-old patient who is disoriented and reports a headache and muscle cramps is hospitalized with possible syndrome of inappropriate antidiuretic hormone (SIADH). The nurse would expect the initial laboratory results to include a(n)
A. elevated hematocrit.
B. decreased serum sodium.
C. low urine specific gravity.
D. increased serum chloride.

Answer: B

When water is retained, the serum sodium level will drop below normal, causing the clinical manifestations reported by the patient. The hematocrit will decrease because of the dilution caused by water retention. Urine will be more concentrated with a higher specific gravity. The serum chloride level will usually decrease along with the sodium level.

==================================================

Question: An expected nursing diagnosis for a 30-year-old patient admitted to the hospital with symptoms of diabetes insipidus is
A. excess fluid volume related to intake greater than output.
B. impaired gas exchange related to fluid retention in lungs.
C. sleep pattern disturbance related to frequent waking to void.
D. risk for impaired skin integrity related to generalized edema.

Answer: C

Nocturia occurs as a result of the polyuria caused by diabetes insipidus. Edema, excess fluid volume, and fluid retention are not expected.

==================================================

Question: Which intervention will the nurse include in the plan of care for a 52-year-old male patient with syndrome of inappropriate antidiuretic hormone (SIADH)?
A. Monitor for peripheral edema.
B. Offer patient hard candies to suck on.
C. Encourage fluids to 2 to 3 liters per day.
D. Keep head of bed elevated to 30 degrees.

Answer: B

Sucking on hard candies decreases thirst for a patient on fluid restriction. Patients with SIADH are on fluid restrictions of 800 to 1000 mL/day. Peripheral edema is not seen with SIADH. The head of the bed is elevated no more than 10 degrees to increase left atrial filling pressure and decrease antidiuretic hormone (ADH) release.

==================================================

Question: The nurse is caring for a patient admitted with diabetes insipidus (DI). Which information is most important to report to the health care provider?
A. The patient is confused and lethargic.
B. The patient reports a recent head injury.
C. The patient has a urine output of 400 mL/hr.
D. The patient's urine specific gravity is 1.003.

Answer: A

The patient's confusion and lethargy may indicate hypernatremia and should be addressed quickly. In addition, patients with DI compensate for fluid losses by drinking copious amounts of fluids, but a patient who is lethargic will be unable to drink enough fluids and will become hypovolemic. A high urine output, low urine specific gravity, and history of a recent head injury are consistent with diabetes insipidus, but they do not require immediate nursing action to avoid life-threatening complications.

==================================================

Question: A 23-year-old patient is admitted with diabetes insipidus. Which action will be most appropriate for the registered nurse (RN) to delegate to an experienced licensed practical/vocational nurse (LPN/LVN)?
A. Titrate the infusion of 5% dextrose in water.
B. Teach the patient how to use desmopressin (DDAVP) nasal spray.
C. Assess the patient's hydration status every 8 hours.
D. Administer subcutaneous DDAVP.

Answer: D

Administration of medications is included in LPN/LVN education and scope of practice. Assessments, patient teaching, and titrating fluid infusions are more complex skills and should be done by the RN.

==================================================

Question: Which information is most important for the nurse to communicate rapidly to the health care provider about a patient admitted with possible syndrome of inappropriate antidiuretic hormone (SIADH)?
A. The patient has a recent weight gain of 9 lb.
B. The patient complains of dyspnea with activity.
C. The patient has a urine specific gravity of 1.025.
D. The patient has a serum sodium level of 118 mEq/L.

Answer: D

A serum sodium of less than 120 mEq/L increases the risk for complications such as seizures and needs rapid correction. The other data are not unusual for a patient with SIADH and do not indicate the need for rapid action

==================================================

Nclex Questions On Elimination

Question: During a home visit, an older adult male client mentions that he has experienced an increase in the frequency of urination at night. Which condition should the nurse discuss as a possible factor related to increased urinary elimination at night? (Select all that apply.)
A. Oliguria
B. Nocturia
C. Infection
D. Residual urine
E. Recognition of bladder fullness

Answer: B. Nocturia
C. Infection
D. Residual urine

==================================================

Question: A breastfeeding mother of a 2-month-old infant is concerned that her son defecates too frequently. Which response by the nurse should address this mother's concern?
A. "Feces containing less water may be difficult for infants to expel."
B. "Frequent bowel movements can occur with breastfeeding."
C. "The increased frequency in defecation means your baby is at risk of weight loss."
D. "Your baby should be able to control defecation by now."

Answer: B. "Frequent bowel movements can occur with breastfeeding."

==================================================

Question: The father of a 3-year-old boy is concerned that his child still wets the bed at night. Which explanation by the nurse is most appropriate regarding bedwetting?
A. "Children often achieve daytime bladder control prior to nighttime control."
B. "Sometimes children experience nocturia."
C. "By 24 months, children are capable of holding urine beyond the urge to void."
D. "Oliguria is not uncommon in children."

Answer: A. "Children often achieve daytime bladder control prior to nighttime control."

==================================================

Question: The daughter of a wheelchair-bound older adult client is concerned because her mother has been experiencing urinary incontinence. Which statement should the nurse use to explain the condition to the daughter?
A. "The kidneys reach maximum size at ages 35 to 40."
B. "Mobility issues may cause urinary incontinence."
C. "Renal blood flow and ability to concentrate urine decrease in older adults."
D. "The frequency of voiding varies in older adults and may cause urinary incontinence."

Answer: B. "Mobility issues may cause urinary incontinence."

==================================================

Question: The nurse educator is planning a presentation on involuntary urinary elimination for a group of new nurse graduates. The nurse educator should include which condition related to the types of involuntary urinary elimination? (Select all that apply.)
A. Anuria
B. Oliguria
C. Enuresis
D. Impaction
E. Incontinence

Answer: C. Enuresis
E. Incontinence

==================================================

Question: The nurse should anticipate conducting which assessment when preparing to provide care for a client experiencing alterations in bowel function? (Select all that apply.)
A. Client interview
B. Skin assessment
C. Renal assessment
D. Abdominal assessment
E. Inguinal area assessment

Answer: A. Client interview
B. Skin assessment
D. Abdominal assessment
E. Inguinal area assessment

==================================================

Question: The nurse is caring for a client who has a positive fecal test for occult blood. The nurse should anticipate which collaborative activity to further identify the cause of the client's problem?
A. Colonoscopy
B. Cystoscopy
C. Direct rectal examination (DRE)
D. Ultrasonic bladder scan

Answer: A. Colonoscopy

==================================================

Question: The nurse is assessing a client who is experiencing lower abdominal pain. Which abnormal finding requires the nurse to evaluate further?
A. Palpable bladder after urination
B. Absence of tenderness on kidney palpation
C. Absence of bruits over the renal arteries
D. Midline urinary meatus

Answer: A. Palpable bladder after urination
Rationale: Normally, the bladder isn't palpable over the pubic bone, especially if the client has just urinated.

==================================================

Question: A nurse is caring for a young woman with a suspected urinary tract infection (UTI). Which finding should confirm the nurse's suspicion?
A. pH of 9.2
B. Specific gravity of 1.012
C. Three white blood cells (WBCs) per low-powered field
D. Clear urine

Answer: A. pH of 9.2

==================================================

Question: A nurse is assessing a client who is complaining of black stools. About which medication that the client might be taking should the nurse inquire?
A. Antacids
B. Iron supplements
C.Antibiotics
D. Stool softener

Answer: B. Iron supplements

==================================================

Question: During an office visit, a client reports infrequent and difficult bowel movements. Which teaching topic should the nurse include when developing the client's plan of care? (Select all that apply.)
A. The importance of staying active
B. The use of laxatives or stool softeners
C. The importance of cooking and storing food correctly
D. The importance of consuming adequate amounts of fluid and fiber
E. The avoidance of raw fruit, vegetables, and meat when traveling abroad

Answer: A. The importance of staying active
B. The use of laxatives or stool softeners
D. The importance of consuming adequate amounts of fluid and fiber

==================================================

Question: The nurse should encourage the client to consume how much fluid each day to promote healthy bowel movements?
A. 1000-2000 mL
B. 3000-4000 mL
C. 2000-3000 mL
D. 4000-5000 mL

Answer: C. 2000-3000 mL

==================================================

Question: A nurse is preparing a client for colonoscopy. Which statement by the client indicates an understanding of the instructions?
A. "I will have to refrain from eating the night before and morning of the procedure."
B. "I will have to take a series of laxatives for one week before the procedure."
C. "I will have a large glass of water on the morning of the procedure."
D. "I may need an enema after the procedure has been completed."

Answer: A. "I will have to refrain from eating the night before and morning of the procedure."

==================================================

Question: When caring for a client with severe dehydration, the nurse should ensure which results are documented?
A. Oxygen saturation
B. Respiratory rate
C. Intake/output
D. Catheter care

Answer: C. Intake/output

==================================================

Question: A nurse is mentoring a new graduate nurse about caring for a Foley catheter. Which action by the new graduate nurse requires immediate intervention?
A. The nurse puts on a pair of non-sterile gloves before inserting the Foley catheter.
B. The nurse provides regular perineal care.
C. The nurse checks the collection system to ensure that it has remained closed.
D. The nurse washes hands before donning gloves.

Answer: A. The nurse puts on a pair of non-sterile gloves before inserting the Foley catheter.

==================================================

Question: The nurse is preparing to provide a newly prescribed laxative medication to a client with chronic constipation. Which should the nurse assess prior to administering the medication? (Select all that apply)
A. The client's fluid and electrolyte balance
B. Whether the client has recently had abdominal surgery
C. The client's blood urea nitrogen (BUN) and creatinine values
D. Whether the client has been experiencing nausea, vomiting, or cramps
E. Preventive measures for constipation to avoid overdependence on laxatives

Answer: B. Whether the client has recently had abdominal surgery
D. Whether the client has been experiencing nausea, vomiting, or cramps
E. Preventive measures for constipation to avoid overdependence on laxatives

==================================================

Question: The home health nurse is caring for a client with recent and ongoing urinary incontinence. The nurse should arrange for which type of referral?
A. Medical supply company
B. Social worker
C. Psychologist
D. Physical therapist

Answer: A. Medical supply company

==================================================

Question: A nurse has noticed a significant rise in catheter-associated infections on the medical floor and is designing a series of new initiatives to help reduce their occurrence. Which current practice should the nurse consider a priority to address with the staff on the medical floor?
A. Placement of Foley catheters in all admitted bed-bound clients
B. Use of ultrasound to assess bladder volume instead of intermittent catheterization
C. Foley catheters being changed only as needed
D. Refusal of healthcare providers to order routine bladder irrigation

Answer: A. Placement of Foley catheters in all admitted bed-bound clients

==================================================

Question: The nurse is assessing a client who presents with abdominal distention and cramping, vomiting, and inability to pass gas. The nurse should prepare for which collaborative intervention?
A. Insertion of a nasogastric tube
B. Removal of a fecal impaction
C. Insertion of a Foley tube
D. Administration of an enema

Answer: A. Insertion of a nasogastric tube

==================================================

Question: The nurse is caring for a client in the emergency department who is complaining of severe gas pain. The nurse should anticipate the administration of which medication to the client?
A. Antidiarrheal agent, such as loperamide
B.Stool softener, such as ducosate sodium
C.Antiflatuent, such as simethicone
D. Bulk-forming agent, such as methylcellulose

Answer: C.Antiflatuent, such as simethicone

==================================================

Nclex Questions On Fluid And Electrolytes

Question: What is the nurse's primary concern regarding fluid & electrolytes when caring for an elderly pt who is intermittently confused?
1. risk of dehydration
2. risk of kidney damage
3. risk of stroke
4. risk of bleeding

Answer: Answer: 1

Rationale 1: As an adult ages, the thirst mechanism declines. Adding this in a pt with an altered level of consciousness, there is an increased risk of dehydration & high serum osmolality.
Rationale 2: The risks for kidney damage are not specifically related to aging or fluid & electrolyte issues.
Rationale 3: The risk of stroke is not specifically related to aging or fluid & electrolyte issues.
Rationale 4: The risk of bleeding is not specifically related to aging or fluid & electrolyte issues.

==================================================

Question: The nurse is planning care for a pt with severe burns. Which of the following is this pt at risk for developing?
1. intracellular fluid deficit
2. intracellular fluid overload
3. extracellular fluid deficit
4. interstitial fluid deficit

Answer: Answer: 1

Rationale 1: Because this pt was severely burned, the fluid within the cells is diminished, leading to an intracellular fluid deficit.
Rationale 2: The intracellular fluid is all fluids that exist within the cell cytoplasm & nucleus. Because this pt was severely burned, the fluid within the cells is diminished, leading to an intracellular fluid deficit.
Rationale 3: The extracellular fluid is all fluids that exist outside the cell, including the interstitial fluid between the cells. Because this pt was severely burned, the fluid within the cells is diminished, leading to an intracellular fluid deficit.
Rationale 4: The extracellular fluid is all fluids that exist outside the cell, including the interstitial fluid between the cells. Because this pt was severely burned, the fluid within the cells is diminished, leading to an intracellular fluid deficit.

==================================================

Question: A pt, experiencing multisystem fluid volume deficit, has the symptoms of tachycardia, pale, cool skin, & decreased urine output. The nurse realizes these findings are most likely a direct result of which of the following?
1. the body's natural compensatory mechanisms
2. pharmacological effects of a diuretic
3. effects of rapidly infused intravenous fluids
4. cardiac failure

Answer: Answer: 1

Rationale 1: The internal vasoconstrictive compensatory reactions within the body are responsible for the symptoms exhibited. The body naturally attempts to conserve fluid internally specifically for the brain & heart.
Rationale 2: A diuretic would cause further fluid loss, & is contraindicated.
Rationale 3: Rapidly infused intravenous fluids would not cause a decrease in urine output.
Rationale 4: The manifestations reported are not indicative of cardiac failure in this pt.

==================================================

Question: A pregnant pt is admitted with excessive thirst, increased urination, & has a medical diagnosis of diabetes insipidus. The nurse chooses which of the following nursing diagnoses as most appropriate?
1. Risk for Imbalanced Fluid Volume
2. Excess Fluid Volume
3. Imbalanced Nutrition
4. Ineffective Tissue Perfusion

Answer: Answer: 1

Rationale 1: The pt with excessive thirst, increased urination & a medical diagnosis of diabetes insipidus is at risk for Imbalanced Fluid Volume due to the pt &'s excess volume loss that can increase the serum levels of sodium.
Rationale 2: Excess Fluid Volume is not an issue for pts with diabetes insipidus, especially during the early stages of treatment.
Rationale 3: Imbalanced Nutrition does not apply.
Rationale 4: Ineffective Tissue Perfusion does not apply

==================================================

Question: A pt recovering from surgery has an indwelling urinary catheter. The nurse would contact the pt's primary healthcare provider with which of the following 24-hour urine output volumes?
1. 600 mL
2. 750 mL
3. 1000 mL
4. 1200 mL

Answer: Answer: 1
Rationale 1: A urine output of less than 30 mL per hour must be reported to the primary healthcare provider. This indicates inadequate renal perfusion, placing the pt at increased risk for acute renal failure & inadequate tissue perfusion. A minimum of 720 mL over a 24-hour period is desired (30 mL multiplied by 24 hours equals 720 mL per 24 hours).

==================================================

Question: A pt is receiving intravenous fluids postoperatively following cardiac surgery. Nursing assessments should focus on which postoperative complication?
1. fluid volume excess
2. fluid volume deficit
3. seizure activity
4. liver failure

Answer: Answer: 1
Rationale 1: Antidiuretic hormone & aldosterone levels are commonly increased following the stress response before, during, & immediately after surgery. This increase leads to sodium & water retention. Adding more fluids intravenously can cause a fluid volume excess & stress upon the heart & circulatory system.
Rationale 2: Adding more fluids intravenously can cause a fluid volume excess, not fluid volume deficit, & stress upon the heart & circulatory system.
Rationale 3: Seizure activity would more commonly be associated with electrolyte imbalances.
Rationale 4: Liver failure is not anticipated related to postoperative intravenous fluid administration.

==================================================

Question: A pt is diagnosed with severe hyponatremia. The nurse realizes this pt will mostly likely need which of the following precautions implemented?
1. seizure
2. infection
3. neutropenic
4. high-risk fall

Answer: Answer: 1
Rationale 1: Severe hyponatremia can lead to seizures. Seizure precautions such as a quiet environment, raised side rails, & having an oral airway at the bedside would be included.
Rationale 2: Infection precautions not specifically indicated for a pt with hyponatremia.
Rationale 3: Neutropenic precautions not specifically indicated for a pt with hyponatremia.
Rationale 4: High-risk fall precautions not specifically indicated for a pt with hyponatremia.

==================================================

Question: A pt is diagnosed with hypokalemia. After reviewing the pt's current medications, which of the following might have contributed to the pt's health problem?
1. corticosteroid
2. thiazide diuretic
3. narcotic
4. muscle relaxer

Answer: Answer: 1
Rationale 1: Excess potassium loss through the kidneys is often caused by such meds as corticosteroids, potassium-wasting diuretics, amphotericin B, & large doses of some antibiotics.
Rationale 2: Excessive sodium is lost with the use of thiazide diuretics.
Rationale 3: Narcotics do not typically affect electrolyte balance.
Rationale 4: Muscle relaxants do not typically affect electrolyte balance.

==================================================

Question: A pt prescribed spironolactone is demonstrating ECG changes & complaining of muscle weakness. The nurse realizes this pt is exhibiting signs of which of the following?
1. hyperkalemia
2. hypokalemia
3. hypercalcemia
4. hypocalcemia

Answer: Answer: 1
Rationale 1: Hyperkalemia is serum potassium level greater than 5.0 mEq/L. Decreased potassium excretion is seen in potassium-sparing diuretics such as spironolactone. Common manifestations of hyperkalemia are muscle weakness & ECG changes.
Rationale 2: Hypokalemia is seen in non-potassium diuretics such as furosemide.
Rationale 3: Hypercalcemia has been associated with thiazide diuretics.
Rationale 4: Hypocalcemia is seen in pts who have received many units of citrated blood & is not associated with diuretic use.

==================================================

Question: The nurse is planning care for a pt with fluid volume overload & hyponatremia. Which of the following should be included in this pt's plan of care?
1. Restrict fluids.
2. Administer intravenous fluids.
3. Provide Kayexalate.
4. Administer intravenous normal saline with furosemide.

Answer: Answer: 1
Rationale 1: The nursing care for a pt with hyponatremia is dependent on the cause. Restriction of fluids to 1,000 mL/day is usually implemented to assist sodium increase & to prevent the sodium level from dropping further due to dilution.
Rationale 2: The administration of intravenous fluids would be indicated in fluid volume deficit & hypernatremia.
Rationale 3: Kayexalate is used in pts with hyperkalemia.
Rationale 4: The administration of normal saline with furosemide is used to increase calcium secretion.

==================================================

Question: When caring for a pt diagnosed with hypocalcemia, which of the following should the nurse additionally assess in the pt?
1. other electrolyte disturbances
2. hypertension
3. visual disturbances
4. drug toxicity

Answer: Answer: 1
Rationale 1: The pt diagnosed with hypocalcemia may also have high phosphorus or decreased magnesium levels.
Rationale 2: The pt with hypocalcemia may exhibit hypotension, & not hypertension.
Rationale 3: Visual disturbances do not occur with hypocalcemia.
Rationale 4: Hypercalcemia is more commonly caused by drug toxicities.

==================================================

Question: A pt with a history of stomach ulcers is diagnosed with hypophosphatemia. Which of the following interventions should the nurse include in this pt's plan of care?
1. Request a dietitian consult for selecting foods high in phosphorous.
2. Provide aluminum hydroxide antacids as prescribed.
3. Instruct pt to avoid poultry, peanuts, & seeds.
4. Instruct to avoid the intake of sodium phosphate.

Answer: Answer: 1
Rationale 1: Treatment of hypophosphatemia includes treating the underlying cause & promoting a high phosphate diet, especially milk, if it is tolerated. Other foods high in phosphate are dried beans & peas, eggs, fish, organ meats, Brazil nuts & peanuts, poultry, seeds & whole grains.
Rationale 2: Phosphate-binding antacids, such as aluminum hydroxide, should be avoided.
Rationale 3: Poultry, peanuts, & seeds are part of a high phosphate diet.
Rationale 4: Mild hypophosphatemia may be corrected by oral supplements, such as sodium phosphate.

==================================================

Question: When analyzing an arterial blood gas report of a pt with COPD & respiratory acidosis, the nurse anticipates that compensation will develop through which of the following mechanisms?
1. The kidneys retain bicarbonate.
2. The kidneys excrete bicarbonate.
3. The lungs will retain carbon dioxide.
4. The lungs will excrete carbon dioxide.

Answer: Answer: 1
Rationale 1: The kidneys will compensate for a respiratory disorder by retaining bicarbonate.
Rationale 2: Excreting bicarbonate causes acidosis to develop.
Rationale 3: Retaining carbon dioxide causes respiratory acidosis.
Rationale 4: Excreting carbon dioxide causes respiratory alkalosis

==================================================

Question: The nurse is caring for a pt diagnosed with renal failure. Which of the following does the nurse recognize as compensation for the acid-base disturbance found in pts with renal failure?
1. The pt breathes rapidly to eliminate carbon dioxide.
2. The pt will retain bicarbonate in excess of normal.
3. The pH will decrease from the present value.
4. The pt's oxygen saturation level will improve.

Answer: Answer: 1
Rationale 1: In metabolic acidosis compensation is accomplished through increased ventilation or "blowing off" C02. This raises the pH by eliminating the volatile respiratory acid & compensates for the acidosis.
Rationale 2: Because compensation must be performed by the system other than the affected system, the pt cannot retain bicarbonate; the manifestation of metabolic acidosis of renal failure is a lower than normal bicarbonate value.
Rationale 3: Metabolic acidosis of renal failure causes a low pH; this is the manifestation of the disease process, not the compensation.
Rationale 4: Oxygenation disturbance is not part of the acid-base status of the pt with renal failure.

==================================================

Question: When caring for a group of pts, the nurse realizes that which of the following health problems increases the risk for metabolic alkalosis?
1. bulimia
2. dialysis
3. venous stasis ulcer
4. COPD

Answer: Answer: 1
Rationale 1: Metabolic alkalosis is cause by vomiting, diuretic therapy or nasogastric suction, among others. A pt with bulimia may engage in vomiting or indiscriminate use of diuretics.
Rationale 2: A pt receiving dialysis has kidney failure, which causes metabolic acidosis.
Rationale 3: A venous stasis ulcer does not result in an acid-base disorder.
Rationale 4: The pt diagnosed with COPD typically has hypercapnea & respiratory acidosis.

==================================================

Question: The nurse is caring for a pt who is anxious & dizzy following a traumatic experience. The arterial blood gas findings include: pH 7.48, PaO2 110, PaCO2 25, & HCO3 24. The nurse would anticipate which initial intervention to correct this problem?
1. Encourage the pt to breathe in & out slowly into a paper bag.
2. Immediately administer oxygen via a mask & monitor oxygen saturation.
3. Prepare to start an intravenous fluid bolus using isotonic fluids.
4. Anticipate the administration of intravenous sodium bicarbonate.

Answer: Answer: 1
Rationale 1: This pt is exhibiting signs of hyperventilation that is confirmed with the blood gas results of respiratory alkalosis. Breathing into a paper bag will help the pt to retain carbon dioxide & lower oxygen levels to normal, correcting the cause of the problem.
Rationale 2: The oxygen levels are high, so oxygen is not indicated, & would exacerbate the problem if given. Intravenous fluids would not be the initial intervention.
Rationale 3: Not enough information is given to determine the need for intravenous fluids.
Rationale 4: Bicarbonate would be contraindicated as the pH is already high.

==================================================

Question: A pt is prescribed 20 mEq of potassium chloride. The nurse realizes that the reason the pt is receiving this replacement is
1. to sustain respiratory function.
2. to help regulate acid-base balance.
3. to keep a vein open.
4. to encourage urine output.

Answer: Answer: 2
Rationale 1: Potassium does not sustain respiratory function.
Rationale 2: Electrolytes have many functions. They assist in regulating water balance, help regulate & maintain acid-base balance, contribute to enzyme reactions, & are essential for neuromuscular activity.
Rationale 3: Intravenous fluids are used to keep venous access not potassium.
Rationale 4: Urinary output is impacted by fluid intake not potassium.

==================================================

Question: An elderly pt does not complain of thirst. What should the nurse do to assess that this pt is not dehydrated?
1. Ask the physician for an order to begin intravenous fluid replacement.
2. Ask the physician to order a chest x-ray.
3. Assess the urine for osmolality.
4. Ask the physician for an order for a brain scan.

Answer: Answer: 3
Rationale 1: It is inappropriate to seek an IV at this stage.
Rationale 2: There is no indication the pt is experiencing pulmonary complications thus a cheat x-ray is not indicated.
Rationale 3: The thirst mechanism declines with aging, which makes older adults more vulnerable to dehydration & hyperosmolality. The nurse should check the pt's urine for osmolality as a 1st step in determining hydration status before other detailed & invasive testing is done.
Rationale 4: There is no data to support the need for a brain scan.

==================================================

Question: An elderly pt who is being medicated for pain had an episode of incontinence. The nurse realizes that this pt is at risk for developing
1. dehydration.
2. over-hydration.
3. fecal incontinence.
4. a stroke.

Answer: Correct Answer: 1
Rationale 1: Functional changes of aging also affect fluid balance. Older adults who have self-care deficits, or who are confused, depressed, tube-fed, on bed rest, or taking medications (such as sedatives, tranquilizers, diuretics, & laxatives), are at greatest risk for fluid volume imbalance.
Rationale 2: There is inadequate evidence to support the risk of over-hydration.
Rationale 3: There is inadequate evidence to support the risk of fecal incontinence.
Rationale 4: There is inadequate evidence to support the risk of a stroke.

==================================================

Question: The nurse assesses a pt's weight loss as being 22 lbs. How many liters of fluid did this pt lose?

Answer: Correct Answer: 10
Rationale: Each liter of body fluid weighs 1 kg or 2.2 lbs. This pt has lost 10 liters of fluid.

==================================================

Question: A postoperative pt with a fluid volume deficit is prescribed progressive ambulation yet is weak from an inadequate fluid status. What can the nurse do to help this pt?
1. Assist the pt to maintain a standing position for several minutes.
2. This pt should be on bed rest.
3. Assist the pt to move into different positions in stages.
4. Contact physical therapy to provide a walker.

Answer: Answer: 3
Rationale 1: The pt should avoid prolonged standing.
Rationale 2: Bed rest can promote skin breakdown.
Rationale 3: The pt needs to be taught how to avoid orthostatic hypotension which would include assisting & teaching the pt how to move from one position to another in stages.
Rationale 4: A physician referral is needed for physical therapy intervention & is not indicated in this situation.

==================================================

Question: A postoperative pt is diagnosed with fluid volume overload. Which of the following should the nurse assess in this pt?
1. poor skin turgor
2. decreased urine output
3. distended neck veins
4. concentrated hemoglobin & hematocrit levels

Answer: Answer: 3
Rationale 1: Poor skin turgor is associated with fluid volume deficit.
Rationale 2: Decreased urine output is associated with fluid volume deficit.
Rationale 3: Circulatory overload causes manifestations such as a full, bounding pulse; distended neck & peripheral veins; increased central venous pressure; cough; dyspnea; orthopnea; rales in the lungs; pulmonary edema; polyuria; ascites; peripheral edema, or if severe, anasarca, in which dilution of plasma by excess fluid causes a decreased hematocrit & blood urea nitrogen (BUN); & possible cerebral edema.
Rationale 4: Increased hemoglobin & hematocrit values are associated with fluid volume deficit.

==================================================

Question: An elderly pt is at home after being diagnosed with fluid volume overload. Which of the following should the home care nurse instruct this pt to do?
1. Wear support hose.
2. Keep legs in a dependent position.
3. Avoid wearing shoes while in the home.
4. Try to sleep without extra pillows.

Answer: Answer: 1
Rationale 1: The home care nurse should instruct this pt about ways to decrease dependent edema, which include wearing support hose, elevating feet when in a sitting position, & resting in a recliner or bed with extra pillows.
Rationale 2: The pt should elevate the legs.
Rationale 3: As long as the shoes are well fitting, there is not reason to avoid wearing them.
Rationale 4: It is appropriate for the pt to use extra pillows to keep the head up while sleeping.

==================================================

Question: A pt with fluid retention related to renal problems is admitted to the hospital. The nurse realizes that this pt could possibly have which of the following electrolyte imbalances?
1. hypokalemia
2. hypernatremia
3. carbon dioxide
4. magnesium

Answer: Answer: 2
Rationale 1: The kidneys are the principal organs involved in the elimination of potassium. Renal failure is often associated with elevations potassium levels.
Rationale 2: The kidney is the primary regulator of sodium in the body. Fluid retention is associated with hypernatremia.
Rationale 3: Carbon dioxide abnormalities are not normally seen in this type of pt.
Rationale 4: Magnesium abnormalities are not normally seen in this type of pt.

==================================================

Question: An elderly pt comes into the clinic with the complaint of watery diarrhea for several days with abdominal & muscle cramping. The nurse realizes that this pt is demonstrating which of the following?
1. hypernatremia
2. hyponatremia
3. fluid volume excess
4. hyperkalemia

Answer: Answer: 2
Rationale 1: Hypernatremia is associated with fluid retention & overload. FVE is associated with hypernatremia.
Rationale 2: This elderly pt has watery diarrhea, which contributes to the loss of sodium. The abdominal & muscle cramps are manifestations of a low serum sodium level.
Rationale 3: This pt is more likely to develop clinical manifestations associated with fluid volume deficit.
Rationale 4: Hyperkalemia is associated with cardiac dysrhythmias.

==================================================

Question: A pt is admitted with hypernatremia caused by being str&ed on a boat in the Atlantic Ocean for five days without a fresh water source. Which of the following is this pt at risk for developing?
1. pulmonary edema
2. atrial dysrhythmias
3. cerebral bleeding
4. stress fractures

Answer: Answer: 3
Rationale 1: Pulmonary edema is not associated with dehydration.
Rationale 2: Atrial dysrhythmias are not a factor for this pt.
Rationale 3: The brain experiences the most serious effects of cellular dehydration. As brain cells contract, the brain shrinks, which puts mechanical traction on cerebral vessels. These vessels may tear, bleed, & lead to cerebral vascular bleeding.
Rationale 4: There have been no activities to support the development or occurrence of stress fractures.

==================================================

Question: The nurse is admitting a pt who was diagnosed with acute renal failure. Which of the following electrolytes will be most affected with this disorder?
1. calcium
2. magnesium
3. phosphorous
4. potassium

Answer: Answer: 4
Rationale 1: This pt will be less likely to develop a calcium imbalance.
Rationale 2: This pt will be less likely to develop a magnesium imbalance.
Rationale 3: This pt will be less likely to develop a phosphorous imbalance.
Rationale 4: Because the kidneys are the principal organs involved in the elimination of potassium, renal failure

==================================================

Question: A pt who is taking digoxin (Lanoxin) is admitted with possible hypokalemia. Which of the following does the nurse realize might occur with this pt?
1. Digoxin toxicity may occur.
2. A higher dose of digoxin (Lanoxin) may be needed.
3. A diuretic may be needed.
4. Fluid volume deficit may occur.

Answer: Answer: 1
Rationale 1: Hypokalemia increases the risk of digitalis toxicity in pts who receive this drug for heart failure.
Rationale 2: More digoxin is not needed.
Rationale 3: A diuretic may cause further fluid loss.
Rationale 4: There is inadequate information to assess for concerns related to fluid volume deficits.

==================================================

Question: A pt is prescribed 40 mEq potassium as a replacement. The nurse realizes that this replacement should be administered
1. directly into the venous access line.
2. mixed in the prescribed intravenous fluid.
3. via a rectal suppository.
4. via intramuscular injection.

Answer: Answer: 2
Rationale 1: Never administer undiluted potassium directly into a vein.
Rationale 2: The intravenous route is the recommended route for diluted potassium.
Rationale 3: The nurse should administer diluted potassium into the pt's intravenous line.
Rationale 4: The nurse should administer diluted potassium into the pt's intravenous line.

==================================================

Question: An elderly pt with a history of sodium retention arrives to the clinic with the complaints of "heart skipping beats" & leg tremors. Which of the following should the nurse ask this pt regarding these symptoms?

1. "Have you stopped taking your digoxin medication?"
2. "When was the last time you had a bowel movement?"
3. "Were you doing any unusual physical activity?"
4. "Are you using a salt substitute?"

Answer: Answer: 4
Rationale 1: Although this pt may be prescribed digoxin this is not the primary focus of this question.
Rationale 2: The pt's bowel habits are not of concern at this time.
Rationale 3: The cardiac & musculoskeletal discomforts being reported are not consistent with physical exertion.
Rationale 4: The pt has a history of sodium retention & might think that a salt substitute can be used. Advise pts who are taking a potassium supplement or potassium-sparing diuretic to avoid salt substitutes, which usually contain potassium.

==================================================

Question: A 35-year-old female pt comes into the clinic postoperative parathyroidectomy. Which of the following should the nurse instruct this pt?
1. Drink one glass of red wine per day.
2. Avoid the sun.
3. Milk & milk-based products will ensure an adequate calcium intake.
4. Red meat is the protein source of choice.

Answer: Answer: 3
Rationale 1: This pt should avoid alcohol.
Rationale 2: This pt can benefit from sun exposure.
Rationale 3: This pt is at risk for developing hypocalcemia. This risk can be avoided if instructed to ingest milk & milk-based products.
Rationale 4: Protein monitoring is not indicated.

==================================================

Question: A pt is admitted for treatment of hypercalcemia. The nurse realizes that this pt's intravenous fluids will most likely be which of the following?
1. dextrose 5% & water
2. dextrose 5% & ? normal saline
3. dextrose 5% & ? normal saline
4. normal saline

Answer: Answer: 4
Rationale 1: If isotonic saline is not used, the pt is at risk for hyponatremia in addition to the hypercalcemia.
Rationale 2: This solution is hypotonic. Isotonic saline is used because sodium excretion is accompanied by calcium excretion through the kidneys.
Rationale 3: This solution is hypotonic. Isotonic saline is used because sodium excretion is accompanied by calcium excretion through the kidneys.
Rationale 4: Isotonic saline is used because sodium excretion is accompanied by calcium excretion through the kidneys.

==================================================

Question: A 28-year-old male pt is admitted with diabetic ketoacidosis. The nurse realizes that this pt will have a need for which of the following electrolytes?
1. sodium
2. potassium
3. calcium
4. magnesium

Answer: Answer: 4
Rationale 4: One risk factor for hypomagnesaemia is an endocrine disorder, including diabetic ketoacidosis.

==================================================

Question: An elderly pt with peripheral neuropathy has been taking magnesium supplements. The nurse realizes that which of the following symptoms can indicate hypomagnesaemia?
1. hypotension, warmth, & sweating
2. nausea & vomiting
3. hyperreflexia
4. excessive urination

Answer: Answer: 1
Rationale 1: Elevations in magnesium levels are accompanied by hypotension, warmth, & sweating.
Rationale 2: Lower levels of magnesium are associated with nausea & vomiting.
Rationale 3: Lower levels of magnesium are associated & hyperreflexia.
Rationale 4: Urinary changes are not noted.

==================================================

Question: A pt is admitted with burns over 50% of his body. The nurse realizes that this pt is at risk for which of the following electrolyte imbalances?
1. hypercalcemia
2. hypophosphatemia
3. hypernatremia
4. hypermagnesemia

Answer: Correct Answer: 2

Rationale 1: Pts who experience burns are not at an increased risk for developing increased blood calcium levels.
Rationale 2: Causes of hypophosphatemia include stress responses & extensive burns.
Rationale 3: Pts who experience burns are not at an increased risk for developing increased blood sodium levels.
Rationale 4: Pts who experience burns are not at an increased risk for developing increased blood magnesium levels.

==================================================

Question: A pt is diagnosed with hyperphosphatemia. The nurse realizes that this pt might also have an imbalance of which of the following electrolytes?
1. calcium
2. sodium
3. potassium
4. chloride

Answer: Answer: 1
Rationale 1: Excessive serum phosphate levels cause few specific symptoms. The effects of high serum phosphate levels on nerves & muscles are more likely the result of hypocalcemia that develops secondary to an elevated serum phosphorus level. The phosphate in the serum combines with ionized calcium, & the ionized serum calcium level falls.

==================================================

Question: The nurse is reviewing a pt's blood pH level. Which of the systems in the body regulate blood pH? Select all that apply.
1. renal
2. cardiac
3. buffers
4. respiratory

Answer: Answer: 1,3
Rationale 1: Three systems work together in the body to maintain the pH despite continuous acid production: buffers, the respiratory system, & the renal system.
Rationale 2: The cardiac system is responsible for circulating blood to the body. It does not help maintain the body's pH.
Rationale 3: Three systems work together in the body to maintain the pH despite continuous acid production: buffers, the respiratory system, & the renal system.
Rationale 4: Three systems work together in the body to maintain the pH despite continuous acid production: buffers, the respiratory system, & the renal system.

==================================================

Question: The nurse observes a pt's respirations & notes that the rate is 30 per minute & the respirations are very deep. The metabolic disorder this pt might be demonstrating is which of the following?
1. hypernatremia
2. increasing carbon dioxide in the blood
3. hypertension
4. pain

Answer: Answer: 2
Rationale 1: Hypernatremia is associated with profuse sweating & diarrhea.
Rationale 2: Acute increases in either carbon dioxide or hydrogen ions in the blood stimulate the respiratory center in the brain. As a result, both the rate & depth of respiration increase. The increased rate & depth of lung ventilation eliminates carbon dioxide from the body, & carbonic acid levels fall, which brings the pH to a more normal range.
Rationale 3: The respiratory rate in a pt exhibiting hypertension is not altered.
Rationale 4: Pain may be manifested in rapid, shallow respirations.

==================================================

Question: The blood gases of a pt with an acid-base disorder show a blood pH outside of normal limits. The nurse realizes that this pt is
1. fully compensated.
2. demonstrating anaerobic metabolism.
3. partially compensated.
4. in need of intravenous fluids

Answer: Answer: 3

Rationale 1: If the pH is restored to within normal limits, the disorder is said to be fully compensated.
Rationale 2: Anaerobic metabolism results when the body's cells become hypoxic.
Rationale 3: If the pH is restored to within normal limits, the disorder is said to be fully compensated. When these changes are reflected in arterial blood gas (ABG) values but the pH remains outside normal limits, the disorder is said to be partially compensated.
Rationale 4: Although the pt may be in need of intravenous fluids, this is not the most correct or definitive answer.

==================================================

Question: A pt's blood gases show a pH greater of 7.53 & bicarbonate level of 36 mEq/L. The nurse realizes that the acid-base disorder this pt is demonstrating is which of the following?
1. respiratory acidosis
2. metabolic acidosis
3. respiratory alkalosis
4. metabolic alkalosis

Answer: Answer: 4
Rationale 1& 2: Respiratory acidosis & metabolic acidosis are both consistent with pH less than 7.35.
Rationale 3: Respiratory alkalosis is associated with a pH greater than 7.45 & a PaCO2 of less than 35 mmHG. It is caused by respiratory related conditions.
Rationale 4: Arterial blood gases (ABGs) show a pH greater than 7.45 & bicarbonate level greater than 26 mEq/L when the pt is in metabolic alkalosis.

==================================================

Question: An elderly postoperative pt is demonstrating lethargy, confusion, & a resp rate of 8 per minute. The nurse sees that the last dose of pain medication administered via a pt controlled anesthesia (PCA) pump was within 30 minutes. Which of the following acid-base disorders might this pt be experiencing?
1. respiratory acidosis
2. metabolic acidosis
3. respiratory alkalosis
4. metabolic alkalosis

Answer: Answer: 1
Rationale 1: Acute respiratory acidosis occurs due to a sudden failure of ventilation. Overdoses of narcotic or sedative medications can lead to this condition.
Rationale 2: The pt condition being described is respiratory not metabolic in nature.
Rationale 3: Acute respiratory acidosis occurs due to a sudden failure of ventilation. Overdoses of narcotic or sedative medications can lead to this condition.
Rationale 4: Acute respiratory acidosis occurs due to a sudden failure of ventilation. Overdoses of narcotic or sedative medications can lead to this condition. The pt condition being described is respiratory not metabolic in nature.

==================================================

Question: The pt has been placed on a 1200 mL daily fluid restriction. The pt's IV is infusing at a keep open rate of 10 mL/hr. The pt has no additional IV medications. How much fluid should the pt be allowed from 0700 until 1500 daily?

Answer: Answer: 540
Rationale: Fluid allowed is calculated by figuring the total daily IV intake (in this case 10 mL/hr × 24 hours = 240 mL/day), subtracting that total from the daily allowance (in this case 1200mL - 240 mL = 960mL). The amount calculated is then distributed as 50% for the traditional day shift, 25%-35% for the traditional evening shift, & the remainder for the traditional night shift. In this case, 50% of 960 is 540 mL.

==================================================

Question: The pt is receiving intravenous potassium (KCL). Which nursing actions are required? Select all that apply.
1. Administer the dose IV push over 3 minutes.
2. Monitor the injection site for redness.
3. Add the ordered dose to the IV hanging.
4. Use an infusion controller for the IV.
5. Monitor fluid intake & output.

Answer: Answer: 2,4,5

==================================================

Question: Which pts are at risk for the development of hypercalcemia? Select all that apply.
1. the pt with a malignancy
2. the pt taking lithium
3. the pt who uses sunscreen to excess
4. the pt with hyperparathyroidism
5. the pt who overuses antacids

Answer: Correct Answer: 1,2,4,5
Rationale 1: Pts with malignancy are at risk for development of hypercalcemia due to destruction of bone or the production of hormone-like substances by the malignancy.
Rationale 2: Lithium & overuse of antacids can result in hypercalcemia. Hypercalcemia can result from hyperparathyroidism which causes release of calcium from the bones, increased calcium absorption in the intestines & retention of calcium by the kidneys.
Rationale 3: The pt who uses sunscreen to excess is more likely to have a vitamin D deficiency which would result in hypocalcemia.
Rationale 4: Hypercalcemia can result from hyperparathyroidism which causes release of calcium from the bones, increased calcium absorption in the intestines & retention of calcium by the kidneys.
Rationale 5: Lithium & overuse of antacids can result in hypercalcemia.

==================================================

Question: The pt who has a serum magnesium level of 1.4 mg/dL is being treated with dietary modification. Which foods should the nurse suggest for this pt? Select all that apply.
1. bananas
2. seafood
3. white rice
4. lean red meat
5. chocolate

Answer: Answer: 1,2,5
Rationale: Serum magnesium level of 1.4 mg/dL suggests mild hypomagnesaemia, so this pt should be counseled to eat foods high in magnesium. Foods high in magnesium include green leafy vegetables, seafood, milk, bananas, citrus fruits, & chocolate. White rice & lean red meat are not included.

==================================================

Question: The pt has a serum phosphate level of 4.7 mg/dL. Which interdisciplinary treatments would the nurse expect for this pt? Select all that apply.
1. IV normal saline
2. calcium containing antacids
3. IV potassium phosphate
4. encouraging milk intake
5. increasing vitamin D intake

Answer: Answer: 1,2
Rationale: Serum phosphate level of 4.7 mg/dL indicates hyperphosphatemia. IV normal saline promotes renal excretion of phosphate.

==================================================

Question: The pt, newly diagnosed with diabetes mellitus, is admitted to the emergency department with nausea, vomiting, & abdominal pain. ABG results reveal a pH of 7.2 & a bicarbonate level of 20 mEq/L. Which other assessment findings would the nurse anticipate in this pt? Select all that apply.
1. tachycardia
2. weakness
3. dysrhythmias
4. Kussmaul's respirations
5. cold, clammy skin

Answer: Answer: 2,3,4
Rationale: Further assessment findings of this condition are weakness, bradycardia, dysrhythmias, general malaise, decreased level of consciousness, warm flushed skin, & Kussmaul's respirations.

Rationale: These ABG results, coupled with the pt's recent diagnosis of diabetes mellitus & history of vomiting would lead the nurse to suspect metabolic acidosis. Further assessment findings of this condition are weakness, bradycardia, dysrhythmias, general malaise, decreased level of consciousness, warm flushed skin, & Kussmaul's respirations.

==================================================

Question: daily body fluid excretion

Answer: skin by diffusion 400ml Total 2450
skin by perspiration 100ml
lungs 300ml
feces 150ml
kidneys 1500ml

==================================================

Question: water intake

Answer: 2500ml daily required to cover loss

==================================================

Question: Isotonic dehydration(hypovolemia)
most common

Answer: water and electrolytes loss in equal proportions
caused by inadequate intake of fluids and solutes
shift in fluid, and excessive losses
Treatment Isotonic Replacement :
0.9% sodium chloride, 5%dextrose in water, and 5% dextrose in 0.225%saline, Ringer's Lactate

==================================================

Question: Hypovolemic signs and symptoms Cardiovascular

Answer: Cardiovascular
Thready, increased pulse
decreased blood pressure and orthostatic hypotension
flat neck and hand veins in dependent position
diminished peripheral pulses

==================================================

Question: Hypovolemic signs and symptoms Respiratory Neuromuscular

Answer: Respiratory
Increase rate and depth
Neuromuscular
decreased central nervous system activity(lethargy to coma) and fever

==================================================

Question: hypovolemia Renal

Answer: Decreased urinary output
Increase urine specific gravity

==================================================

Question: Hypovolemia Integumentary

Answer: Dry skin
poor turgor, tenting present
Dry mouth mucosal membranes

==================================================

Question: Hypovolemia Gastrointestinal

Answer: decreased motility and diminished bowel sounds
constipation
thrist
decreased body weight

==================================================

Question: Lab finding

Answer: Increased serum osmolality
Increased hematocrit
increased BUN
increased serum sodium level

==================================================

Question: Hypertonic dehydration

Answer: Water loss exceeds electrolyte loss
caused by excessive perspiration, hyperventilation, ketoacidosis, prolonged fever, diarrhea, early stage renal failure, and diabetes insipidus. cells shrink
Replacement of water using hypotonic solutions
0.45% sodium chloride, 0.225% sodium chloride, and 0.33% sodium chloride

==================================================

Question: Hypertonic dehydration signs and symptoms

Answer: Hyperactive deep tendon reflexes
Pitting edema

==================================================

Question: Hypotonic dehydration

Answer: electrolyte loss exceeds water loss and causes
cells to swell
caused by chronic illness, excessive fluid replacement
renal failure, chronic malnutrition
Treatment Replacement of electrolytes: 3% sodium chloride, 5%sodium chloride, 10% dextrose in water, 5% dextrose in 0.9% sodium chloride, 5% dextrose in 0.45% sodium chloride and 5% dextrose in Ringer's Lactate

==================================================

Question: dehydration Interventions

Answer: monitor cardiovascular, respiratory, neuromuscular, renal, integumentary, and gastrointestional status
prevent further fluid loss and increase fluid compartment volumes to normal ranges
provide oral rehydration and IV rehydration is severe monitor intake and output
administer medications as prescribed
antidiarrheal, antimicrobial, antiemetic and treat symptoms
Administer O2 as prescribed
monitor electrolyte labs and treat

==================================================

Question: Fluid overload Isotonic hypervolemia

Answer: causes circulatory overload and interstitial edema when severe or when client has poor cardiac function cogestive heart failure, and pulmonary edema
caused by
Inadequately controlled IV therapy
renal failure
long-term corticosteroid therapy

==================================================

Question: Isotonic signs and symptoms

Answer: enlarged liver and ascites

==================================================

Question: fluid overload signs and symptoms cardiovascular

Answer: bounding, increased pulse rate
elevated blood pressure
distended neck and hand veins
elevated central venous pressure

==================================================

Question: Fluid overload Respiratory

Answer: increased shallow respiration
dyspnea
moist crackles on auscultation

==================================================

Question: Fluid overload Neromuscular

Answer: altered level of consciousness
headache
visual disturbances
skeletal muscle weakness
paresthesias

==================================================

Question: Fluid overload integumentary

Answer: pitting edema in dependant areas
skin pale and cool to touch

==================================================

Question: Fluid overload gastrointestinal tract

Answer: increased motility

==================================================

Question: Fluid overload Lab

Answer: Decreased serum osmolality
decreased hematocrit
decreased BUN level
decreased serum sodium level
decreased urine specific gravity

==================================================

Question: Fluid overload intervention

Answer: monitor cardiovascular, respiratory, neuromuscular, renal, integumentary and gastrointestinal status.
Prevent further fluid overload and restore normal fluid balance
administer diurectics; osmotic diuretics typically are prescribed first to prevent severe electrolyte imbalances
Restrict fluid and sodium intake
monitor intake and output
monitor electrolyte values

==================================================

Question: fluid overload hypertonic

Answer: caused by excessive sodium, Raip infusion of hypertonic solution, excessive sodium bicarbonate therapy

==================================================

Question: Fluid overload hypotonic

Answer: water intoxication caused by early renal failure, syndrome of inappropriate antidiuretic hormone secretion
Inadequately controlled IV therapy
Replacement of isotonic fluid loss with hypotonic fluids
irrigation of wounds and body cavities with hypotonic fluids

==================================================

Question: Fluid overload hypotonic signs and symptoms

Answer: Polyuria, diarrhea, nonpitting edema, dysrhythmias, projectile vomiting

==================================================

Question: Sodium NA

Answer: Normal 135 to 145
Common food source
bacon, butter, canned food, cheese( american, cottage)
frankfurters, ketchup, lunch meat, milk, mustard, processed food, snack food, table salt, white and whole wheat bread

==================================================

Question: Hyponatremia

Answer: below 135
caused by increased sodium excretion
excessive diaphoresis
diurectics, vomiting, diarrhea, wound drainage( especially gastrointestinal), renal disease, decreased secretion of aldosterone, inadequate sodium intake, NPO, low salt diet, excessive ingestion of hypotonic fluids or irragation with hypotonic fluids
renal failure, freshwater drowning, syndrome of inappropriate antidiuretic hormone secretion
hyperglycemia, and congestive heart failure

==================================================

Question: Hyponatremia signs and symptoms

Answer: change as vascular volume change
Normovolemic: Rapid pulse rate, normal blood pressure
Hypovolemic: Thready, weak, rapid pulse rate, hypotension, flat neck and hand veins, normal or low central venous pressure
Hypervolemic: Rapid bounding pulse, blood pressure normal to elevated. normal to elevated central venous pressure

==================================================

Question: Hyponatremia signs and symptoms
Respiratory

Answer: shallow, ineffective respiratory movements as a late manifestation related to skeletal muscle weakness

==================================================

Question: Hyponatremia neurmuscular

Answer: generalized muscle weakness that is worse in the extremities
Diminished deep tendon reflexes

==================================================

Question: Hyponatremia Cereberal function

Answer: headache, personality changes, confusion, seizures, coma

==================================================

Question: Hyponatremia gastrointestinal

Answer: increased motility and hyperactive bowel sounds
nausea
abdominal cramping and diarrhea

==================================================

Question: Hyponatremia Renal

Answer: decreased urinary specific gravity
increased urinary output

==================================================

Question: Hyponatremia Interventions

Answer: Monitor cardiovascular, respiratory, neuromuscular, cerebral, renal, and gastrointestinal status
hyponatremia/hypovolemia: IV sodium chloride infusion
hyponatremia/hypervolemia: osmotic diuretics
if caused by inappropriate or excessive secretion of antidiuretic hormone, Use lithium or Demeclocycline
Instruct to take in more sodium
Monitor lithium levels for toxicity

==================================================

Question: Hypernatremia

Answer: levels above 145
caused by
corticosteroids
cushing syndrome, renal failure, hyperaldosteronism
Excessive oral sodium, excessive IV infusion, Decreased water intake, increased water loss,increased metabolism, fever, hyperventilation, infection, excessive diaphoresis, watery dirrahea, diabetes insipids

==================================================

Question: Hypernatremia signs and symptoms
Cardiovascular

Answer: heart rate and blood pressure responde to vascular volume status

==================================================

Question: Hypernatremia Respiratory

Answer: Pulmonary edema if hypervolemia present

==================================================

Question: hypernatremia neuromuscular

Answer: Early: spontaneous muscle twitches; irregular muscle contractions
Late: skeletal muscle weakness, deep tendon reflex diminished or absent

==================================================

Question: Hypernatremia central nervous system

Answer: altered cerebral function is most common manifestations
Normovolemia and hypovolemia agitation, confusion, seizures
hypovolemia lethargy, stupor, coma

==================================================

Question: hypernatremia renal

Answer: increased urinary specific gravity
decreased urinary ouput

==================================================

Question: hypernatremia integumentary

Answer: dry skin
Presence of absence of edema, depending on fluid volume changes

==================================================

Question: hypernatremia Interventions

Answer: monitor cardiovascular, respiratory, neuromuscular, cerebral, renal, and integumentary status.
Adminsiter IV if caused by fluid loss
if caused by inadequate renal excreation of sodium, administer diurectics that promote sodium loss
restrict sodium intake as prescribed

==================================================

Question: hypokalemaia normal Potassium values 3.5 to 5.1 mEq/L

Answer: total body loss of potassium
caused by: excessive use of medications such as diuretics or corticosteroids
Increased secretion of aldosterone, such as in cushing's syndrome
vomiting diarrhea
wound drainage, particularly gastronintestional
prolonged nasogastric suction
excessive diaphrosis
renal disease impairing reabsorption of potassium
inadequate intake NPO
extracellular fluid to intracellular fluid
alkalosis
hyperinsulinism
silution of serum potassium due to water intoxication
IV therapy with poor potassium solution

==================================================

Question: hypokalemia cardiovascular

Answer: assessment
thready, weak, irregular pulse
peripheral pulses weak
orthostatic hypotension
ST depression, shallow flat or inverted T wave, and prominent u wave

==================================================

Question: hypokalemia respiratory

Answer: shallow, ineffective respirations that result from profound weakness of the skeletal nuscles of respiration
Diminished breath sounds

==================================================

Question: hypokalemis neuromuscular

Answer: anxiety, lethargy, confussion, coma
skeletal muscle weakness, eventual flaccid paralysis
loss of tactile discrimination
deep tendon hyporeflex

==================================================

Question: hypokalemia gastrointestinal

Answer: decreased motility, hypoactive to absent bowel sounds
nausea, vomiting, constipation, abdominal distention
paralytic ileus

==================================================

Question: hypokalemia renal

Answer: decreased urinary specific gravity
increased urinary output

==================================================

Question: hypokalemia interventions

Answer: monitor cardiovascular, respiratory, neuromuscular, gastrointestinal, and renal status, and place on a cardiac monitor
monitor electrolytes values
administer potassium supplements not on empty stomach
may need to be discontinued if client complains of abdominal pain, distention, nausea, vomiting, diarrhea, or gastrointestional bleeding.
Liquid potassium chloride has un unpleasant taste and should be taken with juice or another liquid

==================================================

Question: hypokalemia interventions

Answer: potassium never given as IV push or intramuscular or subcutaneous route.
1mEq/ per 10mL of solution is recommended
after adding to IV bag, invert bag to distributed evenly throughout IV solution/hr. never to exceed 20 mEq/hr.
label IV bag properly
maxium infusion rate is 5 to 10 mEq
client receiving more than 10 mEq/hr should be placed on cardiac monitor and monitored during the entire infusion and controlled by an infusion pump

==================================================

Question: hypokalemia interventions

Answer: phlebitis and infiltration precaution: inspection of IV site and stopped immediately if occurs
renal function before administering potassium and monitor intake and output during administration
safety measures for muscle weakness
prescribed potassium sparing diuretic may need to replace potassium losing diuretic
Instruct client about foods high in potassium content
avocado,bananas,cantalope,carrots,fish,mushrooms,
oranges, potatoes, pork, beef, veal, raisins, spinach, strawberries, tomatoes

==================================================

Question: Hyperkalemia

Answer: caused by excessive potassium intake
overingestion of foods, medications, salt substitutes
rapid infusion of potassium containing solutions
decreased potassium excretion
potassium sparing diuretics
renal failure
adrenal insuffiency, (addison's disease)
intracellular fluid in to extracellular fluid
Tissue damage
acidosis
hyperuricemia
hypercatabolism

==================================================

Question: hyperkalemia cardiovascular

Answer: slow, weak, irregular heart rate
decreased blood pressure
Tall peaked T waves, flat P waves, widened QRS complex and prolonged PR intervals

==================================================

Question: hyperkalemia respiratory

Answer: profound weakness of the skeletal muscles leading to respiratory failure

==================================================

Question: hyperkalemia neuromuscular

Answer: Early: muscle twitches, cramps, paresthesias (tingling and burning followed by numbness in the hands and feet and around the mouth)
Late: Profound weakness, ascending flaccid paralysis in the arms and legs (trunk, head, and respiratory muscles become affected when the serum potassium level reaches a lethal level)

==================================================

Question: hyperkalemia Gastrointestinal

Answer: increased motility, hyperactive bowel sounds
Diarrhea

==================================================

Question: hyperkalemia Interventions

Answer: monitor cardiovascular, respiratory, neuromuscular, renal, and gastrointestional status; place on cardiac monitor
Discontinue IV and hold oral potassium supplements
potassium restricted diet
administer potassium excreting diurects if renal function is not impaired

==================================================

Question: hyperkalemia Interventions

Answer: if renal function impaired then administer Kayexalate a cation exchange resin that promotes gastrointestional sodium absorption and potassium excretion
dialysis if potassium level is critically high
IV administration of hypertonic glucose with regular insulin to move excess potassium into the cells
monitor renal function

==================================================

Question: hyperkalemia Interventions

Answer: Blood transfusion should be fresh blood, if possible, stored blood may elevate potassium level, breakdown of old blood causes potassium release
avoid potassium foods
avoid use of salt substitutes

==================================================

Question: hypocalcemia normal values 8.6 to 10mg/dL

Answer: causes: inhibition of calcium absorption from the gastrointestinal tract
inadequate oral intake, lactose intolerance
malabsorption syndromes such as celiac sprue or Crohn's disease
inadequate intake of vitamin D
end stage renal disease
increased calcium excretion
renal failure
diarrhea, steatorrhea, wound drainage, gastrointestional

==================================================

Question: hypocalcemia

Answer: decrease in ionized fraction of calcium
hyperproteinemia,alkalosis, calcium binders chelators
acute pancreatitis, hyperphospatemia, immobility
Removal or destruction of parathyroid glands

==================================================

Question: hypocalcemia cardiac

Answer: decreased heart rate
hypotension, diminished peripheral pulses
Prolonged ST interval, prolonged QT interval

==================================================

Question: hypocalcemia respiratory

Answer: not directly affected but, respiratory failure and arrest may result from decreased respiratory movement because of muscle tetany or seizures

==================================================

Question: hypocalcemia neuromuscular

Answer: irritable skeletal muscles Twitches, cramps, tetany, seizures
painful muscles spasms in the calf or foot during periods of inactivity
paresthesias followed by numbness that may affect the lips, nose, and ears in addition to the limbs
Positive Trousseau's and Chvostek's signs
hyperactive deep tendon relexes
anxiety, irritability

==================================================

Question: hypocalcemia gastrointestional

Answer: increased gastric motility; hyperactive bowel sounds
abdominal cramping, diarrhea

==================================================

Question: hypocalcemia Interventions

Answer: monitor cardiovascular, respiratory, neuromuscular, and gastrointestional status; place the client on a cardiac monitor
administer calcium supplements orally or calcium intravenously
Warm injection to body temperature before administration and administer slowly; monitor for ECG changes, observe for infiltration, and monitor for hypercalcemia
administer medications that increase absorption of calcium

==================================================

Question: hypocalcemia Interventions

Answer: aluminum hydroxide reduces serum phosphorus levels, causing the countereffect of increasing calcium levels.
Vitamin D aids in the absorption of calcium from the intestinal tract
Provide quiet environment to reduce environmental stimuli
initiate seizure precautions
move the client carefully, and monitor for signs of a fracture
Keep 10% calcium gluconate available for treatment of acute calcium deficit
instruct client to consume foods high in calcium
Cheese, collard greens, milk and soy milk, Rhubarb, sardines, spinach, tofu, yogurt

==================================================

Question: hypercalcemia

Answer: increased calcium absorption
excessive oral intake
excessive oral intake of vitamin D
decreased calcium excretion
renal failure
use of thiazide diuretics
Increased bone resorption of calcium
hyperparathyroidism
hyperthyroidism, Malignancy (bone destruction from metastatic tumors)
immobility, use of glucocorticoids
Hemoconcentration
Dehydration, use of lithium, adrenal insufficiency

==================================================

Question: hypercalcemia cardiovascular

Answer: increased heart rate in early phase, bradycardia that can lead to cardiac arrest in the late phase
increased blood pressure
bounding, full peripheral pulses
ECG Shortened ST segment, widened T wave

==================================================

Question: hypercalcemia respiratory

Answer: ineffective respriatory movement as a result of profound skeletal muscle weakness

==================================================

Question: hypercalcemia neuromuscular

Answer: profound muscle weakness
diminished or absent deep tendon reflexes
disorientation, lethargy, coma

==================================================

Question: hypercalcemia renal

Answer: increased urinary output leading to dehyfreation
formation of renal calculi

==================================================

Question: hypercalcemia gastrointestional

Answer: decreased motility and hypoactive bowel sounds
anorexia, nausea, abdominal distention constipation

==================================================

Question: hypercalcemia intervention

Answer: monitor cardiovascular, respiratory, neuromuscular, renal, and gastrointestional status; place on cardiac monitor
Discontinue IV infusions of solutions containing calcium and oral medications containing calcium and vitamin D
discontinue thiazide diuretics and replace with diurectics that enhance the excretion of calcium
administer medications as prescribed that inhibit calcium resorption from the bone, such a phosphorus, calcitonin (Calcimar), bisphosphonates, and prostaglandin synthesis inhibitors (aspirin, nonsteroidal antiinglammatory drugs)

==================================================

Question: hypercalcemia

Answer: prepare the client with severe hypercalcemia for dialysis if medications fail to reduce the serum calcium level
Move carefully and monitor for signs of fracture
monitor for flank or abdominal pain, and strain the urine to check for the presence of urinary stones.
Instruct the client to avoid foods high in calcium
cheese
collard greens, milk and soy milk, rhubarb, sardines, spinach, tofu, yogurt

==================================================

Question: hypomagnesemia normal values 1.6 to 2.6mg/dL

Answer: casuses increased magnesium intake
antacids and laxatives, Excessive IV infusion
Decreased renal excretion as a result of renal insufficiency

==================================================

Question: hypomagnesemia cardiovascular

Answer: Bradycardia, dysrhythmias
Hypotension Prolonged PR interval, widened QRS complexes

==================================================

Question: hypomagnesemia respiratory

Answer: insufficiency with skeletal muscles of respiration are involved.

==================================================

Question: hypomagnesemia neuromuscular

Answer: diminished or absent deep tendon reflexes
skeletal muscle weakness

==================================================

Question: hypomagnesemia central nervous system

Answer: Drowsiness and lethargy that progresses to coma

==================================================

Question: hypomagnesemia Interventions

Answer: IV administration of calcium chloride or calcium gluconate to reverse the effects of magnesium on cardiac muscle.
restrict dietary intake of magnesium
avoid use of laxatives and antacids.

==================================================

Question: hypophosphatemia normal values 2.7 to 4.5mg/dL

Answer: decrease in phosphorus and increase in serum calcium
causes: malnutrition and starvation
increased excretion by hyperparathyroidism, malignancey and use of aluminum hydroxide-based or magnesium based antacids
Intracellular shifts
hyperglycemia, respiratory alkalosis

==================================================

Question: hypophosphatemia cardiovascular

Answer: decreased contractility and cardiac output
slowed peripheral pulses

==================================================

Question: hypophosphatemia respiratory

Answer: shallow respirations

==================================================

Question: hypophosphatemia neuromuscular

Answer: weakness, decreased deep tendon reflexes, Decreased bone density that can result in contractures and alterations in bone shape, Rhabdomyolysis

==================================================

Question: hypophosphatemia central nervous system

Answer: irritability, confusion, seizures

==================================================

Question: hypophosphatemia hematological

Answer: decreased platelet aggregation and increased bleeding
immunosuppression

==================================================

Question: hypophosphatemia interventions

Answer: monitor cardiovascular, respiratory, neuromuscular, central nervous system, and hematological status.
discontinue meds that contribute to hypophosphatemia
administer oral phosphorus with vitamin D
Intravenously administer phosphorus when levels fall below 1mg/dL and critical clinical manifestations

==================================================

Question: hypophosphatemia interventions

Answer: adminster IV phosphorus slowly because of risks associated with hyperphosphatemia
assess renal system before administering
move client carefully, and monitor for signs of fracture
instruct client to increase intake of phosphorus containg foods while decreasing the intake of calcium containg foods.

==================================================

Question: hypophosphatemia interventions

Answer: Increase:
Fish, organ meats, nuts, pork, beef, chicken, whole grain breads and cereals.
decrease: cheese, collard greens, milk and soy milk,rhubarb, sardines,spinach, tofu, yogurt.

==================================================

Question: Hyperohosphatemia

Answer: body tolerates well
increase in phosphorus accompanied by a decreased
serum calcium level. Problems occur in hypocalcemia

==================================================

Question: Hyperohosphatemia

Answer: causes: decreased renal excretion resulting from renal insufficiency
Tumor lysis syndrome
Increased intake dietary intake and or overuse of phosphate containing laxatives or enemas
hypoparathyroidism

==================================================

Question: Hyperphosphatemia cardiac

Answer: hypocalcemia assessmentdecreased heart rate
hypotension, diminished peripheral pulses
Prolonged ST interval, prolonged QT interval

==================================================

Question: hyperphosphatemia respiratory

Answer: not directly affected but, respiratory failure and arrest may result from decreased respiratory movement because of muscle tetany or seizures

==================================================

Question: hyperphosphatemia neuromuscular

Answer: irritable skeletal muscles Twitches, cramps, tetany, seizures
painful muscles spasms in the calf or foot during periods of inactivity
paresthesias followed by numbness that may affect the lips, nose, and ears in addition to the limbs
Positive Trousseau's and Chvostek's signs
hyperactive deep tendon relexes
anxiety, irritability

==================================================

Question: hyperphosphatemia gastrointestional

Answer: increased gastric motility; hyperactive bowel sounds
abdominal cramping, diarrhea

==================================================

Question: hyperphosphatemia Interventions

Answer: management of hypocalcemia
administer phosphate binding medications that increase fecal excretion of phosphorus by binding phosphorus from food in the gastrointestional tract.
avoid phosphate containing medications, laxatives and enemas

==================================================

Question: hyperphosphatemia Interventions

Answer: decrease intake of food high in phosphorus
Fish, organ meats, nuts, pork, beef, chicken, whole grain breads and cereals
instruct client in medication administration: taking phosphate binding medications, emphasizing that they should be taken with meals or immediately after meals.

==================================================

Nclex Questions On Oncology

Question: A 32-year-old woman meets with the nurse on her first official visit since undergoing a left mastectomy. When asked how she is doing, the woman states her appetite is still not good, she is not getting much sleep because she doesn't go to bed until her husband is asleep, and she is really anxious to get back to work. Which of the following nursing interventions should the nurse explore to support the client's current needs?
a) Ask open-ended questions about sexuality issues related to her mastectomy
b) Suggest that the client learn relaxation techniques to help with her insomnia
c) Call the physician to discuss allowing the client to return to work earlier
d) Perform a nutritional assessment to assess for anorexia

Answer: a) Ask open-ended questions about sexuality issues related to her mastectomy
- Correct Answer: A. Ask open-ended questions about sexuality issues related to her mastectomy
Option A: The content of the client's comments suggests that she is avoiding intimacy with her husband by waiting until he is asleep before going to bed. Addressing sexuality issues is appropriate for a client who has undergone a mastectomy.
Option B: Suggesting that she learn relaxation techniques to help her with her insomnia is appropriate; however, the nurse must first address the psychosocial and sexual issues that are contributing to her sleeping difficulties.
Option C: Rushing her return to work may debilitate her and add to her exhaustion.
Option D: A nutritional assessment may be useful, but there is no indication that she has anorexia.

==================================================

Question: One of the most serious blood coagulation complications for individuals with cancer and for those undergoing cancer treatments is disseminated intravascular coagulation (DIC). The most common cause of this bleeding disorder is:
a) Brain metastasis
b) Sepsis
c) Intravenous heparin therapy
d) underlying liver disease

Answer: b) Sepsis
- Correct Answer: B. Sepsis
Option B: Bacterial endotoxins released from gram-negative bacteria activate the Hageman factor or coagulation factor XII. This factor inhibits coagulation via the intrinsic pathway of homeostasis, as well as stimulating fibrinolysis.
Option D: Liver disease can cause multiple bleeding abnormalities resulting in chronic, subclinical DIC; however, sepsis is the most common cause.

==================================================

Question: A pneumonectomy is a surgical procedure sometimes indicated for the treatment of non-small-cell lung cancer. A pneumonectomy involves removal of:
a) One lobe of a lung
b) An entire lung field
c) One or more segments of a lung lobe
d) A small, wedge-shaped lung surface

Answer: b) An entire lung field
- Correct Answer: B. An entire lung field
Option B: A pneumonectomy is the removal of an entire lung field indicated for the treatment of non-small cell lung cancer that has not spread outside of the lung tissue. It is performed on patients who will have adequate lung function in the unaffected lung.
Option D: A wedge resection refers to the removal of a wedge-shaped section of lung tissue. It may be used to remove a tumor and a small amount of normal tissue around it/
Option A: A lobectomy is the removal of one lobe.
Option C: Removal of one or more segments of a lung lobe is called a partial lobectomy.

==================================================

Question: A 36-year-old man with lymphoma presents with signs of impending septic shock 9 days after chemotherapy. The nurse would expect which of the following to be present?
a) low-grade fever, chills, tachycardia
b) Elevated temperature, oliguria, hypotension
c) Flushing, decreased oxygen saturation, mild hypotension
d) High-grade fever, normal blood pressure, increased respirations

Answer: a) low-grade fever, chills, tachycardia
- Correct Answer: A. Low-grade fever, chills, tachycardia
Option A: Nine days after chemotherapy, one would expect the client to be immunocompromised. The clinical signs of shock reflect changes in cardiac function, vascular resistance, cellular metabolism, and capillary permeability. Low-grade fever, tachycardia, and flushing may be early signs of shock.
Option B: Oliguria and hypotension are late signs of shock. Urine output can be initially normal or increased.
Options C and D: The client with impending signs of septic shock may not have decreased oxygen saturation levels and normal blood pressure.

==================================================

Question: Which of the following represents the most appropriate nursing intervention for a client with pruritus caused by cancer or the treatments?
a) Silk sheets
b) Steroids
c) Medicated cool baths
d) Administration of antihistamines

Answer: c) Medicated cool baths
- Correct Answer: C. Medicated cool baths
Option C: Nursing interventions to decrease the discomfort of pruritus include those that prevent vasodilation, decrease anxiety, and maintain skin integrity and hydration. Medicated baths with salicylic acid or colloidal oatmeal can be soothing as a temporary relief.
Option A: Using silk sheets is not a practical intervention for the hospitalized client with pruritus.
Options B and D: The use of antihistamines or topical steroids depends on the cause of pruritus, and these agents should be used with caution.

==================================================

Question: A 56-year-old woman is currently receiving radiation therapy to the chest wall for recurrent breast cancer. She calls her health care provider to report that she has pain while swallowing and burning and tightness in her chest. Which of the following complications of radiation therapy is A. Radiation enteritis likely responsible for her symptoms?
a) Radiation enteritis
b) Stomatitis
c) Esophagitis
d) Hiatal hernia

Answer: c) Esophagitis
- Correct Answer: C. Esophagitis
Option C: Difficulty in swallowing, pain, and tightness in the chest are signs of esophagitis, which is a common complication of radiation therapy of the chest wall.
Option A: Radiation enteritis is a damage to the intestinal lining caused by radiation therapy. Symptoms include diarrhea, rectal pain, and bleeding or mucus from the rectum.
Option B: Stomatitis results from the local effects of radiation to the oral mucosa. Symptoms include mouth ulcers, red patches, swelling, and oral dysaesthesia.
Option D: Hiatal hernia may also cause symptoms of dysphagia and chest pain but is not related to radiation therapy.

==================================================

Question: A male client has an abnormal result on a Papanicolaou test. After admitting, he read his chart while the nurse was out of the room, the client asked what dysplasia means. Which definition should the nurse provide?
a) Alteration in the size, shape, and organization of differentiated cells
b) Increase in the number of normal cells in a normal arrangement in a tissue or an organ
c) Presence of completely undifferentiated tumor cells that don't resemble cells of the tissues of their origin
d) Replacement of one type of fully differentiated cell by another in tissues where the second type normally isn't found

Answer: a) Alteration in the size, shape, and organization of differentiated cells
- orrect Answer: A. Alteration in the size, shape, and organization of differentiated cells
Option A: Dysplasia refers to an alteration in the size, shape, and organization of differentiated cells.
Option B: An increase in the number of normal cells in a normal arrangement in a tissue or an organ is called hyperplasia.
Option C: The presence of completely undifferentiated tumor cells that don't resemble cells of the tissues of their origin is called anaplasia.
Option D: Replacement of one type of fully differentiated cell by another in tissues where the second type normally isn't found is called metaplasia.

==================================================

Question: For a female client with newly diagnosed cancer, the nurse formulates a nursing diagnosis of anxiety related to the threat of death secondary to a cancer diagnosis. Which expected outcome would be appropriate for this client?
a) "Client stops seeking information."
b) "Client uses any effective method to reduce tension."
c) "Client doesn't guess at prognosis."
d) "Client verbalizes feeling of anxiety."

Answer: d) "Client verbalizes feeling of anxiety."
- Correct Answer: D. "Client verbalizes feelings of anxiety."
Option D: Verbalizing feelings is the client's first step in coping with the situational crisis. It also helps the health care team gain insight into the client's feelings, helping guide psychosocial care.
Option A: Seeking information can help a client with cancer gain a sense of control over the crisis.
Option B: This is undesirable because some methods of reducing tension, such as illicit drug or alcohol use, may prevent the client from coming to terms with the threat of death as well as cause physiological harm.
Option C: Suppressing speculation may prevent the client from coming to terms with the crisis and planning accordingly.

==================================================

Question: A male client with a cerebellar brain tumor is admitted to an acute care facility. The nurse formulates a nursing diagnosis of Risk for injury. Which "related-to" phrase should the nurse add to complete the nursing diagnosis statement?
a) Related to psychomotor seizures
b) Related to impaired balance
c) Related to visual field deficits
d) Related to difficulty swallowing

Answer: b) Related to impaired balance
- Correct Answer: B. Related to impaired balance
Option B: A client with a cerebellar brain tumor may suffer injury from impaired balance as well as disturbed gait and incoordination.
Option A: Psychomotor seizures suggest temporal lobe dysfunction.
Option C: Visual field deficits, difficulty swallowing, and psychomotor seizures may result from dysfunction of the pituitary gland, pons, occipital lobe, parietal lobe, or temporal lobe — not from a cerebellar brain tumor.
Option D: Difficulty swallowing suggests medullary dysfunction.

==================================================

Question: A female client with cancer is scheduled for radiation therapy. The nurse knows that radiation at any treatment site may cause a certain adverse effect. Therefore, the nurse should prepare the client to expect:
a) Fatigue
b) Vomiting
c) Hair loss
d) Stomatitis

Answer: a) Fatigue
- Correct Answer: A. Fatigue
Option A: Radiation therapy may cause fatigue, skin toxicities, and anorexia regardless of the treatment site. Fatigue occurs when the treatment damages and destroys not only the healthy cells but also the cancer cells.
Options B, C, and D: Hair loss, stomatitis, and vomiting are site-specific, not generalized, adverse effects of radiation therapy.

==================================================

Question: Nurse April is teaching a client who suspects that she has a lump in her breast. The nurse instructs the client that a diagnosis of breast cancer is confirmed by:
a) Breast self-examination
b) Mammography
c) Fine needle aspiration
d) chest x-ray

Answer: c) Fine needle aspiration
- Correct Answer: C. Fine needle aspiration
Option C: Fine needle aspiration and biopsy provide cells for histologic examination to confirm a diagnosis of cancer. During the procedure, a needle is inserted into the lump and a sample of tissue is taken for examination.
Option A: A breast self-examination, if done regularly, is the most reliable method for detecting breast lumps early.
Option B: Mammography is used to detect tumors that are too small to palpate.
Option D: Chest X-rays can be used to pinpoint rib metastasis.

==================================================

Question: A male client undergoes a laryngectomy to treat laryngeal cancer. When teaching the client how to care for the neck stoma, the nurse should include which instruction?
a) "Keep the stoma dry."
b) "Keep the stoma moist."
c) "Keep the stoma uncovered."
d) "Have a family member perform stoma care initially until you get used to the procedure

Answer: b) "Keep the stoma moist."
- Correct Answer: B. "Keep the stoma moist."
Option B: The nurse should instruct the client to keep the stoma moist, such as by applying a thin layer of petroleum jelly around the edges, because a dry stoma may become irritated.
Option A: Moisture is needed by the stoma to keep the airway moist. The skin around the stoma is kept clean and dry instead.
Option C: The nurse should recommend placing a stoma bib over the stoma to filter and warm air before it enters the stoma.
Option D: The client should begin performing stoma care without assistance as soon as possible to gain independence in self-care activities.

==================================================

Question: A female client is receiving chemotherapy to treat breast cancer. Which assessment finding indicates a fluid and electrolyte imbalance induced by chemotherapy?
a) Serum potassium level of 3.6 mEq/L
b) Blood pressure of 120/64 to 130/72 mmHg
c) Dry oral mucous membranes and cracked lips
d) Urine output of 400 mL in 8 hours

Answer: c) Dry oral mucous membranes and cracked lips
- Correct Answer: C. Dry oral mucous membranes and cracked lips
Option C: Chemotherapy commonly causes nausea and vomiting, which may lead to fluid and electrolyte imbalances. Signs of fluid loss include dry oral mucous membranes, cracked lips, decreased urine output (less than 40 ml/hour), abnormally low blood pressure, and a serum potassium level below 3.5 mEq/L.
Options A, B, and D: These values are within the normal limits.

==================================================

Question: Nurse April is teaching a group of women to perform breast self-examination. The nurse should explain that the purpose of performing the examination is to discover:
a) Fibrocystic masses
b) Changes from previous self-examinations
c) Areas of thickness or fullness
d) Cancerous lumps

Answer: b) Changes from previous self-examinations
- Correct Answer: B. Changes from previous self-examinations
Option B: Women are instructed to examine themselves to discover changes that have occurred in the breast.
Options A, C, and D: Only a physician can diagnose lumps that are cancerous, areas of thickness or fullness that signal the presence of a malignancy, or masses that are fibrocystic as opposed to malignant.

==================================================

Question: A client, age 41, visits the gynecologist. After examining her, the physician suspects cervical cancer. The nurse reviews the client's history for risk factors for this disease. Which history finding is a risk factor for cervical cancer?
a) Pregnancy complicated with eclampsia at age 27
b) Spontaneous abortion at age 19
c) ONset of sporadic sexual activity at age 17
d) Human papillomavirus infection at age 32

Answer: d) Human papillomavirus infection at age 32
- Correct Answer: D. Human papillomavirus infection at age 32
Option D: Like other viral and bacterial venereal infections, human papillomavirus is a risk factor for cervical cancer. Other risk factors for this disease include multiple sex partners, multiple pregnancies, long-term use of oral contraceptives and diethylstilbestrol (DES).
Options A and B: A spontaneous abortion and pregnancy complicated by eclampsia aren't risk factors for cervical cancer.
Option C: Risk factors for this disease include frequent sexual intercourse before age 16.

==================================================

Question: A female client is receiving methotrexate (Mexate), 12 g/m2 I.V., to treat osteogenic carcinoma. During methotrexate therapy, the nurse expects the client to receive which other drug to protect normal cells?
a) Tabloid (thioguanine)
b) Cytosar-U (cytarabine)
c) Wellcovorin (leucovorin or citrovorum factor or folinic acid)
d) Benemid (probenecid)

Answer: c) Wellcovorin (leucovorin or citrovorum factor or folinic acid)
- Correct Answer: C. Wellcovorin (leucovorin or citrovorum factor or folinic acid)
Option C: Leucovorin is administered with methotrexate to protect normal cells, which methotrexate could destroy if given alone.
Options A and B: Cytarabine and thioguanine aren't used to treat osteogenic carcinoma.
Option D: Probenecid should be avoided in clients receiving methotrexate because it reduces renal elimination of methotrexate, increasing the risk of methotrexate toxicity.

==================================================

Question: The nurse is interviewing a male client about his past medical history. Which preexisting condition may lead the nurse to suspect that a client has colorectal cancer?
a) polyps
b) weight gain
c) Hemorrhoids
d) Duodenal ulcers

Answer: a) Polyps
- Correct Answer: A. Polyps
Option A: Colorectal polyps are common with colon cancer. These polyps can develop into cancer over time depending on the type of polyps such as adenomatous polyps and sessile serrated polyps.
Option B: Weight loss — not gain — is an indication of colorectal cancer.
Options C and D: Duodenal ulcers and hemorrhoids aren't preexisting conditions of colorectal cancer

==================================================

Question: Nurse Amy is speaking to a group of women about early detection of breast cancer. The average age of the women in the group is 47. Following the American Cancer Society guidelines, the nurse should recommend that the women:
a) Have a mammogram annually
b) Perform breast self-exams annually
c) Have a hormonal receptor assay annually
d) Have a physician conduct a clinical exam every 2 yrs

Answer: a) Have a mammogram annually
- Correct Answer: A. Have a mammogram annually
Option A: The American Cancer Society guidelines state, "Women older than age 40 should have a mammogram annually and a clinical examination at least annually [not every 2 years].
Option B: All women should perform breast self-examination monthly [not annually].
Option C: The hormonal receptor assay is done on a known breast tumor to determine whether the tumor is estrogen- or progesterone-dependent.
Option D: A physician checkup every 2 years will not detect early signs of breast cancer

==================================================

Question: A male client with a nagging cough makes an appointment to see the physician after reading that this symptom is one of the seven warning signs of cancer. What is another warning sign of cancer?
a) rash
b) Indigestion
c) chronic ache or pain
d) Persistent nausea

Answer: b) Indigestion
- Correct Answer: B. Indigestion
Option B: Indigestion, or difficulty swallowing, is one of the seven warning signs of cancer. The other six are a change in bowel or bladder habits, a sore that does not heal, unusual bleeding or discharge, a thickening or lump in the breast or elsewhere, an obvious change in a wart or mole, and a nagging cough or hoarseness.
Options A and C: Rash and chronic ache or pain seldom indicate cancer.
Option D: Persistent nausea may signal stomach cancer but isn't one of the seven major warning signs.

==================================================

Question: For a female client newly diagnosed with radiation-induced thrombocytopenia, the nurse should include which intervention in the plan of care?
a) Inspecting the skin for petechiae once every shift
b) Placing the client in strict isolation
c) Providing for frequent rest periods
d) Administering aspirin if the temperature exceeds 102 degrees F (38.8 C)

Answer: a) Inspecting the skin for petechiae once every shift
- Correct Answer: A. Inspecting the skin for petechiae once every shift
Option A: Because thrombocytopenia impairs blood clotting, the nurse should inspect the client regularly for signs of bleeding, such as petechiae, purpura, epistaxis, and bleeding gums.
Option B: Strict isolation is indicated only for clients who have highly contagious or virulent infections that are spread by air or physical contact.
Option C: Frequent rest periods are indicated for clients with anemia, not thrombocytopenia.
Option D: The nurse should avoid administering aspirin because it may increase the risk of bleeding.

==================================================

Question: Nurse Lucia is providing breast cancer education at a community facility. The American Cancer Society recommends that women get mammograms:
a) After the first menstrual period and annually thereafter
b) Yearly after age 40
c) Every 3 years between ages 20 and 40 and annually thereafter
d) After the birth of the first child and every 2 years thereafter

Answer: b) Yearly after age 40
- Correct Answer: B. Yearly after age 40
Option B: Breast cancer is a common health problem for women ages 40-49 years old. The American Cancer Society recommends a mammogram yearly for women over age 40.
Options A, C, and D: The other statements are incorrect. It's recommended that women between ages 20 and 40 have a professional breast examination (not a mammogram) every 3 years.

==================================================

Question: Which intervention is appropriate for the nurse caring for a male client in severe pain receiving a continuous I.V. infusion of morphine?
a) Discontinuing the drug immediately if signs of dependence appear
b) Assisting with a naloxone challenge test before therapy begins
c) Obtaining baseline vital signs before administering the first dose
d) Changing the administration route to P.O. if the client can tolerate fluids

Answer: c) Obtaining baseline vital signs before administering the first dose
- Correct Answer: C. Obtaining baseline vital signs before administering the first dose
Option C: The nurse should obtain the client's baseline blood pressure and pulse and respiratory rates before administering the initial dose and then continue to monitor vital signs throughout therapy.
Option A: The nurse shouldn't discontinue a narcotic agonist abruptly because withdrawal symptoms may occur.
Option B: A naloxone challenge test may be administered before using a narcotic antagonist, not a narcotic agonist.
Option D: Morphine commonly is used as a continuous infusion in clients with severe pain regardless of the ability to tolerate fluids.

==================================================

Question: A 35 years old client with ovarian cancer is prescribed hydroxyurea (Hydrea), an antimetabolite drug. Antimetabolites are a diverse group of antineoplastic agents that interfere with various metabolic actions of the cell. The mechanism of action of antimetabolites interferes with:
a) Cell division or mitosis during the M phase of the cell cycle
b) Normal cellular processes during the S phase of the cell cycle
c) The chemical structure of deoxyribonucleic acid (DNA) and chemical binding between DNA molecules (cell cycle-nonspecific)
d) One or more stages of ribonucleic acid (RNA) synthesis, or both (cell cycle-nonspecific)

Answer: b) Normal cellular processes during the S phase of the cell cycle
- Correct Answer: B. Normal cellular processes during the S phase of the cell cycle
Option B: Antimetabolites act during the S phase of the cell cycle, contributing to cell destruction or preventing cell replication. They're most effective against rapidly proliferating cancers.
Option A: Miotic inhibitors interfere with cell division or mitosis during the M phase of the cell cycle.
Option C: Alkylating agents affect all rapidly proliferating cells by interfering with DNA; they may kill dividing cells in all phases of the cell cycle and may also kill nondividing cells.
Option D: Antineoplastic antibiotic agents interfere with one or more stages of the synthesis of RNA, DNA, or both, preventing normal cell growth and reproduction.

==================================================

Question: The ABCD method offers one way to assess skin lesions for possible skin cancer. What does the A stand for?
a) Assessment
b) Arcus
c) Actinic
d) Asymmetry

Answer: d) Asymmetry
- Correct Answer: D. Asymmetry
Option D: When following the ABCD method for assessing skin lesions, the A stands for "asymmetry," the B for "border irregularity," the C for "color variation," and the D for "diameter."

==================================================

Question: When caring for a male client diagnosed with a brain tumor of the parietal lobe, the nurse expects to assess:
a) Seizures
b) Tactile agnosia
c) Short-term memory impairment
d) Contralateral homonymous hemianopia

Answer: b) Tactile agnosia
- Correct Answer: B. Tactile agnosia
Option B: Tactile agnosia (inability to identify objects by touch) is a sign of a parietal lobe tumor.
Option A: Seizures may result from a tumor of the frontal, temporal, or occipital lobe.
Option C: Short-term memory impairment occurs with a frontal lobe tumor.
Option D: Contralateral homonymous hemianopia suggests an occipital lobe tumor.

==================================================

Question: A female client is undergoing tests for multiple myeloma. Diagnostic study findings in multiple myeloma include:
a) A decreased serum creatinine level
b) A low serum protein level
c) Hypocalcemia
d) Bence Jone protein in the urine

Answer: d) Bence Jones protein in the urine
- Correct Answer: D. Bence Jones protein in the urine
Option D: Bence-Jones protein is an antibody fragment called a light chain that is not detectable in the urine. A presence of Bence Jones may indicate excess light chain production of a single type of antibody by the bone marrow cells.
Option A: The serum creatinine level may also be increased.
Option B: Serum protein electrophoresis shows elevated globulin spike.
Option C: Serum calcium levels are elevated because calcium is lost from the bone and reabsorbed in the serum.

==================================================

Question: A 35-year-old client has been receiving chemotherapy to treat cancer. Which assessment finding suggests that the client has developed stomatitis (inflammation of the mouth)?
a) Rust-colored sputum
b) Red, open sores on the oral mucosa
c) Yellow tooth discoloration
d) White, cottage cheese-like patches on the tongue

Answer: b) Red, open sores on the oral mucosa
- Correct Answer: B. Red, open sores on the oral mucosa
Option B: The tissue-destructive effects of cancer chemotherapy typically cause stomatitis, resulting in ulcers on the oral mucosa that appear as red, open sores.
Option A: Rust-colored sputum suggests a respiratory disorder, such as pneumonia.
Option C: Yellow tooth discoloration may result from antibiotic therapy, not cancer chemotherapy.
Option D: White, cottage cheese-like patches on the tongue suggest a candidal infection, another common adverse effect of chemotherapy.

==================================================

Question: During chemotherapy, an oncology client has a nursing diagnosis of impaired oral mucous membrane related to decreased nutrition and immunosuppression secondary to the cytotoxic effects of chemotherapy. Which nursing intervention is most likely to decrease the pain of stomatitis?
a) Monitoring the client's platelet and leukocyte counts
b) Checking regularly for signs and symptoms of stomatitis
c) Recommending that the client discontinue chemotherapy
d) Providing a solution of hydrogen peroxide and water for use as a mouth rinse

Answer: d) Providing a solution of hydrogen peroxide and water for use as a mouth rinse
- Correct Answer: D. Providing a solution of hydrogen peroxide and water for use as a mouth rinse
Option D: To decrease the pain of stomatitis, the nurse should provide a solution of hydrogen peroxide and water for the client to use as a mouth rinse. (Commercially prepared mouthwashes contain alcohol and may cause dryness and irritation of the oral mucosa.) The nurse also may administer viscous lidocaine or systemic analgesics as prescribed.
Option A: Monitoring platelet and leukocyte counts may help prevent bleeding and infection but wouldn't decrease pain in this highly susceptible client.
Option B: Checking for signs and symptoms of stomatitis also wouldn't decrease the pain.
Option C: Stomatitis occurs 7 to 10 days after chemotherapy begins; thus, stopping chemotherapy wouldn't be helpful or practical. Instead, the nurse should stay alert for this potential problem to ensure prompt treatment.

==================================================

Question: What should a male client over age 52 do to help ensure early identification of prostate cancer?
a) Have a transrectal ultrasound every 5 years
b) Perform monthly testicular self-examinations, especially after age 50
c) Have a digital rectal examination and prostate-specific antigen (PSA) test done yearly
d) Have a CBC and BUN and creatinine levels checked yearly

Answer: c) Have a digital rectal examination and prostate-specific antigen (PSA) test done yearly
- Correct Answer: C. Have a digital rectal examination and prostate-specific antigen (PSA) test done yearly
Option C: The incidence of prostate cancer increases after age 50. The digital rectal examination, which identifies enlargement or irregularity of the prostate, and PSA test, a tumor marker for prostate cancer, are effective diagnostic measures that should be done yearly.
Options A and D: A transrectal ultrasound, CBC, and BUN and creatinine levels are usually done after diagnosis to identify the extent of the disease and potential metastases.
Option B: Testicular self-examinations won't identify changes in the prostate gland due to its location in the body.

==================================================

Question: A male client complains of sporadic epigastric pain, yellow skin, nausea, vomiting, weight loss, and fatigue. Suspecting gallbladder disease, the physician orders a diagnostic workup, which reveals gallbladder cancer. Which nursing diagnosis may be appropriate for this client?
a) Chronic low self-esteem
b) Disturbed body image
c) Anticipatory grieving
d) Impaired swallowing

Answer: c) Anticipatory grieving
- Correct Answer: C. Anticipatory grieving
Option C: Anticipatory grieving is an appropriate nursing diagnosis for this client because few clients with gallbladder cancer live more than 1 year after diagnosis.
Option A: Chronic low self-esteem isn't an appropriate nursing diagnosis at this time because the diagnosis has just been made.
Option B: Although surgery typically is done to remove the gallbladder and, possibly, a section of the liver, it isn't disfiguring and doesn't cause Disturbed body image.
Option D: Impaired swallowing isn't associated with gallbladder cancer.

==================================================

Question: A male client is in isolation after receiving an internal radioactive implant to treat cancer. Two hours later, the nurse discovers the implant in the bed linens. What should the nurse do first?
a) Leave the room and notify the radiation therapy department immediately
b) Put the implant back in place, using forceps and a shield for self-protection, and call for help
c) Pick up the implant with long-handled forceps and place it in a lead-lined container
d) Stand as far away from the implant as possible and call for help

Answer: c) Pick up the implant with long-handled forceps and place it in a lead-lined container
- Correct Answer: C. Pick up the implant with long-handled forceps and place it in a lead-lined container
Option C: If a radioactive implant becomes dislodged, the nurse should pick it up with long-handled forceps and place it in a lead-lined container, then notify the radiation therapy department immediately. The highest priority is to minimize radiation exposure for the client and the nurse; therefore, the nurse must not take any action that delays implant removal.
Options A, B, and D: Standing as far from the implant as possible, leaving the room with the implant still exposed, or attempting to put it back in place can greatly increase the risk of harm to the client and the nurse from excessive radiation exposure.

==================================================

Question: Jenny with an advanced breast cancer is prescribed Nolvadex (tamoxifen). When teaching the client about this drug, the nurse should emphasize the importance of reporting which adverse reaction immediately?
a) Anorexia
b) Headache
c) Hearing loss
d) Vision changes

Answer: d) Vision changes
- Correct Answer: D. Vision changes
Option D: Tamoxifen, a selective estrogen receptor modulator (SERM) causes ocular side effects such as dryness, irritation, and cataracts. The client must report changes in visual acuity immediately because this adverse effect may be irreversible.
Options A and B: Although the drug may cause anorexia, headache, and hot flashes, the client need not report these adverse effects immediately because they don't warrant a change in therapy.
Option C: Tamoxifen isn't associated with hearing loss.

==================================================

Question: A female client with cancer is being evaluated for possible metastasis. Which of the following is one of the most common metastasis sites for cancer cells?
a) Colon
b) Liver
c) Reproductive tract
d) White blood cells (WBCs)

Answer: b) Liver
- Correct Answer: B. Liver
Option B: The liver is one of the five most common cancer metastasis sites. The others are the lymph nodes, lung, bone, and brain.
Options A, C, and D: The colon, reproductive tract, and WBCs are occasional metastasis sites.

==================================================

Question: A 34-year-old female client is requesting information about mammograms and breast cancer. She isn't considered at high risk for breast cancer. What should the nurse tell this client?
a) She should have had a baseline mammogram before age 30
b) When she begins having yearly mammograms, breast self-exams will no longer be necessary
c) She should perform breast self-exam during the first 5 days of each menstrual cycle
d) She should eat a low-fat diet to further decrease her risk of breast cancer

Answer: d) She should eat a low-fat diet to further decrease her risk of breast cancer
- Correct Answer: D. She should eat a low-fat diet to further decrease her risk of breast cancer
Option D: A low-fat diet (one that maintains weight within 20% of recommended body weight) has been found to decrease a woman's risk of breast cancer.
Option A: A baseline mammogram should be done between ages 30 and 40.
Option B: The client should continue to perform monthly breast self-examinations even when receiving yearly mammograms.
Option C: Monthly breast self-examinations should be done between days 7 and 10 of the menstrual cycle.

==================================================

Question: Nurse Brian is developing a plan of care for marrow suppression, the major dose-limiting adverse reaction to floxuridine (FUDR). How long after drug administration does bone marrow suppression become noticeable?
a) 24 hours
b) 2 to 4 days
c) 7 to 14 days
d) 21 to 28 days

Answer: c) 7 to 14 days
- Correct Answer: C. 7 to 14 days
Bone marrow suppression becomes noticeable 7 to 14 days after floxuridine administration. Bone marrow recovery occurs in 21 to 28 days.

==================================================

Question: The nurse is preparing for a female client for magnetic resonance imaging (MRI) to confirm or rule out a spinal cord lesion. During the MRI scan, which of the following would pose a threat to the client?
a) The client lies still
b) The client asks questions
c) The client hears thumping sounds
d) The client wears a watch and wedding band

Answer: d) The client wears a watch and wedding band
- Correct Answer: D. The client wears a watch and wedding band
Option D: During an MRI, the client should wear no metal objects, such as jewelry, because the strong magnetic field can pull on them, causing injury to the client and (if they fly off) to others.
Options A and B: The client must lie still during the MRI but can talk to those performing the test by way of the microphone inside the scanner tunnel.
Option C: The client should hear thumping sounds, which are caused by the sound waves thumping on the magnetic field.

==================================================

Question: Nina, an oncology nurse educator, is speaking to a women's group about breast cancer. Questions and comments from the audience reveal a misunderstanding of some aspects of the disease. Various members of the audience have made all of the following statements. Which one is accurate?
a) Breast cancer requires a mastectomy
b) Men can develop breast cancer
c) Breast cancer is the leading killer of women of childbearing age
d) Mammography is the most reliable method for detecting breast cancer

Answer: b) Men can develop breast cancer
- Correct Answer: B. Men can develop breast cancer
Option B: Men can develop breast cancer, although they seldom do. It is common among older men.
Option A: A mastectomy may not be required if the tumor is small, confined, and in an early stage.
Option C: Lung cancer causes more deaths than breast cancer in women of all ages.
Option D: The most reliable method for detecting breast cancer is monthly self-examination, not mammography.

==================================================

Question: Nurse Mary is instructing a premenopausal woman about breast self-examination. The nurse should tell the client to do her self-examination:
a) On the 1st day of the menstrual cycle
b) On the same day each month
c) Immediately after her menstrual period
d) At the end of her menstrual cycle

Answer: c) Immediately after her menstrual period
- Correct Answer: C. Immediately after her menstrual period
Option C: Premenopausal women should do their self-examination immediately after the menstrual period, when the breasts are least tender and least lumpy.
Options A and D: On the 1st and last days of the cycle, the woman's breasts are still very tender.
Option B: Postmenopausal women because their bodies lack fluctuation of hormone levels, should select one particular day of the month to do breast self-examination.

==================================================

Question: Nurse Kent is teaching a male client to perform monthly testicular self-examinations. Which of the following points would be appropriate to make?
a) Testicular cancer is a highly curable type of cancer
b) Testicular cancer is very difficult to diagnose
c) Testicular cancer is the number one cause of cancer deaths in males
d) Testicular cancer is more common in older men

Answer: a) Testicular cancer is a highly curable type of cancer
- Correct Answer: A. Testicular cancer is a highly curable type of cancer
Option A: Testicular cancer is highly curable, particularly when it's treated in its early stage. Stage I of the disease, a radical inguinal orchiectomy (removal of testicles) is performed first then followed by chemotherapy or radiation therapy.
Option B: Self-examination allows early detection and facilitates the early initiation of treatment.
Option C: The highest mortality rates from cancer among men are in men with lung cancer.
Option D: Testicular cancer is found more commonly in younger men.

==================================================

Question: Rhea has malignant lymphoma. As part of her chemotherapy, the physician prescribes chlorambucil (Leukeran), 10 mg by mouth daily. When caring for the client, the nurse teaches her about adverse reactions to chlorambucil, such as alopecia. How soon after the first administration of chlorambucil might this reaction occur?
a) Immediately
b) 1 week
c) 2 to 3 weeks
d) 1 month

Answer: c) 2 to 3 weeks
- Correct Answer: C. 2 to 3 weeks
Chlorambucil-induced alopecia occurs 2 to 3 weeks after therapy begins. The medication causes structural damage to the scalp hairs resulting in reduced hair growth and complete hair loss (alopecia).

==================================================

Question: A male client is receiving the cell cycle-nonspecific alkylating agent Thioplex (thiotepa), 60 mg weekly for 4 weeks by bladder installation as part of a chemotherapeutic regimen to treat bladder cancer. The client asks the nurse how the drug works. How does thiotepa exert its therapeutic effects?
a) It interferes with deoxyribonucleic acid (DNA) replication only
b) It interferes with ribonucleic acid (RNA) transcription only
c) It interferes with DNA replication and RNA transcription
d) It destroys the cell membranes, causing lysis

Answer: c) It interferes with DNA replication and RNA transcription
- Correct Answer: C. It interferes with DNA replication and RNA transcription.
Option C: Thiotepa is an alkylating agent that works by crosslinking DNA strands by reacting with phosphate groups to stop protein synthesis, RNA, and DNA.
Options A, B, and D: Thiotepa interferes with DNA replication and RNA transcription. It doesn't destroy the cell membrane.

==================================================

Question: Gio, a community health nurse, is instructing a group of female clients about breast self-examination. The nurse instructs the client to perform the examination:
a) At the onset of menstruation
b) Every month during ovulation
c) Weekly at the same time of day
d) 1 week after menstruation begins

Answer: d) 1 week after menstruation begins
- Correct Answer: D. 1 week after menstruation begins
Option D: The breast self-examination should be performed monthly 7 days after the onset of the menstrual period when the breasts are less tender and lumpy.
Options A and B: At the onset of menstruation and during ovulation, hormonal changes occur that may alter breast tissue.
Option C: Performing the examination weekly is not recommended.

==================================================

Question: Nurse Cindy is caring for a client who has undergone a vaginal hysterectomy. The nurse avoids which of the following in the care of this client?
a) Removal of anti-embolism stockings twice daily
b) Checking placement of pneumatic compression boots
c) Elevating the knee gatch on the bed
d) Assisting with range-of-motion leg exercises

Answer: c) Elevating the knee gatch on the bed
- Correct Answer: C. Elevating the knee gatch on the bed
Option C: The nurse should avoid using the knee gatch in the bed, which inhibits venous return, thus placing the client more at risk for deep vein thrombosis or thrombophlebitis.
Options A, B, and D: The client is at risk of deep vein thrombosis or thrombophlebitis after this surgery, as for any other major surgery. For this reason, the nurse implements measures that will prevent this complication. Range-of-motion exercises, anti-embolism stockings, and pneumatic compression boots are helpful.

==================================================

Question: Mina, who is suspected of an ovarian tumor is scheduled for a pelvic ultrasound. The nurse provides which pre-procedure instruction to the client?
a) Wear comfortable clothing and shoes for the procedure
b) Maintain an NPO status before the procedure
c) Drink six to eight glasses of water without voiding before the test
d) Eat a light breakfast only

Answer: c) Drink six to eight glasses of water without voiding before the test
- Correct Answer: C. Drink six to eight glasses of water without voiding before the test
Option C: A pelvic ultrasound requires the ingestion of large volumes of water just before the procedure. A full bladder is necessary so that it will be visualized as such and not mistaken for possible pelvic growth.
Option A: Comfortable shoes and clothing is unrelated to this specific procedure.
Option B: An abdominal ultrasound may require that the client abstain from food or fluid for several hours before the procedure.
Option D: A patient may eat and drink on the day of the exam regardless of quantity.

==================================================

Question: A male client is diagnosed as having a bowel tumor and several diagnostic tests are prescribed. The nurse understands which test will confirm the diagnosis of malignancy?
a) Magnetic resonance imaging
b) Computerized tomography scan
c) Abdominal ultrasound
d) Biopsy of the tumor

Answer: d) Biopsy of the tumor
- Correct Answer: D. Biopsy of the tumor
Option D: A biopsy is done to determine whether a tumor is malignant or benign through the examination of the sample of tissue taken into a body part.
Options A, B, and C: Magnetic resonance imaging, computed tomography scan, and ultrasound will visualize the presence of a mass but will not confirm a diagnosis of malignancy.

==================================================

Question: Vanessa, a community health nurse conducts a health promotion program regarding testicular cancer to community members. The nurse determines that further information needs to be provided if a community member states that which of the following is a sign of testicular cancer?
a) back pain
b) alopecia
c) Heavy sensation in the scrotum
d) Painless testicular swelling

Answer: b) Alopecia
- Correct Answer: B. Alopecia
Option B: Alopecia is not an assessment finding in testicular cancer. Alopecia may occur, however, as a result of radiation or chemotherapy.
Options A, C, and D: Back pain, heavy sensation in the scrotum, and painless testicular swelling are assessment findings in testicular cancer. Back pain may indicate metastasis to the retroperitoneal lymph nodes.

==================================================

Question: The male client is receiving external radiation to the neck for cancer of the larynx. The most likely side effect to be expected is:
a) Diarrhea
b) dyspnea
c) Constipation
d) Sore throat

Answer: d) Sore Throat
- Correct Answer: D. Sore throat
Option D: In general, only the area in the treatment field is affected by the radiation. Skin reactions, fatigue, nausea, and anorexia may occur with radiation to any site, whereas other side effects occur only when specific areas are involved in treatment. A client receiving radiation to the larynx is most likely to experience a sore throat.
Options A and C: May occur with radiation to the gastrointestinal tract.
Option B: Dyspnea may occur with lung involvement.

==================================================

Question: Nurse Joy is caring for a client with an internal radiation implant. When caring for the client, the nurse should observe which of the following principles?
a) Remove the dosimeter badge when entering the client's room
b) Individual's younger than 16 yr may be allowed to go in the room as long as they are 6 feet away from the client
c) Limit the time with the client to 1 hour per shift
d) Do not allow pregnant women into the client's room

Answer: d) Do not allow pregnant women into the client's room
- Correct Answer: D. Do not allow pregnant women into the client's room
Options B and D: Children younger than 16 years of age and pregnant women are not allowed in the client's room to avoid radiation exposure that may harm the children and the developing baby.
Option A: The dosimeter badge must be worn when in the client's room.
Option C: The time that the nurse spends in a room of a client with an internal radiation implant is 30 minutes per 8-hour shift.

==================================================

Question: A cervical radiation implant is placed in the client for treatment of cervical cancer. The nurse initiates what most appropriate activity order for this client
a) Out of bed ad lib
b) Ambulation to the bathroom only
c) Bed rest
d) Out of bed in a chair only

Answer: c) Bed Rest
- Correct Answer: C. Bed rest
Option C: The client with a cervical radiation implant should be maintained on bed rest in the dorsal position to prevent movement of the radiation source. The head of the bed is elevated to a maximum of 10 to 15 degrees for comfort. The nurse avoids turning the client on the side. If turning is absolutely necessary, a pillow is placed between the knees and, with the body in straight alignment, the client is logrolled.

==================================================

Question: The nurse is caring for a female client experiencing neutropenia as a result of chemotherapy and develops a plan of care for the client. The nurse plans to:
a) Teach the client and family about the need for hand hygiene
b) Insert an indwelling urinary catheter to prevent skin breakdown
c) Restrict fluid intake
d) Restrict all visitors

Answer: a) Teach the client and family about the need for hand hygiene
- Correct Answer: A. Teach the client and family about the need for hand hygiene
Option A: In the neutropenic client, meticulous hand hygiene education is implemented for the client, family, visitors, and staff to avoid transmission-based infection.
Option B: Invasive measures such as an indwelling urinary catheter should be avoided to prevent infections.
Option C: Fluids should be encouraged.
Option D: Not all visitors are restricted, but the client is protected from persons with known infections.

==================================================

Nclex Questions On Pediatric Nursing

Question: To establish a good interview relationship with an adolescent, which strategy is most appropriate?

1. Asking personal questions unrelated to the situation
2. Writing down everything the teen says
3. Asking open-ended questions
4. Discussing the nurse's own thoughts and feelings about the situation

Answer: 3. Asking open-ended questions

RATIONALE: Open-ended questions allow the adolescent to share information and feelings. Asking personal questions not related to the situation jeopardizes the trust that must be established because the adolescent may feel as though he's being interrogated with unnecessary questions. Writing everything down during the interview can be a distraction and doesn't allow the nurse to observe how the adolescent behaves. Discussing the nurse's thoughts and feelings may bias the assessment and is inappropriate when interviewing any client

==================================================

Question: A chronically ill school-age child is most vulnerable to which stressor?

1. Mutilation anxiety
2. Anticipatory grief
3. Anxiety over school absences
4. Fear of hospital procedures

Answer: 3. Anxiety over school absences

RATIONALE: The school-age child is becoming industrious and attempts to master school-related activities. Therefore, school absences are likely to cause extreme anxiety for a school-age child who's chronically ill. Mutilation anxiety is more common in adolescents. Anticipatory grief is rare in a school-age child. Fear of hospital procedures is most pronounced in preschool-age children.

==================================================

Question: When developing a care plan for an adolescent, the nurse considers the child's psychosocial needs. During adolescence, psychosocial development focuses on:

1. becoming industrious.
2. establishing an identity.
3. achieving intimacy.
4. developing initiative.

Answer: 2. establishing an identity.

RATIONALE: According to Erikson, the primary psychosocial task during adolescence is to establish a personal identity while overcoming role or identity confusion. The adolescent attempts to establish a group identity by seeking acceptance and approval from peers, and strives to attain a personal identity by becoming more independent from his family. Becoming industrious is the developmental task of the school-age child; achieving intimacy is the task of the young adult; and developing initiative is the task of the preschooler.

==================================================

Question: A nurse notes that an infant develops arm movement before fine-motor finger skills and interprets this as an example of which pattern of development?

1. Cephalocaudal
2. Proximodistal
3. Differentiation
4. Mass-to-specific

Answer: 2. Proximodistal

RATIONALE: Proximodistal development progresses from the center of the body to the extremities, such as from the arm to the fingers. Cephalocaudal development occurs along the body's long axis; for example, the infant develops control over the head, mouth, and eye movements before the upper body, torso, and legs. Mass-to-specific development, sometimes called differentiation, occurs as the child masters simple operations before complex functions and moves from broad, general patterns of behavior to more refined ones.

==================================================

Question: A teenage mother brings her 1-year-old child to the pediatrician's office for a well-baby checkup. She says that her infant can't sit alone or roll over. An appropriate response by the nurse would be:

1. "This is very abnormal. Your child must be sick."
2. "Let's see about further developmental testing."
3. "Don't worry, this is normal for her age."
4. "Maybe you just haven't seen her do it."

Answer: . "Let's see about further developmental testing."

RATIONALE: Stating that further developmental testing is necessary is appropriate because at age 12 months a child should be sitting up and rolling over. Therefore, this child may have developmental problems. Saying the infant's behavior is abnormal or suggesting that the mother hasn't seen her infant do these milestones isn't therapeutic and can cut off communication with the mother. Telling the mother that the infant's behavior is normal misleads the mother with false reassurance.

==================================================

Question: The mother of an 11-month-old infant reports to the nurse that her infant sleeps much less than other children. The mother asks the nurse whether her infant is getting sufficient sleep. What should be the nurse's initial response?

1. Reassure the mother that each infant's sleep needs are individual.
2. Ask the mother for more information about the infant's sleep patterns.
3. Instruct the mother to decrease the infant's daytime sleep to increase his nighttime sleep.
4. Inform the mother that her infant's growth and development are appropriate for his age, so sleep isn't a concern.

Answer: 2. Ask the mother for more information about the infant's sleep patterns.

RATIONALE: The nurse needs more information about the infant's sleep patterns to rule out potential problems before determining whether the infant is getting enough sleep. The nurse shouldn't offer advice or reassurance without knowing more about the infant's specific sleep habits.

==================================================

Question: A nurse observes a 2½-year-old child playing with another child of the same age in the playroom on the pediatric unit. What type of play should the nurse expect the children to engage in?

1. Associative play
2. Parallel play
3. Cooperative play
4. Therapeutic play

Answer: 2. Parallel play

RATIONALE: Two-year-olds engage in parallel play, in which they play side by side but rarely interact. Associative play is characteristic of preschoolers, in which they are all engaged in a similar activity but there is little organization. School-age children engage in cooperative play, which is organized and goal-directed. Therapeutic play is a technique that can be used to help understand a child's feelings; it consists of energy release, dramatic play, and creative play.

==================================================

Question: An infant who weighs 7.5 kg is to receive ampicillin (Omnipen) 25 mg/kg I.V. every 6 hours. How many milligrams should the nurse administer per dose? Record your answer using one decimal place.

Answer:

milligrams

Answer: 187.5 milligrams

RATIONALE: The nurse should calculate the correct dose using the following equation:
25 mg/kg × 7.5 kg = 187.5 mg

==================================================

Question: When making ethical decisions about caring for preschoolers, a nurse should remember to:

1. provide beneficial care and avoid harming the child.
2. make decisions that will prevent legal trouble.
3. do what she would do for her own child or loved ones.
4. be sure to do what the physician says.

Answer: 1. provide beneficial care and avoid harming the child.

RATIONALE: Nurses must provide beneficial care and avoid harming all clients. A nurse shouldn't base any decision solely on the desire to prevent legal trouble, on her own feelings for her loved ones, or what the physician says.

==================================================

Question: An emergency department nurse suspects neglect in a 3-year-old boy admitted for failure to thrive. Signs of neglect in the child would include:

1. slapping, kicking, and punching others.
2. poor hygiene and weight loss.
3. loud crying and screaming.
4. pulling hair and hitting

Answer: 2. poor hygiene and weight loss.

RATIONALE: Signs of neglect include poor hygiene and weight loss because neglect can involve failure to provide food, bed, shelter, health care, or hygiene. Slapping, kicking, pulling hair, hitting, and punching are examples of forms of physical abuse, not neglect. Loud crying and screaming are normal findings in a 3-year-old boy.

==================================================

Question: When developing a care plan for a child, the nurse identifies which Eriksonian stage as corresponding to Freud's oral stage of psychosexual development?

1. Initiative versus guilt
2. Autonomy versus shame and doubt
3. Trust versus mistrust
4. Industry versus inferiority

Answer: 3. Trust versus mistrust

RATIONALE: Freud defined the first 2 years of life as the oral stage and suggested that the mouth is the primary source of satisfaction for the developing child. Erikson posited that infancy (from birth to age 12 months) is the stage of trust versus mistrust, during which the infant learns to deal with the environment through the emergence of trustfulness or mistrust. Initiative versus guilt corresponds to Freud's phallic stage. Autonomy versus shame and doubt corresponds to Freud's anal/sensory stage. Industry versus inferiority corresponds to Freud's latency period

==================================================

Question: An infant, age 10 months, is brought to the well-baby clinic for a follow-up visit. The mother tells the nurse that she has been having trouble feeding her infant solid foods. To help correct this problem, the nurse should:

1. point out that tongue thrusting is the infant's way of rejecting food.
2. instruct the mother to place the food at the back and toward the side of the infant's mouth.
3. advise the mother to puree foods if the child resists them in solid form.
4. suggest that the mother force-feed the child if necessary.

Answer: 2. instruct the mother to place the food at the back and toward the side of the infant's mouth.

RATIONALE: The nurse should instruct the mother to place the food at the back and toward the side of the infant's mouth because it encourages swallowing. Tongue thrusting is a physiologic response to food placed incorrectly in the mouth. Offering pureed foods wouldn't encourage swallowing, which is a learned behavior. Force-feeding is inappropriate because it may be frustrating for both the mother and child and may cause the child to gag and choke when attempting to reject the undesired food; also, it may lead to a higher-than-normal caloric intake, resulting in obesity.

==================================================

Question: A nurse is caring for an adolescent who underwent surgery for a perforated appendix. When caring for this adolescent, the nurse should keep in mind that the main life-stage task for an adolescent is to:

1. resolve conflict with parents.
2. develop an identity and independence.
3. develop trust.
4. plan for the future

Answer: 2. develop an identity and independence.

RATIONALE: An adolescent strives for a sense of independence and identity. During this time, conflicts are heightened, not resolved. Trust begins to develop during infancy and matures during the course of development. Adolescents rarely finalize plans for the future; this normally happens later in adulthood

==================================================

Question: What is a normal systolic blood pressure for a 3-year-old child?
1. 60 mm Hg
2. 93 mm Hg
3. 120 mm Hg
4. 150 mm Hg

Answer: 2. 93 mm Hg

RATIONALE: The normal range for systolic blood pressure in preschoolers is 82 to 110 mm Hg. The normal range for diastolic blood pressure is 50 to 78 mm Hg.

==================================================

Question: A child, age 3, is admitted to the pediatric unit with dehydration after 2 days of nausea and vomiting. The mother tells the nurse that her child's illness "is all my fault." How should the nurse respond?

1. "Maybe next time you'll bring the child in sooner."
2. "Tell me why you think this is your fault."
3. "Try not to cry in front of the child. It'll only upset her."
4. "Don't be so upset. Your child will be fine."

Answer: 2. "Tell me why you think this is your fault."

RATIONALE: Having the mother explain why she feels the illness is her fault is appropriate because many parents feel responsible for their child's illness and may need instruction about the actual cause of the illness. Pointing out that the mother could have brought the child in sooner could increase the mother's feelings of guilt. Telling the mother not to cry or be upset ignores her feelings

==================================================

Question: What should a nurse do to ensure a safe hospital environment for a toddler?

1. Place the child in a youth bed.
2. Move stacking toys out of reach.
3. Pad the crib rails.
4. Move the equipment out of reach.

Answer: 4. Move the equipment out of reach.

RATIONALE: Moving the equipment out of reach ensures a safe environment because toddlers are curious and may try to play with items within their reach. Toddlers in a strange hospital environment still need the security of a crib. Stacking toys don't need to be moved out of reach because they don't present a safety hazard and are appropriate for this age-group. Padded crib rails are necessary only if seizure activity is present

==================================================

Question: An infant is hospitalized for treatment of inorganic failure to thrive. Which nursing action is most appropriate for this child?

1. Encouraging the infant to hold a bottle
2. Keeping the infant on bed rest to conserve energy
3. Rotating caregivers to provide more stimulation
4. Maintaining a consistent, structured environment

Answer: 4. Maintaining a consistent, structured environment

RATIONALE: The nurse caring for an infant with inorganic failure to thrive should strive to maintain a consistent, structured environment because it reinforces a caring feeding environment. Encouraging the infant to hold a bottle would reinforce an uncaring feeding environment. The infant should receive social stimulation rather than be confined to bed rest. The number of caregivers should be minimized to promote consistency of care

==================================================

Question: A toddler develops acute otitis media and is ordered cefpodoxime proxetil (Vantin) 5 mg/kg P.O. every 12 hours. If the child weighs 22 lb (10 kg), how many milligrams will the nurse administer with each dose?

1. 50 mg
2. 100 mg
3. 110 mg
4. 220 mg

Answer: 1. 50 mg

RATIONALE: The dose is 5 mg/kg and the child weighs 10 kg. To determine the dose, the nurse would calculate: 5 mg/1 kg × 10 kg = 50 mg per dose.

==================================================

Question: According to Erikson's psychosocial theory of development, an 8-year-old child would be in which stage?

1. Trust versus mistrust
2. Initiative versus guilt
3. Industry versus inferiority
4. Identity versus role confusion

Answer: 3. Industry versus inferiority

RATIONALE: In middle childhood, the 6- to 12-year-old child is mastering the task of industry versus inferiority. The trust versus mistrust task is in infancy (birth to 1 year). In early childhood, the 1- to 3-year-old child is in the stage of initiative versus guilt. Identity versus role confusion occurs during adolescence.

==================================================

Question: A nurse is preparing a child, age 4, for cardiac catheterization. Which explanation of the procedure is appropriate?

1. "Don't worry. It won't hurt."
2. "The test usually takes an hour."
3. "You must sleep the whole time that the test is being done."
4. "The special medicine will feel warm when it's put in the tubing."

Answer: 4. "The special medicine will feel warm when it's put in the tubing."

RATIONALE: To prepare a 4-year-old child without increasing anxiety, the nurse should provide concrete information in small amounts about nonthreatening aspects of the procedure. Therefore, saying the special medicine will feel warm is most appropriate. Saying that it won't hurt may prevent the child from trusting the nurse in the future. Explaining the time needed for the procedure wouldn't provide sufficient information. Stating that the child will need to sleep isn't true and could provoke anxiety.

==================================================

Question: A child has just been admitted to the facility and is displaying fear related to separation from his parents, the room being too dark, being hurt while in the hospital, and having many different staff members come into the room. Based on the nurse's knowledge of growth and development, the child is likely:

1. 7 to 12 months old (an infant).
2. 1 to 3 years old (a toddler).
3. 6 to 12 years old (a school-age child).
4. 12 to 18 years old (an adolescent

Answer: 2. 1 to 3 years old (a toddler).

RATIONALE: Toddlers show fear of separation from their parents, the dark, loud or sudden noises, injury, strangers, certain persons, certain situations, animals, large objects or machines, and change in environment. Infants show fear of strangers, the sudden appearance of unexpected and looming objects (including people), animals, and heights. School-age children show fear of supernatural beings, injury, storms, the dark, staying alone, separation from parents, things seen on television and in the movies, injury, tests and failure in school, consequences related to unattractive physical appearance, and death. Adolescents show fear of inept social performance, social isolation, sexuality, drugs, war, divorce, crowds, gossip, public speaking, plane and car crashes, and death.

==================================================

Question: A toddler is in the hospital. The parents tell the nurse they're concerned about the seriousness of the child's illness. Which response to the parents is most appropriate?

1. "Please try not to worry. Your child will be fine."
2. "If you look around, you'll see other children who are much sicker."
3. "What seems to concern you about your child being hospitalized?"
4. "It must be difficult for you when your child is ill and hospitalized."

Answer: 4. "It must be difficult for you when your child is ill and hospitalized."

RATIONALE: Expressing concern is the most appropriate response because it acknowledges the parents' feelings. False reassurance, such as telling parents not to worry, isn't helpful because it doesn't acknowledge their feelings. Encouraging parents to look at how ill other children are also isn't helpful because the focus of the parents is on their own child. Asking what the concern is merely reinforces the parents' concern without addressing it.

==================================================

Question: A mother of a 4-year-old child asks the nurse how to talk with her daughter about strangers. The little girl is very friendly and her mother is concerned that her child could be abducted. The nurse should tell the mother:

1. to talk with her daughter about what she should do if a stranger talks to her.
2. that she lives in a safe town and shouldn't worry.
3. to talk with her daughter about bad people and remind her to tell Mommy if someone she doesn't know talks to her.
4. contact social services, which is better equipped to respond to her questions

Answer: 1. to talk with her daughter about what she should do if a stranger talks to her.

RATIONALE: Preschoolers can begin to take a role in their own safety. They must be taught what a stranger is and what to do if a stranger approaches them. Living in a safe town doesn't eliminate the need to warn a child about talking to strangers. Although it's appropriate for the mother to talk with her daughter about strangers and have the daughter tell her if a stranger approaches her, the child needs to be aware of what to do at the time that the situation occurs, not only afterward. Contacting social services isn't appropriate because the nurse is capable of answering the mother's questions.

==================================================

Question: An 8-month-old infant is admitted with a febrile seizure. The infant weighs 17 lb (7.7 kg). The physician orders ceftriaxone (Rocephin), 270 mg I.M. every 12 hours. (The safe dosage range is 50 to 75 mg/kg daily.) The pharmacy sends a vial containing 500 mg, to which the nurse adds 2 ml of preservative-free normal saline solution. The nurse should administer how many milliliters?

1. None because this isn't a safe dosage
2. 0.08 ml
3. 1.08 ml
4. 1.8 ml

Answer: 3. 1.08 ml

RATIONALE: Because the infant weighs 17 lb (7.7 kg), the safe dosage range is 385 to 578 mg daily. The ordered dosage, 540 mg daily, is safe. To calculate the amount to administer, the nurse may use the following fraction method:
500 mg/2 ml = 270 mg/X ml
500X = 270 × 2
500X = 540
X = 540/500
X = 1.08 ml

==================================================

Question: A nurse is teaching the parents of a 6-month-old infant about usual growth and development. Which statements about infant development are true? Select all that apply.

1. A 6-month-old infant has difficulty holding objects.
2. A 6-month-old infant can usually roll from prone to supine and supine to prone positions.
3. A teething ring is appropriate for a 6-month-old infant.
4. Stranger anxiety usually peaks at 12 to 18 months.
5. Head lag is commonly noted in infants at age 6 months.
6. Lack of visual coordination usually resolves by age 6 months

Answer: 2. A 6-month-old infant can usually roll from prone to supine and supine to prone positions.
3. A teething ring is appropriate for a 6-month-old infant.
6. Lack of visual coordination usually resolves by age 6 months.

RATIONALE: Gross motor skills of the 6-month-old infant include rolling from front to back and back to front. Teething usually begins around age 6 months; therefore, a teething ring is appropriate. Visual coordination is usually resolved by age 6 months. At age 6 months, fine motor skills include purposeful grasps. Stranger anxiety normally peaks at 8 months of age. The 6-month-old infant also should have good head control and no longer display head lag when pulled up to a sitting position.

==================================================

Question: How should a nurse position a 4-month-old infant when administering an oral medication?

1. Seated in a high chair
2. Restrained flat in the crib
3. Held on the nurse's lap
4. Held in the bottle-feeding position

Answer: 4. Held in the bottle-feeding position

RATIONALE: The nurse should hold an infant in the bottle-feeding position when administering an oral medication by placing the child's inner arm behind the back, supporting the head in the crook of the elbow, and holding the child's free hand with the hand of the supporting arm. A 4-month-old infant can't sit unsupported in a high chair. Administering medication to an infant lying flat could cause choking and aspiration. Holding the infant in the lap may cause the medication to spill.

==================================================

Question: Which relaxation strategy would be effective for a school-age child to use during a painful procedure?

1. Having the child keep his eyes shut at all times
2. Having the child hold his breath and not yell
3. Having the child take a deep breath and blow it out until told to stop
4. Being honest with the child and telling him the procedure will hurt a lot

Answer: 3. Having the child take a deep breath and blow it out until told to stop

RATIONALE: Having the child take a deep breath and blow it out is a form of distraction and will help the child cope better with the procedure. A child may prefer to keep his eyes open, not shut, during a procedure so he can see what is going on and can anticipate what is going to happen. Letting a child yell during a procedure is a form of helpful distraction. In addition, holding the breath isn't beneficial and could have adverse effects (such as feeling dizzy or faint). The nurse should prepare a child for a procedure by using nonpain descriptors and not suggesting pain. For example, the nurse might say, "Sometimes this feels like pushing or sticking, and sometimes it doesn't bother children at all."

==================================================

Question: A school-age child presents to the office for a routine examination. Given the child's developmental level, a nurse should give highest priority to:

1. allowing the child to change into a gown while she isn't in the room.
2. allowing the child to play with medical equipment before the examination begins.
3. asking the parents to leave the room during the child's examination.
4. encouraging the child to hold a stuffed animal during the examination.

Answer: 1. allowing the child to change into a gown while she isn't in the room.

RATIONALE: School-age children tend to be very modest. The nurse should allow them to change into gowns while she isn't in the examination room. Children shouldn't have to take off their underwear for routine medical examinations. Playing with medical equipment is characteristic of younger children. The nurse shouldn't ask parents to leave the room unless the child requests that they not be present. A school-age child may feel too old to hold a stuffed animal during the examination.

==================================================

Question: Ten days after cardiac surgery, an 18-month-old child is recovering well. The child is alert and fairly active and is playing well with the parents. Discharge is planned soon. The nurse notes that the parents are still very reluctant to allow the child to do anything without help. What is the best initial action for the nurse to take?
1. Reemphasize the need for autonomy in toddlers
2. Provide opportunities for autonomy when the
parents are not present
3. Reassess the parent's needs and concerns
4. Discuss the success of the surgery and how well
the child is doing

Answer: 3. Before the nurse can teach the parents, it
will be necessary to reassess their needs and concerns. The question asks for the best initial action. Initially, the nurse should assess. Later, the nurse may emphasize the toddler's need for autonomy. The nurse may provide the child with opportunities to develop autonomy, although it would be better to teach the parents. The nurse may also discuss the success of the surgery and how well the child is doing, but this is not the initial action.

==================================================

Question: A 3-year-old child has all of the following abilities. Which did he acquire most recently?
1. Walking
2. Throwing a large ball
3. Riding a tricycle
4. Stating his name

Answer: 3. Riding a tricycle is 3-year-old behavior. Remember, "three years, three wheels." Children start to walk at about 1 year of age. Throwing a large ball and stating his name are 2-year-old behaviors. Remember to use developmental trends when determining the most recently acquired behavior—head to tail and simple to complex. Look for a complex lower body behavior.

==================================================

Question: The parents of a 3-year-old child are leaving for the evening. Which behavior would the nurse expect the child to exhibit?
1. Wave goodbye to the parents
2. Cry when the parents leave
3. Hide his/her head under the covers
4. Ask to go to the playroom

Answer: 2. It is normal for a 3-year-old to cry when the parents leave. The child will probably not wave goodbye even though he/she is able to. The child is not likely to hide under the covers. The child will likely be too upset to ask to go to the playroom.

==================================================

Question: A 5-year-old child had major surgery several days ago and is allowed to be up. When planning diversional activity, which action by the nurse is most appropriate?
1. Give the child a book to read.
2. Play a board game with the child.
3. Encourage the child to play house with other children.
4. Turn on the television so the child can watch cartoons

Answer: 3. Five-year-old children like cooperative play, such as playing house. The other activities are solitary activities. Note that the child is several days postsurgery. Most 5-year-olds are not able to read a book by themselves. Playing a board game with a child is not wrong, but it is a solitary activity. Most 5-year-olds would prefer to play with other children. There is almost always a better alternative than turning on the television. This child is several days postsurgery and is able to be up and play with others.

==================================================

Question: In planning care for an 18-month-old child, the nurse would expect him to be able to do which of the following?
1. Button his shirt and tie his shoes
2. Feed himself and drink from a cup
3. Cut with scissors
4. Walk up and down stairs

Answer: 2. An 18-month-old should be able to feed himself and drink from a cup. He may be messy. A 5- or 6-year-old can usually button a shirt and tie shoes. Cutting with scissors is appropriate for a preschool child. A 2-year-old child can go up and down stairs with both feet on the same step, and
a 3-year-old child can go up and down stairs by alternating feet.

==================================================

Question: Which activity would best occupy a 12-month- old child while the nurse is interviewing the parents?
1. String of large snap beads and a large plastic bowl
2. Riding toy
3. Several small puzzles
4. Paste, paper, and scissors

Answer: 1. Stringing large beads is appropriate for 12 months. Note that the beads are large and therefore not subject to being swallowed. A riding toy and small puzzles would be more appropriate for a toddler. Paste, paper, and scissors are appropriate for a preschooler when used with supervision.

==================================================

Question: A 10-year-old girl is being treated for rheumatic fever. Which would be an appropriate activity while she is on bed rest?
1. Stringing large wooden beads
2. Engaging in a pillow fight
3. Making craft items from felt
4. Watching television

Answer: 3. Craft work allows her to accomplish something while meeting her needs for rest. Industry is the developmental task for school-age children. The joint pains with rheumatic fever tend to be in the large joints, not the small ones, so craft work using finger activity would probably not be painful. Stringing large wooden beads is appropriate for younger children. Pillow fighting requires too much energy for a child on bed rest and is not appropriate for a hospital environment. Watching television is a solitary activity with no sense of accomplishment

==================================================

Question: A hospitalized 2.5-year-old child has a temper tantrum while her mother is bathing her. Her mother asks the nurse how she should handle this behavior. Which information should be included in the nurse's reply?
1. Temper tantrums in a hospitalized child indicate regression.
2. Tantrums suggest a poorly developed sense of trust.
3. Discipline is necessary when a child has a temper tantrum.
4. This behavior is a normal response to limit setting in a child of this age.

Answer: 4. Temper tantrums are a normal response to limit setting in a 2-year-old child. Answer 1 might be correct if the child were older. However, temper tantrums in a 2-year-old child do not indicate regression; rather, they are normal for this age. Tantrums are not suggestive of a poorly developed sense of trust; they are normal. Ignoring the tantrum is preferable to discipline when a 2-year- old has a tantrum.

==================================================

Question: A 2-year-old child is hospitalized for a fractured femur. During his first two days in the hospital, he lies quietly, sucks his thumb, and does not cry. Which is the best interpretation of his behavior?
1. He has made a good adjustment to being in the hospital.
2. He is comfortable with the nurses caring for him.
3. He is experiencing anxiety.
4. He does not have a good relationship with his parents.

Answer: 3. The child's behavior is typical of the despair phase of toddler responses to anxiety. The child should cry. Lying quietly, sucking his thumb, and saying nothing are suggestive of severe anxiety, a bad adjustment to the hospital, and no comfort with the nurses. This anxiety response does not suggest a poor relationship with his parents. In fact, his severe separation anxiety may be because he is so close to his parents.

==================================================

Nclex Questions On Shock

Question: What will the nurse identify as symptoms of hypovolemic shock in a patient?
Note: Credit will be given only if all correct choices and no incorrect choices are selected.
Standard Text: Select all that apply.
1. Temperature of 97.6°F (36.4°C)
2. Restlessness
3. Decrease in blood pressure of 20 mm Hg when the patient sits up
4. Capillary refill time greater than 3 seconds
5. Sinus bradycardia of 55 beats per minute

Answer: Correct Answer: 2,3,4
Rationale 1: Fever will increase oxygen demands but is unrelated to hypovolemic shock unless prolonged fever has caused severe dehydration, reducing the circulating blood volume. Hypovolemic shock reduces temperatures by peripheral shunting of blood away from the extremities and reducing the core metabolic rate.
Rationale 2: Due to decreased blood flow to the brain and peripheral areas when blood is shunted to maintain the vital organs, cerebral hypoxia occurs, leading to a change in mental status.
Rationale 3: Orthostatic hypotension is a manifestation of hypovolemic shock.
Rationale 4: Due to decreased blood flow to the brain and peripheral areas when blood is shunted to maintain the vital organs, capillary refill time will be reduced.
Rationale 5: Bradycardia is not present. The compensatory response is to increase the heart rate to circulate the blood faster to make up for the fluids that are not present in hypovolemic shock.

==================================================

Question: Which laboratory finding should cause the nurse to suspect that a patient is developing hypovolemic shock?
1. Serum sodium of 130 mEq/L (130 mmol/L)
2. Metabolic acidosis validated by arterial blood gases
3. Serum lactate of 3 mmol/L
4. SvO2 greater than 80%

Answer: Correct Answer: 2
Rationale 1: The sodium level in hypovolemic shock is elevated above the normal values of 135 to 145 mEq/L, not reduced.
Rationale 2: Metabolic acidosis is present due to an accumulation of carbonic acid, leaving a bicarbonate deficit from decreased tissue perfusion.
Rationale 3: Serum lactate is greater than 4 mmol/L as a result of tissue ischemia, hypoxia, and breakdown from decreased blood flow with hypovolemic shock.
Rationale 4: SvO2 (mixed venous oxygen saturation) would be less than 60% due to decreased circulating blood volume or decrease in cells to carry the oxygen. Therefore, O2 is carried less efficiently and decreased, not increased.

==================================================

Question: The nurse recognizes that which patient would be most likely to develop hypovolemic shock? A patient with:
1. Decreased cardiac output
2. Severe constipation, causing watery diarrhea
3. Ascites
4. Syndrome of inappropriate ADH (SIADH)

Answer: Correct Answer: 3
Rationale 1: Although ECG changes reflect the effectiveness of the heart's pumping when circulating the blood, it is not a risk factor for hypovolemic shock, which reflects a decreased circulating volume from either blood or fluid losses within the intravascular system.
Rationale 2: Severe constipation does not affect the circulating blood volume.
Rationale 3: Third spacing shifts move the fluids from the intravascular space into the interstitial space, causing a drop in the circulating blood volume. Therefore, third spacing is a risk factor for the development of hypovolemic shock.
Rationale 4: Overhydration does not lead to hypovolemic shock. It leads to fluid overload, which might cause cardiogenic shock, congestive heart failure, and pulmonary edema.

==================================================

Question: Which finding indicates that a patient is experiencing increased peripheral resistance and vasoconstriction?
1. Strong bounding pulse with deep red coloring
2. Pale, cool extremities with decreased pulses
3. Increased venous engorgement with strong pulses
4. Faster than normal capillary refill time

Answer: Correct Answer: 2
Rationale 1: An increased blood supply would increase color and bounding pulses as seen with vasodilation (blood engorgement) and is not present with increased peripheral resistance and vasoconstriction.
Rationale 2: Increased peripheral resistance causes the blood supply to decrease and results in decreased blood to the tissues, which causes pallor and decreased skin temperatures. The pulses would decrease in intensity with a decreased blood supply.
Rationale 3: Venous engorgement would not result from vasoconstriction of the arteries. Strong pulses would not be present with vasoconstriction from increased peripheral resistance.
Rationale 4: Capillary refill times are delayed or slowed due to decreased blood flow through the vessels caused by the vasoconstriction from increased peripheral resistance.

==================================================

Question: Which solution would be the most appropriate initial volume replacement for a patient with severe GI bleeding?
1. 200 mL of normal saline (NS) per hour for 5 hours
2. A liter of Ringer's lactate (RL) over 15 minutes
3. Two liters of D5W over half an hour
4. 500 mL of 0.45% normal saline (1/2 NS) over half an hour

Answer: Correct Answer: 2
Rationale 1: This is not an adequate amount of fluid replacement.
Rationale 2: The patient requires immediate infusion of an adequate amount of fluid. Fluid resuscitation begins with 500 to 1,000 mL of an isotonic solution.
Rationale 3: This is a hypotonic solution and would not help with fluid resuscitation.
Rationale 4: This is a hypotonic solution and would not help with fluid resuscitation.

==================================================

Question: Which life-threatening complications would the nurse anticipate developing in the patient being treated for hypovolemic shock?
Note: Credit will be given only if all correct choices and no incorrect choices are selected.
Standard Text: Select all that apply.
1. Fluid volume overload
2. Renal insufficiency
3. Cerebral ischemia
4. Gastric stress ulcer
5. Pulmonary edema

Answer: Correct Answer: 2,3
Rationale 1: Fluid volume overload is not an identified complication of hypovolemic shock.
Rationale 2: Renal insufficiency is a serious complication because of the prerenal etiology of hypovolemia.
Rationale 3: Early identification and correction of the fluid volume deficit in hypovolemic shock is necessary to prevent cerebral ischemia.
Rationale 4: Although physiologic stress can increase the risk for the development of stress ulcers, it is not considered one of the common or life-threatening complications of hypovolemic shock.
Rationale 5: Pulmonary edema is not an identified complication of hypovolemic shock.

==================================================

Question: The nurse, caring for a patient in hypovolemic shock, will not utilize a hypotonic solution for fluid resuscitation because hypotonic solutions:
1. Move quickly into the interstitial spaces and can cause third spacing
2. Stay longer to expand the intravascular space but deplete intracellular fluid levels
3. Do not stay in the intravascular space long enough to expand the circulating blood volume
4. Need a smaller bore needle to run at a slower rate to keep the intravascular space low

Answer: Correct Answer: 3
Rationale 1: Hypotonic solutions do not cause third spacing.
Rationale 2: Hypotonic solutions do not stay in the intravascular space long enough to expand the circulating blood volume.
Rationale 3: Hypotonic solutions do not stay in the intravascular space long enough to expand the circulating blood volume.
Rationale 4: The bore size of the needle does not affect the displacement or shifting of fluids.

==================================================

Question: The nurse should warm intravenous fluids when a rapid infuser is being utilized to prevent which complication?
1. Hemorrhagic shock
2. Hypothermia
3. Sepsis
4. Cardiogenic shock

Answer: Correct Answer: 2
Rationale 1: Hemorrhagic shock is caused by a loss of cells or blood volume and is not a result of infusing fluids too quickly.
Rationale 2: Hypothermia can result when providing room temperature fluids at a faster pace than the body can warm them.
Rationale 3: Bacterial contamination can be avoided by sterile technique, and sepsis is not caused by the rate or temperature of the fluid being administered.
Rationale 4: Cardiogenic shock results from poor ventricular functioning, not from the temperature of the intravenous fluids being administered too rapidly.

==================================================

Question: Which finding indicates that rehydration is complete and hypovolemic shock has been successfully treated in a patient?
1. CVP = 8 mm Hg
2. MAP = 45 mm Hg
3. Urinary output of 0.1 mL/kg/hr
4. Hct = 54%

Answer: Correct Answer: 1
Rationale 1: A CVP reading of 8 mm Hg is within normal range and rehydration has been restored.
Rationale 2: The mean arterial pressure (MAP) should be between 60 to 70 mm Hg as evidence of positive fluid resuscitation efforts.
Rationale 3: Urinary output to reflect adequate rehydration begins at 0.5 to 1 mL/kg/hr. Therefore, 0.1 mL is too small and renal insufficiency may be present due to inadequate circulating blood volume.
Rationale 4: Hematocrit (Hct) is the percentage of the number of RBCs per fluid volume. The normal range is 35% to 45% for an adult. The higher percentage represents a decreased fluid-to-cell ratio, which implies a fluid deficit and rehydration is not complete. An Hct of 54% is critical and increases the risk of clots, strokes, and other vessel obstruction from potential hemolysis and sluggishness of cellular movements.

==================================================

Question: A patient is demonstrating pulmonary edema, hypotension, and delayed capillary refill. The nurse suspects the patient is experiencing which type of shock?
1. Hypovolemic
2. Cardiogenic
3. Anaphylactic
4. Obstructive

Answer: Correct Answer: 2
Rationale 1: Pulmonary edema would not be present in hypovolemic shock.
Rationale 2: In cardiogenic shock, there is a low cardiac output, hypotension, and pulmonary edema.
Rationale 3: Pulmonary edema would not be present in anaphylactic shock.
Rationale 4: Pulmonary edema would not be present in obstructive shock

==================================================

Question: A patient, experiencing vasodilation, is diagnosed with distributive shock. The nurse will assess the patient for which etiologies?
Note: Credit will be given only if all correct choices and no incorrect choices are selected.
Standard Text: Select all that apply.
1. Sepsis
2. Spinal cord injury
3. Anaphylaxis
4. Hemorrhage
5. Pulmonary embolism

Answer: Correct Answer: 1,2,3
Rationale 1: One etiology of distributive shock is sepsis.
Rationale 2: One etiology of distributive shock is spinal cord injury.
Rationale 3: One etiology of distributive shock is anaphylaxis.
Rationale 4: Hemorrhage is not an etiology of distributive shock.
Rationale 5: Pulmonary embolism is not an etiology of distributive shock.

==================================================

Question: The nurse is concerned that a patient is at risk for developing obstructive shock because of which assessment findings?
Note: Credit will be given only if all correct choices and no incorrect choices are selected.
Standard Text: Select all that apply.
1. Age 80
2. History of atrial fibrillation
3. Bacteremia
4. T3 spinal cord injury
5. Latex allergy

Answer: Correct Answer: 1,2
Rationale 1: Advanced age increases the risk for development of pulmonary emboli, which is one cause of obstructive shock.
Rationale 2: Atrial fibrillation increases the risk for developing pulmonary emboli, which is one cause of obstructive shock.
Rationale 3: Bacteremia increases a patient's risk of developing septic shock and not obstructive shock.
Rationale 4: A spinal cord injury increases the risk for developing distributive shock and not obstructive shock.
Rationale 5: A latex allergy increases the risk for developing distributive shock and not obstructive shock.

==================================================

Question: A patient is experiencing an anaphylactic reaction to a medication. The nurse is concerned that the patient will develop distributive shock because:
1. The release of histamine causes vasodilation with plasma leakage.
2. Sympathetic innervation is interrupted.
3. Microorganisms overwhelm the vascular system.
4. Parasympathetic innervation functions are unopposed.

Answer: Correct Answer: 1
Rationale 1: In an anaphylactic reaction leading to distributive shock, the release of histamine causes vasodilation with plasma leakage. Vasodilation leads to profound hypotension, hypovolemia from fluid extravasation, reduced reload, and reduced cardiac output.
Rationale 2: Sympathetic innervation is not interrupted in an anaphylactic reaction.
Rationale 3: Microorganisms do not overwhelm the vascular system in an anaphylactic reaction.
Rationale 4: Parasympathetic innervation functioning unopposed is not a characteristic of an anaphylactic reaction.

==================================================

Question: A patient is diagnosed with cardiac tamponade. When planning care, the nurse will include interventions to address which type of shock?
1. Obstructive
2. Hypovolemic
3. Distributive
4. Cardiogenic

Answer: Correct Answer: 1
Rationale 1: Cardiac tamponade can lead to obstructive shock.
Rationale 2: Cardiac tamponade will not lead to hypovolemic shock.
Rationale 3: Cardiac tamponade will not lead to distributive shock.
Rationale 4: Cardiac tamponade will not lead to cardiogenic shock.

==================================================

Question: The nurse, caring for a patient recovering from an acute myocardial infarction, is planning interventions to reduce the risk of which type of shock?
1. Cardiogenic
2. Hypovolemic
3. Distributive
4. Obstructive

Answer: Correct Answer: 1
Rationale 1: One etiology of cardiogenic shock is a myocardial infarction.
Rationale 2: Acute myocardial infarction does not cause hypovolemic shock.
Rationale 3: Acute myocardial infarction does not cause distributive shock.
Rationale 4: Acute myocardial infarction does not cause obstructive shock.

==================================================

Question: A patient with cardiomyopathy is demonstrating signs of cardiogenic shock. The nurse realizes that this type of shock is due to:
1. Reduced cardiac output
2. Increased stroke volume
3. Reduced blood volume
4. Blood flow blocked in the pulmonary circulation

Answer: Correct Answer: 1
Rationale 1: In cardiogenic shock, cardiac output is reduced, leading to poor tissue perfusion.
Rationale 2: In cardiogenic shock, stroke volume is decreased.
Rationale 3: There is not a reduction of blood volume in cardiogenic shock.
Rationale 4: There is not a blockage of blood flow through the pulmonary circulation in cardiogenic shock.

==================================================

Question: The nurse is explaining the mechanism of a pulmonary embolism to the family of a patient diagnosed with the disorder. Place in order the steps the nurse will use to instruct the family about this disease process.
Standard Text: Click and drag the options below to move them up or down.
Choice 1. Blood clot causes backup of blood in the right ventricle.
Choice 2. Blood clot blocks blood to the left ventricle.
Choice 3. Left ventricle does not get enough blood to pump through the body.
Choice 4. Amount of blood the heart has to pump to the body drops.
Choice 5. Blood pressure drops.
Choice 6. Amount of blood going to the body drops.

Answer: Correct Answer: 1,2,3,4,5,6
Rationale 1: The obstruction caused by the pulmonary embolism increases the afterload of the right ventricle, causing right ventricular failure.
Rationale 2: The embolus prevents adequate blood flow from the pulmonary circulation to the left ventricle.
Rationale 3: Because blood flow from the pulmonary circulation is blocked, left ventricular preload drops.
Rationale 4: Because left ventricular preload is decreased, there is not enough blood in the heart to pump, causing decreased cardiac output.
Rationale 5: A lack of blood circulating will lead to hypotension.
Rationale 6: When the blood is backed up and is not being pumped into the general circulation, tissue perfusion is reduced.

==================================================

Question: A patient is being treated for pericarditis. The nurse will plan interventions to prevent the onset of which type of shock?
1. Obstructive
2. Hypovolemic
3. Distributive
4. Cardiogenic

Answer: Correct Answer: 1
Rationale 1: Acute pericarditis and the development of fluid accumulation in the pericardial space can lead to the development of obstructive shock.
Rationale 2: Pericarditis is not a risk factor for the development of hypovolemic shock.
Rationale 3: Pericarditis is not a risk factor for the development of distributive shock.
Rationale 4: Pericarditis is not a risk factor for the development of cardiogenic shock.

==================================================

Question: A patient is brought to the emergency department with manifestations of anaphylactic shock. What will the nurse assess as possible causes for this disorder?
Note: Credit will be given only if all correct choices and no incorrect choices are selected.
Standard Text: Select all that apply.
1. Recent bee sting
2. Ingestion of drugs
3. History of latex allergy
4. Recent diagnostic imaging tests
5. Recent myocardial infarction

Answer: Correct Answer: 1,2,3,4
Rationale 1: Venoms such as bee stings can trigger anaphylactic shock.
Rationale 2: Drugs can trigger anaphylactic shock.
Rationale 3: Latex can trigger anaphylactic shock.
Rationale 4: Contrast media for diagnostic tests can trigger anaphylactic shock.
Rationale 5: Myocardial infarction is not a trigger for anaphylactic shock.

==================================================

Question: During an assessment the nurse is concerned that a patient is developing cardiogenic shock. What did the nurse assess in this patient?
Note: Credit will be given only if all correct choices and no incorrect choices are selected.
Standard Text: Select all that apply.
1. Systolic blood pressure 82 mm Hg
2. Capillary refill 10 seconds
3. Crackles bilateral lung bases
4. Heart rate 55 and regular
5. Warm dry skin

Answer: Correct Answer: 1,2,3
Rationale 1: Hypotension is a manifestation of cardiogenic shock.
Rationale 2: Delayed capillary refill is a manifestation of cardiogenic shock.
Rationale 3: Crackles are a manifestation of cardiogenic shock.
Rationale 4: Bradycardia is not a manifestation of cardiogenic shock.
Rationale 5: Warm dry skin is not a manifestation of cardiogenic shock.

==================================================

Question: A patient is brought to the emergency department with hypotension, tachycardia, reduced capillary refill, and oliguria. During the assessment, the nurse determines the patient is experiencing cardiogenic shock because of which additional finding?
1. Jugular vein distention
2. Dry mucous membranes
3. Poor skin turgor
4. Thirst

Answer: Correct Answer: 1
Rationale 1: Jugular vein distention is a manifestation of cardiogenic shock.
Rationale 2: The mucous membranes are not dry in cardiogenic shock.
Rationale 3: The skin turgor is not poor in cardiogenic shock.
Rationale 4: Thirst is not a manifestation of cardiogenic shock.

==================================================

Question: A patient being treated for cardiogenic shock is being hemodynamically monitored. Which findings are consistent with the patient's diagnosis?
Note: Credit will be given only if all correct choices and no incorrect choices are selected.
Standard Text: Select all that apply.
1. Elevated pulmonary arterial wedge pressure
2. Elevated central venous pressure
3. Elevated systemic vascular resistance index
4. Elevated mean arterial pressure
5. Elevated stroke volume

Answer: Correct Answer: 1,2,3
Rationale 1: This finding is consistent with pulmonary vascular congestion.
Rationale 2: This finding is consistent with fluid volume overload.
Rationale 3: This finding is consistent with pulmonary vascular congestion.
Rationale 4: This finding is not consistent with cardiogenic shock.
Rationale 5: This finding is not consistent with cardiogenic shock.

==================================================

Question: The nurse is caring for a patient recovering from a spinal cord injury sustained during a motor vehicle crash. What assessment findings indicate that the patient is developing neurogenic shock?
Note: Credit will be given only if all correct choices and no incorrect choices are selected.
Standard Text: Select all that apply.
1. Hypotension
2. Bradycardia
3. Warm dry skin
4. Abdominal cramps
5. Palpitations

Answer: Correct Answer: 1,2,3
Rationale 1: Hypotension is a manifestation of neurogenic shock because of the loss of autonomic reflexes.
Rationale 2: Bradycardia occurs because of the loss of sympathetic innervation.
Rationale 3: Warm dry skin occurs because of a loss of cutaneous control of sweat glands.
Rationale 4: Abdominal cramping is not a manifestation of neurogenic shock.
Rationale 5: Palpitations are not seen in neurogenic shock.

==================================================

Question: A patient is demonstrating signs of obstructive shock but the cause has yet to be determined. Which finding indicates the patient is experiencing a pulmonary embolism as the cause for obstructive shock?
1. Chest pain
2. Hypotension
3. Tachycardia
4. Oliguria

Answer: Correct Answer: 1
Rationale 1: Chest pain is a symptom associated with a massive pulmonary embolus.
Rationale 2: Hypotension is seen in other causes of obstructive shock and is not a symptom that differentiates the cause as being from a pulmonary embolus.
Rationale 3: Tachycardia is seen in other causes of obstructive shock and is not a symptom that differentiates the cause as being from a pulmonary embolus.
Rationale 4: Oliguria is seen in other causes of obstructive shock and is not a symptom that differentiates the cause as being from a pulmonary embolus.

==================================================

Question: The nurse is preparing medications for a patient being treated for cardiogenic shock. Which medications will the nurse most likely provide to this patient?
Note: Credit will be given only if all correct choices and no incorrect choices are selected.
Standard Text: Select all that apply.
1. Dopamine
2. Norepinephrine
3. Dobutamine
4. Epinephrine
5. Phenylephrine

Answer: Correct Answer: 1,2,3,5
Rationale 1: Dopamine is commonly used in the treatment of cardiogenic shock.
Rationale 2: Norepinephrine is commonly used in the treatment of cardiogenic shock.
Rationale 3: Dobutamine may be used in the patient with cardiogenic shock who has an adequate blood pressure.
Rationale 4: Epinephrine is not used in the treatment of cardiogenic shock.
Rationale 5: Phenylephrine is a vasopressor and may be used in the patient with cardiogenic shock who is receiving dobutamine.

==================================================

Question: A patient with neurogenic shock is demonstrating bradycardia. What action will the nurse take at this time?
1. Limit patient movement.
2. Prepare to administer crystalloids.
3. Administer phenylephrine as prescribed.
4. Administer atropine as prescribed.

Answer: Correct Answer: 4
Rationale 1: Limiting movement will not correct bradycardia in the patient with neurogenic shock.
Rationale 2: Crystalloids are used to correct vasodilation.
Rationale 3: Phenylephrine is used in the patient with neurogenic shock to correct hypotension.
Rationale 4: Bradycardia in neurogenic shock is corrected by the administration of atropine at the dose of 0.5 to 1.0 mg intravenous every 5 minutes to a total dose of 3 mg.

==================================================

Question: A patient is experiencing acute respiratory distress after eating an item of a known food allergy. What interventions will the nurse implement when providing emergency care to this patient?
Note: Credit will be given only if all correct choices and no incorrect choices are selected.
Standard Text: Select all that apply.
1. Administer epinephrine 1:1000 intramuscularly.
2. Apply oxygen via face mask as prescribed.
3. Provide diphenhydramine 25 mg intravenous.
4. Administer vasopressin.
5. Prepare to administer antithrombolytic agents as prescribed.

Answer: Correct Answer: 1,2,3
Rationale 1: Epinephrine produces bronchodilation, improving the respiratory status. The route of administration is initially intramuscular.
Rationale 2: Supplemental oxygen is used in the treatment of anaphylactic shock.
Rationale 3: Hydrogen ion blockers such as diphenhydramine may be administered to block the histamine effects.
Rationale 4: Vasopressin is not used in the treatment of anaphylactic shock.
Rationale 5: Antithrombolytic agents are not used in the treatment of anaphylactic shock.

==================================================

Question: A patient is receiving phenylephrine 50 mcg/min as treatment for shock. Which assessment finding indicates this medication is effective?
1. Blood pressure 110/68 mm Hg
2. Heart rate 110
3. Respiratory rate 12 and regular
4. Decreased peripheral pulses

Answer: Correct Answer: 1
Rationale 1: The expected effect of this medication is an increase in blood pressure.
Rationale 2: Phenylephrine does not increase the heart rate.
Rationale 3: Phenylephrine does not affect the respiratory rate.
Rationale 4: Decreased peripheral pulses is a side/toxic effect of this medication.

==================================================

Question: A patient weighing 220 lbs is prescribed 10 mcg/kg/min of dopamine to improve cardiac output from cardiogenic shock. How many milligrams of dopamine will the patient receive in an hour?

Answer: Correct Answer: 60
Rationale : Determine the patient's weight in kg by dividing 220 lbs by 2.2 or 100 kg. Then multiply the number of mcg of medication the patient is to receive per minute by 100 kg or 10 mcg × 100 kg = 1,000 mcg. This is the dosage the patient will receive in 1 minute. To determine the amount of medication in 1 hour, multiply 1,000 mcg × 60 = 60,000 mcg. Using the conversion 1 mg = 1,000 mcg, divide 60,000 mcg by 1,000 mcg to determine that the patient will receive 60 mg of dopamine in 1 hour.

==================================================

Question: Which assessment finding indicates that an infusion of intravenous epinephrine 4 mcg/min is effective in the treatment of a patient with anaphylactic shock?
1. Reduced wheezing
2. Heart rate 55 and regular
3. Blood pressure 98/50 mm Hg
4. Respiratory rate 28

Answer: Correct Answer: 1
Rationale 1: An expected action for epinephrine is bronchodilation as evidenced by less wheezing.
Rationale 2: Epinephrine increases heart rate.
Rationale 3: Epinephrine increases blood pressure.
Rationale 4: Epinephrine lowers the respiratory rate. This respiratory rate indicates that epinephrine has not been effective.

==================================================

Question: A patient is receiving norepinephrine 30 mcg/min for treatment of refractory shock. Which assessment findings suggest the patient is experiencing peripheral vasoconstriction from the medication?
Note: Credit will be given only if all correct choices and no incorrect choices are selected.
Standard Text: Select all that apply.
1. Decreased peripheral pulses
2. Drop in body temperature
3. Onset of paresthesias
4. Drop in blood pressure
5. Increased cardiac output

Answer: Correct Answer: 1,2,3
Rationale 1: At high doses of norepinephrine, decreased peripheral pulses indicates significant vasoconstriction.
Rationale 2: At high doses of norepinephrine, a drop in body temperature indicates significant vasoconstriction.
Rationale 3: At high doses of norepinephrine, paresthesias indicate significant vasoconstriction.
Rationale 4: This medication does not cause a drop in blood pressure.
Rationale 5: An increase in cardiac output is an expected effect of this medication and does not indicate significant vasoconstriction.

==================================================

Question: A patient is prescribed vasopressin 0.03 units/minute as treatment for septic shock. What action will the nurse take when providing this medication?
1. Provide the vasopressin infusion in addition to a norepinephrine infusion.
2. Infuse through a peripheral line.
3. Utilize a rapid infuser.
4. Administer with 0.9% normal saline.

Answer: Correct Answer: 1
Rationale 1: The dose of 0.03 units/min is usually added to a norepinephrine infusion.
Rationale 2: This medication should always be administered via an infusion pump.
Rationale 3: A rapid infuser is used to deliver large amounts of warmed crystalloid or blood to a patient over a short period of time. It is not used for medication administration.
Rationale 4: This medication does not need to be administered with 0.9% normal saline.

==================================================

Nclex Questions On Uti

Question: 1

Fever, chills, and costovertebral pain are symptoms of a urinary tract infection (acute pyelonephritis), which requires a urine culture first to confirm the diagnosis.

Answer: The client is reporting chills, fever, and left costovertebral pain. Which diagnostic test should the nurse expect the HCP to prescribe first?

1.A midstream urine for culture.
2.A sonogram of the kidney.
3.An intravenous pyelogram for renal calculi.
4.A CT scan of the kidneys.

==================================================

Question: 4

Coffee, tea, cola, and alcoholic beverages are urinary tract irritants.

Answer: The female client in an outpatient clinic is being sent home with a diagnosis of urinary tract infection (UTI). Which instruction should the nurse teach to prevent a recurrence of a UTI?

1.Clean the perineum from back to front after a bowel movement.
2.Take warm tub baths instead of hot showers daily.
3.Void immediately preceding sexual intercourse.
4.Avoid coffee, tea, colas, and alcoholic beverages.

==================================================

Question: 3

The client should be taught to take all the prescribed medication anytime a prescription is written for antibiotics.

Answer: The nurse is discharging a client with a health-care facility acquired urinary tract infection. Which information should the nurse include in the discharge teaching?

1.Limit fluid intake so the urinary tract can heal.
2.Collect a routine urine specimen for culture.
3.Take all the antibiotics as prescribed.
4.Tell the client to void every five (5) to six (6) hours.

==================================================

Question: 3

A long-term complication of glomerulonephritis is it can become chronic if unresponsive to treatment,and this can lead to end-stage renal disease. Maintaining renal function is an appropriate long-term goal.

Answer: The nurse is preparing a plan of care for the client diagnosed with acute glomerulonephritis. Which statement is an appropriate long-term goal?

1.The client will have a blood pressure within normal limits.
2.The client will show no protein in the urine.
3.The client will maintain normal renal function.
4.The client will have clear lung sounds.

==================================================

Question: 3

Glomerular filtration rate (GFR) is approximately 120 mL/min. If the GFR is decreased to 40 mL/min, the kidneys are functioning at about one-third filtration capacity.

Answer: The elderly client is diagnosed with chronic glomerulonephritis. Which laboratory value indicates to the nurse the condition has become worse?

1.The blood urea nitrogen is 15 mg/dL.
2.The creatinine level is 1.2 mg/dL.
3.The glomerular filtration rate is 40 mL/min.
4.The 24-hour creatinine clearance is 100 mL/min.

==================================================

Question: 1.

Blood urea nitrogen (BUN) levels reflect the balance between the production and excretion of urea from the kidneys. Creatinine is a by-product of the metabolism of the muscles and is excreted by the kidneys. Creatinine is the ideal sub-stance for determining renal clearance because it is relatively constant in the body and is the laboratory value most significant in diagnosing renal failure.

Answer: The nurse is caring for a client diagnosed with ARF. Which laboratory values are most significant for diagnosing ARF?

1.BUN and creatinine.
2.WBC and hemoglobin.
3.Potassium and sodium.
4.Bilirubin and ammonia level.

==================================================

Question: 2.

Hypotension, which causes a decreased blood supply to the kidney, is one of the most common causes of pre-renal failure(before the kidney).

Answer: The nurse is caring for a client diagnosed with rule-out ARF. Which condition predisposes the client to developing prerenal failure?

1.Diabetes mellitus.
2.Hypotension.
3.Aminoglycosides.
4.Benign prostatic hypertrophy

==================================================

Question: 4.

Normal potassium level is 3.5 to5.5 mEq/L. A level of 6.8 mEq/L is life threatening and could lead to cardiac dysrhythmias. Therefore, the client may be dialyzed to decrease the potassium level quickly. This requires a health-careprovider order, so it is a collaborative intervention.

Answer: The client diagnosed with ARF has a serum potassium level of 6.8 mEq/L. Which collaborative treatment should the nurse anticipate for the client?

1.Administer a phosphate binder.
2.Type and crossmatch for whole blood.
3.Assess the client for leg cramps.
4.Prepare the client for dialysis.

==================================================

Question: 3.

Carbohydrates are increased to provide for the client's caloric intake and protein is restricted to minimize protein breakdown and to prevent accumulation of toxic waste products.

Answer: The client diagnosed with ARF is admitted to the intensive care unit and placed on a therapeutic diet. Which diet is most appropriate for the client?

1.A high-potassium and low-calcium diet.
2.A low-fat and low-cholesterol diet.
3.A high-carbohydrate and restricted-protein diet.
4.A regular diet with six (6) small feedings a day.

==================================================

Question: 2.

Bed rest reduces exertion and the metabolic rate, thereby reducing catabolism and subsequent release of potassium and accumulation of endogenous waste products (urea and creatinine).

Answer: The client diagnosed with ARF is placed on bed rest. The client asks the nurse, "Why do I have to stay in bed? I don't feel bad." Which scientific rationale supports the nurse's response?

1.Bed rest helps increase the blood return to the renal circulation.
2.Bed rest reduces the metabolic rate during the acute stage.
3.Bed rest decreases the workload of the left side of the heart.
4.Bed rest aids in reduction of peripheral and sacral edema.

==================================================

Question: 2.

These crystals are uremic frost resulting from irritating toxins deposited in the client's tissues. Bathing in cool water will remove the crystals, promote client comfort, and decrease the itching resulting from uremic frost.

Answer: The UAP tells the nurse the client with ARF has a white crystal-like layer on top of the skin. Which intervention should the nurse implement?

1.Have the assistant apply a moisture barrier cream to the skin.
2.Instruct the UAP to bathe the client in cool water.
3.Tell the UAP not to turn the client in this condition.
4.Explain this is normal and do not do anything for the client.

==================================================

Question: 3.

Regular insulin, along with glucose, will drive potassium into the cells,thereby lowering serum potassium levels temporarily.

Answer: The client diagnosed with ARF is experiencing hyperkalemia. Which medication should the nurse prepare to administer to help decrease the potassium level?

1.Erythropoietin.
2.Calcium gluconate.
3.Regular insulin.
4.Osmotic diuretic.

==================================================

Question: 2.

This client's dialysis access is compromised and he or she should be assessed first.

Answer: The nurse in the dialysis center is initiating the morning dialysis run. Which client should the nurse assess first?

1.The client who has hemoglobin of 9.8 g/dL and hematocrit of 30%.
2.The client who does not have a palpable thrill or auscultated bruit.
3.The client who is complaining of being exhausted and is sleeping.
4.The client who did not take antihypertensive medication this morning.

==================================================

Question: 1.

The nurse should place the client's chair with the head lower than thebody, which will shunt blood to the brain; this is the Trendelenburg position.

Answer: The client receiving dialysis is complaining of being dizzy and light-headed. Which action should the nurse implement first?

1.Place the client in the Trendelenburg position.
2.Turn off the dialysis machine immediately.
3.Bolus the client with 500 mL of normal saline.
4.Notify the health-care provider as soon as possible.

==================================================

Question: 3.

Dehydration results in concentrated serum, causing laboratory values to increase because the blood has normal constituents but not enough volume to dilute the values to within normal range or possibly lower.

Answer: The client is admitted to a nursing unit from a long-term care facility with a hematocrit of 56% and a serum sodium level of 152 mEq/L. Which condition is a cause for these findings?

1.Overhydration.
2.Anemia.
3.Dehydration.
4.Renal failure.

==================================================

Question: 1.

A client with a peaked T wave could be experiencing hyperkalemia. Changes in potassium levels can initiate cardiac dysrhythmias and instability.

Answer: The telemetry monitor technician notifies the nurse of the morning telemetry readings. Which client should the nurse assess first?

1.The client in normal sinus rhythm with a peaked T wave.
2.The client diagnosed with atrial fibrillation with a rate of 100.
3.The client diagnosed with a myocardial infarction who has occasional PVCs.
4.The client with a first-degree atrioventricular block and a rate of 92.

==================================================

Question: 2.

Increasing the irrigation fluid will flush out the clots and blood.

Answer: The nurse observes red urine and several large clots in the tubing of the normal saline continuous irrigation catheter for the client who is one (1) day postoperative TURP. Which intervention should the nurse implement?

1.Remove the indwelling catheter.
2.Titrate the NS irrigation to run faster.
3.Administer protamine sulfate IVP.
4.Administer vitamin K slowly.

==================================================

Question: 4.

Clients with acute bacterial prostatitis will frequently experience a sudden onset of fever and chills. Clients with chronic prostatitis have milder symptoms.

Answer: Which data support to the nurse the client's diagnosis of acute bacterial prostatitis?

1.Terminal dribbling.
2.Urinary frequency.
3.Stress incontinence.
4.Sudden fever and chills.

==================================================

Question: 4.

This is a potentially life-threatening problem.

Answer: Which nursing diagnosis is priority for the client who has undergone a TURP?

1.Potential for sexual dysfunction.
2.Potential for an altered body image.
3.Potential for chronic infection.
4.Potential for hemorrhage.

==================================================

Question: 2.

Elevating the scrotum on a towel for support is a task which can be delegated to the UAP.

Answer: The client is one (1) day postoperative TURP. Which task should the nurse delegate to the UAP?

1.Increase the irrigation fluid to clear clots from the tubing.
2.Elevate the scrotum on a towel roll for support.
3.Change the dressing on the first postoperative day.
4.Teach the client how to care for the continuous irrigation catheter.

==================================================

Question: 4.

The nurse should always assess any complaint before dismissing it as a commonly occurring problem.

Answer: The client with a TURP who has a continuous irrigation catheter complains of the need to urinate. Which intervention should the nurse implement first?

1.Call the surgeon to inform the HCP of the client's complaint.
2.Administer the client a narcotic medication for pain.
3.Explain to the client this sensation happens frequently.
4.Assess the continuous irrigation catheter for patency.

==================================================

Question: 3.

This is usually the length of time clients need to wait prior to having sexual intercourse; this is the information the client wants to know.

Answer: The client who is postoperative TURP asks the nurse, "When will I know if I will be able to have sex after my TURP?" Which response is most appropriate by the nurse?

1."You seem anxious about your surgery."
2."Tell me about your fears of impotency."
3."Potency can return in six (6) to eight (8) weeks."
4."Did you ask your doctor about your concern?"

==================================================

Question: 1.

An elevated PSA can be from urinary retention, BPH, prostate cancer, or prostate infarct.

Answer: The client asks, "What does an elevated PSA test mean?" On which scientific rationale should the nurse base the response?

1.An elevated PSA can result from several different causes.
2.An elevated PSA can be only from prostate cancer.
3.An elevated PSA can be diagnostic for testicular cancer.
4.An elevated PSA is the only test used to diagnose BPH.

==================================================

Question: 1, 3, 4

The nurse should assess the drain postoperatively.

The head of the bed should be lowered and the foot should be elevated to shunt blood to the central circulating system.

The surgeon needs to be notified of the change in condition.

Answer: The client returned from surgery after having a TURP and has a P 110, R 24, BP90/40, and cool and clammy skin. Which interventions should the nurse implement?Select all that apply.

1.Assess the urine in the continuous irrigation drainage bag.
2.Decrease the irrigation fluid in the continuous irrigation catheter.
3.Lower the head of the bed while raising the foot of the bed.
4.Contact the surgeon to give an update on the client's condition.
5.Check the client's postoperative creatinine and BUN.

==================================================

Question: 2.

Bladder spasms are common, but being relieved with medication indicates the condition is improving.

Answer: The nurse is caring for a client with a TURP. Which expected outcome indicates the client's condition is improving?

1.The client is using the maximum amount allowed by the PCA pump.
2.The client's bladder spasms are relieved by medication.
3.The client's scrotum is swollen and tender with movement.
4.The client has passed a large, hard, brown stool this morning.

==================================================

Question: 4.

The white blood cell count is elevated;normal is 5,000 to 10,000/mm3.

Answer: The client had surgery to remove a kidney stone. Which laboratory assessment data warrant immediate intervention by the nurse?

1.A serum potassium level of 3.8 mEq/L.
2.A urinalysis shows microscopic hematuria.
3.A creatinine level of 0.8 mg/100 mL.
4.A white blood cell count of 14,000/mm3.

==================================================

Question: 3.

Venison, sardines, goose, organ meats,and herring are high-purine foods, which should be eliminated from the diet to help prevent uric acid stones.

Answer: The client is diagnosed with a uric acid stone. Which foods should the client eliminate from the diet to help prevent re-occurrence?

1.Beer and colas.
2.Asparagus and cabbage.
3.Venison and sardines.
4.Cheese and eggs.

==================================================

Question: 1.

Clients who have urinary incontinenceare often embarrassed, so it is the responsibility of the nurse to approach this subject with respect and consideration.

Answer: The elderly client being seen in the clinic has complaints of urinary frequency,urgency, and "leaking." Which priority intervention should the nurse implement when interviewing the client?

1.Ensure communication is nonjudgmental and respectful.
2.Set the temperature for comfort in the examination room.
3.Speak loudly to ensure the client understands the nurse.
4.Ensure the examining room has adequate lighting.

==================================================

Question: 4.

Use of the bladder training drill is helpful in stress incontinence. The client is instructed to void at scheduled intervals. After consistently being dry, the interval is increased by 15 minutes until the client reaches an acceptable interval.

Answer: The elderly client recovering from a prostatectomy has been experiencing stress incontinence. Which independent nursing intervention should the nurse discuss with the client?

1.Establish a set voiding frequency of every two (2) hours while awake.
2.Encourage a family member to assist the client to the bathroom to void.
3.Apply a transurethral electrical stimulator to relieve symptoms of urinary urgency.
4.Discuss the use of a "bladder drill," including a timed voiding schedule.

==================================================

Question: 2.

When an elderly client's mental status changes to confused and irritable, the nurse should seek the etiology, which may be a UTI secondary to an indwelling catheter. Elderly client soften do not present with classic signs and symptoms of infection.

Answer: The nurse is caring for an elderly client who has an indwelling catheter. Which data warrant further investigation?

1.The client's temperature is 98.0˚F.
2.The client has become confused and irritable.
3.The client's urine is clear and light yellow.
4.The client feels the need to urinate.

==================================================

Question: 3.

The drainage bag should be kept below the level of the bladder to prevent reflux of urine into the renal system; it should not be placed on the bed.

Answer: The nurse is observing the UAP providing direct care to a client with an indwelling catheter. Which data warrant immediate intervention by the nurse?

1.The UAP secures the tubing to the client's leg with tape.
2.The UAP provides catheter care with the client's bath.
3.The UAP puts the collection bag on the client's bed.
4.The UAP cares for the catheter after washing the hands.

==================================================

Question: 4.

The nephrostomy tube should never be clamped or have kinks because an obstruction can cause pyelonephritis.

Answer: Which intervention should the nurse implement when caring for the client with a nephrostomy tube?

1.Change the dressing only if soiled by urine.
2.Clean the end of the connecting tubing with Betadine.
3.Clean the drainage system every day with bleach and water.
4.Assess the tube for kinks to prevent obstruction.

==================================================

Nclex Renal Questions

Question: The nurse has admitted a client with uremia. The nurse questions the client about a history of which health problem?
A) Polycystic kidney disease
B) End-stage renal failure
C) Pyelonephritis
D) Cystitis

Answer: ANS:B

==================================================

Question: A client receiving peritoneal dialysis has outflow that is 200 ml less than the inflow for 2 consecutive exchanges. Which actions would be appropriate for the nurse to take at this time? Select all that apply.
A) Examine the abd dressing
B) Change clients position
C) Lower the drainage bag
D) Continue to monitor the outflow in future exchanges
E) Irrigate the peritoneal dialysis catheter

Answer: ANS: A, B, C

==================================================

Question: The nurse is planning to teach the client with acute glomerulonephritis about necessary dietary restrictions. Which dietary changes should the nurse include in the plan?
A) Limit fluid intake to 500 ml perday
B) Restrict protein intake by limiting meats and other high protein foods
C) Increase intake of high fiber foods, such as bran cereal.
D) Increase intake of potassium-rich foods such as bananas or canteloupe

Answer: ANS: B

==================================================

Question: A client develops a renal disorder after taking an antibiotic that is known to have nephrotoxic effects. The nurse adds a standardized care plan to the client's medical record for which disorder?
A) Polycystic kidney disease
B) Glomerulonephritis
C) Acute renal failure
D) Chronic renal failure

Answer: ANS: C

==================================================

Question: A client with a chronic UTI is scheduled for a number of lab tests. The nurse would note that which test result best evaluates whther the kidneys are being adversely affected?
A) Serum potassium 3.8
B) UA specific gravity 1.015
C) Serum creatinine 2.0
D) Urine culture negative

Answer: ANS: C

==================================================

Question: The nurse is teaching a female client with recurrent cystitis who is scheduled for cystoscopy in the morning. Which preprocedure instructions should the nurse include in the teaching?
A) Take Tylenol and Benadryl the morning of the test to minimize allergic reaction
B) Light breakfast may be eaten
C) This test is only used to diagnose disorders
D) Take a laxative the evening before procedure

Answer: ANS: D

==================================================

Question: The nurse is teaching the client to perform peritoneal dialysis. The nurse reviews which essential action that will help to prevent the major complication of peritoneal dialysis?
A) Monitor postvoid residuals
B) Maintain strict aseptic technique during connection and disconnection
C) Add heparin to dialysate at least once per day
D) Change catheter site dressing twice daily

Answer: ANS: B

==================================================

Question: A client is admitted to the E.D. with suspected urolithiasis. The nurse questions the client about the presence of which symptom that would be expected with urolithiasis?
A) Flank pain
B) Difficult urination
C) Absence of urine
D) Headache

Answer: ANS: A

==================================================

Question: The nurse is caring for a client immediately following a transurethral prostatectomy (TURP). The nurse places high priority on which nursing interventions in the immediate post-op period for this client? Select all that apply.
A) Asses for signs of a UTI
B) Prevent post-op atelectasis
C) Discontinue urinary catheter asap
D) Adjust flow rate of the bladder irrigant to keep catheter patent
E) Monitor the amount of blood in the urine

Answer: ANS: B, D, E

==================================================

Question: The nurse is reviewing health care provider prescriptions for a client diagnosed with end-stage renal failure. Which diet should the nurse expect the provider to prescribe for this client?
A) Increased protein, decreased CHO
B) Restricted protein, increased CHO
C) Increased potassium and sodium
D) Increased phosphorus and magnesium

Answer: ANS: B

==================================================

Question: The nurse is assessing cognitive status on a client who ahd developed urinary incontinence. When questioned by the family, the nurse explains that this assessment will help to determine what condition?
A) Stress incontinence
B) Overflow incontinence
C) Functional incontinence
D) Mixed incontinence

Answer: ANS: C

==================================================

Question: An older adult male reports to the nurse that he has difficulty starting urination and is voiding several times during the night. The nurse suspects that this client has which condition?
A) BPH
B) UTI
C) Urethral strictures
D) Urethritis

Answer: ANS: A

==================================================

Question: Which nursing action is most appropriate when caring for a client with a nursing dx of excess fluid volume r/t renal insufficiency?
A) Teaching clients about sodium content of foods
B) Administering vitamin D supplements
C) Assessing and documenting clients energy level
D) Observing for signs of hypocalcemia

Answer: ANS: A

==================================================

Question: The nurse is caring for a client with recurrent cystitis. Which instruction should be included prior to discharge?
A) Limit fluid daily intake to decrease the urge to void.
B) There is no relationship between sexual intercourse and cystitis
C) Void at least q4h even if the urge is absent
D) Daily cranberry juice has been proven to decrease the number of UTIs

Answer: ANS: C

==================================================

Question: A client has undergone creation of an ileal conduit. Which instruction to the client about urostomy care would be appropriate to include in the teaching plan?
A) Cut the faceplate of the appliance so that the opening is slightly smaller than the stoma
B) Plan to do appliance changes just before bedtime
C) Limit fluids to minimize odor from urine breakdown to ammonia
D) Cleanse the skin around the stoma using gentle soap and water, rinse and dry well

Answer: ANS: D

==================================================

Question: The nurse is reviewing the past medical history of a client diagnosed with chronic renal failure. Which conditions in the client's history most likely contributed to the client's renal failure? Select all that apply.
A) DM type 1
B) HTN
C) SLE
D) Hypothyroidism
E) Hiatal hernia

Answer: ANS: A, B, C

==================================================

Question: The nurse is conducting discharge teaching with a client following insertion of a new AV fistula in the L arm for hemodialysis. The nurse will be sure to include which topics in discussions with this client? Select all that apply.
A) Avoid carrying items with the left arm
B) Assess the fistula daily for redness or drainage
C) BP should be taken in the right arm only
D) Sleep on the left side
E) It is expected to have some bleeding at the access site after a dialysis treatment

Answer: ANS: A, B, C

==================================================

Nclex Rn Cram Plan

Question: Pressure Ulcer Stage 1

Answer: Non- blanchable reddness (erythema)
Superficial (intact skin)
Epidermis tissue showing

==================================================

Question: Pressure Ulcer Stage 2

Answer: Red or pink (erythema) ulcer
Dermis tissue showing
Partial thickness (damage of epidermis and dermis)
Risk for infection
-Hydrocolloid dressing needed for autolytic debridement; should be left on for several days at a time to be effective

==================================================

Question: Pressure Ulcer Stage 3

Answer: Yellowish ulcer
Adipose tissue showing
Full thickness (damage of epidermis, dermis, and deeper tissues)
Wet-to-dry dressing until debrided to establish granular tissue
Consult wound care; debridement needed

==================================================

Question: Pressure Ulcer Stage 4

Answer: Bone or muscle showing
Wet-to-dry dressing until debrided to establish granular tissue
Consult wound care; debridement needed

==================================================

Question: Unstageable Pressure Ulcer

Answer: Black
Unable to assess underlying tissues; wound base is obscured by slough or eschar

==================================================

Question: Morse Fall Scale

Answer: History of falls
Secondary diagnosis
Ambulatory Aids
IV therapy/heparin lock
Gait
Mental status

==================================================

Question: Horizontal infection

Answer: person to person

==================================================

Question: vertical infection

Answer: mother to child

==================================================

Question: C. Diff

Answer: *Contact Precautions
Hand washing with soap and water
Often caused by overuse of antibiotics
Severe and foul-smelling odors
BLEACH

==================================================

Question: Gastroenteritis

Answer: *Contact Precautions
inflammation of the stomach and intestines
may be caused by rotavirus, norovirus
vomiting/diarrhea

==================================================

Question: Croup (Laryngotracheal bronchitis)

Answer: *Contact Precautions
Caused by diptheria virus
Barking cough and stridor
minor: treat with anti-inflammatories
major: treat with epinephrine

==================================================

Question: Rabies

Answer: *Contact Precautions
leads to meningitis

==================================================

Question: Hepatitis A

Answer: *Contact Precautions
Enteric precautions
Fecal/oral route of transmission
Incubates 3 to 5 weeks
Vaccine available (Can give immune globulin after exposure)
HAsAg (this is what the blood test show) Hepatitis A surface Antigen

==================================================

Question: Impetigo

Answer: *Contact Precautions
Common in pediatric population
Honey-crusted lesions on mucous membranes (sometimes on back)

==================================================

Question: Respiratory Syncytial Virus (RSV)

Answer: *Contact Precautions
-unless productive cough exists, then droplet precautions

==================================================

Question: Contact Precautions

Answer: Wear gloves & gown
-VRE/MRSA
-Lice/scabies
-Conjunctivitis (pink eye)

==================================================

Question: Standards Precautions

Answer: Wear gloves to reduce risk of bodily fluid exposure
-Blood-borne infectious diseases: Ebola, Hep B/C, HIV
Anthrax: Inhaled as a white powder, not communicable

==================================================

Question: Droplet Precautions

Answer: Wear gloves, gown, mask & goggles
-Influenza type b (Hib) *vaccine
-Meningococcal meningitis
-Rubella (German measles) *MMR vaccine
-Sepsis
-Pertussis (whooping cough)

==================================================

Question: Mumps

Answer: *Droplet Precautions
-MMR vaccine
Swollen salivary glands
Fever and headache

==================================================

Question: Streptococcus

Answer: *Droplet Precautions
Most commonly respiratory
Pharyngitis
Pneumonia
Pertussis

==================================================

Question: Parvovirus

Answer: *Droplet Precautions
Causes fifth disease (common in children)
Appears as rash on face (looks like a redness from a slap on the face)

==================================================

Question: Airborne Precautions

Answer: Wear an N95 or a surgical mask and use a negative airflow room
-Sudden acute respiratory syndrome (SARS)
-Tuberculosis

==================================================

Question: Herpes Zoster (Shingles)

Answer: *Airborne Precautions
Only possible if chickenpox infection occurred in past
Begins with pain; rash form after

==================================================

Question: Measles (rubeola)

Answer: *Airborne Precautions
Vaccinate to prevent (MMR)
Fever, irritability

==================================================

Question: Varicella zoster (chickenpox)

Answer: *Airborne Precautions
7-day incubation period
Oatmeal baths
Tzanck test
Pain when chewing

==================================================

Question: Yersinia pestis

Answer: Transmitted by rats and fleas
Cause of bubonic plague

==================================================

Question: Shigella

Answer: Similar to salmonella
Contracted by consuming undercooked food
Causes diarrhea

==================================================

Question: Yellow fever

Answer: Transmitted by mosquitoes
Causes headaches and vomiting

==================================================

Question: West Nile virus

Answer: transmitted by mosquitos
may lead to meningitis

==================================================

Question: Lyme disease

Answer: transmitted by ticks
bullseye rash
causes flu-like symptoms

==================================================

Question: Brucellosis

Answer: Contracted by consuming uncooked meats and unpasteurized milk

==================================================

Question: Typhoid fever

Answer: Contracted by consuming contaminated food or water
Causes diarrhea

==================================================

Question: Pinworm

Answer: Transmitted via ingestion
Fecal-oral route
More common children
Place tape on anus during sleep to diagnose

==================================================

Question: Legionnaire's disease

Answer: Contracted by inhaling droplets of contaminated water
Found in fountains and ponds with stagnant water

==================================================

Question: IV infiltration

Answer: Catheter falls out of the vein (third spacing of fluid)
Coolness, redness, swelling, discomfort
Stop the IV infusion, discontinue the IV, apply a warm compress, elevate the extremity

==================================================

Question: IV Thrombophlebitis

Answer: Formation of a clot/inflammation at the catheter site
Warmth, redness, swelling, pain, discoloration of vein
Stop the IV infusion, discontinue the IV, apply a warm compress, elevate the extremity

==================================================

Question: IV Extravasation

Answer: Infiltration of third space with a vesicant medication
Destruction of cells and pain at the site
Stop the IV infusion, discontinue the IV, apply a warm compress, elevate the extremity
A medication may be ordered to soak up vesicant

==================================================

Question: Christianity Religious Beliefs

Answer: Anointing of the oil upon death

==================================================

Question: Judaism Religious Beliefs

Answer: 2-3 hour gap between eating dairy and meat

==================================================

Question: Islam Religious Beliefs

Answer: Face east toward Mecca to pray
Genuflect (kneel) upon praying; assist a patient to floor if able
Do not touch deceased Muslim if not Muslim yourself
Direct eye contact is considered sexually advancing

==================================================

Question: Buddhism Religious Beliefs

Answer: Commonly practiced in China
Karma

==================================================

Question: Hinduism Religious Beliefs

Answer: Reincarnation

==================================================

Question: Jehovah's Witness

Answer: No blood products should be used

==================================================

Question: Anglo Saxon Culture

Answer: Caucasian
Eye contact is considered respectful
May sit closer

==================================================

Question: Mexican culture

Answer: Hot milk concept????

==================================================

Question: Asian Culture

Answer: Stoic in relation to pain
Hot-cold concept (cold food given for a sickness of heat and viceversa?)

==================================================

Question: Native American Culture

Answer: Higher prevalence of diabetes mellitus

==================================================

Question: Muslim Culture

Answer: Do not sit closer together, considered rude
No caring of opposite sex
Men often decision makers

==================================================

Question: Myocardial Infarction (Heart Attack)- Pathophysiology

Answer: Ischemia of heart muscle

==================================================

Question: Myocardial Infarction (Heart Attack)- Causes

Answer: Clotting (embolism)
Atherosclerotic (narrowed coronary)
Vasospastic (Prinzmetal's angina)
-Caused by stimulants (cocaine)

==================================================

Question: Myocardial Infarction (Heart Attack)- Signs & Symptoms

Answer: Men:
Chest pain radiating down left arm
Shortness of breath
Chest pressure
Tachycardia
Jaw Pain

Women/Elderly:
Nausea/Vomiting
Malaise
Cold Sweats
Jaw Pain

==================================================

Question: Myocardial Infarction (Heart Attack)- Interventions

Answer: M- Morphine: treats pain, also aids in vasodilation
O- Oxygen: delivery to point of infarct
N- Nitroglycerin: vasodilator
A- Aspirin/clopidogrel: prevents growth or further exacerbation

==================================================

Question: Heart Failure/Cardiomyopathy- Pathophysiology

Answer: Preload: pressure of blood filling into relaxed ventricles
Afterload: pressure ventricles overcome to push blood out of the heart
Cardiac Output=HRxSV: decrease in HF
-diagnosed via echocardiogram
-transesophageal echocardiogram requires sedation and oxygen
-men & pregnant women have higher cardiac outputs
-ejection fraction: precentage showing function of heart

==================================================

Question: Heart Failure/Cardiomyopathy- Causes

Answer: R-sided: Pulmonary hypertension (vasculature), myocardial infarction
L-sided: Systemic hypertension, myocardial infarction

==================================================

Question: Heart Failure/Cardiomyopathy- Signs & Symptoms

Answer: R-sided: Edema- peripheral, dependent (gravity), generalized, legs, ankles, abdominal ascites, JVD, hypertrophy
L-sided: Fatigue, SOB, decreased cardiac output and ejection fraction leading to hypotension, pulmonary edema- fine crackles and an increase in pulmonary pressure, hypertrophy

==================================================

Question: Heart Failure/Cardiomyopathy- Interventions

Answer: Low-salt diet (>2,000mg per day)
Fluid Restriction
Pharmacology: Diuretics, ACE inhibitors, digoxin
Intra-aortic balloon pump
AICD Defibrillator placement
Surgery/transplant

==================================================

Question: Endocarditis

Answer: inflammation of the inner lining of the heart

==================================================

Question: Pericarditis

Answer: inflammation of the membrane surrounding the heart, the pericardium

==================================================

Question: Valvulitis

Answer: inflammation of a heart valve

==================================================

Question: Endocarditis/Pericarditis/Valvulitis- Pathophysiology

Answer: Inflammation of the layers of the heart
Common in those with a history of grafts, IV drugs users, and previous heart surgery patients

==================================================

Question: Endocarditis/Pericarditis/Valvulitis- Signs & Symptoms

Answer: Elevated WBC, fever, pain
May decrease cardiac output, causing hypotension
Pericardial friction rub

==================================================

Question: Endocarditis/Pericarditis/Valvulitis- Interventions

Answer: IV antibiotics

==================================================

Question: Cardiac Tamponade- pathophysiology

Answer: Medical emergency
Pericardial effusion (fluid build-up)
-May be blood (trauma)

==================================================

Question: Cardiac Tamponade- Signs & Symptoms

Answer: SOB
Tachycardia
Narrowing pulse pressure (Systolic - Diastolic)
Muffled heart sounds
Pulsus paradoxus (BP drops more than 10 mmHg during inhalation)

==================================================

Question: Cardiac Tamponade- Interventions

Answer: Pericardiocentesis
-Patient supine, may cause pneumothorax

==================================================

Question: Aortic Aneurysm/Aortic Dissection- Pathophysiology

Answer: Bulging of the aorta (aneurysm)
Tearing away of the aortic lining (dissection)

==================================================

Question: Aortic Aneurysm/Aortic Dissection- Signs & Symptoms

Answer: Thoracic: Back pain indicates emergent rupture (PRIORITY)
Abdominal (AAA): Palpable pulsating mass; NEVER palpate it again.

==================================================

Question: Aortic Aneurysm/Aortic Dissection- Interventions

Answer: BP management (beta blockers)
Interventions radiology
Surgical graft repair
-Assess for post-op pre-renal acute kidney injury (oliguria)

==================================================

Question: Marfan Syndrome- Pathophysiology

Answer: Genetic
Abnormal weakening of the vessel lining, affects the connective tissue

==================================================

Question: Marfan Syndrome- Signs & Symptoms

Answer: Frequents aneurysms
Tall body with thin and long fingers

==================================================

Question: Marfan Syndrome- Interventions

Answer: Screening and preventions to treat the various complications

==================================================

Question: Heart Sounds

Answer: S1/S2: normal "lub dub'
S3: extra heart sound after s2
S4: fluid overload (normal in pregnant women)

==================================================

Question: Arterial

Answer: -Has a pulse
-Dangle the artery
***Intermittent claudication

==================================================

Question: Intermittent claudication

Answer: Pain in the calves
Light ambulation helps with increased blood flow
Pentoxifylline may help

==================================================

Question: Peripheral arterial disease/arterial insufficiency

Answer: -diabeties mellitus
-decreased peripheral sensation
inspect feet daily
lotion prevent cracking
wear cotton socks
cotton between toes prevents friction
snug-fitting shoes
take care cutting toenails

==================================================

Question: Venous

Answer: elevate the veins
ambulate (venous valves prevent blood from pooling)
-varicose veins if valves fail
DVT prophylaxis
-antiembolism stockings
-sequential compression devices
-subq heparin
Venous stasis ulcer/stasis dermatitis
-painful ulcer on lower extremities

==================================================

Question: Shock- Pathophysiology

Answer: Decreased perfusion to vital organs and tissues

==================================================

Question: Shock- Signs & Symptoms

Answer: Acute kidney injury (first organ to fail)
-Decreased urine output
Elevated lactate

==================================================

Question: Shock- Interventions

Answer: Fluids
Vasopressors
Treat the underlying condition

==================================================

Question: Cardiogenic Shock- Causes) Acute MI

Answer: Decreased cardiac output
Ischemia, Dysrhythmias, Myocarditis, Endocarditis, Cardiomyopathy, HF, MI

==================================================

Question: Cardiogenic Shock- Signs & Symptoms) Heart is Weak

Answer: Low BP, Low Cardiac Output, High HR, Weak/Thready Pulse, Chest Pain, Cool-Pale Skin (Cyanosis), Confusion/Agitation, Crackles, Tachypnea, Oliguria

==================================================

Question: Cardiogenic Shock- Treatment

Answer: 1. Immediate EKG
2. Supplemental Oxygen
3. Pain Control
4. Immediate Re-perfusion - BP Support Meds: Dopamine, Norepinephrine, Dobutamine, Pressors
5. Monitor for tissue perfusion
6. Watch for signs of fluid overload in lungs

==================================================

Question: Hypovolemic Shock- Causes

Answer: -dehydration/fluid volume deficit
-hemorrhage (blunt trauma, pregnancy)
-severe vomiting, diarrhea

==================================================

Question: Hypovolemic Shock- Signs & Symptoms

Answer: Loss of Fluid
Low BP, High HR, Low CO, High SVR, Weak-Thready Pulse, Cool-Pale Skin, Oliguria, Orthostatic Hypotension, Confusion

==================================================

Question: Hypovolemic Shock- Treatment

Answer: 1. Trendlenburg
2. Fluids: NS or LR until blod can be matched
3. Monitor fluid overload (JVD, pulm. edema, RR)
4. Monitor VSq15
5. Supplemental Oxygen
6. Monitor Oxygen
7. Meds: Vasoactive drugs, Desmopressin-DI, Insulin- hyperglycemia, antiemetics/diarrheals

==================================================

Question: Septic Shock- Cause

Answer: Response to untreated infection

==================================================

Question: Septic Shock- Signs & Symptoms

Answer: Vasodilation (blood volume is not diminished)
Persistant low BP, that doesn't respond to IV fluids; warm flushed skin, hyperthermia - may become hypothermic later on, high CO, HIGH HR, high RR, low SVR

==================================================

Question: Septic Shock- Treatment

Answer: Antibiotics within first hour, vasopressors, fluids, and oxygen

==================================================

Question: Neurogenic Shock- Pathophysiology

Answer: Inability of SNS to stimulate nerve impulses

==================================================

Question: Neurogenic Shock- Causes

Answer: Spinal cord injury (T6 or higher), drugs, spinal anesthesia

==================================================

Question: Neurogenic Shock- Signs & Symptoms

Answer: Vasodilation (blood volume is not diminished)
Low BP, Low SVR, Low HR, Low CO, Warm/Dry extremities but cold body, hypothermia

==================================================

Question: Neurogenic Shock- Treatment

Answer: Keep spine immobilized, IV fluids to increased CO; watch fluid overload , vasopressors, atropine to increase HR , assess urine output

==================================================

Question: Anaphylactic Shock- Cause

Answer: allergen via injection, inhalation, oral, contact

==================================================

Question: Anaphylactic Shock- Sign & Symptoms

Answer: Vasodilation (blood volume is not diminished)
Bronchoconstrcition, dyspnea, wheezing, high HR, low BP, sweeling, itchy

==================================================

Question: Anaphylactic Shock- Treatment

Answer: Airway! Trendelenberg, Epinephrine, Albuterol, Remove Allergen

==================================================

Question: Hemodynamic Monitoring: Arterial Line (red)

Answer: Catheter typically radial or femoral artery
Continuous blood pressure
ABG blood draws

==================================================

Question: Hemodynamic Monitoring: Central Venous Pressure (blue)

Answer: Sensor via central line catheter
-sits in a venae cavae
Good indicator of preload and pressure from the right side of the heart
Often used in HF
Normal CVP is 2-6
-Elevated: worsening HF
-Decreased: hypovolemia

==================================================

Question: Hemodynamic Monitoring: Pulmonary Pressure/Pulmonary Wedge Pressure (yellow)

Answer: Sensor via Swan-Ganz catheter
-sits in the pulmonary artery
Good indicator of pulmonary hypertension
Can inflate balloon for pulmonary wedge pressure
-Never inflate for prolonged periods of time
-Never remove specialized syringe to inflate

==================================================

Question: Sinus Tachycardia

Answer: Assess for underlying causes:
-Substances (caffeine, stimulants)
-Medical (infection, anemia)

==================================================

Question: Sinus Bradycardia

Answer: May affect blood pressure
May be expected due to medications ( beta blockers, digoxin)
Symptomatic bradycardia (hypotension leading to dizziness)
-Atropine

==================================================

Question: Supraventricular Tachycardia (SVT)

Answer: Assess for underlying causes.
-Electrolyte imbalances
Vagal/Valsalva maneuver (bear down)
-Baroreceptors cause lowering of the heart rate
-Do not perform in patients with underlying heart conditions.
Adenosine
-Push rapid: half-life <10 seconds
-Central line push preferred
-Chemical cardioversion
-Expect ventricular asystole for a few seconds
Cardioverison (synchronized shock)
-Do not confuse with defibrillation
-Sedate the patient
Cardiac ablation
-Destroys tissue causing electrical activity

==================================================

Question: Atrial Fibrillation

Answer: No discernable/multiple P waves
Risk for clotting/clotting events
-Prevention (aspirin, warfarin, clopidogrel)
-Risk for pulmonary embolism, stroke, heart attack

==================================================

Question: Atrial Flutter

Answer: Sawtooth pattern
Same risks as with atrial fibrillation: clots/clotting events

==================================================

Question: Preventricular Contractions (PVCs)

Answer: Wide QRS complex
May be caused by stimulants
Six or more per minute requires a call to the HCP
Prevent or treat with amiodarone
Assess for hyperkalemia

==================================================

Question: Ventricular Tachycardia

Answer: Code Blue
Pulse or Pulseless
-Assess carotid
Torsades
-Type of V. Tach
Assess for hypomagnesemia
Assess for long QT syndrome
No pulse
-CPR first
-Defibrillation (shock) second

==================================================

Question: Ventricular Fibrillation

Answer: Code Blue
Always pulseless
CPR first
Defibrillation second

==================================================

Question: Asystole

Answer: Code Blue
Flatline
CPR first

==================================================

Question: Code Blue

Answer: V. Tach, V. Fib, Asystole
CPR
-2 inches for an adult
- 1 inch with the palm of hand for children
-1 inch with two fingers for infants
Avoid any pauses in compressions
Defibrillations
-Biphasic: 200 joules
-Monophasic: 360 joules
Medications: epinephrine, amiodarone

==================================================

Question: ST elevation

Answer: Injury to myocardial tissue

==================================================

Question: T wave inversion

Answer: Ischemia to myocardial tissue

==================================================

Question: URTIs - Rhinitis

Answer: Common cold
Viral
Rhinorrhea (runny nose)

==================================================

Question: URTIs - Sinusitis

Answer: Frontal sinus: forehead
Maxillary sinus: zygomatic bone (cheek)

==================================================

Question: URTIs - Pharyngitis

Answer: Sore throat, cough
Bacterial

==================================================

Question: URTIs- Influenza

Answer: Fever, muscle aches, malaise
Viral

==================================================

Question: Strep Throat - Pathophysiology

Answer: Caused by the streptococcus bacterium through droplet transmission
Diagnosed via throat culture

==================================================

Question: Strep Throat - Signs & Symptoms

Answer: Fever, sore throat, enlarged lymph nodes
Tonsillar exudate (whitish tonsils)

==================================================

Question: Strep Throat - Interventions

Answer: Early treatment with antibiotics; prevent progression to rheumatic fever or post-strep glomerulonephritis
Antibiotics may be given even with a negative culture (protect the patient).

==================================================

Question: Rheumatic Fever - Pathophysiology

Answer: Complication secondary to streptococcus infection
May progress to rheumatic heart disease (permanent condition)
Heart damage increases risk for subsequent infections (endocarditis)
Prophylactic antibiotics common for procedures (dental)

==================================================

Question: Rheumatic Fever - Signs & Symptoms

Answer: Systemic inflammation
Muscles, joints, & the brain

==================================================

Question: Rheumatic Fever - Interventions

Answer: Treat strep throat early
Monitor and treat inflammation

==================================================

Question: Epiglottitis - Pathophysiology

Answer: Most common in pediatric clients
Inflammation of the epiglottis
Potential airway loss (medical emergency: prioritization)

==================================================

Question: Epiglottitis - Signs & Symptoms

Answer: Difficulty swallowing and stridor (early signs)
Drooling, loss of consciousness (late signs)
Quieting stridor (late sign) *red flag

==================================================

Question: Epiglottitis - Interventions

Answer: Assess and treat inflammation early
Emergency intubation may be needed if it progresses to an airway obstruction
Keep tracheostomy kit bedside

==================================================

Question: Cystic Fibrosis - Pathophysiology

Answer: Genetic disorder with no cure
Diagnosed via sweat test (measures chloride in the sweat)
Abnormally thick mucus secretions lead to lung disease
May affect multiple organs ( lungs, liver, kidneys, pancreas, intestines)

==================================================

Question: Cystic Fibrosis - Signs & Symptoms

Answer: Cough with or without blood/phlegm
Frequent lung infections
Pancreatitis
Difficulty gaining weight (GI/endocrine related)
Steatorrhea (fatty stools)

==================================================

Question: Cystic Fibrosis - Interventions

Answer: Vaccinations (pneumococcal, flu)
Chest physiotherapy (helps break up mucus); often performed by the respiratory therapist
-Postural drainage (vibrations on the back)
-Flutter valve (blow out; expiratory effort)
Lung transplant (last resort)

==================================================

Question: Asthma- Pathophysiology

Answer: Commonly diagnosed in childhood
Often triggered by environmental factors (pollution, pollen, etc.)
Chronic inflammation and/or constriction of the airway obstruction

==================================================

Question: Asthma- Signs & Symptoms

Answer: Coughing and shortness of breath (dyspnea)
Three or more ED visits in past year
Wheezing (auscultate)
-If diminishing, signs of worsening condition
Loss of airway (status asthmatics): medical emergency

==================================================

Question: Asthma- Interventions

Answer: Reduce triggers (environmental, lifestyle, etc.)
Pharmacological (bronchodilators, anti-inflammatory agents)
Emergency intubation (status asthmatics)

==================================================

Question: Chronic Obstructive Pulmonary Disease (COPD)- Pathophysiology

Answer: Chronic inflammatory disease (bronchitis)
Emphysema (ballooning out of the alveoli, dysfunctional)
Risk with smoking

==================================================

Question: Chronic Obstructive Pulmonary Disease (COPD)- Signs & Symptoms

Answer: Dyspnea, especially exhalation (pursed lip)
Barrel chest (larger diameter anterior-posterior)
Chronic respiratory acidosis
Chronic hypoxia (88-92%)

==================================================

Question: Chronic Obstructive Pulmonary Disease (COPD)- Interventions

Answer: Low oxygen supplementation
Bronchodilators/inhaled glucocorticoids

==================================================

Question: Atelectasis - Pathophysiology

Answer: Partial collapse of the lung (alveoli)
Risk with post-op patients
May lead to pneumonia if not treated or prevented

==================================================

Question: Atelectasis - Signs & Symptoms

Answer: Soft or absent lung sounds at base
Desaturations via pulse ox and/or serum oxygen

==================================================

Question: Atelectasis - Interventions

Answer: Deep breathing and incentive spirometry
Splint chest with a pillow during coughing or painful breathing

==================================================

Question: Pneumonia - Pathophysiology

Answer: caused by the pneumococcal bacterium
Droplet transmission
May lead to fluid accumulation (lung consolidation)

==================================================

Question: Pneumonia - Signs & Symptoms

Answer: Fever, chills, dyspnea, cough, increased WBC
Chest pain (pleuritic); pain upon breathing
Hemoptysis (blood in cough, green

==================================================

Question: Pneumonia - Interventions

Answer: Pleurocentesis/thoracentesis
Chest tube placement to suction out the air

==================================================

Question: Chest tube - Purpose

Answer: Removal of air or fluid from the pleural cavity
Suction out blood (chest cavity/mediastinum) post-cardiac surgery
Oscillating water-seal chamber: normal

==================================================

Question: Chest tube - Complications

Answer: Continuous bubbling in water-seal chamber: not normal (air leak)
No output may be a sign of a clogged tube: a risk for blood build-up
Tube accidentally removed from patient: Apply petrolatum (Vaseline) gauze.
Tube accidentally removed from the chest tube: Place tube in sterile water.
Do not strip a chest tube; you may cause a clot to break.
Do not clamp the chest tube; you may cause a tension pneumothorax.

==================================================

Question: Mechanical Ventilation - Purpose

Answer: Assisted ventilation of oxygen and carbon dioxide
Endotracheal or tracheostomy
Commonly used in acute respiratory distress syndrome (ARDS)

==================================================

Question: Mechanical Ventilation - Settings

Answer: Respiratory Rate
Tidal Volume: Volume in a normal breath
FiO2
PEEP - Keeps alveoli open

==================================================

Question: Mechanical Ventilation

Answer: Can ventilate on 3 modes:
Assist control- Complete control of all four settings, used for critically ill patients and those in the operating room
Synchronized Intermittent Mechanical Ventilation (SIMV)- A demand/backup setting (Example: The ventilator is set at 14 breaths per minute; if patient breaths are below that, the machine will kick in.)
Pressure Support- Also called CPAP setting, provides inward pressure to aid breathing but does not trigger a rate or tidal volume; used before extubation to test for readiness to wean

==================================================

Question: Mechanical Ventilation Complications

Answer: High-Pressure alarm: obstruction, biting tube, mucous plug, kinking tube
Low-Pressure alarm: air leak, disconnection
Ventilator-associated pneumonia (VAP)

==================================================

Question: Ventilator-associated pneumonia (VAP)

Answer: Elevate head of bed
Administer proton-pump inhibitors
Oral care q4 hours
Monitor residuals/distention, and do not overfeed.
Maintain endotracheal cuff pressure

==================================================

Question: BiPAP

Answer: Pressure support during inhalation and exhalation
Commonly the last resort before intubation is necessary
May deliver added oxygen if needed
Improvement of oxygenation is the goal (even if the patient is in restraints, do not think safety; physiology is the priority).

==================================================

Question: CPAP

Answer: Continuous pressure support
Used for sleep apnea

==================================================

Question: Rebreather Mask

Answer: Reservoir bag holds patient's carbon dioxide
Patient breathes back come Co2
Use in respiratory alkalosis (similar to using a brown paper bag).

==================================================

Question: Non-Rebreather Mask

Answer: Carbon dioxide escapes the mask during exhalation
Patient breathes higher concentration of oxygen

==================================================

Question: Simple Mask/Venturi

Answer: Venturi masks utilize color-coded jets that deliver set amounts of oxygen.
Regulate oxygen liter flow via the wall oxygen piping
RN may start without an order; LPN may initiate if order exists.

==================================================

Question: Nasal Cannula

Answer: Use humidification to reduce irritation
RN may start without an order; LPN may initiate if order exists.

==================================================

Question: Early Hypoxia

Answer: Anxiety
Pallor (whitish skin)
Discoloration of mucous membranes (dark-skinned patients)

==================================================

Question: Late Hypoxia

Answer: Delirium
Mottled skin appearance
Clubbing of fingers (chronic hypoxia)

==================================================

Question: Anoxia

Answer: Complete lack of oxygen supply to cellular tissue
After 5 minutes, permanent damage is likely (myocardial infarction, anoxic brain injury)

==================================================

Question: Carbon Monoxide (CO) Poisoning

Answer: CO competes with oxygen in hemoglobin (carboxyhemoglobin)
Leads to oxygen deprivation of tissues
Early sign: a dull headache
Treat with oxygen supplementation

==================================================

Question: Tracheal

Answer: *Normal Lung Sound
High Pitch
Heard over the Trachea

==================================================

Question: Bronchovesicular

Answer: *Normal Lung Sound
Medium to low pitch
Heard over the middle lobes/center of the lungs

==================================================

Question: Bronchial

Answer: *Normal Lung Sound
Medium pitch
Heard over the upper lobes

==================================================

Question: Vesicular

Answer: *Normal Lung Sounds
Low pitch
Heard over the lower lobes

==================================================

Question: Crackles (rales)

Answer: Popping sound (bubble wrap)
Consolidation in the lungs
Fine crackles: pulmonary edema
Coarse crackles: pneumonia

==================================================

Question: Wheezing (Rhonchi)

Answer: Musical in quality
Continuous
Typically louder during expiration
Common in asthma and COPD patients

==================================================

Question: Stridor

Answer: High-pitched sound during inspiration
A sign of obstruction (airway compromise)
Common symptom in laryngotracheobronchitis (croup)

==================================================

Question: Pleural Friction Rub

Answer: Crunching sound
Common symptom in inflammatory diseases of the lungs, especially the pleural lining

==================================================

Question: Allen test

Answer: Test for collateral blood flow (radial and ulnar arteries)
Occlude both arteries, pump first causing pallor, then open one of the arteries
The hand should pink up when one of the arteries is opened
Failed Allen test denotes no radial arterial line in that hand

==================================================

Question: Respiratory Acidosis Pathophysiology

Answer: Retention of Co2
Patient is not breathing enough
Obstruction (COPD, emphysema)
Respiratory distress/ARDS
Brainstem trauma

==================================================

Question: Respiratory Acidosis - Signs & Symptoms

Answer: Hypercapnia
Respiratory distress/bradypnea
High Co2 , Low pH

==================================================

Question: Respiratory Acidosis - Interventions

Answer: Treat the underlying problem
CPAP, BiPAP
Intubation (last resort)

==================================================

Question: Respiratory Alkalosis - Pathophysiology

Answer: Loss of Co2
Patient is breathing too often
Anxiety, fear, pain

==================================================

Question: Respiratory Alkalosis - Signs & Symptoms

Answer: Hypocapnia
Hyperventilation
Paresthesias, dizziness

==================================================

Question: Respiratory Alkalosis - Interventions

Answer: Rebreather mask/brown bag
Treat the underlying condition (pain meds, anxiolytics)

==================================================

Question: Metabolic Acidosis - Pathophysiology

Answer: Decrease in bicarb
Production of too much acid (lactic acidosis, ketoacidosis)
Kidneys not producing enough bicarb ( chronic renal failure)
Long-term diarrhea may cause

==================================================

Question: Metabolic Acidosis - Signs & Symptoms

Answer: Kussmaul breathing (ketoacidosis)
Arrhythmias
Coma, death (if left untreated)

==================================================

Question: Metabolic Acidosis - Interventions

Answer: Correct the underlying problem (provide oxygen if needed)
Intravenous bicarbonate

==================================================

Question: Metabolic Alkalosis - Pathophysiology

Answer: Increase in bicarb
Long-term vomiting may cause (also chronic NG tube suctioning)

==================================================

Question: Metabolic Alkalosis - Signs & Symptoms

Answer: Confusion
Muscle twitching
Paresthesias

==================================================

Question: Metabolic Alkalosis - Interventions

Answer: Treat the underlying problem
Condition is secondary to a broader problem

==================================================

Question: Hypoxemia

Answer: Low oxygenation in the blood
Early signs: pallor, mucous membrane discoloration, anxiety, irritability
Late signs: delirium, begins to affect organs

==================================================

Question: Hypoxia

Answer: Low oxygenation to the tissues
Pulse Ox: Infrared technology reads hemoglobin concentration. Falsely elevated in carbon monoxide poisoning; carboxyhemoglobin
Early signs: shortness of breath, tachypnea, anxiety
Late signs: delirium, clubbing of fingers (chronic)

==================================================

Question: Osmosis:

Answer: Water moves into higher solute concentrations (water follows salt)

==================================================

Question: Diffusion

Answer: Movement of molecules from an area of higher concentration to an area of lower concentration.

==================================================

Question: Hyperosmolar

Answer: Higher concentration than blood

==================================================

Question: Iso-osmolar

Answer: Same concentration as blood

==================================================

Question: Hypo-osmolar

Answer: Lower concentration than blood (diluted)

==================================================

Question: 3% and 7% NaCl (Crystalloid)

Answer: Hypertonic solution (higher concentration than blood)
Used in severe hyponatremia and increased intracranial pressure
Causes water to shift into the vasculature
Risks to dehydrate the surrounding cells
Do not use in hypertension (remember, low salt).

==================================================

Question: 0.9% NaCl (Normal Saline) *Crystalloid

Answer: Isotonic solution (same concentration as blood)
Used in dehydration and hypovolemia (fill up the tank)

==================================================

Question: 0.45% NaCl (Half-Normal Saline) *Crystalloid

Answer: Hypotonic solution (lower concentration than blood)
Often combined with dextrose to maintain iso-osmolality

==================================================

Question: Lactated Ringer's (LR) *Crystalloid

Answer: Isotonic solution
Contains potassium, chloride, sodium, and calcium
Used in trauma, volume loss, and especially burns
Alkalizing agent used to counteract acidosis (contains bicarb)

==================================================

Question: Dextrose 5% and 10% *Crystalloid

Answer: Glucose-containing solution
Becomes hypotonic once the dextrose is metabolized out
Typically combined with other salines
Used in hypoglycemia (often severe infection)
Contained in vitals such as D5 (50%), pushed IV for hypoglycemia protocol

==================================================

Question: Albumin (Colloid)

Answer: Responsible for 60% of blood osmolality (maintaining blood concnetration)
Serum protein created by the liver
Also given via IV like crystalloids
Commonly in liver failure, burn victims, and general hypoalbuminemia

==================================================

Question: Platelets

Answer: Given for thrombocytopenia (cancer, infection, sepsis, liver failure, heparin-induced thrombocytopenia, disseminated intravascular coagulation)AQ

==================================================

Question: Fresh Frozen Plasma

Answer: Contains everything, but red blood cells
Patient has normal hemoglobin, does not require RBCs

==================================================

Question: Packed Red Blood Cells (PRBCs)

Answer: Given for anemia (low hemoglobin)
Hemorrhage

==================================================

Question: Whole Blood

Answer: Entire composition of blood (plasma and RBCs)

==================================================

Question: Transfusion Reaction- Allergic: Minor

Answer: Localized rash (individualized rash), pruritus
-Slow the drip rate and notify the HCP for an antihistamine (diphenhydramine)

==================================================

Question: Transfusion Reaction- Allergic: Major

Answer: Diffuse or generalized rash (all over), difficulty breathing, sweating, complaints of feeling hot
-Stop the drip, flush normal saline through alternate tubing, and call the HCP
-Epinephrine may be needed
-Send remaining blood to blood bank for analysis

==================================================

Question: Hemolytic/Cytotoxic Reaction

Answer: Incompatible blood (ABO mismatch)
Flank/back pain
Fever
Dark-colored urine (cola-colored)
-Stop the drip, flush normal saline through alternate tubing, and call the HCP
-Epinephrine may be needed
-Send remaining blood to blood bank for analysis

==================================================

Question: White Blood Cells (WBCs)

Answer: 5,000-10,000
If high: leukocytosis
-Active infection
If low: neutropenia/leukopenia
-Risk for infection

==================================================

Question: Red Blood Cells (RBCs)

Answer: 4.2-5.5
Reticulocytes (baby RBCs)
If high: polycythermia vera
-Risk for clotting events (MI, PE, CVA)/poor circulation
If low: anemia

==================================================

Question: Hemoglobin (HgB)

Answer: 12-16
Oxygen-carrying component of blood
Transfusion likely under 8 (especially 7)

==================================================

Question: Hematocrit (HcT)

Answer: 37-50
Percentage of RBCs to total blood volume
If high: dehydration, polycythemia, hypovolemia
If low: fluid overload, anemia, hypervolemia

==================================================

Question: Platelets (Plts)

Answer: 150,00-300,000
If low: thrombocytopenia
May be caused by numerous factors (heparin-induced, chemotherapy, bone marrow suppression)
Bleeding risk (no razors, no hard edges, no rectal temps, no enemas, no suppositories, no hard-bristle toothbrushes)

==================================================

Question: Sodium

Answer: 135-145
Water follows salt (osmosis)
If high: hypernatremia
Dehydration
Follow a low-sodium diet
If low: hyponatremia
Fluid retention
Very low sodium (water intoxication) is a potential medical emergency leading to a seizures
Replete with sodium chloride

==================================================

Question: Potassium

Answer: 3.5-5
If high: hyperkalemia
Cardiac dysrhythmias (PVCs)
If low: hypokalemia
May be caused by vomiting, diarrhea, and GI suctioning
Muscle weakness, malaise, dysrhythmias
May lead to digoxin toxicity
Replete with potassium chloride

==================================================

Question: Calcium

Answer: 8-10
If high: hypercalcemia
CNS-related issues (irritability, paresthesias)
May lead to hypercalcemic crisis and death (typically secondary to bone cancer)
If low: hypocalecemia
Chvostek's sign/Trousseau's sign
Muscle tetany
Replete calcium chloride/gluconate (reversal agent for hypermagnesemia as well)

==================================================

Question: Magnesium

Answer: If high: hypermagnesmia
Decrease in deep tendon reflexes
Potential adverse effect of magnesium sulfate for preeclampsia
If low: hypomagnesemia
Muscle weakness or tetany

==================================================

Question: Phosphorus

Answer: Inverted relationship to calcium; one goes up, the other goes down
If high: hyperphosphatemia
May lead to osteopenia/osteoporosis
Carbonated beverages are high in phosphorus (colas)
If low: hypophosphatemia
May be due to alcoholism
Replete with IV phosphorus

==================================================

Question: Glucose

Answer: 70-100
If high: hyperglycemia
-The 3 P's (polydipsia, polyuria, polyphagia)
If low: hypoglycemia
Diaphoresis, tachycardia, clammy skin, confusion, death

==================================================

Question: Albumin

Answer: 3.5-5
If high: hyperalbuminemia
Dehydration
If low: hypoalbuminemia
Malnutrition, liver failure
Replete with IV albumin

==================================================

Question: Creatinine

Answer: 0.1-1.5
If high: azotemia
Acute kidney injury (formerly called acute renal failure)
Chronic renal failure (CKD)

==================================================

Question: BUN (Blood, Urea, Nitrogen)

Answer: 8-20
If high: azotemia
Acute kidney injury/chronic renal failure
Heart failure/dehydration

==================================================

Question: Liver Enzymes (ALT, AST)

Answer: If high: liver injury

==================================================

Question: Troponin

Answer: By-product of dead heart muscle
If high: myocardial injury/infarction

==================================================

Question: Brain Natriuretic Peptide (BNP)

Answer: Hormone released by ventricles when stretched
If high: heart failure

==================================================

Question: Creatine Kinease

Answer: Inflammation of muscles (MI, rhabdomyolysis, autoimmunity)

==================================================

Question: Total Cholesterol: Less than 200

Answer: If high: hypercholesterolemia
Risk for cardiovascular disease (CVD)

==================================================

Question: HDL: More than 40-60

Answer: Good cholesterol (carries away LDL)
Diet and exercises encourage high HDL.

==================================================

Question: LDL: Less than 150

Answer: Bad cholesterol
If high: hyperlipidemia
Risk for CVD

==================================================

Question: D-Dimer

Answer: Indicates abnormal clotting (thrombus, coagulation, active clotting)
Elevation may indicate DVT/PE/DIC

==================================================

Question: C-Reactive Protein

Answer: Indicates inflammation in the body (acute)

==================================================

Question: Erythrocyte Sedimentation Rate (ESR)

Answer: Indicates inflammation in the body (chronic)

==================================================

Question: aPTT

Answer: normal: 20-40 seconds (heparin)
therapeutic: 40-80 seconds

==================================================

Question: PT/INR

Answer: normal: 0.5-1.5 seconds
therapeutic: 2-3 seconds

==================================================

Question: digoxin

Answer: sub-therapeutic/normal: 0
therapeutic: 0.5-1.5

==================================================

Question: lithium

Answer: sub-therapeutic/normal: 0
therapeutic:0.6/1.4

==================================================

Question: theophylline

Answer: sub-therapeutic/normal: 0
therapeutic: 10-20

==================================================

Question: phenytoin

Answer: sub-therapeutic/normal: 0
therapeutic: 10-20

==================================================

Question: metoprolol

Answer: Beta blocker - suffix "lol"
Lowers heart rate and decrease force of contraction
Treats hypertension, dysrhythmias
-May mask signs of hypoglycemia
-Careful with asthma clients
-Contraindicated in bradycardia & heart block

==================================================

Question: losartan

Answer: Angiotensin Receptor Blocker - suffix "sartan"
Decreases retention of water and sodium
Treats hypertension and diabetic neuropathy
-Used if ACE inhibitor cannot be tolerated
-Hyperkalemia risk

==================================================

Question: pentoxifylline

Answer: Vasodilator
Smooth muscle relaxation (blood vessels)
Pain due to PAD (intermittent claudication)

==================================================

Question: verapamil, diltiazem, nicardipine

Answer: Calcium channel blocker - suffix "dipine"
Decreases force of contraction
Treats hypertension, angina, cardiac dysrhythmias
-No grapefruit juice

==================================================

Question: clonidine

Answer: Alpha agonist
Lowers heart rate, vasodilators
Treats hypertension
-Side effects: dry mouth

==================================================

Question: nitroglycerin, isosorbide dinitrate

Answer: Nitrate
Vasodilator, decreases preload
Treats hypertension and angina
-Sublingual: 3 times, 5 minutes apart
-If second dose fails, head to ER
-Risk for orthostatic hypotension
-Risk for headache; treat with acetaminophen
-Do not take with erectile dysfunction meds (sildenafil)
-Given via IV for severe HTN

==================================================

Question: captopril

Answer: Angiotensin Converting Enzyme (ACE) Inhibitor ; suffix "pril"
Decreases retention of water and sodium
Treats hypertension
-Side effect: cough
-Hyperkalemia risk

==================================================

Question: hydralazine

Answer: Vasodilator
Strong vasodilator, decreases preload
Hypertensive crisis
-Oral or IV

==================================================

Question: nitroprusside

Answer: Nitrate
Strong vasodilator, decreases preload
Hypertensive crisis
-Slowly lower BP or MAP (too quickly may cause body to go into shock)
-May cause cyanide poisoning (caution)

==================================================

Question: dopamine

Answer: Catecholamine/Vasopressor
Increases force of contraction and heart rate
Treats shock
-Close hemodynamic monitoring necessary (ICU)
-Renal dose: low dose to increase GFR

==================================================

Question: dobutamine

Answer: Adrenergic agonist/vasopressor/positive inotrope
Increases force of contraction and heart rate
Shock, heart failure

==================================================

Question: epinephrine

Answer: Catecholamine/vasopressor/adrenergic agonist
Cardiovascular and CNS stimulant, increases force of contraction
Shock, cardiac emergency
-EpiPen for anaphylaxis; head to EF after administration
-Injection can go through clothes
-Close hemodynamic monitoring
-Risk for hyperglycemia

==================================================

Question: norepinephrine

Answer: Catecholamine/vasopressor/adrenergic agonist
Strong vasoconstrictor
Shock
-May cause limb ischemia (purple toes, necrosis)

==================================================

Question: Inotropic

Answer: contractility (dopamine)

==================================================

Question: Chronotropic

Answer: time (digoxin, atropine)

==================================================

Question: Dromotropic

Answer: conduction (nervous system of heart)

==================================================

Question: hydrochlorothiazide

Answer: Thiazide diuretic
Increases urine production
Treats hypertension, fluid overload
-Check I/O, Weight Changes, and Electrolyte Imbalances

==================================================

Question: furosemide, bumetanide

Answer: Loop diuretic
Increases urine output including sodium and potassium
Treats fluid volume excess, edema, pulmonary edema, hypertension
-Contraindicated in chronic kidney disease (CKD)
-Risk for hypokalemia
-Push slowly; risk for ototoxicity

==================================================

Question: triamterene, eplerenone, spironolactone

Answer: Potassium-sparing diuretic
Increases urine output with retention of potassium
Treats fluid volume excess, edema, hypertension
-Risk for hyperkalemia (avoid salt substitutes, avoid bananas)

==================================================

Question: mannitol

Answer: Osmotic diuretic
Increases urine output
Treats increased intracranial pressure and intraocular pressure
-Contraindicated in CKD

==================================================

Question: acetazolamide

Answer: Carbonic anhydrase inhibitor
Increases urine output with sodium, potassium, and bicarb
Used for glaucoma, pulmonary edema
-Risk for orthostasis

==================================================

Question: digoxin for HF

Answer: cardiac glycoside
lowers heart rate, increases force of contraction
used for symptomatic heart failure, atrial fibrillation
-do not administer if HR is below 60
-risk for digitalis toxicity (anorexia, N/V/D, headache); hypokalemia may trigger toxicity

==================================================

Question: lovastatin

Answer: Statin; suffix "statin"
Increases HDL, lowers LDL, lowers cholesterol
Used for hypercholesterolemia and hyperlipidemia
-Contraindicated in liver disease
-Risk for rhabdomyolysis (medical emergency)

==================================================

Question: fenofibrate

Answer: Fibrate
Increases HDL, lowers LDL, lowers cholesterol
Used for hypercholesterolemia and hyperlipidemia

==================================================

Question: niacin

Answer: Nicotinic acid (vitamin B3)
Increases HDL, lowers LDL, lowers cholesterol
Used for hypercholesterolemia and hyperlipidemia
-Over the counter supplement
-Added to other therapies

==================================================

Question: Hypocoagulation

Answer: thin blood

==================================================

Question: Hypercoagulation

Answer: thick blood

==================================================

Question: Heparin, enoxaparin

Answer: Anticoagulant; suffix "parin"
Blocks mechanism in clotting cascade
Prevention of blood clots (DVT)
-Lab values: aPTT
-Antidote: protamine sulfate
-SQ injection: Rotate injection sites; never massage
-For inpatient use
-Risk for heparin-induced thrombocytopenia (HIT)

==================================================

Question: warfarin

Answer: Anticoagulant
Blocks mechanism in clotting cascade
Prevention of blood clots (atrial fibrillation)
-Lab values: PT/INR
-Antidote: vitamin K
-For outpatient or inpatient use
-Take warfarin while going off of heparin (use will overlap)

==================================================

Question: clopidogrel, aspirin

Answer: Antiplatelet
Block platelet aggregation
Prophylaxis for clotting events (MI, PE, CVA)
-Risk for salicylate poisoning (aspirin)
-Risk for bleeding (assess)

==================================================

Question: alteplase, tissue plasminogen activator (tPA)

Answer: Thrombolytic/fibrinolytic
Enzyme that breaks down clots
Diagnosed acute ischemic stroke, myocardial infarction, pulmonary embolism
-Given within the golden window (3 hours)
-Hemorrhagic stroke must be ruled out (obtain CT of head)
-Caution in the elderly

==================================================

Question: aminocaproic acid

Answer: Hemostat
Aids in clotting cascade, prevents breakdown of formed clots
Used for hemorrhage, DIC, controlling bleeding, overdose of fibrinolysis
-Commonly used post-op to prevent hemorrhage

==================================================

Question: quinidine, procainamide

Answer: Antiarrhythmic
Decreases conduction
Used for atrial dysrhythmias

==================================================

Question: lidocaine

Answer: Antiarrhythmic
Decreases threshold or membrane response
Used in ventricular dysrhythmias (V.fib)
-Lidocaine toxicity: CV/CNS

==================================================

Question: amiodarone

Answer: Antiarrhythmic
Prolongs action potential duration
Used in life-threatening dysrhythmias
-Prophylaxis for fatal rhythms
-Risk for pulmonary toxicity

==================================================

Question: adenosine

Answer: Antiarrhythmic
Decreases conduction through AV node
Used in supraventricular tachycardia (SVT) and ventricular tachycardia
-Hemodynamic monitoring required (keep crash cart close)
-Central preferred, peripheral allowed
-Push fast followed by 10 mL NS flush (FAST)

==================================================

Question: sodium-polystyrene sulfonate

Answer: Potassium removing resin
Causes excretion of potassium in stool
Used for hyperkalemia
-Slow onset
-Assess for constipation
-Hypokalemia risk
-Oral or by enema

==================================================

Question: Addison's disease - Pathophysiology

Answer: Adrenal insufficiency
Hypocortisolism/hypoadrenalism

==================================================

Question: Addison's disease - Signs & Symptoms

Answer: Fatigue, weakness
Weight loss, anorexia
Increased pigmentation of skin
Painful muscles/joints
Inability to cope with stress, intolerance to cold
Hyponatremia (salt cravings), hyperkalemia

==================================================

Question: Addison's disease - Interventions

Answer: Lifelong cortisol replacement

==================================================

Question: Addisonian Crisis - Pathophysiology

Answer: Medical emergency
Cortisol levels dangerously low
Often triggered by infection or stress

==================================================

Question: Addisonian Crisis - Signs & Symptoms

Answer: Hypotension
Nausea/Vomiting
Fever, chills
Skin rash

==================================================

Question: Addisonian Crisis - Interventions

Answer: Immediate cortisol administration

==================================================

Question: Cushing's Syndrome - Pathophysiology

Answer: Elevated cortisol levels
Hypercortisolism/hyperadrenalism

==================================================

Question: Cushing's Syndrome - Signs & Symptoms

Answer: Weight gain, central obesity (abdominal)
Moon face
Thinning skin, easily bruised
Fatigue, muscle weakness
Depression, anxiety
Hypertension
Hypernatremia (polydipsia), hypokalemia

==================================================

Question: Cushing's Syndrome - Interventions

Answer: Stop steroid medications
Removal of the adrenal glands (adrenalectomy)

==================================================

Question: Hyperparathyroidism - Pathophysiology

Answer: Excessive release of parathyroid hormone
Hypercalcemia caused by the body pulling calcium from the bones

==================================================

Question: Hyperparathyroidism - Signs & Symptoms

Answer: CNS issues (irritability, fatigue, confusion)
Kidney stones
Osteopenia/osteoporosis: may lead to fractures

==================================================

Question: Hyperparathyroidism - Interventions

Answer: Pharmacological (calcitonin, bisphosphonates)
Resection of the parathyroid

==================================================

Question: Hypoparathyroidism - Pathophysiology

Answer: Decreased production of parathyroid hormone
Leads to hypocalcemia
Common complication post-thyroidectomy

==================================================

Question: Hypoparathyroidism - Signs & Symptoms

Answer: Muscle tetany, cramping
Paresthesias
Chvostek's sign/Trousseau's sign

==================================================

Question: Hypoparathyroidism - Interventions

Answer: Pharmacological (calcitriol, vitamin D, calcium gluconate)
Decrease intake of phosphorus

==================================================

Question: Hyperthyroidism - Pathophysiology

Answer: Decreased thyroid-stimulating hormone (TSH) from the brain
Excessive release of thyroid hormones
Radioactive iodine uptake test (diagnostic)

==================================================

Question: Hyperthyroidism - Signs & Symptoms

Answer: Body function sped up (metabolism)
Weight loss
Heat intolerance
Thyroid storm (thyrotoxicosis): medical emergency

==================================================

Question: Hyperthyroidism - Interventions

Answer: Pharmacological (methimazole, propythiouracil, iodine)
Radioactive iodine: results in ablation of thyroid function
Thyroidectomy

==================================================

Question: Hypothyroidism - Pathophysiology

Answer: Increased TSH
Decreased production of thyroid hormones

==================================================

Question: Hypothyroidism - Signs & Symptoms

Answer: Fatigue
Cold intolerance
Weight gain
Muscle weakness
Myxedema coma (throat tightness): medical emergency

==================================================

Question: Hypothyroidism - Interventions

Answer: Lifelong hormone replacement

==================================================

Question: Graves' Disease - Pathophysiology

Answer: Autoimmune disorder leading to overactivity of the thyroid gland
Mimics hyperthyroidism
More common in women over 20

==================================================

Question: Graves' Disease - Signs & Symptoms

Answer: Similar to hyperthyroidism
Exothalmos (bulging eyeballs)

==================================================

Question: Graves' Disease - Interventions

Answer: Similar to hyperthyroidism
Immunomodulators

==================================================

Question: Hashimoto's Thyroiditis - Pathophysiology

Answer: Autoimmune disorder leading to underactivity of the thyroid gland
Mimics hypothyroid
More common in women
May be caused by high intake of selenium or iodine

==================================================

Question: Hashimoto's Thyroiditis - Signs & Symptoms

Answer: Similar to hypothyroidism

==================================================

Question: Hashimoto's Thyroiditis - Interventions

Answer: Similar to hypothyroidism

==================================================

Question: Goiter - Pathophysiology

Answer: Enlarged thyroid
Lack of iodine in the diet
Tumor or nodules on thyroid

==================================================

Question: Goiter - Signs & Symptoms

Answer: Visible enlargement of the neck
May be benign
Dizziness when raising arms above heads
Dysphagia
Respiratory distress: medical emergency

==================================================

Question: Goiter: Interventions

Answer: Monitoring
Surgical

==================================================

Question: Hypoglycemia - Pathophysiology

Answer: Not enough glucose
Using ginseng while on insulin
Beta blockers with insulin
Diabetes mellitus (DM) patient exercising more than normal
Too much insulin

==================================================

Question: Hypoglycemia - Signs & Symptoms

Answer: Hypotension, tachycardia
Anxiety, diaphoresis
Cold, clammy
CNS issues (irritability, fatigue)

==================================================

Question: Hypoglycemia - Interventions

Answer: 15/15 rule
-Intervention for slight hypoglycemia
-15g of carbs followed by a 15-minute assessment
Simple carbs (juice)
D5/dextrose (IV infusion or push)
Glucagon (severe hypoglycemia)

==================================================

Question: Hyperglycemia - Pathophysiology

Answer: Diabetes mellitus (DM)
Total parenteral nutrition (TPN)
Long-term steroid use

==================================================

Question: Hyperglycemia - Signs & Symptoms

Answer: 3 P's - (polydipsia, polyuria, polyphagia)

==================================================

Question: Hyperglycemia - Interventions

Answer: Diet and exercise
Oral antidiabetic agents
Insulin

==================================================

Question: Diabetes Type 1

Answer: insulin dependent
possibly caused by coxsackievirus
typically presents in children (juvenile)
no production of insulin from pancreas

==================================================

Question: Diabetes Type 2

Answer: Insulin resistant
Obesity (poor cellular response to insulin)
Lack of exercise (sedentary)
Poor diet

==================================================

Question: Dawn Phenomenon

Answer: Higher glucose level in morning

==================================================

Question: Somogyi effect

Answer: Rebound hyperglycemia due to hypoglycemic event overnight
Encourage checking glucose at night and bedtime snack

==================================================

Question: Comorbidity with Cardiovascular Disease (CVD)

Answer: May lead to CVD, increasing risk of myocardial infarction

==================================================

Question: Diabetic nephropathy

Answer: May lead to chronic kidney disease

==================================================

Question: Diabetic retinopathy

Answer: Begins with blurred vision
Patient may lost vision

==================================================

Question: Foot ulcers/infection

Answer: Decreased sensation due to arterial insufficiency
Wet-to-dry dressings for open wounds
Prevention is key

==================================================

Question: Diabetic Ketoacidosis (DKA) - Pathophysiology

Answer: Glucose cannot get into cells for energy
Body switches to fat energy (breakdown of ketones)
Uncontrolled type 1 DM

==================================================

Question: Diabetic Ketoacidosis (DKA) - Signs & Symptoms

Answer: 3 P's
Fruit-scented breath
Nausea/Vomiting, Weakness
Kussmaul breathing (fast and deep)
-Compensatory for metabolic acidosis)

==================================================

Question: Diabetic Ketoacidosis (DKA) - Diagnostic Testing

Answer: Glucose level >600
Ketones in urine

==================================================

Question: Diabetic Ketoacidosis (DKA) - Interventions

Answer: Fluid (priority) and electrolyte replacement
Insulin

==================================================

Question: Hyperosmolar Hyperglycemic Nonketotic Syndrome (HHNS) - Pathophysiology

Answer: Kidneys excrete too much water in attempt to rid glucose
Dehydration
Prevalent in type 2 DM
Prevalent in elderly (decreased thirst mechanism)

==================================================

Question: Hyperosmolar Hyperglycemic Nonketotic Syndrome (HHNS) - Signs & Symptoms

Answer: Glucose > 600
3 P's
Hot and dry (may see fever)
Sleepy and confused
May lead to seizures, coma, and death

==================================================

Question: Hyperosmolar Hyperglycemic Nonketotic Syndrome (HHNS) - Interventions

Answer: Fluids (priority)
Insulin
Potassium may be needed

==================================================

Question: Glycosylated Hemoglobin (A1C)

Answer: Gives a 3-month outlook on glucose management
Displays adherence of meds, diet, and exercise
Higher A1C indicates unregulated diabetes

==================================================

Question: Basal Dose/Scheduled Dose

Answer: Outside the scope of the nurse to change scheduled medications
Doses meant to stabilize the patient long term

==================================================

Question: Prandinal Dose

Answer: Extra dose of insulin at mealtime
15g of carbs equal 1 carbohydrate exchange
Typically 1 unit per exchange

==================================================

Question: SICK Day Rules

Answer: S - Sugar: Check more often (q4hr)
I - Insulin: May need more
C - Carbs: Pay attention to diet.
K - Ketones: Watch for signs of DKA.

==================================================

Question: Insulin Infusion Pump

Answer: Do not confuse with subQ
Needle change every 3 days
Sterile technique
Cleanse insertion site with alcohol

==================================================

Question: Diabetes Insipidus - Pathophysiology

Answer: Lack of antidiuretic hormone (ADH)

==================================================

Question: Diabetes Insipidus - Signs & Symptoms

Answer: Frequent urination
Dehydration leading to hypovolemia
May lead to hypotension (dizziness)
Rebound tachycardia
Hyperosmolar blood: concentrated lab values

==================================================

Question: Diabetes Insipidus - Interventions

Answer: Pharmacological (vasopressin, desmopressin)

==================================================

Question: Syndrome of Inappropriate Antidiuretic Hormone (SIADH) - Pathophysiology

Answer: Excessive release of ADH

==================================================

Question: Syndrome of Inappropriate Antidiuretic Hormone (SIADH) - Signs & Symptoms

Answer: Decreased urine output (oliguria)
Fluid retention leading to hypervolemia
May lead to hypertension
Hypo-osmolar blood: diluted lab values

==================================================

Question: Syndrome of Inappropriate Antidiuretic Hormone (SIADH) - Interventions

Answer: Water restriction (ice chips)
Diuretics
Vasopressin Antagonist

==================================================

Question: prednisone, hydrocortisone

Answer: Glucorticosteroid, mineralocorticosteriod, hormone agonist
Replacement hormone, anti-inflammatory agent, suppresses immune system
Used for Adrenal insufficiency (Addison's disease), inflammation, and organ transplants
-Short-term side effects: CNS stimulation
-Long-term side effects: Immunosuppression, Weight gain(water), Hyperglycemia, Osteoporosis

==================================================

Question: Aminoglutethimide

Answer: steroid hormone antagonist
slows down the production of adrenal steroids
used for hypercortisolism (Cushing's syndrome)
-Short-term therapy until surgery
-May cause orthostatic hypotension

==================================================

Question: aspart

Answer: rapid acting insulin
causes glucose to move into the cells
used for type 1 and 2 diabetes
-may cause hypokalemia
-may be pushed with D5 for hyperkalemia
-risk for hypoglycemia

==================================================

Question: lispro
glulisine
regular insulin

Answer: short acting insulin
causes glucose to move into the cells
used for type 1 and 2 diabetes
-may cause hypokalemia
-may be pushed with D5 for hyperkalemia
-risk for hypoglycemia

==================================================

Question: NPH/isophane

Answer: intermediate acting insulin
causes glucose to move into the cells
used for type 1 and 2 diabetes
-may cause hypokalemia
-may be pushed with D5 for hyperkalemia
-risk for hypoglycemia

==================================================

Question: Detemir, glargine

Answer: long acting insulin
causes glucose to move into the cells
used for type 1 and 2 diabetes
-may cause hypokalemia
-may be pushed with D5 for hyperkalemia
-risk for hypoglycemia

==================================================

Question: glyburide, glipizide

Answer: Sulfonylureas
Stimulates pancreatic cells to release insulin, decreases sugar release insulin, decreases sugar release by liver
Used for Type 2 Diabetes
-Cross hypersensitivity (allergy) with sulfa antibiotics (SMZ TMP)
-Contraindicated in severe liver and renal disease

==================================================

Question: metformin

Answer: Oral antidiabetic agent
Decreases sugar release by liver, increases cell sensitivity to insulin
Used for Type 2 Diabetes
-Nephrotoxic, hold dose for diagnostics with contrast (CT, cardiac cath, etc.)
-Contraindicated in severe liver and renal disease

==================================================

Question: exenatide

Answer: Antidiabetic agent
Increases release of insulin from pancreas, decreases release of glucagon by pancreas
Used for Type 2 Diabetes
-SubQ injection

==================================================

Question: glucagon

Answer: Glucose elevator
Stimulates the release of glucose by the liver
Severe hypoglycemia
-May be given IM for outpatient emergencies (similar to EpiPen, stick through clothes into thigh)

==================================================

Question: somatropin

Answer: Growth hormone
Stimulates growth when epiphyses of bones not closed
Low intrinsic GH in children, Turner's syndrome

==================================================

Question: vasopressin

Answer: Antiduretic hormone
Retention of fluid, vasocontricts
Diabetes insipidus, code situations

==================================================

Question: levothyroxine (T4)

Answer: Thyroid hormone
Stimulates metabolism
Hypothyroidism
-Take in AM on empty stomach
-Lifelong therapy
-Watch for thyroid storm

==================================================

Question: propylthiouracil (PTU), iodine, methimazole

Answer: Antithyroid compound
Inhibits thyroid hormons (T4 and T3)
Used for hyperthyroidism, thyrotoxic crisis, Grave's disease
-Iodine may be radioactive (destroys tissue), but not a cure
-If radioactive, caution to surrounding people for 72 hours
*Caution in bodily fluids (bathroom use)
-Risk for neutropenia with PTU

==================================================

Question: Calcitonin / Alendronate / Risedronate

Answer: Antihypercalcemic, bisphosphonates
Inhibits body's ability to pull calcium from the kidneys
Used for hyperparathyroidism, hypercalcemia, Paget;s disease, osteopenia, osteoporosis
-Do not lie down after taking (GERD risk).
-Take bisphosphonates on empty stomach
-Bisphosphonates commonly taken in postmenopausal women

==================================================

Question: calcitritol, vitamin D

Answer: Antihypocalcemic
Stimulates bone growth
Used for hypoparathyroidism, hypocalcemia, bone disease
-UV light (sunlight) needed to work

==================================================

Question: Hepatology/Liver Anatomy

Answer: Blood through portal liver vein rich in nutrients from GI tract
Portal vein and hepatic artery deliver blood to Kupffer cells for filtration

==================================================

Question: Hepatology/Liver Physiology

Answer: Glucose metabolism
Ammonia conversion
Protein metabolism
Fat metabolism
Vitamin and iron storage
Bile formation
Medication metabolism

==================================================

Question: Hepatology/Liver Interventions

Answer: Lifelong cortisol replacement

==================================================

Question: Hepatitis - Pathophysiology

Answer: Caused by a virus (HAV, HBV, HCV)
Hep A (HAV): transmitted by food (contact, uncooked food, fecal-oral)
Hep B (HBV): transmitted via bodily fluids (blood)
Hep C (HCV): transmitted via bodily fluids (blood)
Subsequent hepatitis infections (Hep D and Hep E) are secondary to HBV or HCV

==================================================

Question: Hepatitis - Signs & Symptoms

Answer: Fatigue
Nausea/vomiting
Hep C may lead to liver damage and liver cancer

==================================================

Question: Hepatitis - Interventions

Answer: Vaccination (Hep A and Hep B)
Treat the symptoms and ride it out

==================================================

Question: Liver Failure/Cirrhosis - Pathophysiology

Answer: Late stages of scarring (fibrosis) of the liver
Hardening and thickening
Chronic alcohol (ETOH) use
Hepatitis
Fatty liver disease

==================================================

Question: Liver Failure/Cirrhosis - Signs & Symptoms

Answer: Easy bruising and bleeding
Jaundice (icterus) of skin and sclera
Ascites and swelling of the legs
Muscle wasting
Decrease in chest/axillary hair
Late stage cirrhosis; fetor hepaticus (musty breath smell) identifies increased blood ammonia

==================================================

Question: Liver Failure/Cirrhosis - Diagnositics

Answer: Liver panel (ALT, AST)
Bilirubin levels (elevated)
Clotting factors (elevated)

==================================================

Question: Liver Failure/Cirrhosis - Interventions

Answer: Avoid alcohol
Weight loss
Liver transplant

==================================================

Question: Hepatic Encephalopathy

Answer: *Liver Complications
Build-up of toxins normally cleaned by the liver (ammonia)
Leads to confusion, drowsiness, slurred speech, and delirium

==================================================

Question: Portal Hypertension

Answer: *Liver Complications
Cirrhosis slows flow of blood through portal vein
Leads to splenomegaly, esophageal varices, gastric varices
May require a transjugular intrahepatic portosystemic shunt (TIPS) surgery

==================================================

Question: Pancreatitis - Pathophysiology

Answer: Pancreatic enzymes become activated while inside the organ, causing damage
Alcoholism, smoking
Cholelithiasis (gallstones) most common cause of acute pancreatitis in adults
Cystic fibrosis

==================================================

Question: Pancreatitis - Signs & Symptoms

Answer: Upper abdominal pain radiating to the back, worse after eating
Nausea and vomiting
Fever
Cullen's sign (superficial edema and bruising at the umbilicus)
Grey-Turner's sign (bruising on the flank, sign of bleeding)

==================================================

Question: Pancreatitis - Diagnostic Testing

Answer: Elevated amylase and lipase
CT/MRI
Endoscopic retograde cholangiopancreatography (ERCP)
-May include fluoroscopy

==================================================

Question: Pancreatitis - Interventions

Answer: Bowel rest for the pancreas (NPO)
High-carb diet with no fatty acids when diet is restored
Pain meds
IV fluids
Antibiotics
Whipple procedure (pancreaticoduodenectomy)

==================================================

Question: Cholangitis - Pathophysiology

Answer: Infection of the common bile duct
Medical emergency
Secondary to cholelithiasis/cholecystitis

==================================================

Question: Cholangitis - Signs & Symptoms

Answer: Charcot's triad:
Jaundice (rapid onset)
Abdominal pain
Fever

==================================================

Question: Cholangitis - Interventions

Answer: ERCP with lithotripsy

==================================================

Question: Chloelithiasis (Gallstones)

Answer: Stones are usually made of bile
Risk factors: female, fertile, 40, overweight
Diagnosed by ultrasound
Lithotripsy may be effective in breaking up stones
Ursodiol used to break up stones
May lead to infection

==================================================

Question: Cholecystitis

Answer: Right upper quadrant (RUQ) rebound tenderness (Murphy's sign)

==================================================

Question: Chloecystectomy

Answer: Typically laproscopically
Low-fat diet post-surically

==================================================

Question: Stomatitis

Answer: Inflamed sore in the mouth
Avoid spicy food
Avoid hot and cold

==================================================

Question: gastroesophageal reflux disease (GERD)

Answer: do not lie supine after meals; sit up 3-4 hours after eating
small frequent meals
no spicy food, no caffeine, no alcohol, no smoking

==================================================

Question: fundoplication

Answer: surgical wrap of the fundus of the stomach around the esophagus
treats advanced GERD or hiatal hernia

==================================================

Question: gastrectomy (subtotal or total)

Answer: loss of intrinsic factor (needed for B12 absorption)
patients require B12 (cobalamin) injections for life; can lead to pernicious anemia if B12 not administered

==================================================

Question: dumping syndrome

Answer: ingest fluids between meals, not with
risk after gastric bypass (gastrectomy)

==================================================

Question: pyloric stenosis

Answer: narrowing of pylorus (bottom of the stomach)
infants: projectile vomiting after feeding
corrected surgically

==================================================

Question: achalasia

Answer: abnormal narrowing of the lower part of the esophagus
dysphagia is common
dilation of the esophagus on the top
barium swallow to diagnose

==================================================

Question: Nasogastric (NG) tube

Answer: Measure from the nose to the back of the throat to the xiphoid process
Placed by RNs or LPNs with specialized certification
OG tube passed through mouth instead of nose
Cannot use without x-ray confirmation

==================================================

Question: Sengstaken-Blakemore tube

Answer: Used in ruptured esophageal or gastric varcies

==================================================

Question: PEG tube

Answer: Typically placed when patients cannot feed orally
Common in post-stroke or comatose patients

==================================================

Question: Stomach Decompression

Answer: Suction to the wall for decompression
Initiated for GI bleeding, pancreatitis, etc. (stop bowel movements)
Document color and amount as output (UAP may perform)
Chronic suctioning may lead to hypokalemia

==================================================

Question: Tube Feedings

Answer: Feedings have 24-hour shelf life
Monitor for signs of intolerance (nausea, bloating, fullness)
Keep head of bed (HOB) elevated at all times during feedings
Assessing residual and effectiveness of feedings:
-50% or more residual being aspirated: discard, halve the rate on pump
-50% or less residual being aspirated: push back in, keep the rate on pump
Confirmation
-Aspiration of contents of pH
-Gold standard is x-ray
Feedings cannot begin until x-ray confirmation

==================================================

Question: total parenteral nutrition (TPN)

Answer: Used when oral and NG tube intake are not possible
Intravenous nutrition
Risk for hyperglycemia, infection, and fluid overload

==================================================

Question: Appendicitis - Pathophysiology

Answer: Inflammation and infection
May lead to rupture

==================================================

Question: Appendicitis - Signs & Symptoms

Answer: Right lower quadrant (RLQ) pain - McBurney's point
Nausea/vomiting
Fever

==================================================

Question: Appendicitis - Interventions

Answer: Appendectomy
Easy delegation to LPN or new RN

==================================================

Question: Peritonitis - Pathophysiology

Answer: Risk of peritoneal dialysis
Sign of rupture of organs (medical emergency)
Sign of internal bleeding (medical emergency)

==================================================

Question: Peritonitis - Signs & Symptoms

Answer: Abdominal pain and distention
Rigid/board-like abdomen
Cloudy output (peritoneal dialysis)
May lead to sepsis/septic shock

==================================================

Question: Peritonitis - Interventions

Answer: Surgical abdominal washout
Antibiotics

==================================================

Question: Inflammatory Bowel Disease - Pathophysiology

Answer: Colitis, ulcerative colitis, gastroenteritis, diverticulitis, Crohn's disease
Bacterial, viral, fungal, autoimmune

==================================================

Question: Inflammatory Bowel Disease - Signs & Symptoms

Answer: Abdominal cramping, diarrhea
Blood in stool (fecal occult)
Change in bowel habits

==================================================

Question: Inflammatory Bowel Disease - Interventions

Answer: Avoid tobacco, caffeine, alcohol, and tea (GI stimulants)
Avoid popcorn, nuts, and gas-producing foods
Low-residue diet while in hospital (no need at discharge)
-Less frequent and looser stool
No gluten (celiac disease)

==================================================

Question: GI Bleeding - Pathophysiology

Answer: Common cause is helicobacter pylori
Peptic ulcers, duodenal ulcers

==================================================

Question: GI Bleeding - Signs & Symptoms

Answer: Upper GI: black, tarry stools (melena)
Lower GI: bright red stool
Esophagogastroduodenoscopy (EGD) to diagnose upper GI
Colonscopy to diagnose lower GI

==================================================

Question: GI Bleeding - Interventions

Answer: NPO until bleeding is controlled

==================================================

Question: Bowel Obstruction - Pathophysiology

Answer: Fecal obstruction
Intussusception (common in pediatric patients)
Volvulus (twisting of the bowels)

==================================================

Question: Bowel Obstruction - Signs & Symptoms

Answer: Sudden onset of abdominal cramps and vomiting
Drawing of legs up to chest (intussusception)
Watery discharge with no stool
Can lead to infection or perforation if left untreated

==================================================

Question: Bowel Obstruction - Interventions

Answer: Push fluids (2-3 L per day is recommened for any human being)
Increase fiber for constipation
Laxatives and enemas
Air enemas (intussusception)

==================================================

Question: Dehiscence

Answer: Montgomery strap used to prevent
Opening of a surgical wound
Intestines may protrude from opening
Apply gentle pressure and call HCP

==================================================

Question: Hemochromatosis

Answer: Abnormal collection of iron in organs

==================================================

Question: Wilson's Disease

Answer: Abnormal collection of copper in organs

==================================================

Question: Colostomy/Ileostomy - Indications

Answer: Inflammatory bowel diseases (providing bowel rest)
Colorectal cancer (bypass resected bowel)

==================================================

Question: Colostomy/Ileostomy - Stoma Care

Answer: Swelling and slight bleeding are normal after placement
Prevent excoriation post-op
Red or pink is normal color for stoma
Purple, maroon, or black is sign of cyanosis
Refer to stoma nurse for body image issues

==================================================

Question: Colostomy/Ileostomy - Diet

Answer: Formed stool: colostomy
Loose stool: ileostomy
Roughage may obstruct the stoma (caution)
Ileostomies require increased water intake

==================================================

Question: Colonscopy - Indications

Answer: Age 50 or older
Every 5 years (more frequently if problems present)
Secondary prevention (screening)
Early identification of colorectal cancer (polyps)

==================================================

Question: Colonscopy - Procedure Care

Answer: Bowel prep (polyethylene glycol)
NPO (8-12 hours prior)
Strong benzodiazepine (midazolam) with fentanyl (moderate sedation)
-Anterograde amnesia is the goal (forget the procedure)
RN monitors vital signs, especially the respiratory rate and O2 saturation

==================================================

Question: Colonscopy - Post-procedure Care

Answer: Same-day procedure (patient must urinate before discharge)
Slowly advance diet after procedure

==================================================

Question: aluminum, hydroxide, sucralfate (*Upper GI Tract)

Answer: antacid
increases pH within stomach
used for GERD, esophagitis, hiatal hernia, and peptic ulcers
-take separate from other drugs, food, etc. (30 minutes before or after )
-coats the stomach

==================================================

Question: ondansetron (*Upper GI Tract)

Answer: Antiemetic
decreases the stimulation causing nausea and vomiting
for post-operative and chemo-related nausea and vomiting

==================================================

Question: omeprazole (*Upper GI Tract)

Answer: proton pump inhibitor; suffix "prazole"
suppresses acid production in the stomach
used for GERD, and peptic ulcers
-common for prophylactic prevention of aspiration pneumonia
*NG tube, tracheostomy, intubated patient
-May lead to osteoporosis

==================================================

Question: ranitidine (*Upper GI Tract)

Answer: H2-receptor antagonist; suffix "tidine"
suppresses acid production in the stomach
used for GI ulcers, GERD, and esophagitis

==================================================

Question: metoclopramide (*Upper GI Tract)

Answer: prokinetic agent
increases GI peristalsis
used for gastroparesis, nausea/vomiting, post surgery or chemotherapy
-commonly used post-op or prevent constipation (opioids)

==================================================

Question: pancrelipase, amylase (*Upper GI Tract)

Answer: digestive enzymes
breaks down food into nutrient components for absorption
used for enzyme replacement for cystic fibrosis or pancreatic insufficiency
-do not crush or chew tablets
-take with meals (no need if not eating)

==================================================

Question: ursoidol (*Upper GI Tract)

Answer: breaks down cholesterol-formed gallstones
used for cholesterol gallstones

==================================================

Question: simethicone (*Lower GI Tract)

Answer: Antiflatulent
Decreases gas production
Used for pain and discomfort from GI upset
-Take after meals

==================================================

Question: loperamide (*Lower GI Tract)

Answer: Antidiarrheal
Suppresses GI peristalsis
Used for diarrhea
-Risk for abuse

==================================================

Question: magnesium hydroxide, lactulose (*Lower GI Tract)

Answer: Laxatives
Softens stool, induces bowel movements
Used for constipation, and hepatic encephalopathy
-Lactulose used to discard ammonia through stool
*Assess for constipation first (it may not work)
-Castor oil contraindicated in pregnancy
*May lead to preterm labor

==================================================

Question: sulfasalazine (*Lower GI Tract)

Answer: Aminosalicylate
Decreases acid and inflammation in colon
Used for inflammatory bowel disease (IBD)
-Contains salicylate (aspirin)
*avoid in children (reye's syndrome)
*bleeding risk

==================================================

Nclex Rn Practice Questions Exam Cram

Question: A client with a renal failure is prescribed a low potassium diet. Which food choice would be best for this client?

A. 1 cup beef broth
B. 1 baked potato
C. 1/2 cup raisins
D. 1 cup rice

Answer: D (1 cup of rice)
( Answer D is correct because one cup of rice is considered a low-potassium food. The foods in answer A, B, and C are incorrect because they contain higher amounts of potassium)

==================================================

Question: An appropriate nursing intervention for the client with borderline personality disorder is:

A. Observing the client for signs of depression or suicidal thinking
B. Allowing the client to lead unit group sessions
C. Restricting the client's activity to the assigned unit of care throughout hospitalization
D. Allowing the client to select a primary caregiver

Answer: A (observing the client for signs of depression or suicidal thinking)
(Clients with borderline personality frequently suffer from depression and suicidal thinking and should be assessed for risk of self-injury. Answers B and D are incorrect choices because they allow the client too much control of the therapeutic environment. Answer C is incorrect because the client's activities do not have to be restricted to the unit after the level of depression has been determined )

==================================================

Question: Which of the following is an expected finding in the assessment of a client with bulimia nervosa

A. Extreme weight loss
B. Presence of lanugo over body
C. Erosion of tooth enamel
D. Muscle wasting

Answer: C (Erosion of tooth enamel)
(Erosion of tooth enamel caused by frequent self-induced vomiting is an expected finding in a client with bulimia nervosa. Answers A, B, and D are expected findings in the client with anorexia nervosa; therefore, they are incorrect.)

==================================================

Question: Assuming that all have achieved normal cognitive and emotional development, which of the following children is at greatest risk for accidental poisoning?

A. One-year-old
B. Four-year-old
C. Eight-year-old
D. Twelve-year-old

Answer: B (Four-year-old)
(Because of their increased mobility, manual dexterity and curiosity, the four year old is at greater risk for accidental poisoning. Other accidental injuries in this age group include being struck by a car, falls, burns, and drowning. Answer A is incorrect because the one-year-old lacks the developmental skill to be at risk for accidental poisoning. Answers C and D are incorrect because the eight-year-old and the twelve-year-old are at less risk because they are aware of the dangers of accidental poisoning)

==================================================

Question: Which term describes the play activity of the preschool aged child?

A. Cooperative
B. Associative
C. Parallel
D. Solitary

Answer: B (Associative)
(Play of the preschool aged child is described as associative. At this stage, children are more interested in playing with other children than they are with playing with toys. The child may talk to other children and exchange toys or play games without any rules. Answer A describes the play of a school-aged child. Answer C describes the play of an infant.)

==================================================

Question: The nurse is ready to begin an exam on a nine-month-old infant who is sitting quietly on his mother's lap. Which should the nurse do first?

A. Check the Babinski reflex
B. Listen to the heart and lung sounds
C. Palpate the abdomen
D. Check tympanic membranes

Answer: B (Listen to the heart and lung sounds)
(While the infant is quiet, the nurse should begin the exam by listening to the heart and lungs. If the nurse elicits the Babinski reflex , palpates the abdomen, or checks the tympanic membranes, the infant may cry and it will be difficult to adequately listen to the heart and lungs; therefore answers A,C, and D are incorrect.)

==================================================

Question: In terms of cognitive development, a three-year-old would be expected to:

A. Think abstractly
B. Use magical thinking
C. Understand conservation of matter
D. See things from the perspective of others

Answer: B (Use magical thinking)
(A three-year-old is expected to use magical thinking, such as believing that a toy bear is a real bear. Answers A, C, and D are incorrect because of abstract thinking, conservation of matter, and the ability to look at things from the perspective of others are cognitive abilities of an older child)

==================================================

Question: Which of the following describes the language development of a two-year-old?

A. Doesn't understand yes and no
B. Understands the meaning of all words
C. Can combine three or four words
D. Repeatedly asks "why?"

Answer: C (can combine three or four words)
(The two year old can combine three to four words. Answers A and B are incorrect because the two-year-old understands yes and no, but does not understand the meaning of all the words. Answer D is incorrect because seeking information and asking "why?" is typical of the three-year old)

==================================================

Question: A client who has been receiving Urokinase (uPA) for deep vein thrombosis is noted to have dark brown urine in the urine collection bag. Which action should the nurse take immediately?

A. Prepare an injection of vitamin K
B. Irrigate the urinary catheter with 50 mL of normal saline
C. Offer the client additional oral fluids
D. Withhold the medication and notify the physician

Answer: D (Withhold the medication and notify the physician)
(Urokinase is a thrombolytic agent used in the treatment of deep vein thrombosis, pulmonary embolus, or myocardial infarction. The presence of dark brown or rust-colored urine suggests bleeding. The nurse should withhold the medication, call the doctor immediately, and prepare to administer Amicar. Answer A is correct because vitamin K is not the antidote for urokinase. Answers B and C are incorrect because they do not address the adverse problem of bleeding)

==================================================

Question: Which of the following can occur with the frequent use of calcium based antacids?

A. Constipation
B. Hyperperistalsis
C. Delayed gastric emptying
D. Diarrhea

Answer: A (Constipation)
(The client taking calcium-based antacids will frequently develop constipation. Answers B, C, and D are not associated with the use of calcium-based antacids; therefore, they are incorrect.)

==================================================

Question: Which statement made by the student nurse indicates the need for further teaching regarding the administration of heparin?

A. "I will administer the medication 1-2 inches away from the umbilicus."
B. "I will not massage the injection site after administering the heparin."
C. "I will check the PTT before administering the heparin."
D. "I will need to gently aspirate when I give the heparin."

Answer: D ("I will need to gently aspirate when I give the heparin.")
(The nurse should not aspirate when giving heparin; therefore, answer D indicates a need for further teaching regarding heparin administration. Answers A, B, and C indicate the student nurse understands the the correct administration of heparin and are, therefore, incorrect answers. )

==================================================

Question: To correctly assess the oxygen saturation level of an adult client, the pulse oximeter should not be placed on the:

A. Finger
B. Earlobe
C. Extremity with noninvasive BP cuff
D. Nose

Answer: C (Extremity with noninvasive BP cuff)
(To obtain a correct oxygen saturation reading using pulse oximetry, the probe should not be placed on the arm with a noninvasive BP cuff or intraarterial line. Suitable sites are the finger, earlobe, or nose; therefore, Answers A, B, and D are incorrect.)

==================================================

Question: While caring for an elderly patient with hypertension, the nurse notes the following vital signs: BP of 140/40, pulse 129, respirations 36. The nurse's initial action should be to:

A. Report the findings to the physician
B. Recheck the vital signs in one hour
C. Ask the patient if he is in pain
D. Compare the current vital signs with those on admission

Answer: A (Report the findings to the physician)
(The client is exhibiting a widened pulse pressure, tachycardia, tachypnea. The first nursing action after obtaining these vital signs is to notify the physician for additional orders. Answers B, C, and D can be done after the physician is notified; therefore, they are incorrect choices as a first action.)

==================================================

Question: The nurse is preparing s client with an axillopopliteal bypass graft for discharge. The client should be taught to avoid:

A. Using a recliner to rest
B. Resting in supine position
C. Sitting in a straight chair
D. Sleeping in right Sim's position

Answer: C (Sitting in a straight chair)
(The client with the axillo-popliteal graft should avoid activities that can occlude the femoral artery graft. Sitting in the straight chair and wearing tight clothes are prohibited for this reason. Answers A, B, and D are incorrect because resting in a supine position, resting in a recliner, and sleeping in the right Sim's position are allowed.)

==================================================

Question: The doctor has ordered antithrombotic stockings to be applied to the legs of a client with peripheral vascular disease. The nurse knows the antithrombotic stockings should be applied:

A. Before the client arises in the morning
B. With the client in a standing position
C. After the client has bathed and applied lotion to the legs
D. Before the client retires in the evening

Answer: A (Before the client arises in the morning)
(The best time to apply antithrombotic stockings to the client is in the morning before the client arises. If the physician orders them later in the day, the client should return to bed, wait 30 minutes, and apply the stockings. Answers B, C, and D are incorrect because there is likely )

==================================================

Question: The nurse has just received the change of shift report and is preparing to make rounds. Which client should the nurse assess first?

A. A client recovering from a stroke with an oxygen saturation rate of 99%
B. A client three days port-coronary artery bypass graft with an oral temperature of 100.2 degrees Fahrenheit
C. A client admitted one hour ago with rales and shortness of breath
D. A client being prepared for discharge following a right colectomy

Answer: C (A client admitted one hour ago with rales and shortness of breath)

==================================================

Question: A client with a femoral popliteal bypass graft is assigned to a semiprivate room. The most suitable roommate for this client is the client with:

A. Hypothyroidism
B. Diabetic ulcers
C. Gastroenteritis
D. Bacterial pneumonia

Answer: A (Hypothyroidism)

==================================================

Question: The nurse is teaching the client regarding use of sodium warfarin. Which statement made by the client would require further teaching?

A. "I will have blood drawn every month."
B. "I will assess my skin for rash."
C. "I take aspirin for a headache."
D. "I will use an electric razor to shave."

Answer: C ("I take aspirin for a headache.")

==================================================

Question: The client returns to the recovery room following repair of an abdominal aneurysm. Which finding would require further investigation?

A. Pedal pulses regular
B. Urinary output 20mL in the past hour
C. Blood pressure 108/50
D. Oxygen saturation 97%

Answer: B (Urinary output 20mL in the past hour)

==================================================

Question: The nurse is doing bowel and bladder retraining for the client with paraplegia. Which of the following is not a factor for the nurse to consider?

A. Diet pattern
B. Mobility
C. Fluid intake
D. Sexual function

Answer: D (Sexual function)

==================================================

Nclex Uworld Practice Questions

Question: 2. Increasing temperature, decreasing pulse, decreasing respirations, increasing blood pressure

A change in vital signs may be a late sign of increased intracranial pressure. Trends include increasing temperature and blood pressure and decreasing pulse and respirations. Respiratory irregularities also may occur.

Answer: The nurse is caring for the client with increased intracranial pressure. The nurse would note which trend in vital signs if the intracranial pressure is rising?

1. Increasing temperature, increasing pulse, increasing respirations, decreasing blood pressure
2. Increasing temperature, decreasing pulse, decreasing respirations, decreasing blood pressure
3. Decreasing temperature, decreasing pulse, increasing respirations, decreasing blood pressure
4. Decreasing temperature, increasing pulse, decreasing respirations, increasing blood pressure

==================================================

Question: 2, 3, 4
The nurse should maintain ICP by elevating the head of the bed 15 - 20 degrees and monitoring neurologic status. An ICP >15 mmHg with 20 to 25 mmHg as upper limits of normal indicates increased ICP, and the nurse should notify the HCP. Coughing and range of motion exercises will increase ICP and should be avoided in the early postoperative stage.

Answer: The nurse has established a goal to maintain intracranial pressure (ICP) within the normal range for a client who had a craniotomy 12 hours ago. What should the nurse do? Select all that apply.

1. Encourage the client to cough to expectorate secretions.
2. Elevate the head of the bed 15 - 20 degrees.
3. Contact the HCP if ICP is >15 mmHg.
4. Monitor neurologic status using the Glasgow Coma Scale.
5. Stimulate the client with active range-of-motion exercises.

==================================================

Question: 3
The clear drainage must be analyzed to determine whether it is nasal drainage or CSF. The nurse should not give the client tissues because it is important to know how much leakage of CSF is occurring. Compressing the nares will obstruct the drainage flow. It is inappropriate to tilt the head back, which would allow the fluid to drain down the throat and not be collected for a sample. It is inappropriate to administer an antihistamine because the drainage may not be from postnasal drip.

Answer: What should the nurse do first when a client with a head injury begins to have clear drainage from the nose?

1. Compress the nares
2. Tilt the head back
3. Collect the drainage
4. Administer an antihistamine for postnasal drip

==================================================

Question: 4
Leakage of cerebrospinal fluid from the ears or nose may accompany basilar skull fractures. CSF can be distinguished from other body fluids because the drainage will separate into bloody and yellow concentric rings on dressing material, called a halo sign. The fluid also tests positive for glucose.

Answer: A client has clear fluid leaking from the nose following a basilar skull fracture. Which finding would alert the nurse that cerebrospinal fluid is present?

1. Fluid is clear and tests negative for glucose.
2. Fluid is grossly blood in appearance and has a pH of 6
3. Fluid clumps together on the dressing and had a pH of 7
Fluid separates into concentric rings and tests positive for glucose.

==================================================

Question: 1, 2, 4
Hemiparesis is a weakness of one side of the body that may occur after a stroke. It involves weakness of the face and tongue, arm, and leg on one side. These clients are also aphasic: unable to discriminate words and letters. They are generally very cautions and get anxious when attempting a new task. Complete bilateral paralysis does not occur in hemiparesis. The client with right-sided hemiparesis has weakness of the right arm and leg and needs assistance with feeding, bathing, and ambulating.

Answer: The nurse is assigned to care for a client with complete right-sided hemiparesis from a stroke. Which characteristics are associated with this condition? Select all that apply.

1. The client is aphasic.
2. The client has weakness on the right side of the body.
3. The client has complete bilateral paralysis of the arms and legs.
4. The client has weakness on the right side of the face and tongue.
5. The client has lost the ability to move the right arm but is able to walk independently
6. The client has lost the ability to ambulate independently, but is able to feed and bathe himself or herself without assistance.

==================================================

Question: 4
Homonymous hemianopsia is loss of half of the visual field. The client with homonymous hemianopsia should have objects placed in the intact field of vision, and the nurse also should approach the client from the intact side. The nurse instructs the client to scan the environment to overcome the visual deficit and does client teaching from within the intact field of vision. The nurse encourages the use of personal eyeglasses, if they are available.

Answer: The nurse has instructed the family of a client with stroke who has homonymous hemianopsia about measures to help the client overcome the deficit. Which statement suggests that the family understand the measures to use when caring for the client?

1. We need to discourage him from wearing eyeglasses.
2. We need to place objects in his impaired field of vision.
3. We need to approach him from the impaired field of vision.
4. We need to remind him to turn his head to scan the lost visual field.

==================================================

Question: 3
Thrombolytic enzyme agents are used for clients with a thrombotic stroke to dissolve emboli, this reastablishing cerebral perfusion. They do not increase vascular permeability, cause vasoconstriction, or prevent further hemorrhage.

Answer: What is the expected outcome of thrombolytic drug therapy for stroke?

1. Increased vascular permeability
2. Vasoconstriction
3. Dissolved emboli
4. Prevention of hemorrhage

==================================================

Question: 3, 1, 4, 2
To protect the client from falling, the nurse first should ease the client to the floor. It is important to protect the head and maintain a patent airway since altered breathing and excessive salivation can occur. The assessment of the postictal period should include level of consciousness and vital signs. The nurse should record details of the seizure once the client is stable. The events preceding the seizure, timing with descriptions of each phase, body parts affected and sequence of involvement, and autonomic signs should be recorded.

Answer: The nurse sees a client walking in the hallway who begins to have a seizure. What should the nurse do in order of priority from first to last? All options must be used.
1. Maintain a patent airway.
2. Record the seizure activity observed.
3. Ease the client to the floor.
4. Obtain vital signs.

==================================================

Question: 1, 2, 5, 6
Seizure precautions may vary from agency to agency, but they generally have some common features. Usually, an airway, oxygen, and suctioning equipment are kept available at the bedside. The side rails of the bed are padded, and the bed is kept in the lowest position. The client has an intravenous access in place to have a readily accessible route if antiseizure medications must be administered, and as part of the routine assessment the nurse should be checking the patency of the catheter. The use of padded tongue blades is highly controversial, and they should not be kept at the bedside. Forcing a tongue blade into the mouth during a seizure more likely will harm the client who bites down during seizure activity. Risks include blocking the airway from improper placement, chipping the client's teeth, and subsequent risk of aspirating tooth fragments. If the client has an aura before the seizure, it may give the nurse enough time to place an oral airway before seizure activity begins.

Answer: The nurse is instituting seizure precautions for a client who is being admitted from the emergency department. Which measures should the nurse include in planning for the client's safety? Select all that apply.

1. Padding the side rails of the bed.
2. Placing an airway at the bedside.
3. Placing the bed in the high position
4. Putting a padded tongue blade at the head of the bed
5. Placing oxygen and suction equipment at the bedside
6. Flushing the intravenous catheter to ensure that the site is patent.

==================================================

Question: 3
After a supratentorial surgery, the head is kept at a 30-45 degree angle. The head and neck should not be angled either anteriorly or laterally but rather should be kept in a neutral (midline) position. This promotes venous return through the jugular veins, which will help prevent a rise in intracranial pressure.

Answer: The nurse is caring for a client who has undergone a craniotomy and has a supratentorial incision. The nurse should place the client in which position postoperatively?

1. Head of bed flat, head and neck midline.
2. Head of bed flat, head turned to the nonoperative side
3. Head of bed elevated 30 to 45 degrees, head and neck midline
4. Head of bed elevated 30 to 45 degrees, head turned to the operative side

==================================================

Question: 3. Place the client on droplet precautions.

The client with suspected bacterial meningitis should be placed on droplet precaution isolation until the causative agent has been identified and appropriate treatment is initiated. Meningococcal meningitis and Haemophilus influenzae type B meningitis are highly transmissible to others, and the client must remain on droplet isolation until these can be ruled out. Precautions can usually be discontinued 24 hours after beginning antibiotic therapy. Viral meningitis and other types of bacterial meningitis (ie, other than meningococcal meningitis) usually do not require droplet precautions.

(Option 1) Although assessment is a priority and meningeal signs should be checked, the nurse can only safely perform these assessments once droplet precautions are in place.

(Options 2 and 4) A peripheral IV catheter should be inserted to provide fluids. Subsequently, preparation for lumbar puncture is needed. However, placing the client on isolation is a priority to protect the nurse and other clients and care providers.

Educational objective:
The client with suspected bacterial meningitis should be placed on droplet precaution isolation until the causative agent has been identified and appropriate treatment is initiated.

Answer: A nurse is evaluating an acutely ill client with suspected meningitis. The nurse should take what action first?

1. Check for Kernig's and Brudzinski's signs
2. Establish IV access
3. Place the client on droplet precautions
4. Prepare the client for lumbar puncture

==================================================

Question: 3
Clear, colorless fluid draining from the nose or ears after head trauma is suspicious for cerebrospinal fluid (CSF) leakage (Option 3). When the drainage is clear, dextrose testing can be used to determine if the drainage is CSF. However, the presence of blood would make this test unreliable as blood also contains glucose.

This child is at risk for intracerebral bleeding and meningitis. Vascular compromise may occur with even minimal head trauma; therefore, the nurse should evaluate any changes in level of consciousness and temperature as well as assess the head and neck for subcutaneous bleeding. The nurse should anticipate a CT scan of the head and neck and prophylactic antibiotics.

(Option 1) Iron ingestion is the major concern with vitamin toxicity in children. However, children's formulations contain minimal or no iron. As a result, ingestion of an unknown quantity is unlikely to cause serious toxicity. This child should be seen second.

(Option 2) A low-grade fever is common after immunizations; this child can be seen last.

(Option 4) Although infection and pain are important, this child can be seen third.

Educational objective:
The child with head trauma who is leaking cerebrospinal fluid (CSF) is at risk for meningitis and intracerebral bleeding. If the drainage is clear, a dextrose test is used to determine if the drainage is CSF. The nurse should assess for signs of bleeding (eg, change in level of consciousness) and infection (eg, increased temperature) and anticipate a CT scan and prophylactic antibiotics.

Answer: Four children are brought to the emergency department. Which child should be assessed first?

1. A 13-month-old who ingested an unknown quantity of children's multivitamins
2. A 15-month-old with a fever of 100.5 F (38.1 C) after being vaccinated
3. A 3-year-old with a forehead laceration and colorless nasal drainage
4. A 4-year-old with enlarged tonsillar lymph nodes who is crying in pain

==================================================

Question: 4
A ruptured cerebral aneurysm is a surgical emergency with a high mortality rate. Cerebral aneurysms are usually asymptomatic unless they rupture; they are often called "silent killers" as they may go undetected for many years before rupturing without warning signs. The distinctive description of a cerebral aneurysm rupture is the abrupt onset of "the worst headache of my life" that is different from previous headaches (including migraines). Immediate evaluation for a possible ruptured aneurysm is critical for any client experiencing a severe headache with changes in or loss of consciousness, neurologic deficits, diplopia, seizures, vomiting, or a stiff neck. Early identification and prompt surgical intervention help increase the chance for survival.

(Options 1, 2, and 3) A change in level of consciousness, increased blood pressure, or a feeling of illness should be investigated but alone does not indicate an emergency.

Educational objective:
Sudden onset of a severe headache described as "the worst headache of my life" is characteristic of a ruptured cerebral aneurysm and should be treated as an emergency.

Answer: A client comes to the emergency department with diplopia and recent onset of nausea. Which statement by the client would indicate to the nurse that this is an emergency?

1. "I am very tired, and it's hard for me to keep my eyes open."
2. "I don't feel good, and I want to be seen."
3. "I have not taken my blood pressure medicine in over a week."
4. "I have the worst headache I've ever had in my life."

==================================================

Question: 1, 3, 5, 6
The Valsalva maneuver (straining during defecation) involves holding the breath while bearing down on the perineum to pass a stool. Straining to have a bowel movement is to be avoided in clients recently diagnosed with increased intracranial pressure, stroke, or head injury as straining increases intra-abdominal and intrathoracic pressure, which raises the intracranial pressure (Option 1).

The vagus nerve is stimulated when bearing down; this temporarily slows the heart and decreases cardiac output, leading to potential cardiac complications in clients with heart disease (Option 3).

Straining increases intra-abdominal and intrathoracic pressure and should be avoided in clients diagnosed with portal hypertension related to cirrhosis due to the risk of variceal bleeding (Option 5).

The maneuver increases intraocular pressure and is contraindicated in clients with glaucoma and recent eye surgery (Option 6).

(Option 2) The otherwise healthy client recovering from reconstructive augmentation mammoplasty is not at risk for complications related to the Valsalva maneuver.

(Option 4) The client recently diagnosed with pancreatic cancer is not at risk for complications related to the Valsalva maneuver.

Educational objective:
The Valsalva maneuver is contraindicated in the client diagnosed with increased intracranial pressure, stroke, head injury, heart disease, glaucoma, eye surgery, abdominal surgery, and liver cirrhosis.

Answer: The clinic nurse educator is developing a teaching plan for the following 6 clients. The nurse should instruct which client to avoid the Valsalva maneuver when defecating? Select all that apply.

1. 22-year-old man with a head injury sustained during a college football game
2. 30-year-old woman recently hospitalized for reconstructive augmentation mammoplasty
3. 56-year-old man 2 weeks post myocardial infarction
4. 68-year-old woman recently diagnosed with pancreatic cancer
5. 74-year-old man with portal hypertension related to alcohol-induced cirrhosis
6. 82-year-old woman 1 week post cataract surgery

==================================================

Question: 3
Unexpected and projectile vomiting without nausea can be a sign of increased ICP, especially in the client with a history of increased ICP. The unexpected vomiting is related to pressure changes in the cranium. The vomiting can be associated with headache and gets worse with lowered head position. The most appropriate action is to obtain a full set of vital signs and contact the HCP immediately.

(Option 1) Documentation is important, but it is not the priority action.

(Option 2) The head of the bed should be raised, not lowered, for clients with suspected increased ICP. Raising the head of the bed to 30 degrees helps to drain the cerebrospinal fluid via the valve system without lowering the cerebral blood pressure.

(Option 4) The vomiting is caused not by nausea but by pressure changes in the cranium. Anti-nausea medications are often not effective. Decreasing intracranial pressure will help the vomiting.

Educational objective:
Notify the HCP of signs/symptoms of increased ICP, including unexpected vomiting. The vomiting is often projectile, associated with headache, and gets worse with lowering the head position.

Answer: A client is admitted to the hospital for severe headaches. The client has a history of increased intracranial pressure (ICP), which has required lumbar punctures to relieve the pressure by draining cerebrospinal fluid. The client suddenly vomits and states, "That's weird, I didn't even feel nauseated." Which action by the nurse is the most appropriate?

1. Document the amount of emesis
2. Lower the head of the bed
3. Notify the health care provider (HCP)
4. Offer anti-nausea medication

==================================================

Question: 2, 3, 5
A concussion is considered a minor traumatic brain injury and results from blunt force or an acceleration/deceleration head injury. Typical signs of concussion include:

A brief disruption in level of consciousness
Amnesia regarding the event (retrograde amnesia)
Headache
These clients should be observed closely by family members and not participate in strenuous or athletic activities for 1-2 days. Rest and a light diet are encouraged during this time.

(Options 1 and 4) The following manifestations indicate more serious brain injury and are not expected with simple concussion:

Worsening headaches and vomiting (indicate high intracranial pressure)
Sleepiness and/or confusion (indicate high intracranial pressure)
Visual changes
Weakness or numbness of part of the body
Educational objective:
Expected neurological changes with a concussion include brief loss of consciousness, retrograde amnesia, and headache. These clients should be observed closely by family members and not participate in strenuous or athletic activities for 1-2 days.

Answer: The nurse is caring for a client after a motor vehicle accident. The client's injuries include 2 fractured ribs and a concussion. The nurse notes which of the following as expected neurological changes for the client with a concussion? Select all that apply.

1. Asymmetrical pupillary constriction
2. Brief loss of consciousness
3. Headache
4. Loss of vision
5. Retrograde amnesia

==================================================

Question: 2
Meningitis is an inflammation of the meninges covering the brain and spinal cord. The key clinical manifestations of bacterial meningitis include fever, severe headache, nausea/vomiting, and nuchal rigidity. Other symptoms include photophobia, altered mental status, and other signs of increased intracranial pressure (ICP).

In a hypotensive client with sepsis, the priority of care is fluid resuscitation to increase the client's blood pressure (Option 2).

In addition to IV fluid administration, interventions and prescriptions for a client with sepsis and meningitis may include:

Administer vasopressors.
Obtain relevant labs and blood cultures prior to administering antibiotics.
Administer empiric antibiotics, preferably within 30 minutes of admission (Option 1). This client will continue to decline without antibiotic therapy.
Prior to a lumbar puncture (LP), obtain a head CT scan as increased ICP or mass lesions may contraindicate a LP due to the risk of brain herniation (Option 4).
Assist with a LP for cerebrospinal fluid (CSF) examination and cultures (Option 3). CSF is usually purulent and turbid in clients with bacterial meningitis. CSF cultures will allow for targeted antibiotic therapy.
Educational objective:
For bacterial meningitis with sepsis, fluid resuscitation is the priority. Blood cultures should be drawn before starting antibiotics. After a head CT scan is performed to rule out increased intracranial pressure and mass lesions, cerebrospinal fluid cultures should be drawn via lumbar puncture.

Answer: An adult client with altered mental status and fever has suspected bacterial meningitis with sepsis. Blood pressure is 80/60 mm Hg. Which prescribed intervention should the nurse implement first?

1. Administer IV antibiotics
2. Infuse bolus of IV normal saline
3. Prepare to assist with lumbar puncture
4. Transport client for head CT scan

==================================================

Question: 2
Levetiracetam (Keppra) is a medication often used to treat seizures in various settings. It has minimal drug-drug interactions compared to phenytoin and is often the preferred antiepileptic medication.

(Option 1) Chemotherapy and radiation therapy would kill tumor cells and reduce tumor size.

(Option 3) Hospitalized clients and clients with malignancy are at higher risk for venous thromboembolism. These clients would benefit from anticoagulation (eg, heparin, enoxaparin, rivaroxaban, apixaban).

(Option 4) Dexamethasone, a corticosteroid, is used to treat cerebral edema associated with a brain injury/tumor by decreasing inflammation.

Educational objective:
Levetiracetam (Keppra) is a medication often used to treat seizures in various settings. Corticosteroids are used to reduce inflammation and cerebral edema in clients with brain injury and tumors.

Answer: A client with a brain tumor is admitted for surgery. The health care provider prescribes levetiracetam. The client asks why. What is the nurse's response?

1. "It destroys tumor cells and helps shrink the tumor."
2. "It prevents seizure development."
3. "It prevents blood clots in legs."
4. "It reduces swelling around the tumor."

==================================================

Question: Cushing's triad is related to increased intracranial pressure (ICP). Early signs include change in level of consciousness. Later signs include bradycardia, increased systolic blood pressure with a widening pulse pressure (difference between systolic and diastolic), and slowed irregular (Cheyne-Stokes) respirations. Cushing's triad is a later sign that does not appear until the ICP is increased for some time. It indicates brain stem compression.

The skull cannot expand after the fontanels close at age 18 months, so anything taking up more space inside the skull (eg, hematoma, tumor, swelling, etc.) is a concern for causing pressure on the brain tissue/brain stem and potential herniation. In this scenario, hidden head trauma causing an intracranial bleed must be ruled out with diagnostic testing. The client's intoxication could blunt an accurate history or presentation of a head injury.

(Option 1) Atropine is used to stimulate the sinoatrial node in bradycardia with systemic symptoms. An electrocardiogram (ECG) should be obtained prior to administering atropine. In this client, there is no evidence of a cardiac etiology or systemic symptoms of poor perfusion/circulation from the bradycardia.

(Option 2) Nifedipine (Procardia) is a calcium channel blocker that is a potent vasodilator. However, all components must be considered in this scenario as to the etiology of the hypertension rather than just treating that sign. Ruling out a cerebral cause of the hypertension is most important.

(Option 4) The nurse would continue to perform neurologic assessments (including GCS). However, it is more important to obtain appropriate diagnostic tests and initiate treatment for the changing neurologic symptoms than to just monitor and document. In addition, the nurse should be performing these assessments more frequently than hourly in this rapidly changing client.

Educational objective:
Cushing's triad/reflex indicates increased intercerebral pressure. Classic signs include bradycardia, rising systolic blood pressure, widening pulse pressure, and irregular respirations (such as Cheyne-Stokes).

Answer: A highly intoxicated client was brought to the emergency department after found lying on the sidewalk. On admission, the client is awake with a pulse of 70/min and blood pressure of 160/80 mm Hg. An hour later, the client is lethargic, pulse is 48/min, and blood pressure is 200/80 mm Hg. Which action does the nurse anticipate taking next?

1. Administer atropine for bradycardia
2. Administer nifedipine for hypertension
3. Have CT scan performed to rule out an intracranial bleed
4. Perform hourly neurologic checks with Glasgow coma scale (GCS)

==================================================

Question: A client with a neurological injury (eg, head trauma, stroke) is at risk for cerebral edema and increased intracranial pressure (ICP), a life-threatening situation. The client with atrial fibrillation may also be taking anticoagulants (eg, warfarin, rivaroxaban, apixaban, dabigatran), making a life-threatening intracranial bleed even more dangerous. The nurse should perform a neurologic assessment (eg, level of consciousness, pupil response, vital signs) immediately.

(Option 1) Autonomic dysreflexia (eg, throbbing headache, flushing, hypertension) is a life-threatening condition caused by sensory stimulation that occurs in clients who have a spinal cord injury at T6 or higher. This is not the priority assessment as this client's injury is at L3. This client likely has acute urinary retention and needs catheterization.

(Option 2) Phenytoin toxicity commonly presents with neurologic manifestations such as gait disturbance, slurred speech, and nystagmus. These are expected symptoms and therefore are not a priority.

(Option 3) A brain tumor can also cause increased intracranial pressure; clients report morning headache, nausea, and vomiting. Dexamethasone (Decadron) can be prescribed short-term to decrease the surrounding edema. A tumor usually grows more slowly than a possible hematoma and is therefore not the priority assessment.

Educational objective:
Constant headache, decreased mental status, and sudden-onset emesis indicate increased intracranial pressure.

Answer: The nurse receives report for 4 clients in the emergency department. Which client should be seen first?

1. 30-year-old with a spinal cord injury at L3 sustained in a motor cycle accident who reports lower abdominal pain and difficulty urinating
2. 33-year-old with a seizure disorder admitted with phenytoin toxicity who reports slurred speech and unsteady gait
3. 65-year-old with suspected brain tumor waiting to be admitted for biopsy who reports throbbing headache and had emesis of 250 mL
4. 70-year-old with atrial fibrillation and a closed-head injury waiting for brain imaging who reports a headache and had emesis of 200 mL

==================================================

Nclex Vital Signs

Question: 1. A nurse assesses an oral temperature for an adult patient. The patient's temperature is 37.5°C (99.5°F). What term would the nurse use to report this temperature?

a. Febrile

b. Hypothermia

c. Hypertension

d. Afebrile

Answer: d. Afebrile

==================================================

Question: 2. A nurse is assessing the vital signs of patients who presented at the emergency department. Based on the knowledge of age-related variations in normal vital signs, which patients would the nurse document as having a normal vital sign? Select all that apply.

a. A 4-month old infant whose temperature is 38.1°C (100.5°F)

b. A 3-year old whose blood pressure is 118/80

c. A 9-year old whose temperature is 39°C (102.2°F)

d. An adolescent whose pulse rate is 70 bpm

e. An adult whose respiratory rate is 20 bpm

f. A 72-year old whose pulse rate is 42 bpm

Answer: a. A 4-month old infant whose temperature is 38.1°C (100.5°F)

d. An adolescent whose pulse rate is 70 bpm

e. An adult whose respiratory rate is 20 bpm

f. A 72-year old whose pulse rate is 42 bpm

==================================================

Question: 3. A patient who is febrile may lose body heat through perspiration. The nurse recognizes that this is an example of what mechanism of heat loss?

a. Evaporation

b. Convection

c. Radiation

d. Conduction

Answer: a. Evaporation

==================================================

Question: 4. The rectal temperature, a core temperature, is considered to be one of the most accurate routes. In which cases would taking a rectal temperature be contraindicated? Select all that apply.

a. A newborn who has hypothermia

b. A child who has pneumonia

c. An older patient who is post myocardial infarction (heart attack)

d. A teenager who has leukemia

e. A patient receiving erythropoietin to replace red blood cells

f. An adult patient who is newly diagnosed with pancreatitis

Answer: a. A newborn who has hypothermia

c. An older patient who is post myocardial infarction (heart attack)

d. A teenager who has leukemia

e. A patient receiving erythropoietin to replace red blood cells

==================================================

Question: 5. While taking an adult patient's pulse, a nurse finds the rate to be 140 beats/min. What should the nurse do next?

a. Check the pulse again in 2 hours.

b. Check the blood pressure.

c. Record the information.

d. Report the rate to the primary care provider.

Answer: d. Report the rate to the primary care provider.

==================================================

Question: 6. A patient complains of severe abdominal pain. When assessing the vital signs, the nurse would not be surprised to find what assessments? Select all that apply.

a. An increase in the pulse rate

b. A decrease in body temperature

c. A decrease in blood pressure

d. An increase in respiratory depth

e. An increase in respiratory rate

f. An increase in body temperature

Answer: a. An increase in the pulse rate

e. An increase in respiratory rate

==================================================

Question: 7. Two nurses are taking an apical-radial pulse and note a difference in pulse rate of 8 beats per minute. The nurse would document this difference as which of the following?

a. Pulse deficit

b. Pulse amplitude

c. Ventricular rhythm

d. Heart arrhythmia

Answer: a. Pulse deficit

==================================================

Question: 8. The nurse instructor is teaching student nurses about the factors that may affect a patient's blood pressure. Which statements accurately describe these factors? Select all that apply.

a. Blood pressure decreases with age.

b. Blood pressure is usually lowest on arising in the morning.

c. Women usually have lower blood pressure than men until menopause.

d. Blood pressure decreases after eating food.

e. Blood pressure tends to be lower in the prone or supine position.

f. Increased blood pressure is more prevalent in African Americans.

Answer: b. Blood pressure is usually lowest on arising in the morning.

c. Women usually have lower blood pressure than men until menopause.

e. Blood pressure tends to be lower in the prone or supine position.

f. Increased blood pressure is more prevalent in African Americans.

==================================================

Question: 9. A patient is having dyspnea. What would the nurse do first?

a. Remove pillows from under the head

b. Elevate the head of the bed

c. Elevate the foot of the bed

d. Take the blood pressure

Answer: b. Elevate the head of the bed

==================================================

Question: 10. A student nurse is learning to assess blood pressure. What does the blood pressure measure?

a. Flow of blood through the circulation

b. Force of blood against arterial walls

c. Force of blood against venous walls

d. Flow of blood through the heart

Answer: b. Force of blood against arterial walls

==================================================

Question: 11. Prioritization: Place the following descriptions of the phases of Korotkoff sounds in order from Phase I - Phase V.

a. Characterized by muffled or swishing sounds that may temporarily disappear; also known as the auscultatory gap

b. Characterized by distinct, loud sounds as the blood flows relatively freely through an increasingly open artery

c. The last sound heard before a period of continuous silence, known as the second diastolic pressure

d. Characterized by the first appearance of faint but clear tapping sounds that gradually increase in intensity; known as the systolic pressure

e. Characterized by a distinct, abrupt, muffling sound with a soft, blowing quality; considered to be the first diastolic pressure

Answer: d. Characterized by the first appearance of faint but clear tapping sounds that gradually increase in intensity; known as the systolic pressure

a. Characterized by muffled or swishing sounds that may temporarily disappear; also known as the auscultatory gap

b. Characterized by distinct, loud sounds as the blood flows relatively freely through an increasingly open artery

e. Characterized by a distinct, abrupt, muffling sound with a soft, blowing quality; considered to be the first diastolic pressure

c. The last sound heard before a period of continuous silence, known as the second diastolic pressure

==================================================

Question: 12. A patient has a blood pressure reading of 130/90 mm Hg when visiting a clinic. What would the nurse recommend to the patient?

a. Follow-up measurements of blood pressure

b. Immediate treatment by a physician

c. No action, because the nurse considers this reading is due to anxiety

d. A change in dietary intake

Answer: a. Follow-up measurements of blood pressure

==================================================

Question: 13. A nurse is documenting a blood pressure of 120/80 mm Hg. The nurse interprets the 120 to represent:

a. The rhythmic distention of the arterial walls as a result of increased pressure due to surges of blood with ventricular contraction

b. The lowest pressure present on arterial walls while the ventricles relax

c. The highest pressure present on arterial walls while the ventricles contract

d. The difference between the pressure on arterial walls with ventricular contraction and relaxation

Answer: c. The highest pressure present on arterial walls while the ventricles contract

==================================================

Question: 14. It is important to have the appropriate cuff size when taking the blood pressure. What error may result from a cuff that is too large or too small?

a. An incorrect reading

b. Injury to the patient

c. Prolonged pressure on the arm

d. Loss of Korotkoff sounds

Answer: a. An incorrect reading

==================================================

Question: 15. A patient has intravenous fluids infusing in the right arm. When taking a blood pressure on this patient, what would the nurse do in this situation?

a. Take the blood pressure in the right arm

b. Take the blood pressure in the left arm

c. Use the smallest possible cuff

d. Report inability to take the blood pressure

(Taylor 621-622)Taylor, Carol. Fundamentals of Nursing. Wolters Kluwer Health, 10/2014. VitalBook file.

Answer: b. Take the blood pressure in the left arm

==================================================

Nclhelp

Question: Latitudes

Answer: Must be 18 to enroll
How Guests Earn Points
Guests can earn points as follows:
One (1) point for every cruise night (ex: a 7 night cruise earns 7 rewards points)
One (1) additional point per night when booking nine or more months prior to sailing.
Note: system automatically calculates that the reservation has been booked for 9 months to the day.
One (1) additional point per night when booking a Latitudes Insider Offer (LATITUDE promo code must be on the reservation when guests sail for points to be awarded).
One (1) additional point per night for reserving a full fare Suite or in The Haven (not including Mini-Suites)*
*Note: Points for booking a suite are only applicable when the full fare suite is booked when the reservation is initially made. Suite Points do not apply to upgrades/upsells.

How many points are needed to reach each tier level? Tier Rewards Points
Bronze 1 - 19
Silver 20 - 47
Gold 48 - 75
Platinum 76+
Special milestone gifts are provided for guests who achieve 250, 500, 700 and 1000 points.

==================================================

Question: Allergies

Answer: We can accommodate guests with these dietary needs.
They should be advised to notify us of the allergy so that the
information can be sent to the ship.
Once on board the guest should meet with the Group Service
Coordinator/Access Officer or the Restaurant Manager to further
discus their dietary need.
Soy milk and Lactaid milk are available on board.
Guests may only bring non-perishable, pre-packaged, prepared
snacks to be stored in the guest's cabin (juice boxes, cookies,
crackers, etc.).
Guests can not bring pre-cooked food (in any stage: frozen, chilled,
etc.), food that needs to be cooked/prepared, processed or stored
by ship staff.
Action Required: add special request, then e-mail Access Desk.

==================================================

Question: Managing Calls for Guests with Disabilities

Answer: When a reservation is booked for a guest with medical needs, if necessary,
please send an e-mail to accessdesk@ncl.com with the following information:
The subject line MUST include the ship, sail date, reservation
number, and the guest's name.
The body of the e-mail MUST include as much detail about the
guests medical issue as possible as well as a contact number for the
guest.
Providing this level of detail will allow the Access Desk to handle these emails
as efficiently as possible.
The Access Desk will then contact the travel agent/guest.

==================================================

Question: Blind/Sight Impaired, Service Dogs

Answer: In accordance with the ADA, Norwegian Cruise Line DOES NOT accept
"Emotional Support" dogs as service dogs.

must book at least two weeks in
advance

==================================================

Question: Diabetic Guests

Answer: Guest must bring their own medication (insulin, needles/syringes and testing
supplies).
Guests can obtain a Sharp's container on board form the Guest Service

Only request a refrigerator if guest is taking insulin and sailing on Norwegian
Spirit (all other ship have refrigerator on the cabins).

Recommend (not mandatory) that guest bringing needles/syringes on board,
bring a doctor's note or prescription

==================================================

Question: Pregnancy

Answer: Norwegian will not accept guests who will have entered their 24th week of
pregnancy by the time their travel with Norwegian Cruise Line concludes.

==================================================

Question: Special Needs at Sea

Answer: 1-800-513-4515

==================================================

Question: Booking a Wheelchair Accessible Stateroom

Answer: 1. After a ship, sailing date and category have been determined, the Stateroom
availability will display. Click on
2. Check off the box next to Wheelchair Accessible in the available attributes
and click on
The system will display a list of staterooms designed for Wheelchair
guests.
3. Check off the desired stateroom and click Select.

==================================================

Ncoems Drug List

Question: Adenosine

Answer: 6/12/12mg RIVP

==================================================

Question: Albuterol

Answer: 2.5-5mg in 3cc x3

==================================================

Question: Amiodarone

Answer: VF/PVT: 300mg, rp@150
VTW/P: 150mg in 100cc D5W over 10min

==================================================

Question: Atropine

Answer: 0.5mg ev 3-5min up to 3mg

==================================================

Question: Calcium Chloride

Answer: 1g

==================================================

Question: Valium

Answer: 5mg, 2mg ev 3-5min up to 10mg

==================================================

Question: Cardizem

Answer: 20mg (1/2 dose if >60)

==================================================

Question: Dopamine

Answer: 2-20 mcg/kg/min, titr to BP 90 sys

==================================================

Question: Epinephrine 1:1,000

Answer: 0.3mg

==================================================

Question: Epinephrine 1:10,000

Answer: 1mg, rp ev 3-5min

==================================================

Question: Etomidate

Answer: 0.3mg/kg

==================================================

Question: Fentanyl

Answer: 50-75mcg, 25mcg ev 20min, max 200mcg

==================================================

Question: Lasix

Answer: 40mg

==================================================

Question: Glucagon

Answer: 1-2mg IM, rp in 15min if gc≥69

==================================================

Question: Haldol

Answer: 2-5mg IM

==================================================

Question: Atrovent

Answer: 0.5mg per neb

==================================================

Question: Toradol

Answer: 30mg IV or 60mg IM

==================================================

Question: Lidocaine

Answer: 1.5mg/kg

==================================================

Question: Magnesium Sulfate

Answer: 2g over 10min(Respiratory)
over 2min(Torsades)
over 2-3min(Eclampsia)

==================================================

Question: Solu-medrol

Answer: 125mg

==================================================

Question: Versed

Answer: (Seizure) 2-2.5mg ev 3-5mins to max 10mg, 5mg IM

==================================================

Question: Morphine

Answer: 4mg, 2mg ev 5-10min, max 10mg

==================================================

Question: Narcan

Answer: 0.4-2mg

==================================================

Question: Nitroglycerin

Answer: 0.4mg SL ev 5min

==================================================

Question: Zofran

Answer: 4mg, rp in 15min

==================================================

Question: Phenergan

Answer: 12.5mg, rp 1x as needed

==================================================

Question: Rocuronium

Answer: 1mg/kg

==================================================

Question: Sodium Bicarb

Answer: 50mEq

==================================================

Question: Succinylcholine

Answer: 1.5mg/kg

==================================================

Question: Vasopressin

Answer: 40 units

==================================================

Question: Vecuronium

Answer: 0.1mg/kg

==================================================

Question: Calcium Gluconate

Answer: 2g

==================================================